You are on page 1of 265

Curbside Consultation in Gastroenterology

Series
Series Editor, Francis A. Farraye, MD
Editors

Joseph Leung, MD, FRCP, FACP, FASGE, MACG


Mr. & Mrs. C.W. Law Professor of Medicine
University of California, Davis School of Medicine
Chief of Gastroenterology
Veterans Affairs Northern California Health Care System
Sacramento, California

Simon K. Lo, MD
Clinical Professor of Medicine, David Geffen School of Medicine
Director, Pancreatic and Biliary Diseases Program
Director, Gastrointestinal Endoscopy
Cedars-Sinai Medical Center
Los Angeles, California
www.Healio.com/books
Copyright © 2014 by SLACK Incorporated

All rights reserved. No part of this book may be reproduced, stored in a retrieval system or transmitted in any form or by any
means, electronic, mechanical, photocopying, recording or otherwise, without written permission from the publisher, except
for brief quotations embodied in critical articles and reviews.

The procedures and practices described in this publication should be implemented in a manner consistent with the profes-
sional standards set for the circumstances that apply in each specific situation. Every effort has been made to confirm the
accuracy of the information presented and to correctly relate generally accepted practices. The authors, editors, and publisher
cannot accept responsibility for errors or exclusions or for the outcome of the material presented herein. There is no expressed
or implied warranty of this book or information imparted by it. Care has been taken to ensure that drug selection and dos-
ages are in accordance with currently accepted/recommended practice. Off-label uses of drugs may be discussed. Due to
continuing research, changes in government policy and regulations, and various effects of drug reactions and interactions, it
is recommended that the reader carefully review all materials and literature provided for each drug, especially those that are
new or not frequently used. Some drugs or devices in this publication have clearance for use in a restricted research setting
by the US Food and Drug and Administration or FDA. Each professional should determine the FDA status of any drug or
device prior to use in their practice.

Any review or mention of specific companies or products is not intended as an endorsement by the author or publisher.

SLACK Incorporated uses a review process to evaluate submitted material. Prior to publication, educators or clinicians pro-
vide important feedback on the content that we publish. We welcome feedback on this work.

Published by: SLACK Incorporated


6900 Grove Road
Thorofare, NJ 08086 USA
Telephone: 856-848-1000
Fax: 856-848-6091
www.slackbooks.com

Contact SLACK Incorporated for more information about other books in this field or about the availability of our books from
distributors outside the United States.

Library of Congress Cataloging-in-Publication Data

Curbside consultation in endoscopy : 49 clinical questions / editors, Joseph Leung, Simon K. Lo. -- Second edition.
p. ; cm.
Includes bibliographical references and index.
I. Leung, J. W. C., editor. II. Lo, Simon, editor.
[DNLM: 1. Endoscopy, Digestive System--methods. 2. Digestive System Diseases--diagnosis. 3. Digestive System Diseases-
-therapy. WI 141]
RC78.7.E5
616.07'545--dc 3
2014005637

For permission to reprint material in another publication, contact SLACK Incorporated. Authorization to photocopy items
for internal, personal, or academic use is granted by SLACK Incorporated provided that the appropriate fee is paid directly to
Copyright Clearance Center. Prior to photocopying items, please contact the Copyright Clearance Center at 222 Rosewood
Drive, Danvers, MA 01923 USA; phone: 978-750-8400; website: www.copyright.com; email: info@copyright.com
Dedication
To our families.

Joseph Leung, MD, FRCP, FACP, FASGE, MACG


Simon K. Lo, MD
Contents
Dedication ................................................................................................................................................... v
Acknowledgments ....................................................................................................................................xiii
About the Editors...................................................................................................................................... xv
Contributing Authors .............................................................................................................................xvii
Foreword by Stephen M. Schutz, MD ...................................................................................................xxi
Preface ................................................................................................................................................... xxiii
Introduction ............................................................................................................................................xxv

Section I Upper Endoscopy....................................................................... 1

Question 1 An 81-Year-Old Healthy Male Is Found to Have A 12-cm Long Segment Barrett’s
Esophagus and Several Tiny Raised Lesions, With Moderate to High Grade
Dysplasia in One Location. What Do I Tell The Patient? What Therapy Is
Appropriate? .................................................................................................................... 3
Paulo Sakai, MD, PhD, FASGE and Fauze Maluf-Filho, MD, PhD, FASGE
Question 2 A 46-Year-Old Female With Cirrhosis Came in With Severe Upper GI Bleeding.
Her Proximal Stomach Is Filled With Large Clots and Gastric Varices Are
Suspected. How Can I Tell for Certain Endoscopically, and What Is the
Treatment? ....................................................................................................................... 7
Anne Thai, MD and Joseph Leung, MD, FRCP, FACP, FASGE, MACG
Question 3 What Are the Essential Tools for Removing Esophagogastric Foreign Bodies,
and When Should I Apply These Devices? .............................................................. 13
Luo-wei Wang, MD, PhD and Zhao-shen Li, MD
Question 4 You Were Called by the ER Physician Regarding a Female Who Attempted to
Commit Suicide by Drinking Toilet Cleaning Solution. There Were Obvious
Burns Around the Mouth and Tongue. What Would You Do Next?.................. 21
Rajesh Gupta, MD, DM and D. Nageshwar Reddy, MD, DM, FRCP, DSc
Question 5 What Is the Current Recommendation for Endoscopic Surveillance of
Barrett’s Esophagus, and How Can We Improve the Results With the Current
Technologies? ............................................................................................................... 25
Paulo Sakai, MD, PhD, FASGE and Fauze Maluf-Filho, MD, PhD, FASGE
Question 6 What Should I Do if I Have Inadvertently Perforated the Viscus During an
Endoscopy? .................................................................................................................... 29
Gregory Haber, MD
Question 7 What Is the Current Expert Opinion on What Hemostasis Technique to Use
in Treating a Visible Vessel or Dieulafoy Lesion? When Should I Involve a
Surgeon? ......................................................................................................................... 33
Brintha K. Enestvedt, MD, MBA and Michael L. Kochman, MD, FACP
Question 8 What Do People Do for the Treatment of Achalasia These Days? Does it Matter
if the Symptoms Are Severe or if the Patient Is Elderly? ....................................... 37
Drew Ingram, MD and Joseph Leung, MD, FRCP, FACP, FASGE, MACG
viii  Contents

Question 9 A 65-Year-Old Male With a Long-Standing GERD Complained of Recent Onset


of Progressive Difficulty Swallowing (Especially Solids) and Subjective Weight
Loss. How Should I Evaluate the Patient? ................................................................ 41
Emmanuel C. Gorospe, MD, MPH, FACP and
Kenneth K. Wang, MD, FACG, AGAF, FASGE
Question 10 Several Trips to the ER for a 56-Year-Old Mechanic for SOB and Iron Deficient
Anemia Revealed Positive FOBT, but EGD and Colonoscopies Did not Identify
the Bleeding Source. He Takes NSAIDs Regularly for Arthritis Pain. Is There a
Role for Enteroscopy? .................................................................................................. 45
Neel K. Mann, MD, MPH
Question 11 A 59-Year-Old Professor Had Dyspepsia and Subjective Weight Loss. He Was
Treated Previously for H pylori Infection. EGD and Biopsies Showed Chronic
Inflammation With Intestinal Metaplasia and One Focal Area of Low-Grade
Dysplasia. What Do I Do?. ......................................................................................... 51
Chun-Ying Wu, MD, PhD, MPH, LL.M., LL.B. and
Francis K.L. Chan, MD, FACG
Question 12 A 38-Year-Old Mildly Obese Female Presented With GERD Symptoms and Was
Found to Have a Persistently Elevated Amylase Level Around 250 Iu/L (Normal
< 125) but Normal Lipase. How Should I Manage This Patient?.......................... 53
Sooraj Tejaswi, MD, MSPH
Question 13 A 35-Year-Old Female Presented With Epigastric Pain, Difficulty Swallowing,
and Subjective Fever 1 Month After Gastric Bypass Surgery. She Was Seen in the
ER Because of Recent Hematemesis. What Do I Do?. ........................................... 57
Andrew Yen, MD, FACG and Joseph Leung, MD, FRCP, FACP, FASGE, MACG
Question 14 A 45-Year-Old Alcoholic Was Admitted With Abdominal Pain, Coffee Ground
Emesis, and Mild Anemia. Upper Endoscopy Shows a Few Polypoid Masses in
the Gastric Body and Pigmentations in the Antrum. What Should I Do?......... 61
Sooraj Tejaswi, MD, MSPH
Question 15 A 45-Year-Old Patient With Known Hepatitis C Was Admitted Because of
Massive Upper GI Bleeding. What Should I Do?.................................................... 65
Cara Torruellas, MD, MPH and
Joseph Leung, MD, FRCP, FACP, FASGE, MACG
Question 16 Bougienage, Balloon Dilation, Cautery Cutting, Stenting, and Steroid Injection
Have All Been Described to Treat Benign Strictures of the GI Tract. Is There a
Right Way of Doing This?........................................................................................... 69
Jeffrey L. Conklin, MD, FACP

Section II Colonoscopy ............................................................................ 73

Question 17 Which of My Patients With Ulcerative Colitis Need “Surveillance” Colonoscopy?


How Should the Procedure Be Performed?.............................................................. 75
Michael F. Picco, MD, PhD and Francis A. Farraye, MD, MSc
Question 18 I Had a Difficult Colonoscopy and No Matter What I Tried, I Just Could not
Reach the Cecum. I Maxed out on IV Sedations and Have no Anesthesiology
Support. Can You Help Me? ....................................................................................... 81
Felix W. Leung, MD, FACG
Contents  ix

Question 19 A 68-Year-Old Male With Significant Comorbidities Presented With Weight


Loss and New Onset Vomiting. Investigation Showed Colonic Obstruction
With Poorly Differentiated Adenocarcinoma Involving the Sigmoid Colon. Do I
Consider Stenting? ....................................................................................................... 87
Todd H. Baron, MD, FASGE
Question 20 A 58-Year-Old Female Underwent a Screening Colonoscopy That Showed a
2.5-cm Flat Polyp in the Proximal Ascending Colon. I Tattooed the Area and
Biopsied the Polyp, Which Was Initially Reported as Hyperplastic. What Should
I Do? ............................................................................................................................... 89
Michael Bourke, MBBS, FRACP and Farzan Fahrtash Bahin, MBBS (Hons)
Question 21 What Is the Role of Chromoendoscopy in Screening Colonoscopy? Will it Help
to Differentiate Different Types of Polyps, and Is it Useful for Proximal Serrated
Polyps?............................................................................................................................ 95
Ihab I. El Hajj, MD, MPH and Charles J. Kahi, MD, MSc
Question 22 I Have Trouble Doing a Complete Colonoscopy in Some of My Patients Because
of Poor Bowel Preparation. What Should I Do to Insure a Properly Prepared
Colon?........................................................................................................................... 101
Han-Mo Chiu, MD, PhD

Section III Endoscopic Retrograde


Cholangiopancreatography ..............................................107

Question 23 A 55-Year-Old Veteran Had a Ruptured Gallbladder and a Difficult


Cholecystectomy. Postoperatively, the Patient Had Persistent Bile Leakage
Despite Months of Multiple Biliary Stenting. What Can I Do? ......................... 109
Joseph Leung, MD, FRCP, FACP, FASGE, MACG and Catherine Ngo, MD
Question 24 A 65-Year-Old Male on Plavix Following a Recent MI and Coronary Stenting
Was Seen in the ER With Fever, Pain, Jaundice, and a WBC of 18,000. US
Showed a Dilated Bile Duct of 15 mm and a Gallbladder Full of Stones. MRCP
Revealed a 15-mm Stone in the Distal CBD. What Would You Do?................. 113
Wei-Chih Liao, MD, PhD
Question 25 What Is the Best Endoscopic Method to Diagnose Pancreatic Cancer? What Is
the Best Way to Sample Tissue to Diagnose Suspected Bile Duct Cancer? ...... 117
John G. Lee, MD
Question 26 A 68-Year-Old Man With a Klatskin’s Tumor Was Referred for Consideration of
ERCP and Palliative Drainage. How Should I Manage the Patient? ................. 121
Joseph Leung, MD, FRCP, FACP, FASGE, MACG
Question 27 An Elderly Patient With Multiple Medical Problems Presents With Acute
Pancreatitis and Stones in the Gallbladder. LFTs Show AST of 90 and ALT of
95, Both Declining. MRCP Shows no CBD Stones or Ductal Dilation. Is ERCP
Indicated?.....................................................................................................................125
Stuart Sherman, MD
Question 28 I Have a Patient With Biliary-Type Pain, but no Other Evidence of Bile Duct
Disease (Normal Diameter Duct on Ultrasound, Normal Liver Function Tests).
How Should Such a Patient Be Managed? .............................................................. 127
Martin L. Freeman, MD, FACG, FASGE
x  Contents

Question 29 How Can We Accurately Determine the Appropriate Length of the Stent to Use
in a Patient With a Bile Duct Stricture? ................................................................. 133
Joseph Leung, MD, FRCP, FACP, FASGE, MACG and Erina Foster, MD
Question 30 A 56-Year-Old Female Presents With Recurrent RUQ Pain, Impaired LFTs,
Elevated ALKP, and Transaminases. She Had a Prior ERCP and Possible Biliary
Papillotomy for Similar Complaints but She Does not Recall the Diagnosis.
What Should I Do? ..................................................................................................... 137
Joseph Leung, MD, FRCP, FACP, FASGE, MACG
Question 31 My Accountant, Whose Ulcerative Colitis Is in Good Control on Maintenance
5 ASA Therapy, Called Because His LFTs Were Elevated. He Had no History of
Jaundice, but New Onset of Itching. What Should I Do? .................................... 143
Andrew Yen, MD and Joseph Leung, MD, FRCP, FACP, FASGE, MACG
Question 32 My 35-Year-Old Patient Who Had a Cholecystectomy 5 Years Ago During Her
Gastric Bypass Surgery Presents With Abdominal Pain and Impaired LFTs. CT
Showed a 15-mm CBD With a Distal CBD Stone. ERCP Was Unsuccessful Using
a Pediatric Scope. Can You Help?............................................................................ 147
Simon K. Lo, MD
Question 33 A 48-Year-Old Male Had Liver Transplantation 9 Months Ago for Chronic Hep
C Cirrhosis. He Presented Recently With Recurrence of Jaundice and Impaired
LFTs. How Should I Manage This Patient? ............................................................ 151
Milan Bassan, MBBS, FRACP and Paul Kortan, MD, FRCPC, FASGE, AGAF
Question 34 A 68-Year-Old Banker Who Had a Prior Whipple for Periampullary Cancer
Had Recurrent Abdominal Pain and Fever for 3 Years. Each Resolved With
Antibiotics. LFTs Are Impaired and CT Scan Shows Dilated Intrahepatic Ducts
and a Tubular Structure in the Pancreas. What Should I Do?. .......................... 159
Jason B. Samarasena, MD and John G. Lee, MD
Question 35 A 78-Year-Old Vietnamese Woman Who Collapsed at Home Was Found to Be
Febrile and in Shock With a Systolic BP of 80 mm Hg. She Is Mildly Jaundiced
With Abdominal Guarding. How Would You Manage Her? ............................. 163
Joseph Leung, MD, FRCP, FACP, FASGE, MACG and Catherine Ngo, MD
Question 36 My 68-Year-Old Patient With MS Presents With Dyspeptic Symptoms and Loose
Stools. Imaging Showed a Dilated Pancreatic Duct and Extensive Calcification
in the Head of the Pancreas. Symptoms Responded Partly to Pancreatic Enzyme
Supplement. What Should I Do Next? .................................................................... 169
Mohan Ramchandani, MD, DM and D. Nageshwar Reddy, MD, DM, FRCP, DSc
Question 37 A 12-Year-Old Patient Has a 5-Year History of Recurrent “Acute Pancreatitis,”
Managed Conservatively With Pancreatic Enzymes Supplement. CT Reveals
Mildly Dilated Pancreatic Duct With a Possible Stone in the Head of the
Pancreas. What Else Can I Do? ............................................................................... 175
Joseph Leung, MD, FRCP, FACP, FASGE, MACG
Question 38 A 25-Year-Old Female Presents With Recurrent Abdominal Pain and Mildly
Elevated Serum Amylase and Lipase Levels, Normal LFTs. Abdominal Imaging
Shows Normal Size CBD, PD, and no Obvious Stones in the Gallbladder. What
Should I Do Next? ...................................................................................................... 179
James J. Farrell, MD and Harry Aslanian, MD
Contents  xi

Question 39 What Is the Best Treatment for Duodenal or Ampullary Adenoma, and What Is
the Current Recommendation for Surveillance After Treatment? .................... 183
Richard A. Kozarek, MD
Question 40 What Should Be the Sequence of Investigations for a Patient With Recurrent,
Unexplained Pancreatitis? ........................................................................................ 189
Stuart Sherman, MD
Question 41 A 56-Year-Old Female With Nausea and Vomiting Has a Mildly Elevated
Bilirubin but Normal Amylase and Lipase. CT Shows Thickening of the
Duodenum and Mildly Dilated CBD. Attempted ERCP Shows a Tight Ulcerative
Stenosis of 1st/2nd Part of the Duodenum. What Should I Do? ....................... 193
Todd H. Baron, MD, FASGE

Section IV Endoscopic Ultrasound.......................................................197

Question 42 How Do We Manage a Patient With a 10-cm Pseudocyst Who Is Currently


Asymptomatic? ...........................................................................................................199
Richard A. Kozarek, MD
Question 43 Do All Pancreatic Cysts Need EUS Imaging, and Do They All Need to Be
Aspirated for Analysis? Are There Even Standard Criteria to Determine Their
Origins? ........................................................................................................................203
Won Jae Yoon, MD and William R. Brugge, MD
Question 44 Our Endosonographer Frequently Detects Vague Hypoechoic Pancreatic Lesions,
but Does not Perform Fine Needle Aspiration—He Recommends Follow Up
EUS in 3 Months. How Many Follow Up Exams Are Needed? Are Fine Needle
Aspirations Unsafe? ...................................................................................................207
Jason B. Samarasena, MD and Kenneth J. Chang, MD, FACG, FASGE
Question 45 When Is EUS Necessary for a Newly Diagnosed Cancer of the Esophagus,
Stomach, Colon, or Pancreas? .................................................................................. 211
Thomas J. Savides, MD
Question 46 A 35-Year-Old Mechanic Is Complaining of New Onset Epigastric Pain
After Taking Motrin. EGD Shows Mild, Nonerosive Antral Gastritis, and a
1-cm Submucosal Mass in the Proximal Antrum. Biopsies Showed Chronic
Inflammation. What Should I Do? ......................................................................... 215
Laith H. Jamil, MD

Section V Capsule Endoscopy ...............................................................219

Question 47 In True Obscure GI Bleeding, What Should I Do if a Capsule Endoscopy and


Upper and Lower Endoscopies Are Unrevealing? ................................................ 221
Anupam Singh, MD and David R. Cave, MD, PhD
Question 48 What Is a Reliable Method to Estimate the Location of a Lesion Found on
Capsule Endoscopy?...................................................................................................225
Lucinda A. Harris, MS, MD and Jonathan A. Leighton, MD
xii  Contents

Question 49 If I Suspect a Small-Bowel Lesion, How Do I Choose Among Small Bowel Series,
Capsule Endoscopy, CT Enterography, and Double-Balloon Enteroscopy as the
Diagnostic Test? ..........................................................................................................229
Ravi K. Ghanta, MD and
Jamie S. Barkin, MD, MACP, MACG, FASGE, AGAF
Financial Disclosures ............................................................................................................................. 235
Acknowledgments
To our many friends and colleagues and some of our GI fellows/junior faculty who have helped
research and put together the answers to many of the questions. We deeply appreciate their sup-
port.
About the Editors
Joseph Leung, MD, FRCP, FACP, FASGE, MACG is currently the Mr. & Mrs. C.W. Law
Professor of Medicine at the University of California, Davis School of Medicine and the Chief of
Gastroenterology for the VA Northern California Health Care System. Dr. Leung is a Fellow of
the Royal College of Physicians of Edinburgh, Glasgow and London, as well as the Hong Kong
College of Physicians and Hong Kong Academy of Medicine. He is also a Fellow of the American
College of Physicians, American College of Gastroenterology, and the American Society for
Gastrointestinal Endoscopy.
Dr. Leung has pioneered a number of therapeutic endoscopy procedures including epinephrine
injection for ulcer hemostasis, urgent nasobiliary drainage for acute suppurative cholangitis, the
design and development of the Cotton-Leung stent for biliary drainage and palliation of malig-
nant obstructive jaundice, and the water exchange colonoscopy for colorectal cancer screening.
Dr. Leung has spent the last 25 years improving ERCP training for GI trainees. He has
organized many teaching endoscopy workshops locally, nationally, and internationally and par-
ticipated as teaching faculty in many more. His current research interest includes the use of a
mechanical simulator for ERCP training and methods to improve proximal adenoma detection on
colonoscopy. He has published RCTs on the impact of coached EMS practice on clinical ERCP
performance of trainees.
Dr. Leung received the ACG Senior Governor Award in 2004 and ASGE Master Endoscopist
Award in 2005 and Master of American College of Gastroenterology in 2010. He was a former
Associate Editor for Gastrointestinal Endoscopy and currently Executive Editor-in-Chief for the
Journal of Interventional Gastroenterology. He is the author or coauthor of more than 400 peer-
reviewed articles, abstracts, and book chapters, as well as 5 books.

Simon K. Lo, MD is the Director of Endoscopy at Cedars-Sinai Medical Center, where he is


also the director of the Pancreatic Diseases Program and head of the Interventional Endoscopy
Training. He is a Clinical Professor of Medicine at David Geffen School of Medicine at UCLA.
He obtained his MD degree from New York University. He completed a gastroenterology fellow-
ship at the Integrated GI Training Program at UCLA. In 1989, he refined his biliary endoscopy
skill under renowned Professor Kees Huibregtse at the Academic Medical Center in Amsterdam.
He has served on the ASGE and numerous other international society committees and was elected
Governor of the American College of Gastroenterology for multiple terms.
Known for his extensive endoscopy experience and clinical interests, Dr. Lo has evaluated
many new technologies to treat GERD, obesity, and mucosal lesions. He pioneered in several
small bowel procedures including different platforms of capsule endoscopy and device-assisted
enteroscopy. In spite of his diverse skills, he is most noted for his work in endoscopy for pancreatic
and biliary diseases. For 21 years, he has organized and co-hosted a highly regarded symposium
on pancreatic and biliary endoscopy that features the most recognized biliary endoscopists in the
world. He is currently devoting most of his time to building a program to integrate basic science
and clinical services for improvement in the understanding and treatment of pancreatitis and
pancreatic cancer.
Central to the advancement of health care is the willingness to learn and the commitment to
teach. Dr. Lo has learned from some of the most noted figures in endoscopy and GI sciences,
while engaging in many educational events in endoscopy here and abroad. He has spoken exten-
sively in many countries and has trained numerous gastroenterologists and GI interventionists over
the years. He has also published many articles in the field. He is exceedingly grateful to have the
opportunity to work with Dr. Joseph Leung on this very practical and valuable book on endoscopy.
Contributing Authors
Harry Aslanian, MD (Question 38) Francis K.L. Chan, MD, FACG (Question 11)
Interventional Endoscopy and Pancreatic Choh-Ming Li Professor of Medicine
Diseases Department of Medicine & Therapeutics
Yale School of Medicine The Chinese University of Hong Kong
New Haven, Connecticut Shatin, N.T., Hong Kong

Farzan Fahrtash Bahin, MBBS (Hons) Kenneth J. Chang, MD, FACG, FASGE
(Question 20) (Question 44)
University of Sydney Professor and Chief, Gastroenterology &
Westmead Hospital Endoscopy Unit Hepatology
Westmead Hospital Executive Director, H. H. Chao Comprehensive
New South Wales, Australia Digestive Disease Center
Endowed Chair, GI Endoscopic Oncology
Jamie S. Barkin, MD, MACP, MACG, FASGE, University of California, Irvine
AGAF (Question 49) Irvine, California
Division of Gastroenterology (D-49)
University of Miami, Miller School Han-Mo Chiu, MD, PhD (Question 22)
Miami, Florida Clinical Associate Professor
Department of Internal Medicine
Todd H. Baron, MD, FASGE (Questions 19, 41) National Taiwan University Hospital
Mayo Clinic Taipei, Taiwan
Rochester, Minnesota
Jeffrey L. Conklin, MD, FACP (Question 16)
Milan Bassan, MBBS, FRACP (Question 33) Clinical Professor of Medicine
St. Michael’s Hospital Director, Center for Esophageal Disorders
Toronto, Ontario, Canada and GI Motility Lab
Division of Digestive Diseases
Michael Bourke, MBBS, FRACP (Question 20) David Geffen College of Medicine at UCLA
Professor of Medicine Los Angeles, California
University of Sydney
Department of Gastroenterology and Ihab I. El Hajj, MD, MPH (Question 21)
Hepatology Fellow, Gastroenterology and Hepatology
Westmead Hospital Indiana University School of Medicine
New South Wales, Australia Indianapolis, Indiana

William R. Brugge, MD (Question 43) Brintha K. Enestvedt, MD, MBA (Question 7)


Professor of Medicine Assistant Professor
Massachusetts General Hospital Division of Gastroenterology
Boston, Massachusetts Oregon Health & Science University
Portland, Oregon
David R. Cave, MD, PhD (Question 47)
Professor of Medicine
Director, Clinical Gastroenterology Research
University of Massachusetts Medical Center
Worcester, Massachusetts
xviii  Contributing Authors

Francis A. Farraye, MD, MSc (Question 17) Gregory Haber, MD (Question 6)


Clinical Director, Section of Gastroenterology Director, Division of Gastroenterology
Boston Medical Center Director, Center for Advanced Therapeutic
Professor of Medicine Endoscopy
Boston University School of Medicine NSLIJ/Lenox Hill Hospital
Boston, Massachusetts New York City, New York

James J. Farrell, MD (Question 38) Lucinda A. Harris, MS, MD (Question 48)


Interventional Endoscopy and Pancreatic Associate Professor of Medicine
Diseases Division of Gastroenterology
Yale School of Medicine Mayo School of Medicine
New Haven, Connecticut Scottsdale, Arizona

Erina Foster, MD (Question 29) Drew Ingram, MD (Question 8)


Gastroenterologist Attending Physician
Kaiser Permanente Department of Gastroenterology and
Vacaville, California Hepatology
Mercy Hospital of Folsom
Martin L. Freeman, MD, FACG, FASGE Folsom, California
(Question 28)
Professor of Medicine Laith H. Jamil, MD (Question 46)
Director, Division of Gastroenterology Assistant Clinical Professor
Hepatology and Nutrition University of The David Geffen School of Medicine at
Minnesota UCLA
Minneapolis, Minnesota Associate Director of Interventional Endoscopy
Cedars-Sinai Medical Center
Ravi K. Ghanta, MD (Question 49) Los Angeles, California
Tennova Digestive Disease Center
Floor Powell, Tennessee Charles J. Kahi, MD, MSc (Question 21)
Associate Professor of Clinical Medicine
Emmanuel C. Gorospe, MD, MPH, FACP Indiana University School of Medicine
(Question 9) Gastroenterology Section Chief
Instructor in Medicine Richard L. Roudebush VAMC
Fellow in Gastroenterology and Hepatology Indianapolis, Indiana
Mayo Clinic
Rochester, Minnesota Michael L. Kochman, MD, FACP (Question 7)
Professor of Medicine in Surgery
Rajesh Gupta, MD, DM (Question 4) Co-Director Gastrointestinal Oncology
Senior Consultant Endoscopy Training Director
Gastroenterology and Hepatology Gastroenterology Division
Asian Institute of Gastroenterology Hospital of the University of Pennsylvania
Hyderabad, India Philadelphia, Pennsylvania

Paul Kortan, MD, FRCPC, FASGE, AGAF


(Question 33)
St. Michael’s Hospital
Toronto, Ontario, Canada
Contributing Authors  xix

Richard A. Kozarek, MD (Questions 39, 42) Catherine Ngo, MD (Questions 23, 35)
Executive Director Division of Gastroenterology and Hepatology
Digestive Disease Institute UC Davis Medical Center
Virginia Mason Medical Center Section of Gastroenterology
Seattle, Washington Sacramento VA Medical Center
Northern California Health Care System
John G. Lee, MD (Questions 25, 34) Sacramento, California
UC Irvine Medical Center
Irvine, California Michael F. Picco, MD, PhD (Question 17)
Associate Professor of Medicine
Jonathan A. Leighton, MD (Question 48) Consultant, Division of Gastroenterology
Professor of Medicine Mayo Clinic
Chair, Division of Gastroenterology Jacksonville, Florida
Mayo Clinic in Arizona
Scottsdale, Arizona Mohan Ramchandani, MD, DM (Question 36)
Asian Institute of Gastroenterology
Felix W. Leung, MD, FACG (Question 18) Hyderabad, India
Sepulveda Ambulatory Care Center
Veterans Affairs Greater Los Angeles D. Nageshwar Reddy, MD, DM, FRCP, DSc
Healthcare System (Questions 4, 36)
North Hills, California Chairman
David Geffen School of Medicine at University Asian Institute of Gastroenterology
of California at Los Angeles Hyderabad, India
Los Angeles, California
Paulo Sakai, MD, PhD, FASGE (Questions 1, 5)
Zhao-shen Li, MD (Question 3) Associate Professor
Professor of Medicine Department of Gastroenterology of São Paulo
Director, Digestive Endoscopy Center University
Changhai Hospital São Paulo, Brazil
Second Military Medical University
Shanghai, China Jason B. Samarasena, MD (Questions 34, 44)
UC Irvine Medical Center
Wei-Chih Liao, MD, PhD (Question 24) Irvine, California
National Taiwan University Hospital
Taipei, Taiwan Thomas J. Savides, MD (Question 45)
Professor of Clinical Medicine
Fauze Maluf-Filho, MD, PhD, FASGE University of California, San Diego
(Questions 1, 5)
Associate Professor Stuart Sherman, MD (Questions 27, 40)
Cancer Institute of São Paulo University Professor of Medicine and Radiology
Department of Gastroenterology of São Paulo Clinical Director of Gastroenterology
University Indiana University School of Medicine
São Paulo, Brazil Indianapolis, Indiana

Neel K. Mann, MD, MPH (Question 10) Anupam Singh, MD (Question 47)
Associate Director, Small Bowel Endoscopy Fellow, Division of Gastroenterology,
Cedars-Sinai Medical Center University of Massachusetts Medical Center
Los Angeles, California Worcester, Massachusetts
xx  Contributing Authors

Sooraj Tejaswi, MD, MSPH (Questions 12, 14) Luo-wei Wang, MD, PhD (Question 3)
Assistant Clinical Professor Changhai Hospital
Division of Gastroenterology and Hepatology Second Military Medical University
University of California, Davis Medical Center Shanghai, China
Sacramento, California
Chun-Ying Wu, MD, PhD, MPH, LL.M., LL.B.
Anne Thai, MD (Question 2) (Question 11)
Gastroenterology Professor of Internal Medicine
Mills-Peninsula Medical Group National Yang-Ming University
Mid Peninsula Endoscopy Center Taipei, Taiwan
San Mateo, California Division of Gastroenterology
Taichung Veterans General Hospital
Cara Torruellas, MD, MPH (Question 15) Taichung, Taiwan
GI Fellow
Department of Gastroenterology and Andrew Yen, MD (Questions 13, 31)
Hepatology Assistant Clinical Professor of Medicine
UC Davis Medical Center UC Davis Medical Center
Sacramento, California Assistant Chief of Gastroenterology
Sacramento VA Medical Center
Kenneth K. Wang, MD, FACG, AGAF, FASGE Sacramento, California
(Question 9)
Mayo Clinic Won Jae Yoon, MD (Question 43)
Rochester, Minnesota Gastrointestinal Unit
Massachusetts General Hospital
Boston, Massachusetts
Foreword
We live in the Internet Age—everyone’s plugged in, on-line, and busy. Our attention spans
have shrunk too, and it’s difficult to read textbooks or long journal articles. Still, no matter how
pressing our time constraints may be, we must give our patients excellent care even when they pose
challenging clinical problems. Thankfully, Drs. Joseph Leung and Simon Lo have written a book
that is perfect for endoscopists who are constantly on the go, like me and probably you.
In fact, this excellent tome is best described as a FAQ (frequently asked questions) section for
endoscopy. Like the FAQ portion of a Web site, this book saves time by addressing important
questions that all gastroenterologists ask from time to time. Unlike most Web sites, though, Drs.
Leung and Lo have gathered experts from around the world to answer those questions.
In this second edition, some new and challenging questions have been added including how
to improve cancer detection and surveillance, tackle complex pancreaticobiliary problems, and
manage complications following GI surgery. This book continues to provide concise and clinically
relevant answers for busy clinicians.
Who are the experts? The table of contents is a few pages back, and a cursory examination of
the list of authors will show that Joseph and Simon have assembled a first-rate team. Reading any
chapter will reveal that those experts focused on providing answers that are as informative as a
reference text and as user-friendly as a curbside consultation from a friend.
Thank you Joseph Leung and Simon Lo for a book that is tailor made for busy endoscopists. It
is perfect for the Internet Age.

Stephen M. Schutz, MD
Partner, Boise Digestive Health Clinic
Governor, American College of Gastroenterology
Boise, Idaho
Preface
In the field of digestive endoscopy, we face different challenges in our everyday practice. Some
are common problems that are difficult to manage; some are uncommon problems where there are
no set guidelines or treatment strategy. Curbside Consultation in Endoscopy: 49 Clinical Questions,
Second Edition is a synopsis of our everyday problems where experts give their opinion in the evalu-
ation or management of a patient under their care. The readers will be able to gain the insights and
expertise from those who manage such clinical problems on a regular basis. The first edition of
Curbside Consultation in Endoscopy was published in 2006 and has gained wide popularity among
practicing gastroenterologists and trainees. In the second edition, we have challenged our panel of
experts with a new set of questions with even tougher clinical problems while maintaining some
of the very popular questions from the first edition. We hope to share this experience with our
readers. The book contains expert opinions on important clinical problems. However, some issues
remain controversial and hopefully will generate interesting discussion. We hope you will find
this new edition and new set of questions interesting and that it helps you in your daily practice.

Joseph Leung, MD, FRCP, FACP, FASGE, MACG


Simon K. Lo, MD
Introduction
In the practice of gastroenterology, there is an increasing armamentarium of endoscopic pro-
cedures and the choice of treatment can be variable even for common problems. Some of these
techniques are quite challenging and questions are often raised as to the appropriate application
of these procedures. Although recent advances in endoscopy may deem promising, many are still
lacking controlled data to support their wide spread adoption in every day practice.
Curbside Consultation in Endoscopy: 49 Clinical Questions, Second Edition is set up to provide
practicing gastroenterologists with expert opinions and assistance in managing different gastroin-
testinal (GI) conditions. We have invited a strong team of endoscopy experts and posed them chal-
lenging GI problems. In the following chapters, experts discuss their approach and tricks-of-trade
in the management of some of these difficult GI problems. The book is divided into several sec-
tions including upper endoscopy, colonoscopy, endoscopic retrograde cholangiopancreatography,
endoscopic ultrasound, and capsule endoscopy. The questions posed are common or difficult prob-
lems that we encounter daily and what one would sometimes want to seek a quick advice from a
more experienced colleague. This book is a synopsis of experts’ opinion rather than an exhaustive
review of the literature. The answers are based on their practice experience rather than research or
hypothetical treatment in the daily management of their patients. Some have given their personal
preference; others have tried to summarize the current practice reported in the literature.
For the busy practicing gastroenterologists, we hope this book will provide an excellent update
of the current management of common GI conditions. For the trainees, you will find some practi-
cal tips to improve your skills in the care of your every day patients.

Joseph Leung, MD, FRCP, FACP, FASGE, MACG


Simon K. Lo, MD
SECTION I
UPPER ENDOSCOPY
1
QUESTION

AN 81-YEAR-OLD HEALTHY MALE IS FOUND TO HAVE A 12-CM LONG


SEGMENT BARRETT S ESOPHAGUS AND SEVERAL TINY RAISED LESIONS,
WITH MODERATE TO HIGH GRADE DYSPLASIA IN ONE LOCATION. WHAT
DO I TELL THE PATIENT? WHAT THERAPY IS APPROPRIATE?

Paulo Sakai, MD, PhD, FASGE and Fauze Maluf-Filho, MD, PhD, FASGE

Barrett’s esophagus (BE) is defined as the presence of specialized columnar metaplasia at the
distal esophagus in response to the chronic reflux of gastroduodenal content into the esophagus.
Compared with other patients with gastroesophageal reflux disease, patients with BE present
with longer acid and bile exposure. It has been recently acknowledged that the risk of adenocarci-
noma associated with BE was overestimated.1 One case of cancer for approximately 1000 patients
with BE per year is expected. Long segments of BE (> 3 cm) have a higher risk of cancer when
compared with shorter segments. Endoscopic surveillance with routine biopsies is recommended
for early detection of adenocarcinoma in these patients. The presence of cellular atypia confined
to the epithelium is called dysplasia and is considered to be a marker for the development of
invasive adenocarcinoma in BE. When high-grade dysplasia (HGD) is detected and confirmed
by a senior pathologist, the patient should be made aware of the 30% to 40% risk that an invasive
adenocarcinoma was missed in the endoscopy. The incidence of cancer after diagnosis of HGD
may be in approximately 28% to 30% of patients, which definitely is an indication for treatment. 2
Controversy exists regarding the optimal management of BE with HGD. There are 3 options:
1) esophagectomy to prevent cancer and cure early cancer, 2) endoscopic ablation and resection
to remove the neoplastic mucosa to prevent cancer and cure mucosal cancer, and 3) surveillance
esophagogastroduodenoscopy (EGD) every 3 months with 4-quadrant biopsies every 1 cm.
Endoscopic ultrasound (EUS) for local staging and lymph node detection should also be consid-
ered in this last option.
For some investigators, particularly among surgeons, BE with HGD has been considered
an indication for esophagectomy because of the increased risk of cancer. The patient should
be referred to high-volume surgical consultation centers where more than 50 esophagectomies
are performed every year. In these centers, the mortality rate related to this extensive surgery is
lower than 5%. However, the endoscopic therapy may be an attractive and less invasive treatment

Leung J, Lo SK, eds. Curbside Consultation in Endoscopy:


3 49 Clinical Questions, Second Edition (pp 3-6)
© 2014 SLACK Incorporated
4  Question 1

A B

C D
Figure 1-1. (A) Long segment of BE with HGD lesion (arrowheads), (B, C) band-ligation mucosec-
tomy, and (D) final aspect of local resection.

alternative because the risk of lymph node involvement or hematogenous dissemination is absent
in HGD and negligible in cancers restricted to the mucosal layer. Endoscopic therapy may be
performed through local endoscopic mucosal resection or through endoscopic ablation using pho-
todynamic therapy (PDT), argon plasma coagulation (APC), or, more recently, radiofrequency
ablation (RFA).
Endoscopic mucosal resection (EMR) is safe and effective for complete HGD local resection
and early BE cancer. Usually, 2 endoscopic techniques are applied: cap technique and band-
ligation technique (Figure 1-1). In both techniques, piecemeal resection is required when the
lesion is larger than 10 to 15 mm. More recently, another technique called endoscopic submucosal
dissection (ESD) has been used for the resection of a large lesion en bloc, but specific accessories
and training are required for this procedure. For BE more than 3 cm in length, the complete
circumferential removal of the mucosa may cause an esophageal stricture. One of the most
frequently studied endoscopic therapies for HGD in BE is PDT. It is an expensive method with
limited availability, and long-term results are not well described. There is a 40% rate of esophageal
stenosis, and small foci of invasive cancer may be left untreated. Ablative therapies, such as PDT
and APC, do not provide a specimen for histopathological evaluation and usually the depth of
eradication is limited. Residual BEs under restored squamous epithelium after endoscopic tissue
What Do I Tell The Patient With Barrett's Esophagus? What Therapy Is Appropriate?   5

Figure 1-2. Radiofrequency ablation on remaining BE 3 months post-mucosecto-


my.

ablative therapy may occur in 20% to 30% of cases. Several articles have described the long-term
results of RFA for the treatment of HGD, intestinal metaplasia, and its efficacy to prevent the
development of invasive cancer.3,4 In a multicenter trial involving 19 US institutions, 127 patients
with BE underwent RFA. Complete remission of HGD was obtained in 81%, compared with 19%
of the patients in the sham group (P <.001). Still, in the treated group the risk of progression to
invasive cancer was 2.4% compared with 19% in the sham group. Approximately 6% of the patients
in the treated group developed esophageal stenosis that responded to a mean of 2.6 dilation
sessions.3 In another study,4 23 patients with BE and HGD/intramucosal cancer underwent
endoscopic mucosectomy for visible lesions and RFA for metaplasia ablation. With a mean of
2.6 RFA sessions, complete remission of HGD/intramucosal cancer and RFA were obtained in
100% of the cases.
With a 12-cm long segment BE and several raised lesions with HGD in a healthy person,
the esophagectomy could be considered. Although preoperative morbidity is significant, surgical
resection of HGD in BE may provide excellent long-term survival, with acceptable quality of life.5
On the other hand, the combination of endoscopic resection and RFA may be another option.
With the aid of acetic acid chromoendoscopy and narrow band imaging, the suspicious areas could
be targeted and endoscopically resected. The remaining metaplastic epithelium should be ablated
by radiofrequency approximately 2 to 3 months after the endoscopic mucosectomy (Figure 1-2).
Even considering that without radiofrequency ablation approximately 11% of patients would have a
recurrence or development of metachronous HGD or an early stage cancer in long-term follow-up,
these can be treated again by endoscopic resection.6 Therefore, we emphasize the importance of a
strict endoscopic follow-up to detect and treat any recurrent lesion. Ideally, all possible columnar
mucosa with intestinal metaplasia should be eradicated, and this approach would be recommended
in patients with invisible or multiple neoplastic areas in a shorter BE.
6  Question 1

Conclusion
We can offer for this patient 3 options7: 1) esophagectomy as a definite treatment, 2) endoscopic
resection of suspicious lesions combined with RFA of the remaining intestinal metaplasia epithe-
lium (this endoscopic approach offers less morbidity, but strict endoscopic follow-up still seems
to be warranted), and 3) surveillance EGD every 3 months, with 4-quadrant biopsies every 1 cm
associated with EUS for local staging and lymph node detection.

References
1. Hvid-Jensen F, Pedersen L, Drewes AM, Sørensen HT, Funch-Jensen P. Incidence of adenocarcinoma among
patients with Barrett’s esophagus. N Engl J Med. 2011 Oct 13;365(15):1375-1383.
2. Wang KK, Wongkeesong M, Buttar NS. American Gastroenterological Association medical position statement:
Role of the gastroenterologist in the management of esophageal carcinoma. Gastroenterol. 2005;128(5):1468-
1470.
3. Shaheen NJ, Sharma P, Overholt BF, et al. Radiofrequency ablation in Barrett’s esophagus with dysplasia. N Engl J
Med. 2009;360(22):2277-2288.
4. Pouw RE, Wirths K, Eisendrath P, et al. Efficacy of radiofrequency ablation combined with endoscopic resection
for Barrett’s Esophagus with early neoplasia. Clin Gastroenterol Hepatol. 2010;8(1):23-29.
5. Headrick JR, Nichols III FC, Miller DL, et al. High-grade esophageal dysplasia: long-term survival and quality of life
after esophagectomy. Ann Thorac Surg. 2002;73(6):1697-1703.
6. Ell C, May A, Pech O, et al. Curative endoscopic resection of early esophageal adenocarcinomas (Barrett´s cancer).
Gastrointest Endosc. 2007;65(1):3-10.
7. ASGE Standards of Practice Committee, Evans JA, Early DS, Fukami N, et al; Standards of Practice Committee of
the American Society for Gastrointestinal Endoscopy. The role of endoscopy in Barrett’s esophagus and other
premalignant conditions of the esophagus. Gastrointest Endosc. 2012;76(6):1087-94. doi: 10.1016/j.gie.2012.08.004.
2
QUESTION

A 46-YEAR-OLD FEMALE WITH CIRRHOSIS CAME IN WITH


SEVERE UPPER GI BLEEDING. HER PROXIMAL STOMACH IS FILLED
WITH LARGE CLOTS AND GASTRIC VARICES ARE SUSPECTED.
HOW CAN I TELL FOR CERTAIN ENDOSCOPICALLY, AND
WHAT IS THE TREATMENT?

Anne Thai, MD and Joseph Leung, MD, FRCP, FACP, FASGE, MACG

The patient presents with severe upper gastrointestinal bleeding (UGIB). The initial approach
to the patient is to determine the cause of bleeding. One should conduct a detailed history
and physical exam. The history may reveal common causes of UGIB, such as peptic ulcer dis-
ease, Mallory-Weiss tear, esophagitis, and esophageal or gastric varices. Because most patients
presenting with severe UGIB often cannot give a coherent detailed history, the physical exam
should determine the hemodynamic stability of the patient and the need for airway protection, in
addition to the urgency of resuscitation and the need for intensive care monitoring. Further exami-
nation should look for stigmata of chronic liver disease such as spider angiomata, ascites, caput
medusae, and palmar erythema. Laboratory studies may reveal thrombocytopenia and elevated
international normalized ratio (INR) as a result of advanced chronic liver disease. Adequate resus-
citation with IV fluids and/or blood products is critical.
If variceal bleeding is suspected, pharmacologic therapy with either intravenous octreotide
given in a 50-μg bolus, followed by an octreotide drip at 50 μg per hour, or Terlipressin, a synthetic
analogue of vasopressin, given at 2 mg IV every 4 hours, should be initiated immediately. If
endoscopy confirms variceal bleeding, the octreotide drip should be continued for a minimum of
72 hours, and the Terlipressin drip can be titrated as appropriate to 1 mg IV every 4 hours when
the bleeding is controlled. Beta-blockers should not be used in the acute setting because this may
exacerbate the hemodynamic instability by attenuating the body’s innate response to volume loss.
Early use of proton pump inhibitors (PPIs) should also be considered if the cause of bleeding is
unclear.

Leung J, Lo SK, eds. Curbside Consultation in Endoscopy:


7 49 Clinical Questions, Second Edition (pp 7-11)
© 2014 SLACK Incorporated
8  Question 2

Appropriate antibiotics such as intravenous ciprofloxacin should be given to cirrhotic patients


with upper gastrointestinal (GI) hemorrhage as short-term prophylaxis against spontaneous
bacterial peritonitis and other infectious complications. Ceftriaxone is given in cases with
advanced cirrhosis where there is a high prevalence of quinolone-resistant bacteria.1 Short-term
antibiotic prophylaxis has been shown to decrease mortality. Antibiotics should be discontinued
after 7 days, and oral antibiotic is given when the patient can tolerate oral intake.
Ideally, an esophagogastroduodenoscopy (EGD) should be performed within 12 hours after
appropriate resuscitation. With all GI bleeds, 2 large-bore IV catheters must be placed and
adequate fluids and blood products should be given to stabilize the patient hemodynamically.
However, one must be careful because over fluid resuscitation can lead to an increase in portal
hypertension and subsequent worsening of variceal bleeding. The hemoglobin level should be
maintained around 8 gm/dL, a higher level for those with significant underlying coronary artery
disease. Correcting any underlying bleeding tendency with platelets (goal > 56,000) or fresh frozen
plasma may help with hemostasis. Significant upper GI bleeding warrants airway protection,
including elective intubation prior to emergency endoscopy. In severe cases, the patient should be
admitted to the intensive care unit (ICU) for close monitoring.
If urgent endoscopy reveals the presence of large clots in the proximal stomach or active
bleeding impairing visualization, attempts to clear the clots should be made. If endoscopic
visualization cannot be established because of continued active bleeding and gastric varices are
suspected, balloon tamponade should be used in an attempt to achieve initial hemostasis. Either a
Sengstaken-Blakemore tube (C.R. Bard, Inc) or Minnesota tube (C.R. Bard, Inc) should be placed
with a gastric balloon inflated inside the stomach and pulled with adequate traction against the
gastric fundus to achieve tamponade. Given advances in endoscopic therapies and the less common
presentation of active gastric variceal bleed, physicians may be unfamiliar with this device. As a
brief review, the patient should always be intubated before placement of the tube because of a high
risk of aspiration and need for airway protection. Inflate the tube with air to ensure balloon integ-
rity before placing it. To assist with the passage of the tube either orally or nasally, place the tube
in a bucket of ice, followed by generous lubrication. Advance the tube to approximately 50 cm and
inflate the gastric balloon with approximately 500 cc of water (containing 5 cc of methylene blue),
and pull the balloon until resistance is encountered. Proper placement can be verified through
auscultation or radiography. The tube needs to be pulled with a 1- to 2-lb weight, which equates
to approximately 500 mL to 1 L of fluid hung on a traction device. If active bleeding continues,
the esophageal balloon needs to be inflated to approximately 40 mm Hg and decreased gradually
to a pressure of 25 mm Hg. When assessing for cessation of bleeding, the esophageal balloon can
be deflated every 6 hours for a few minutes. The gastric balloon should remain inflated at all
times. The length of time the balloon stays in place will depend on the patient’s clinical course, but
should not exceed 48 hours. After the bleeding stops and the patient is hemodynamically stable,
repeat endoscopy should be performed.
Suctioning during endoscopy is ineffective in removing large clots because these can block
the relatively small suction channel of the endoscope. Alternatively, clots can be removed using a
large-bore gastric lavage tube with copious irrigation and suction or drainage. If upper endoscopy
reveals that the bleeding has stopped and the views are satisfactory, endoscopic procedures may be
performed. To assist in cleansing the stomach for optimal endoscopy, intravenous erythromycin
250 mg or metoclopramide 10 mg can be given 30 minutes prior to the endoscopy. Attempts can
be made to remove any overlying clots by additional endoscopic irrigation. Repositioning of the
patient to the right lateral position may help to move the clots away from the fundus for better
visualization. Once the clots are removed, appropriate endoscopic therapy can be performed. In
stable patients where the diagnosis of gastric varices is in doubt, endoscopic ultrasound is helpful
to clarify the diagnosis. However, endoscopic ultrasound is not an appropriate examination in the
acute bleed setting.
How Can I Tell for Certain Endoscopically, and How Should I Treat it?   9

If gastric varices are deemed to be the source of bleeding, gastric variceal obliteration with tis-
sue adhesives should be attempted where available. N-butyl-2-cyanoacrylate is available for use in
Europe and Asia. In the United States, 2-octyl cyanoacrylate has been approved by the US Food
and Drug Administration for tissue closure and has been reported to be effective for achieving
initial hemostasis and preventing rebleeding from fundal varices. 2 The technique requires skill
and caution to avoid damage to the endoscope. Complications, although rare, may include embolic
events (ie, cerebral stroke, portal vein embolism, splenic infarction, cardiac emboli, and pulmonary
embolism associated with the cyanoacrylate injections).3 To prevent glue sticking and damage to
the endoscope, lipiodol should be used to coat the tip of the endoscope, and the biopsy channel
should be flushed. A disposable sclerotherapy injection needle and catheter should be primed and
preloaded with a mixture of 1:1 cyanoacrylate and lipiodol prior to insertion into the endoscope.4
After puncturing the varix lumen with the needle, the tissue adhesive is injected in 1-mL aliquots
by using normal saline solution to flush the tissue adhesive into the varix until the varix is firm to
palpation with the tip of the catheter.3
Gastric varices can be caused by isolated splenic vein thrombosis due to neoplasms or chronic
pancreatitis, or more commonly due to increased portal hypertension from advanced cirrhosis.
The Sarin Endoscopic Classification categorizes varices according to endoscopic location.
Gastric varices coexisting with esophageal varices are type 1 gastroesophageal varices (GOV1),
which extend along the lesser curvature, and type 2 gastroesophageal varices (GOV2), which
extend to the greater curvature. Isolated gastric varices are type 1 isolated gastric varices (IGV1),
existing only in the fundus, and type 2 isolated gastric varices (IGV2), existing sporadically
in the gastric antrum, body, pyloric region, or first part of the duodenum.5 Sclerotherapy has
been more successful in GOV than IGV.6 Injection of sclerosants may be performed using
the intravariceal or paravariceal injection technique. Important complications of endoscopic
sclerotherapy include fever, retrosternal pain, dysphagia, injection-induced bleeding, ulceration,
perforation, mediastinitis, pleural effusion, fistula, adult respiratory distress syndrome, and
infectious complications.7 Transjugular intrahepatic portosystemic shunt (TIPS) has been shown
to be highly effective for the control of gastric variceal hemorrhage. Threshold for TIPS place-
ment for bleeding from gastric varices should be lower than bleeding from esophageal varices, and
TIPS should be placed if endoscopic therapy is not possible or after a failed attempt of endoscopic
treatment.8
Balloon-occluded retrograde transvenous obliteration (BRTO) is an alternative to TIPS that
has been shown to significantly decrease rebleeding rates in all patients and increase survival rates
in Child-Pugh class A patients.9 BRTO was first introduced in the 1980s. There are varying
techniques of BRTO with the same underlying principle of direct injection of sclerosants into the
varices, while simultaneously obstructing the outflow of the varices. BRTO is widely used in Asia
for the management of gastric varices. In the United States, the accessibility of BRTO is limited.

Conclusion
Management of the cirrhotic patient with severe upper GI bleeding from suspected gastric
varices should include appropriate evaluation, resuscitation, and stabilization. The patient should
be admitted to the ICU for close monitoring, including potential tracheal intubation for airway
protection prior to endoscopy. Judicious transfusions of blood products and appropriate antibiot-
ics should be given. Balloon tamponade should be considered if the patient is actively bleed-
ing to achieve initial hemostasis. IV erythromycin or metoclopramide should be given prior to
endoscopy to improve visualization during the procedure. Fundal gastric varices should be treated
with pharmacological and endoscopic therapy, and tissue adhesives should be used if available.
10  Question 2

Figure 2-1. Algorithm for evaluating and managing a patient with gastric variceal bleeding.

Familiarization with therapeutic devices available at one’s institution is critical. TIPS should be
considered urgently if endoscopic therapy is not amendable or if bleeding recurs (Figure 2-1).

References
1. Bernard B, Grange JD, Khac EN, Amiot X, Opolon P, Poynard T. Antibiotic prophylaxis for the prevention of bacterial
infections in cirrhotic patients with gastrointestinal bleeding: a meta-analysis. Hepatology. 1999;29:1655-1661.
2. Rengstorff DS, Binmoeller KF. A pilot study of 2-octyl cyanoacrylate injection for treatment of gastric fundal
varices in humans. Gastrointest Endosc. 2004;59:553-558.
3. Tripathi D, Ferguson JW, Therapondos G, et al. Review article: recent advances in the management of bleeding
gastric varices. Aliment Pharmacol Ther. 2006;24:1-17.
4. Dhiman RK, Chawla Y, Taneja S, et al. Endoscopic sclerotherapy of gastric variceal bleeding with N-butyl-2-
cyanoacrylate. J Clin Gastroenterol. 2002;35:222-227.
5. Sarin SK, Lahoti D, Saxena SP, Murthy NS, Makwana UK. Prevalence, classification and natural history of gastric
varices: a long-term follow-up study in 568 portal hypertension patients. Hepatology. 1992;16:1343-1349.
6. Park WG, Yeh RW, Triadafilopoulos G. Injection therapies for variceal bleeding disorders of the GI tract.
Gastrointest Endosc. 2008;67:313-323.
How Can I Tell for Certain Endoscopically, and How Should I Treat it?   11

7. Qureshi W, Adler DG, Davila R, et al. ASGE Guideline: the role of endoscopy in the management of variceal
hemorrhage, updated July 2005. Gastrointest Endosc. 2005;62:651-655.
8. Garcia-Tsao G, Sanyal AJ, Grace ND, Carey WD. Prevention and management of gastroesophageal varices and
variceal hemorrhage in cirrhosis. Am J Gastroenterol. 2007;102:2086-2102.
9. Ninoi T, Nakamura K, Kaminou T, et al. TIPS versus transcatheter sclerotherapy for gastric varices. Am J Roentgenol.
2004;183(2):369-376.
3
QUESTION

WHAT ARE THE ESSENTIAL TOOLS FOR


REMOVING ESOPHAGOGASTRIC FOREIGN BODIES,
AND WHEN SHOULD I APPLY THESE DEVICES?

Luo-wei Wang, MD, PhD and Zhao-shen Li, MD

Foreign-object ingestion and food-bolus impaction occur commonly. The majority of foreign
bodies (ie, foreign objects and food bolus) that reach the stomach will pass spontaneously.
However, 10% to 20% of cases will require nonoperative intervention, and approximately 1% will
require surgical procedures.1 Most cases of foreign body ingestion occur in the pediatric age group,
with a peak incidence between 6 months and 6 years. 2 In adults, foreign object ingestion occurs
more commonly among patients with psychiatric illnesses, mental retardation, alcoholic intoxica-
tion, pica, and those seeking secondary gain with access to a medical facility. 3,4
Since the first report in 1972 on the removal of a foreign body with a flexible endoscope by
McKechnie,5 there has been an increasing application of this method because of its advantages,
which include avoidance of surgeries for most patients, a reduced cost, easy accessibility to the
esophagus with endoscopy, excellent visualization and simultaneous diagnosis of underlying
diseases, and a low morbidity.

Preparation Before Endoscopic


Removal of Foreign Bodies
Prior to endoscopic extraction of a foreign body, information regarding the type, form, and size
of the foreign body is required to plan the strategy of removal and to select the appropriate instru-
ments. If necessary, it may be useful to conduct a dry run of the procedure before performing it
on the patient. A plain radiograph of the upper GI tract with contrast study may be necessary. If
a perforation is suspected, water-soluble contrast such as Gastrografin (Bracco Diagnostics Inc) is

Leung J, Lo SK, eds. Curbside Consultation in Endoscopy:


13 49 Clinical Questions, Second Edition (pp 13-19)
© 2014 SLACK Incorporated
14  Question 3

A B
Figure 3-1. Accessories used to remove foreign bodies. (A) Biopsy forceps, (B) biopsy forceps with spike.
(continued)

preferred. If an esophageal foreign body is suspected, a plain radiograph of the chest, including the
neck region, should be taken to rule out foreign bodies impacted at the cricopharyngeal sphincter.
Children and uncooperative adults often require endotracheal intubation and general anesthesia
to insure that the procedure is carried out safely and successfully.

Endoscopes and Accessories


Standard flexible endoscopes are used in adult patients. A smaller, flexible nasoendoscope with
an outer diameter of 6 mm and a special oral retainer is used in children younger than 3 years.
A double-channel endoscope is used to extract complex and ultra-long objects such as dental
prostheses and chopsticks. Accessories used to remove foreign bodies include rat-tooth forceps,
snare, W-shaped forceps, retrieval basket, tripod forceps, biopsy forceps, and alligator jaws forceps
(Figure 3-1). A latex protector hood (Figure 3-2) or an overtube (Figure 3-3) is used to protect
the upper GI tract during removal of sharp foreign bodies. Patients with psychiatric illness need
general anesthesia with close monitoring of the heart rate and blood pressure. Other patients may
need IV sedation with diazepam when necessary.

When Should I Apply Endoscopy and the Accessories?


Standard and therapeutic endoscopes are used. Equipment that is essential includes rat-tooth
and alligator forceps, polypectomy snare, polyp grasper, retrieval net, overtubes of esophageal and
gastric lengths, and a foreign body protector hood. Different endoscopic methods and equipment
are used depending on the types of foreign bodies. Food bolus can usually be removed en bloc
(Figure 3-4) or in a piecemeal fashion using the retrieval basket (Figure 3-5), rat-tooth forceps, or
snare. When the endoscope, upon air insufflation and distention of the esophageal lumen, can be
steered around the food bolus and into the stomach, the endoscope can be pulled back and used
to gently push the food bolus into the stomach. Food boluses should always be pushed carefully
What Are the Essential Tools for Removing Esophagogastric Foreign Bodies?  15

C D

E F
Figure 3-1 (continued). (C) Alligator-rat tooth forceps, (D) Dormia basket, (E) polypectomy snare, and (F)
tripod forceps.

into the stomach under direct vision. Coins can be removed easily with a foreign body forceps
(rat-tooth, alligator jaws) (Figure 3-6), a snare, or retrieval net.6 Fish bones (Figure 3-7) and
dental prostheses (Figure 3-8) are sharp-pointed objects commonly ingested by adults and elderly
men. Sharp-pointed objects lodged in the esophagus present a medical emergency. Endoscopic
retrieval of these objects is accomplished with retrieval forceps (rat-tooth or alligator jaws forceps)
or a snare. The risk of mucosal injury during sharp-object retrieval can be minimized by orienting
the object with the point trailing during extraction and by using an overtube or a protector hood
attached to the end of the endoscope. Webb6 reported a 98.8% success rate of endoscopic removal
of foreign bodies and a 1.2% failure rate. Li et al7 reported a success rate for removal of esophageal
foreign bodies with a flexible endoscope in 94.1% of cases, with a failure rate of 5.9%. Most of
the foreign bodies that fail endoscopic removal are dental prostheses, iron slices, or complex and
16  Question 3

Figure 3-2. Latex protector hood during


removal of sharp foreign bodies.

Figure 3-3. Remove sharp foreign bodies with


the overtube.

ultra-long objects. Dental prostheses always impact into the esophageal mucosa, usually involving
the muscular layer. In clinical practice, if the risk of esophageal perforation and bleeding is high,
as in those cases with dental prostheses deeply embedded into the esophageal wall, it is better to
avoid endoscopic attempts and to refer the patient directly to surgery.
What Are the Essential Tools for Removing Esophagogastric Foreign Bodies?  17

Figure 3-4. Food bolus removed en bloc using


the retrieval basket.

Figure 3-5. Food bolus removed in a piece-


meal fashion using the retrieval basket.

Long objects are difficult to remove endoscopically because of difficulty in orientation and
grasping. In a retrospective analysis of 542 cases of foreign body ingestion, Velitchkov et al found
that all 17 spoon handles longer than 6 cm remained within the stomach and required surgical
removal.8

Complications
A complication rate of up to 5% is reported for endoscopic treatment, including mucosal
laceration, bleeding, pyrexia, and in severe cases, esophageal perforation.9 The mucosal laceration
and bleeding can be treated immediately by endoscopy. Patients with pyrexia should be covered
with antibiotics for 2 days. However, patients with esophageal perforation will need surgery.10
18  Question 3

Figure 3-6. Coins removed with rat-tooth


forceps.

Figure 3-7. Fish bones removed with rat-tooth


forceps.

Figure 3-8. Dental prostheses removed with


snare.
What Are the Essential Tools for Removing Esophagogastric Foreign Bodies?  19

References
1. Eisen GM, Baron TH, Dominitz JA, et al. Guideline for the management of ingested foreign bodies. Gastrointest
Endosc. 2002;55:802-806.
2. Hachimi-Idrissi S, Corne L, Vandenplas Y. Management of ingested foreign bodies in childhood: our experience
and review of the literature. Eur J Emerg Med. 1998;5:319-323.
3. Blaho KE, Merigian KS, Winbery SL, et al. Foreign body ingestions in the emergency department: case reports and
review of treatment. J Emerg Med. 1998;16:21-26.
4. Kamal I, Thompson J, Paquette DM. The hazards of vinyl glove ingestion in the mentally retarded patient with
pica: new implications for surgical management. Can J Surg. 1999;42:201-204.
5. McKechnie JC. Gastroscopic removal of a phytobezoar. Gastroenterology. 1972;62:1047-1051.
6. Webb WA. Management of foreign bodies of the upper gastrointestinal tract: update. Gastrointest Endosc.
1995;41:39-51.
7. Li ZS, Sun ZX, Zou DW, et al. Endoscopic management of foreign bodies in the upper-GI tract: experience with
1088 cases in China. Gastrointest Endosc. 2006;64:485-492.
8. Velitchkov NG, Grigorov GI, Losanoff JE, et al. Ingested foreign bodies of the gastrointestinal tract: retrospective
analysis of 542 cases. World J Surg. 1996;20:1001-1005.
9. Silva RG, Ahluwalia JP. Asymptomatic esophageal perforation after foreign body ingestion. Gastrointest Endosc.
2005;61:615-619.
10. Management of ingested foreign bodies and food impactions. ASGE Standards Practice Committee. Gastrointest
Endosc. 2011:73:1085-1091.
4
QUESTION

YOU WERE CALLED BY THE ER PHYSICIAN REGARDING


A FEMALE WHO ATTEMPTED TO COMMIT SUICIDE BY DRINKING
TOILET CLEANING SOLUTION. THERE WERE OBVIOUS BURNS
AROUND THE MOUTH AND TONGUE.
WHAT WOULD YOU DO NEXT?

Rajesh Gupta, MD, DM and D. Nageshwar Reddy, MD, DM, FRCP, DSc

Ingestion of a corrosive substance is a common medical emergency. In adults, it is generally


deliberate with suicidal intent. The first task while attending such a patient is to assess his or her
hemodynamic stability. Whenever respiratory symptoms are present, the airway must be secured,
either by endotracheal intubation or tracheostomy. When the patient is stable hemodynamically,
the detailed history and physical examination is performed. There is poor correlation between
symptoms and extent of injury, as many patients are asymptomatic initially, and symptoms may
be delayed for several hours. After examination and laboratory investigations, chest and upright
abdominal radiographs are advised. If radiographs are normal and clinical suspicion of perforation
is high, water-soluble contrast enhanced computed tomography (CT) of the chest or abdomen is
advised.1,2 Several studies have documented the efficacy and safety of early esophagogastroduo-
denoscopy (EGD) in corrosive-induced GI injuries. Early EGD assesses not only the grade of
injury but helps in triaging and planning further management (Figure 4-1).3 The patients with
normal EGD, comprising approximately 40% to 80% of patients, can be safely discharged. Those
with grade I and grade IIA injuries (edema, erythema, erosions, and circumferential ulcers) can be
started on a liquid diet and can be discharged after observation of 48 to 72 hours. Patients with
grade IIB and III injuries (circumferential ulcers—multiple deep ulcers with brown, black, or gray
discoloration) need close monitoring for the next 2 to 3 days because the possibility of short- and
long-term complications is very high. Of this group of patients, 70% to 100% develop strictures.
Patients with a grade IV injury need emergency surgery.4 EGD is not recommended if the patient
has evidence of perforation or delayed presentation (more than 7 days) after corrosive ingestion.
Animal studies have shown that immediate neutralization (within 30 minutes) of corrosive
substance may reduce the extent of injury. However, this apparent benefit has not been proven in
human studies. In fact, many experts believe that heat produced during the neutralization reac-
tion may exacerbate the extent of injury. Based on current data, the use of neutralizing agents or

Leung J, Lo SK, eds. Curbside Consultation in Endoscopy:


21 49 Clinical Questions, Second Edition (pp 21-23)
© 2014 SLACK Incorporated
22  Question 4

A B
Figure 4-1. (A) Endoscopic view showing
grade I damage (diffuse erythema, superficial
exudate) of esophageal mucosa. (B) Endoscopic
view showing grade II (moderate ulceration)
damage of esophageal mucosa. (C) Endoscopic
view showing grade III damage (severe, deep
ulceration, and spontaneous hemorrhage) of
esophageal mucosa. (Reprinted from Poley
JW, Steyerberg EW, Kuipers EJ, et al. Ingestion
of acid and alkaline agents: outcome and
prognostic value of early upper endoscopy.
Gastrointest Endosc. 2004;60(3):372-377, with
C permission from Elsevier.)

emetics is not recommended. There is no evidence to suggest that glucocorticoids prevent stric-
ture formation. In fact, the administration of glucocorticoids may mask the signs of peritonitis or
mediastinitis and increase the risk of infection. There are no studies to suggest a beneficial effect
of the prophylactic use of antibiotics. Most experts recommend that antibiotics should be used only
if there is a specific indication.
Nutrition is an important issue in patients with acute corrosive injuries. Oral feeding can be
started early if EGD is normal or shows grade I or grade IIA injuries. If the patient is unable to
swallow or perforation is suspected, parenteral nutrition may be started or feeding jejunostomy
may be created. There are no data to suggest that oral feeding is injurious in acute corrosive
injuries. A nasogastric tube insertion for enteral feeding may be used if oral feeds are not toler-
ated. Feeding jejunostomy is recommended if the patient continues to have severe odynophagia or
absolute dysphagia.
The role of early dilatation in acute corrosive injuries is controversial. There are no data to sug-
gest that early dilatation prevents esophageal stricture. On the contrary, early dilatation increases
the risk of complications and may accelerate fibrosis and stricture formation. The dilatation of an
esophageal stricture is initiated after 4 to 6 weeks of corrosive injury. The role of the retrievable
self-expanding metal stent (SEMS) in corrosive stricture has been recently evaluated. Although
the early response appears promising, the long-term results are not encouraging. Hence, the use of
SEMS must be limited to a select group of patients with refractory esophageal stricture. Because
the ingestion of corrosive agents in adults is usually a suicide attempt, psychiatric evaluation and
counseling is recommended.
What Would You Do for Obvious Burns Around the Mouth and Tongue?  23

References
1. Kikendall JW. Caustic ingestion injuries. Gastroenterol Clin North Am. 1991;20(4):847-857.
2. Cellor JP, Fogel RP, Boland R. Liquid caustic ingestion. Spectrum of injury. Arch Intern Med. 1980;140(4):501-504.
3. Zargar SA, Kochhar R, Mehta S, Mehta SK. The role of fiberoptic endoscopy in the management of corrosive inges-
tion and modified endoscopic classification of burns. Gastrointest Endosc. 1991;37(2):165-169.
4. Estrera A, Taylor W, Mills LJ, Platt MR. Corrosive burns of the esophagus and stomach: a recommendation for an
aggressive surgical approach. Ann Thorac Surg. 1986;41(3):276-283.
5
QUESTION

WHAT IS THE CURRENT RECOMMENDATION FOR


ENDOSCOPIC SURVEILLANCE OF BARRETT'S ESOPHAGUS,
AND HOW CAN WE IMPROVE THE RESULTS WITH
THE CURRENT TECHNOLOGIES?

Paulo Sakai, MD, PhD, FASGE and Fauze Maluf-Filho, MD, PhD, FASGE

Several observational studies have suggested that patients with Barrett's esophagus (BE), in
whom adenocarcinoma was detected in a surveillance program, have lesions at an earlier stage.
The 5-year survival rate has been improved, compared with patients who have not undergone
routine endoscopic surveillance.1 Therefore, current practice guidelines recommend endoscopic
surveillance of patients with BE in an attempt to detect cancer at an early and potentially curable
stage.
A cellular atypia confined to the epithelium is called dysplasia and is considered a histological
marker for the development of invasive adenocarcinoma in BE. According to the American
Society for Gastrointestinal Endoscopy (ASGE) guideline, 2 for patients with established BE of
any length but with no dysplasia, after 2 consecutive examinations within 1 year, an acceptable
interval for additional surveillance is at every 3 to 5 years. No surveillance and epithelium ablation
with radiofrequency are 2 other possible strategies that can be considered in select patients with
nondysplastic BE. If the presence or degree of dysplasia is indeterminate, the opinion of an expert
GI pathologist is recommended. If there is evidence of acute inflammation due to gastroesophageal
acid reflux, repeat biopsy should be performed after 8 weeks of effective acid-suppression therapy.
Surveillance in patients with confirmed low-grade dysplasia (LGD) is recommended and a follow-
up esophagogastroduodenoscopy (EGD) at 6 months should be performed, with biopsies in the
area of dysplasia. If LGD is confirmed, management includes surveillance at 12 months (4-quad-
rant biopsies every 1 to 2 cm) and yearly thereafter as long as dysplasia persists. Endoscopic resec-
tion or ablation with radiofrequency should be considered in these cases. Patients with high-grade
dysplasia (HGD) are at significant risk for prevalent or incidental cancer. HGD, confirmed by
an expert GI pathologist, can be treated either by esophagectomy or by endoscopic management,

Leung J, Lo SK, eds. Curbside Consultation in Endoscopy:


25 49 Clinical Questions, Second Edition (pp 25-28)
© 2014 SLACK Incorporated
26  Question 5

Figure 5-1. (A) Endoscopic


view of Barrett´s esophagus
using white light and (B) the
same lesion examined with
the NBI system.

A B

which may consist of either endoscopic therapy or heightened surveillance. Patients who are sur-
gical candidates may elect to have definitive therapy. Patients who elect surveillance endoscopy
should undergo follow-up every 3 months for at least 1 year, with multiple biopsies obtained at
1-cm intervals. After 1 year of no cancer detection, the interval of surveillance may be lengthened
if there are no dysplastic changes on 2 subsequent endoscopies performed at 3-month intervals.
Ablation of the metaplastic and dysplastic epithelia should also be considered for patients with
confirmed HGD. The role of endoscopic ultrasound (EUS) in patients with HGD BE is not clear.
Some advocate the use of EUS to detect unrecognized malignant periesophageal lymphadenopa-
thies.
There is minor variation between societal recommendations regarding surveillance of HGD.
The American College of Gastroenterology (ACG) guidelines recommend a repeat endoscopy,
with biopsies for patients in whom HGD is diagnosed; focal HGD (less than 5 crypts) can be
followed by 3-month surveillance endoscopy, whereas endoscopic or surgical intervention is
recommended for multifocal HGD.
Surveillance programs may become difficult in the general clinical practice, and many
clinicians may not follow current practice guidelines. The strategy to perform a systematic jumbo
biopsy protocol at 1- to 2-cm intervals, plus biopsy of any mucosal abnormalities along the entire
length of the BE, has proven to miss cancer in patients with HGD. By using newer endoscopic
techniques, we can target the biopsies to areas with a higher probability of harboring dysplasia or
cancer, thus improving the efficiency and sampling in surveillance programs. Techniques such as
magnification endoscopy, chromoendoscopy with different stains, and instillation of 1.5% acetic
acid have been examined and found to improve the accuracy of BE detection and early neoplasia
compared with standard endoscopy in clinical trials.
Recently, narrow-band imaging (NBI) was developed, which is a special imaging technique
that uses filter systems that allow passage of spectral components of light, mainly blue color. This
new endoscopic technique using high-definition television (HDTV) improves visualization and
more details of the mucosal surface and capillary system without the use of dyes (Figure 5-1).
In several published studies, NBI in combination with magnification endoscopy seems to be a
promising tool in the diagnosis of BE and abnormalities of structural surface patterns, as well as
irregular microvascular patterns, predicting dysplastic areas with high sensitivity and specificity. 3
One study compared NBI with high-resolution endoscopy in patients with known BE undergoing
surveillance. The majority of HGD and early cancer could be detected by using chromoendoscopy
and high-resolution endoscopy alone, with a statistically similar detection rate when compared
with NBI alone.4
The NBI technique is useful for BE surveillance, and the instillation of acetic acid on the
mucosal surface can enhance the contrast, disclosing areas of subtle mucosal abnormalities to
target biopsies. However, the use of high magnification with NBI may be confusing because the
meaning of mucosal and vascular patterns requires a learning curve, and one single international
How Can We Improve the Results With the Current Technologies?   27

Figure 5-2. (A) Endoscopic


view of Barrett´s
esophagus, using a high-
resolution endoscope and
(B) the view after instilling
1.5% acetic acid.

A B

A B C
Figure 5-3. (A) Endoscopic view of BE with HGD, (B) instillation of 1.5% acetic acid, and (C) NBI view.

classification would be desirable. In the future, this new generation of endoscope should be
considered for routine use.
In clinical practice, the NBI system is not always available, and we have recommended the use
of high-resolution endoscope and 1.5% acetic acid instillation in the distal esophagus in patients
with BE, washing out with water after 2 minutes (Figure 5-2). A careful examination can disclose
subtle mucosal abnormalities, such as ulceration, erosion, plaque, nodule, or other luminal irregu-
larity in the Barrett´s segment that should be biopsied. A recent article5 confirmed that the use of
acetic acid had 95% sensitivity and 80% specificity for the detection of neoplastic lesions at BE in
a large cohort of patients. In that study, chromoendoscopy with acetic acid proved to be superior
to white light for the detection of HGD in BE (Figure 5-3). The same group suggested that this
strategy is more cost-effective compared with protocol-guided biopsies, probably because of the
potential to reduce the number of biopsies taken during endoscopic examination of these patients.6

References
1. Wang KK, Wongkeesong M, Buttar NS. American Gastroenterological Association technical review on the role
of the gastroenterologist in the management of esophageal carcinoma. Gastroenterology. 2005;128:1471-1505.
2. ASGE Standards of Practice Committee, Evans JA, Early DS, Fukami N, et al; Standards of Practice Committee of the
American Society for Gastrointestinal Endoscopy. The role of endoscopy in Barrett’s esophagus and other pre-
malignant conditions of the esophagus. Gastrointest Endosc. 2012;76(6):1087-1094. doi:10.1016/j.gie.2012.08.004.
3. Goda K, Tajiri H, Ikegami M, et al. Usefulness of magnifying endoscopy with narrow band imaging for the
detection of specialized intestinal metaplasia in columnar-lined esophagus and Barrett´s adenocarcinoma.
Gastrointest Endosc. 2007;65:36-46.
28  Question 5

4. Kara MA, Peters FP, Rosmolen WD, et al. High-resolution endoscopy plus chromoendoscopy or narrow-band
imaging in Barrett´s esophagus: a prospective randomized crossover study. Endoscopy. 2005;37:929-936.
5. Longcroft-Wheaton G, Duku M, Mead R, Poller D, Bhandari P. Acetic acid spray is an effective tool for the endoscopic
detection of neoplasia in patients with Barrett’s esophagus. Clin Gastroenterol Hepatol. 2010;8(10):843-847.
6. Bhandari P, Kandaswamy P, Cowlishaw D, Longcroft-Wheaton G. Acetic acid-enhanced chromoendosco-
py is more cost-effective than protocol-guided biopsies in high-risk Barrett’s population. Dis Esophagus.
2012;25(5):386-392.
6
QUESTION

WHAT SHOULD I DO IF I HAVE


INADVERTENTLY PERFORATED THE
VISCUS DURING AN ENDOSCOPY?

Gregory Haber, MD

Anticipating higher-risk procedures should prepare the endoscopist for appropriate manage-
ment and a game plan if a perforation actually occurs. High-risk procedures would include the
following:
● Esophagoscopy with a history of dysphagia, especially transfer dysphagia, which occurs with
a Zenker’s diverticulum or a known stricture
● Large paraesophageal hernia
● Prior radiation
● Anastomotic narrowing
● Altered anatomy, as with colonic interposition, Billroth II, Roux-en-Y anastomosis, or hepati-
cojejunostomy and pancreaticojejunostomy
● Use of an oblique or side-viewing endoscope, as with echoendoscopes or duodenoscopes, when
forward visualization is limited
● Presence of luminal content, such as ingested material, blood, or fluid that obscures the desired
orifice
● Large balloon dilation, as for achalasia
● Endoscopic mucosal resection or submucosal dissection of flat lesions or submucosal tumors
If a high-risk situation is known, the scheduling of the procedure should not be at the end of the
day or after hours, when radiological or surgical backup may not be readily available. Contacting
the supportive services in advance to be on hand or to consult the patient in preparation for alter-
native management is prudent. As always, consideration of a referral to a center of expertise is
an option if your experience with the high-risk procedure is limited or if the necessary backup is
Leung J, Lo SK, eds. Curbside Consultation in Endoscopy:
29 49 Clinical Questions, Second Edition (pp 29-32)
© 2014 SLACK Incorporated
30  Question 6

not available in your institution. Likewise, a Friday procedure is not ideal because follow-up on a
weekend is less desirable.

Recognition
Part of recognition is having the tactile sense of the tension being exerted on the wall of the
gut. This is particularly relevant when advancing around sharp angulations in the gut. Examples
include the following:
● Gastrojejunostomy, especially entering the afferent limb, which has a more angulated take off
with short afferent limbs in retrocolic anastomoses
● Enteroscopy with a history of prior surgery or adhesions
● Diverticular disease with tethered sigmoid loops
● Advancing from the hepaticojejunostomy or choledochojejunostomy to the pancreaticojejunos-
tomy within a short fixed loop of jejunum
● Traversing the ligament of Treitz either in a prograde or retrograde direction
● Reversing direction in an enteroenterostomy
Sensing the force being exerted on the wall of the gut is of paramount importance in preven-
tion of perforation. This requires minimizing the resistance at the level of the mouth by ensuring
adequate lubrication around the mouth guard and on the shaft of the scope. If an overtube is
being used, the scope should be similarly well lubricated and the overtube irrigated as needed for
hydrophilic linings. If working with a trainee or if asked to assist another endoscopist, when first
taking over the procedure, the scope should be withdrawn from the point where the endoscopist
is stuck back to a relatively straight position, so as to appreciate the variability in resistance in
reaching the level of obstruction to passage.
The mucosal lining creates a “red out” when the lens of the scope is flush against the wall at
difficult turns; therefore, when no luminal view is available, at least the sliding of the mucosa must
be appreciated as the scope is advanced. If the view is lost due to mucus or foreign material, a water
flush may help and serves to lubricate the mucosa for easier sliding of the scope tip along the wall.
When blanching of the mucosa occurs, the scope needs to be withdrawn in a very slow manner,
as there may be a paradoxic forward force due to unlooping along the shaft when this is done.
A perforation can be caused by the tip of the scope rupturing the gut wall or a bend in the
shaft of the scope tearing the wall, usually in a longitudinal fashion. When a perforation by the
tip occurs, there is initially a loss of view, as the scope is in the wall. At times, due to the increased
resistance at the level of the muscularis propria, there may be time to withdraw, leaving a mucosal
tear only and aborting the procedure before perforation occurs.
The view after a perforation depends on the level of the perforation as one enters the retropha-
ryngeal space, the mediastinum, peritoneal cavity, retroperitoneum, or retrorectal space. Entry
into the peritoneal cavity is easily recognized because the organs, such as the liver or gut, are seen.
However, this may not occur when the mesenteric border of the gut is traversed because only the
fat may be seen. If there are adhesions, which is often the case because prior surgery is a risk factor,
dissection of the associated fibrous tissue may appear like “candy floss.” There may be little to see
if blood or tissue obscures the lens. When entering the retroperitoneum, a loss of view also occurs,
and it is important not to push forward without a luminal view. There may be less resistance to
advancement after the intial perforation, which may give a false sense of security that the scope
has finally traversed a difficult turn, when in fact one may be pushing through the mediastinum
or neck parallel to the true lumen. Once again, irrigation with water and withdrawal to obtain a
better view is mandatory.
What Should I Do if I Have Inadvertently Perforated the Viscus During Endoscopy?   31

Management
Initial management of a perforation no longer dictates an immediate call to a surgeon but rather
an assessment should be made regarding the size and location, and whether it may be remediable
with endoscopic tools. Part of this assessment relates to the clinical indication. For example, if an
anastomotic stricture for the index procedure is likely to require multiple dilations and a perfora-
tion occurs early on, this may tip the scales in favor of surgery for the immediate complication, as
well as for definitive therapy of the underlying problem.
A perforation above the level of the upper esophageal sphincter (eg, through the valecular fossa
or a Zenker’s diverticulum) is always due to the scope tip. This area is difficult to evaluate, as
there are usually pharyngeal contractions and gagging. The best way to assess this is with a clear
cap on the scope to splay apart the folds for better visualization. If access and orientation allow an
adequate field of view, clips with small stems (ie, < 5 mm) may be applied, especially in a diverticu-
lum. Conservative measures include placement of a nasogastric tube under endoscopic vision so as
not to inadvertently advance the tube when it is lodged in the mouth of the perforation. Antibiotics
and acid suppression are routinely given for any perforation in the upper GI tract. Unfortunately,
only a minority of perforations is suitable for endoscopic management, and most require surgery
with a transcervical drain.
Perforations along the body of the esophagus allow for better viewing and access, whereas at the
gastroesophageal junction, the sphincter and puckering of folds may obscure the view of the per-
foration. Once again, a large friction cap on the scope tip, such as a 16-mm oblique Olympus cap,
should open up the area for suitable inspection. In the esophagus, in addition to clip closure, there
is the option of using a covered metal or a solid plastic stent to bridge the perforation and prevent
food and salivary contamination of the mediastinum. Stent options include covered metal stents,
such as the ultraflex or Alveolus (Alveolus Inc) or a solid plastic polyflex stent. Unfortunately,
migration is a major drawback, and attempts to secure the stent include clipping of the upper stent
wire to the esophageal wall or tying a suture line to the top of the stent, which is then brought
out through the nose and secured for a few days to allow the stent to become embedded with a
hyperplastic inflammatory reaction around the stent.
The concern regarding endoscopic therapies is that if they do not succeed, the outcomes of
delayed surgery are generally worse compared with early operation. After endoscopic measures, a
limited radiologic examination with barium is useful to assess the effectiveness of the treatment in
preventing a leak. If the patient is clinically stable with a minimal leak on radiographic studies in
the first 12 hours, ongoing conservative treatment can continue. A white blood cell (WBC) count
less than 15,000 and a temperature less than 99°C can be expected in the first 24 to 48 hours but
should improve thereafter if the endoscopic therapy is adequate.
When the perforation involves the stomach, jejunum, ileum, or colon, with a punched out
hole into the peritoneal cavity, in addition to mechanical closure of the hole with clips, the
pneumoperitoneum may cause respiratory distress and needs to be dealt with as well. Placement
of clips should begin at the ends of the torn tissue because frequently the 12- to 14-mm wingspan
of most clips will not span the defect in the middle. Working from the ends of the tear with
aspiration to reduce distention should allow apposition of the opposing walls of the torn tissue.
If the bowel lumen collapses into the peritoneal cavity due to air loss, the position of the patient
may be changed to elevate the area of the perforation so that the air tends to remain in that part of
the bowel. As air is absorbed slowly and is irritating to the peritoneal lining, air insufflation should
immediately be exchanged for carbon dioxide, if it is available. In addition, large amounts of air
in the peritoneal cavity may impair diaphragmatic movement, and decompression using a simple
16- or 18-Fr angiocath sleeve can bring about immediate relief. One can expect mild degrees of
peritoneal irritation, but if frank peritonitis occurs, surgical intervention will be necessary. When
32  Question 6

the endoscopic therapy is adequate, there may be a low-grade fever or mild leukocytosis, but the
patient should not become tachycardic or toxic and should maintain bowel sounds.
If the tear occurs at an anastomotic site, the narrowed-stricture lumen may be tight enough
to hold a covered stent in place for a sufficient time to allow healing. Conservative measures are
more successful when there are adhesions and adherent loops of bowel that tend to localize the
spillage from the perforation and may spontaneously wall off the area. The same is true with
retroperitoneal perforations in the duodenal sweep or rectum. The lack of free peritoneal air
reduces the clinical impact and improves the chance for definitive endoscopic treatment.
With larger perforations, as can occur with endoscopic submucosal dissection or use of a cap
to suck the tissue prior to snare resection, loops have been used with limited success to entrap the
folds around the perforation and then to cinch them together. One described method requires
a double-channel endoscope. An endoloop is passed down one channel and through the second
channel; clips are deployed over the loop to secure the loop to the margins of the perforation. The
arms of the closed clips should have enough space between them to allow movement of the loop,
and as the loop is tightened, the tissue folds are drawn together with the clips, securing the tissue
to the loop. Another approach has been the use of mesentery to fill the gap of the perforation and
then clipping the margins of the perforated wall to the mesentery. This requires a grasping forceps
to pull the mesentery into the hole of the perforation and then applying multiple clips around the
circumference to hold the mesentery in place.

Conclusion
The presence of air in the peritoneal cavity no longer needs to be considered the clarion
call for surgical intervention. With increasing frequency, in appropriately selected patients and
perforations, endoscopic therapy and associated supportive measures can provide definitive
treatment.
7
QUESTION

WHAT IS THE CURRENT EXPERT OPINION ON


WHAT HEMOSTASIS TECHNIQUE TO USE IN TREATING A
VISIBLE VESSEL OR DIEULAFOY LESION?
WHEN SHOULD I INVOLVE A SURGEON?

Brintha K. Enestvedt, MD, MBA and Michael L. Kochman, MD, FACP

Advances in endoscopic therapy have greatly improved patient outcomes in the management of
upper gastrointestinal (UGI) bleeding due to peptic ulcer disease and other UGI arterial lesions,
such as Dieulafoy lesions. The prevalence of high-risk stigmata of bleeding for patients with peptic
ulcers is approximately 5% to 10% for active arterial spurting and 20% to 25% for nonbleeding
visible vessels (Figure 7-1). Endoscopic therapy reduces the rate of rebleeding from active arterial
spurting from 80% to 90% (from medical therapy alone) to 15% to 20% and from 40% to 50%
down to 10% for visible vessels. Endoscopic therapy also decreases blood transfusion requirements
and length of hospital stay.1
A variety of modalities are available for the management and treatment of these lesions,
including thermal contact devices, mechanical hemostasis (clips), and epinephrine injection. For
the initial diagnosis and treatment of suspected UGI bleeding, a therapeutic endoscope with a
large suction channel (3.7 mm) is recommended. The mainstay of therapy for bleeding peptic
ulcers with high-risk stigmata (active arterial spurting and visible vessel) is a combination of
endoscopic therapy with epinephrine injection and either mechanical hemostasis or contact ther-
mal therapy. Epinephrine injection alone has been demonstrated to be inferior to combination
endoscopic therapy in achieving hemostasis, improving rebleeding rates, decreasing hospital stay,
and need for operative intervention.2 The stigmata of an adherent clot in an ulcer bed warrants
targeted irrigation in an attempt at dislodgement so as to apply therapy to the underlying vessel.
Contact thermal devices include multipolar probes and heater probes. Electrical bipolar probe
devices, such as the commonly used multipolar probes (gold probe), achieve hemostasis by heating
contacted tissue via passage of electrical current through the electrodes attached to the tip or sides
of the tip of the probe. Resistance to further coagulation occurs with increasing desiccation of

Leung J, Lo SK, eds. Curbside Consultation in Endoscopy:


33 49 Clinical Questions, Second Edition (pp 33-36)
© 2014 SLACK Incorporated
34  Question 7

Figure 7-1. (A) Duodenal ulcer with visible


vessel. (B) Coaptive footprint post treatment
of visible vessel with bipolar cautery following
epinephrine injection.

coagulated tissue and thereby theoretically limits the depth of injury. Optimally, a 10-Fr probe
is used and coagulation may be performed at 15 to 20 watts, and 30 Joules (continuous) applied
after initial irrigation of bleeding site. The duration of the therapy and the pulse frequency varies
by report, but it is important that flattening of the bleeding lesion be achieved with the probe
tip to cause coaptive thermal coagulation. A general recommendation is to apply approximately
10 seconds of tamponade with firm pressure to achieve flattening of the vessel. Heater probes
apply energy via a ceramic tip, which can heat more quickly. These are not limited in effect
by tissue resistance and can achieve greater depth of coagulation; however, their use carries a
higher risk of gastrointestinal perforation. For active arterial spurting and nonbleeding visible
vessels, epinephrine injection (1:10,000 in normal saline) is applied in 0.5- to 2.0-mL aliquots in
4 quadrants along the ulcer base prior to application of thermal contact therapy or clip application.3
Injection therapy can often reduce the rate of active bleeding to allow for improved and optimal
visualization for application of contact thermal therapy or clip. However, it is clear that injection
therapy alone as definitive treatment is not recommended.4
Mechanical hemostasis with hemoclips is also an effective treatment for arterial bleeding
lesions. Hemoclips are metallic grasping devices that are passed through the biopsy channel of
the endoscope, as part of a dedicated clip application system, achieving hemostasis via mechanical
What Hemostasis Technique to Use in Treating a Visible Vessel?   35

Figure 7-2. Gastric Dieulafoy lesion with active


oozing.

compression of a bleeding vessel. One advantage of these devices is ease of use. Several clips may
be necessary to achieve adequate hemostasis.
Dieulafoy lesions (exulceratio simplex, caliber persistent vessel) are rare, accounting for
approximately 1.9% of all patients with acute gastrointestinal hemorrhage. They are often present
with severe intermittent bleeding and can often evade diagnosis due to their discontinuous pattern
of bleeding. These lesions are characterized by the presence of a relatively large artery in close
proximity to the mucosal surface in the absence of mucosal ulceration (Figure 7-2). They are most
commonly seen in the proximal stomach within 6 cm of the gastroesophageal junction along the
lesser curvature of the stomach; however, they can be found throughout the gastrointestinal tract.
Endoscopic criteria for diagnosis include 1) active arterial spurting or micropulsatile streaming
from a < 3-mm mucosal defect (essentially the vessel itself), 2) visualization of a protruding vessel
with or without active bleeding within a diminutive mucosal defect with normal surrounding
mucosa, or 3) a densely adherent clot with a narrow point of attachment to a diminutive mucosal
defect or normal-appearing mucosa, and the absence of ulceration. Once identified, endoscopic
therapy is highly successful, with hemostasis achieved in 96% of cases. Similar to the management
of peptic ulcer disease, given the high risk of rebleeding, combination endoscopic therapy with
epinephrine injection and thermal contact or hemoclip application is recommended. Band ligation
has also been applied with success. Placement of a submucosal tattoo can aid in reidentification of
the notoriously elusive lesion should rebleeding occur.5
Endoscopic therapy for bleeding ulcers and Dieulafoy lesions is highly effective. Given the
advances in endoscopic therapy and interventional radiology (arterial embolization) in the effective
management of bleeding from the UGI tract, surgery is reserved for cases in which maximal
endoscopic therapy has failed or interventional radiology is not available. However, it is advis-
able to have a surgeon involved in cases where initial endoscopic hemostasis is ineffective, where
the presentation itself was life threatening, and in patients with lesions that are at high risk for
rebleeding. Often, endoscopic therapy is repeated before a definitive surgical or interventional
radiology treatment occurs in such cases.
36  Question 7

References
1. Laine L, Jensen DM. Management of patients with ulcer bleeding. Am J Gastroenterol. 2012;107(3):345-360.
2. Barkun AN, Bardou M, Kuipers EJ, et al. International consensus recommendations on the management of
patients with nonvariceal upper gastrointestinal bleeding. Ann Intern Med. 2010;152(2):101-113.
3. Gralnek IM, Barkun AN, Bardou M. Management of acute bleeding from a peptic ulcer. N Engl J Med.
2008;359(9):928-937.
4. Marmo R, Rotondano G, Piscopo R, Bianco MA, D’Angella R, Cipolletta L. Dual therapy versus monotherapy in
the endoscopic treatment of high-risk bleeding ulcers: a meta-analysis of controlled trials. Am J Gastroenterol.
2007;102(2):279-289; quiz 469.
5. Lara LF, Sreenarasimhaiah J, Tang SJ, Afonso BB, Rockey DC. Dieulafoy lesions of the GI tract: localization and
therapeutic outcomes. Dig Dis Sci. 2010;55(12):3436-3441.
8
QUESTION

WHAT DO PEOPLE DO FOR THE TREATMENT


OF ACHALASIA THESE DAYS? DOES IT MATTER
IF THE SYMPTOMS ARE SEVERE OR IF THE
PATIENT IS ELDERLY?

Drew Ingram, MD and Joseph Leung, MD, FRCP, FACP, FASGE, MACG

Achalasia is an esophageal motility disorder characterized by the absence of esophageal peri-


stalsis and ineffective lower esophageal sphincter (LES) relaxation. It is an uncommon disorder
that affects both sexes and all races equally. Onset can occur in any decade of life but is rare before
adolescence. Although the cause remains unknown, achalasia is thought to develop secondary to
inflammation of the myenteric plexus, which thereby disrupts LES relaxation.
Acetylcholine is the neurotransmitter that causes contraction of the smooth muscle of the lower
esophagus, whereas nitric oxide and vasoactive intestinal polypeptide (VIP) promote relaxation. In
achalasia, it is believed that inflammation of the myenteric plexus results in the loss of inhibitory
signals to the lower esophagus. The unopposed stimulation of smooth muscle leads to a loss of
peristalsis and failure of the LES to relax. An ongoing hypothesis suggests that a viral infection
is responsible for triggering this inflammation.1 Furthermore, studies suggest that in genetically
susceptible individuals, autoantibodies to the myenteric neurons are formed, which may be the
cause for the chronic loss of the esophageal inhibitory signals.1 The resultant dysmotility eventu-
ally leads to the development of symptoms.
The clinical manifestations of achalasia vary, but commonly patients present with dysphagia to
both solids and liquids. Patients may also experience heartburn, regurgitation of undigested food,
weight loss, and retrosternal chest pain. Respiratory symptoms secondary to aspiration, along with
progression to esophageal squamous cell carcinoma, are other potential sequelae. Thus, timely
diagnosis is critical.
When achalasia is suspected, a combination of radiologic, manometric, and endoscopic studies
may be used. A barium swallow typically reveals a dilated esophagus with constriction of the LES,
classically known as the “bird’s beak” appearance. Subsequently, esophageal manometry is often
performed, revealing aperistalsis and impaired relaxation of a hypertensive LES. These findings

Leung J, Lo SK, eds. Curbside Consultation in Endoscopy:


37 49 Clinical Questions, Second Edition (pp 37-39)
© 2014 SLACK Incorporated
38  Question 8

are characteristic of achalasia. With the advent of high-resolution manometry (HRM), it is now
possible to classify achalasia into three subtypes (I, II, III), each of which has been found to have
a different response to available therapies. Although confirmatory studies are still needed, this
discovery may help practitioners to choose the best treatment modality based on HRM subtype.1-3
Although a barium study and manometry may suggest achalasia, esophagogastroduodenoscopy
(EGD) should be performed to visualize the esophagus, as well as the cardia of the stomach. A
tumor at the lower esophagus or cardia may invade the myenteric plexus, leading to secondary
achalasia, or “pseudoachalasia.” Another, less common cause of secondary achalasia is infection
by Trypanosoma cruzi, or Chagas disease. The infection is endemic to rural, undeveloped areas of
Central and South America. Based on the results of the esophagram, manometry, and EGD, a
diagnosis of achalasia can be made, and treatment options are then considered.
It is important to note that achalasia has no known cure. Treatment is palliative and focuses
on reducing LES pressure to relieve symptoms. Current treatment options include medications,
botulinum toxin injection, pneumatic balloon dilation, and surgical myotomy. Recently, peroral
endoscopic myotomy (POEM) has also been used for the management of this disease.
Regarding medications, both calcium channel blockers and nitrates have been used. Calcium
channel blockers cause LES relaxation by inhibiting the uptake of calcium into muscle cells, while
nitrates replace the depleted nitric oxide of the LES. The medications are taken 30 to 60 minutes
before meals, and the improvement in dysphagia is variable, with some studies reporting 50% effi-
cacy.³ Moreover, common unwanted side effects, such as hypotension, headache, and peripheral
edema, as well as the development of tolerance to these medications, continue to limit their use
and further intervention is often required.
Endoscopically, the use of botulinum toxin A (Botox) and pneumatic dilation have both been
used to reduce the pressure gradient at the LES. When injected into the lower esophagus, botuli-
num toxin inhibits the stimulatory effect of acetylcholine on the smooth muscle, thereby decreas-
ing LES pressure. Botulinum toxin is commonly packaged in 100-unit (U) vials, which is diluted
in 4 mL of normal saline. Using a sclerotherapy needle, the Botox is injected in a 4-quadrant man-
ner into the LES, delivering roughly 20 to 25 U into each quadrant. Doses greater than 25 U have
not demonstrated greater benefit in the treatment of achalasia. Results with Botox have shown
success at decreasing LES tone and improving esophageal emptying. However, this effect will
diminish over the course of several months and repeat injections may be necessary. Furthermore,
the development of antibodies to Botox further limits the long-term efficacy of this treatment.
Contraindications to botulinum toxin include pregnancy, allergy to albumin, and reaction to pre-
vious injections. Side effects can include reflux, skin rash, and chest discomfort.
Pneumatic dilation of the LES is another endoscopic alternative that has been recently shown
to have a high rate of success as well as a good safety profile in the European Achalasia Trial.³
Prior to performing pneumatic dilation, adequate sedation is essential, as this procedure may be
uncomfortable. When the patient is sedated, the endoscope is advanced into the stomach and a
soft-tip guidewire is placed across the gastroesophageal (GE) junction. A Rigiflex 30-mm dilation
balloon (Boston Scientific) is advanced over the guidewire and positioned across the GE junction.
Under fluoroscopy, the balloon is inflated slowly with air to reach the predetermined pressure. A
“waist” is often seen on the partially inflated balloon, suggestive of increased muscle tone. With
full balloon insufflation, this waist formation resolves, indicating adequate dilation. The balloon
is then deflated, and a final fluoroscopic image is taken to rule out any free air that may suggest
a perforation. A good portion of patients will have improvement in symptoms after the initial
dilation to 30 mm, but some may require repeat dilation with a 35-mm balloon. If symptoms still
persist, dilation with a 40-mm balloon can be considered. The main risk of pneumatic dilation is
esophageal perforation, which occurs at a rate of less than 3% when performed by an experienced
endoscopist. The risk of perforation is highest with the first dilation. Therefore, we recommend
performing initial dilation under fluoroscopy with a 30-mm balloon.
What Do People Do for the Treatment of Achalasia These Days?   39

Surgical myotomy as a treatment for achalasia involves an incision through the muscle of the
LES. Myotomy has a very high rate of success in treating dysphagia, and the response usually lasts
for many years. When performed laparoscopically in the hands of an experienced surgeon, the
procedure is well-tolerated with low rates of complication. Although surgical myotomy improves
dysphagia, gastroesophageal reflux can occur after the procedure. For this reason, many surgeons
will perform a Dor fundoplication at the time of myotomy. Recently, an alternative to surgical
myotomy has been explored.
Peroral endoscopic myotomy (POEM) is a newly developed endosurgical method to treat
achalasia. In the distal esophagus, endoscopic access is gained into the submucosal space and
blunt dissection is performed to create a tunnel, through which the circular muscle fibers of the
LES are encountered and dissected under endoscopic visualization. General anesthesia is used
to ensure adequate sedation and patients are monitored in the hospital following the procedure.
Recent reports show a high success rate (94% to 100%) with minimal complications.4 Although
long-term follow-up is still needed, this novel therapy provides an exciting new option for the
endoscopic management of achalasia.
After a diagnosis of achalasia is made, the best approach to therapy should be decided based on
the patient’s age and comorbidities.5 In addition, surgical myotomy may be more difficult to per-
form in patients who have undergone prior botulinum toxin injections due to resultant scar tissue,
making it important to identify surgical candidates at the onset of their disease course. In a young,
healthy achalasia patient, referral to an experienced surgeon for laparoscopic myotomy is the treat-
ment of choice. For patients aged 50 or older, pneumatic dilation and botulinum toxin injections
should be considered, especially if the patient is not the best surgical candidate. Pneumatic dila-
tion has a longer effect than Botox injection; however, there is a risk of perforation. The use of
botulinum toxin has a lower rate of complications, but it may become expensive because it requires
frequent endoscopies given its limited effect. Hence, in the patient with contraindications to surgi-
cal myotomy and whose life expectancy is greater than 5 years, we prefer dilation on the basis of its
longer therapeutic effect. Notably, for those who present with severe symptoms and a manometric
diagnosis of vigorous achalasia (type III), it is advisable to consider a surgical myotomy or Botox
injection, as these patients do not respond well to pneumatic dilation.
If patients begin to experience dysphagia or other manifestations of achalasia after a prior
successful treatment, a repeat EGD with possible intervention should be performed. Again,
patients should be educated that the treatment options for achalasia do not cure the disorder and
that symptoms can recur at any time. Monitoring for squamous cell esophageal carcinoma is also
recommended in patients with achalasia for more than 15 years. Although the optimal screening
interval and technique has yet to be determined, a combination of advanced imaging techniques,
such as narrow band imaging and Lugol’s stain, have been used with success to direct biopsies and
brushings to rule out underlying malignancies.4

References
1. Chuah SK, Hsu PI, Wu KL, Wu DC, Tai WC, Changchien CS. 2011 Update on esophageal achalasia. World J
Gastroenterol. 2012;18(14):1573-1578. doi:10.3748/wjg.v18.i14.1573.
2. Lynch KL, Pandolfino JE, Howden CW, Kahrilas PJ. Major complications of pneumatic dilation and Heller
myotomy for achalasia: single-center experience and systematic review of the literature. Am J Gastroenterol.
2012;107(12):1817-1825. doi:10.1038/ajg.2012.332.
3. Richter JE, Boeckxstaens GE. Management of achalasia: surgery or pneumatic dilation. Gut. 2011;60(6):869-876.
4. Rohof WO, Boeckxstaens GE. Treatment of the patient with achalasia. Curr Opin Gastroenterol. 2012;28(4):389-394.
5. Vaezi MF, Pandolfino JE, Vela MF. ACG clinical guideline: diagnosis and management of achalasia. Am J
Gastroenterol. 2013; 10(8):1238-1249.
9
QUESTION

A 65-YEAR-OLD MALE WITH LONG-STANDING GERD


COMPLAINED OF RECENT ONSET OF PROGRESSIVE DIFFICULTY
SWALLOWING (ESPECIALLY SOLIDS) AND SUBJECTIVE WEIGHT LOSS. HOW
SHOULD I EVALUATE THE PATIENT?

Emmanuell C. Gorospe, MD, MPH, FACP


and Kenneth K. Wang, MD, FACG, AGAF, FASGE

Dysphagia (or difficulty in swallowing) is an alarming symptom that requires further


investigation. As in any case of dysphagia, the patient’s history and predisposing factors for the
development of dysphagia are vital in determining the correct diagnostic algorithm. With an
accurate history alone, the cause of dysphagia can be determined in up to 80% of patients.1
The initial step is to establish the patient’s type of dysphagia. This case is suggestive of an
esophageal dysphagia, in contrast to an oropharyngeal dysphagia, which usually presents as
difficulty in initiating a swallow, inability to safely transfer food into the esophagus, and choking
after swallowing. The second important aspect of the history deals with the type of food that
induces dysphagia. Difficulty with solid food alone usually suggests anatomic obstruction.
Dysphagia with both solid and liquid meals often indicates a motility disorder. However, severe
mechanical obstruction may progress to cause dysphagia for both solids and liquids. Lastly, the
timing and progression of dysphagia are also important. Intermittent, nonprogressive symptoms
suggest lower esophageal rings or webs. Progressive symptoms may be indicative of benign or
malignant strictures. Nevertheless, motility disorders can manifest as either intermittent or
progressive symptoms.
Physical examination has limited contribution in the evaluation of dysphagia. Its value lies
in determining the presence of weight loss, malnutrition, oropharyngeal masses, and lymphade-
nopathy, which are ominous signs in someone presenting with dysphagia. Other important details
that the physical exam can reveal include thyromegaly, neurologic deficits, cutaneous signs of
scleroderma, and scars from prior thoracic or head and neck surgery, or radiation therapy.
To assist in your diagnostic workup for this patient, a list of differential diagnoses for esophageal
dysphagia, including their common risk factors, is provided in Table 9-1.

Leung J, Lo SK, eds. Curbside Consultation in Endoscopy:


41 49 Clinical Questions, Second Edition (pp 41-44)
© 2014 SLACK Incorporated
42  Question 9

Table 9-1

Differential Diagnoses for Esophageal Dysphagia


Category Diagnosis Common Risk Factors
Benign Peptic GERD, reflux esophagitis
strictures Caustic Alkali or strong acid ingestion
Drug induced Pill ingestion, inadequate
esophageal clearance
Radiation induced Radiation of neck and thoracic
cancers
Post-ablation or resection PDT, RFA, EMR of Barrett s esophagus
Mucosal rings Schatzki s ring GERD, other causes unknown
and webs Eosinophilic esophagitis Associated allergic conditions
Malignant Esophageal Barrett s dysplasia
strictures adenocarcinoma
Squamous carcinoma Tobacco and alcohol consumption
Distant malignant Multiple causes
metastases
Intramural Leiomyoma Benign tumor, risk factors unknown
lesions Esophageal lymphoma Immunosuppresion, HIV
Glandular cell tumor Risk factors unknown
Extramural Vascular compression Aberrant right subclavian artery,
lesions aortic compression
Mediastinal/Cardiac/ Multiple causes
Pulmonary masses
Anatomic Hiatal Hernia Obesity, advanced age
abnormalities Epiphrenic diverticulum Esophageal motility disorder
Traction diverticulum Mediastinal disease
Systemic Scleroderma Autoimmune condition
disorders Pemphigus/pemphigoid Autoimmune condition
conditions
Lichen planus Female, advanced age, oral
involvement
Motility Idiopathic achalasia Risk factors unclear
disorders Secondary/Pseudo Amyloid, Chagas disease,
achalasia malignancy
Hypomotility disorders Diabetes, amyloid, scleroderma
Abbreviations: GERD: gastroesophageal reflux disease; PDT: photodynamic therapy; RFA: radio-
frequency ablation; EMR: endoscopic mucosal resection
How Should I Evaluate the Patient With Long-Standing GERD?   43

Figure 9-1. Stepwise diagnostic algorithm for progressive, solid food dysphagia.

In a patient with long-standing reflux, strictures are the likely cause of dysphagia. The majority
of esophageal strictures are benign. They are formed from collagen deposition and fibrous tissue
formation in the setting of esophageal injury from acid reflux. Peptic strictures account for 75%
of benign esophageal strictures.2 In terms of morphology, strictures vary in size, thickness, and
circumferential symmetry. Simple strictures are described to be short, straight, and amenable to
the passage of a diagnostic upper endoscope. Complex strictures are usually asymmetric, longer
than 2 cm, and narrower. They may not allow for straightforward passage of a standard upper
endoscope.3
In this case, the patient’s history of reflux and solid food dysphagia are highly suggestive of a
structural obstruction, likely a stricture. His advanced age, progressive swallowing difficulty, and
subjective weight loss raise the suspicion for a neoplastic process. Further clues that may point to
a malignant condition include rapid progression of symptoms and weight loss disproportionate to
the degree of dysphagia. Given these concerns, we recommend an upper endoscopy as the best
initial evaluation for this patient. A stepwise diagnostic algorithm for progressive, solid food
dysphagia is shown in Figure 9-1.
The choice between barium esophagram and endoscopy as the initial diagnostic test has been
a topic of debate in previous years. There have been no clear evidenced-based data to support
either approach. As an initial test, we recommend upper endoscopy, as virtually all patients
with esophageal dysphagia will need endoscopic and even histologic evaluation in view of the
rising incidence of esophageal cancer and eosinophilic esophagitis. In a patient complaining of
odynophagia (ie, painful swallowing) in addition to dysphagia, endoscopy is even more important
because direct visualization and biopsy acquisition are necessary to differentiate infectious, drug-
induced, radiation-associated, or reflux esophagitis.
In the setting of strictures, an upper endoscopy has the benefit of direct visualization compared
with a barium esophagram. It allows for the determination of stricture location, length, diameter,
and possibly even etiology. It has the dual advantage of obtaining biopsies and possibly treating
the stricture at the same time. In evaluating distal esophageal strictures and rings, patience during
44  Question 9

endoscopy is essential. Subtle narrowing, which can be missed during routine endoscopy, may be
detected with ideal luminal distention. This can be accomplished by applying constant and gentle
air insufflation with the tip of an endoscope at the distal esophagus.
Barium esophagram may also be considered as an initial diagnostic test. A properly done
esophagram can accurately demonstrate stricture location and configuration. It can help to
visualize mucosal rings and webs that were not visualized during endoscopy, usually due to
inadequate luminal distention. Rings, webs, and subtle strictures can be detected in the prone
position, preferably during a Valsalva maneuver. An esophagram with a 13-mm barium tablet
can determine the clinical significance of a stricture. A 13-mm narrowing usually correlates with
the onset of dysphagia.4 Viewing esophageal contours in a contrast-enhanced esophagram allows
for better appreciation of extraluminal compression, compared to endoscopy. If an extraluminal
etiology is suspected, a computed tomography scan may be used to identify masses and sometimes
malignant infiltration near the gastroesophageal junction, such as in the case of pseudoachalasia.
Although we present a stepwise diagnostic approach in Figure 9-1, combining radiologic
imaging and endoscopic studies is often done in our practice, especially in patients with significant
concern for malignancy. The majority of malignant strictures can be differentiated from benign
strictures during endoscopy. The presence of friable mucosa, a fungating mass, and ulceration
are endoscopic features of a neoplastic process. If in doubt, endoscopic ultrasonography may be
useful in detecting cancer invasion and spread, if the equipment and expertise are available at the
time of the exam. Once malignancy is established, we promptly move forward to cancer staging
to direct the best therapeutic approach. More often than not, esophageal cancer is diagnosed in
an advanced stage when the patient has severe dysphagia and weight loss. At that stage, palliation
may be the only option.
Despite the available endoscopic and imaging tools, progressive solid food esophageal dysphagia
may not always present with an anatomic obstruction. If the endoscopy and esophagram are
unremarkable, you should consider intramucosal lesions, connective tissue disease, or esophageal
motility disorders.

References
1. Castell DO, Donner MW. Evaluation of dysphagia: a careful history is crucial. Dysphagia. 1987;2(2):65-71.
2. Marks RD, Richter JE. Peptic strictures of the esophagus. Am J Gastroenterol. 1993;88(8):1160-1173.
3. Ferguson DD. Evaluation and management of benign esophageal strictures. Dis Esophagus. 2005;18(6):359-364.
4. Ott DJ, Chen YM, Wu WC, Gelfand DW, Munitz HA. Radiographic and endoscopic sensitivity in detecting lower
esophageal mucosal ring. AJR Am J Roentgenol. 1986;147(2):261-265.
10 QUESTION

SEVERAL TRIPS TO THE ER FOR A 56-YEAR-OLD MECHANIC FOR SOB


AND IRON DEFICIENT ANEMIA REVEALED POSITIVE FOBT, BUT EGD AND
COLONOSCOPIES DID NOT IDENTIFY THE BLEEDING SOURCE. HE TAKES
NSAIDS REGULARLY FOR ARTHRITIS PAIN.
IS THERE A ROLE FOR ENTEROSCOPY?

Neel K. Mann, MD, MPH

This is a classic description of symptomatic obscure–occult gastrointestinal (GI) bleed. The


patient is a gentleman with persistent anemia, and the GI tract is the suspected source of bleed-
ing despite prior negative but appropriate endoscopy. This case of obscure GI bleeding can be
further categorized into an obscure–occult GI bleed because there are no overt signs of GI bleed-
ing, although the patient is dependent upon iron supplementation. The usual approach in the
diagnostic algorithm1 of an obscure GI bleed would be a second-look endoscopy, as up to 12% to
25% of lesions can actually be found within reach of a standard endoscope.2 These lesions include
Cameron erosions within a hernia sac, gastric antral vascular ectasias (GAVE; Figure 10-1), duo-
denal arteriovenous malformation (AVM) or ulcerations, as well as a cecal Dieulafoy. Despite not
being cost effective, a second look endoscopy/colonoscopy is warranted, especially if the initial
colonoscopy did not have an adequate preparation, or not enough biopsies were obtained in the
distal duodenum to rule out celiac sprue at the time of the upper endoscopy.
When the repeat endoscopy has been completed and is negative, it is logical to rule out under-
lying small-bowel pathologies as the cause of bleeding. If the patient does not have obstructive
symptoms, it would be appropriate to consider capsule endoscopy (CE) because it can safely
examine the entire small bowel with a diagnostic yield of approximately 60% to 70%. In addi-
tion, the yield for capsule endoscopy is higher and best utilized in cases of male gender, inpatient
status, greater than a 4-g drop in hemoglobin, overt GI bleeding, and history of connective tissue
disease.3 Additionally, some studies suggest that if the capsule is deployed within 24 to 72 hours
of the sentinel bleed, then there will be a higher diagnostic yield.4 I try to deploy the capsule as
close to the timing of the sentinel bleed as possible. Finally, it is well proven that CE has a higher
diagnostic yield compared to barium small-bowel follow through or push enteroscopy.5

Leung J, Lo SK, eds. Curbside Consultation in Endoscopy:


45 49 Clinical Questions, Second Edition (pp 45-49)
© 2014 SLACK Incorporated
46  Question 10

Figure 10-1. GAVE on capsule endoscopy.

Figure 10-2. Angiectasia on capsule


endoscopy.

The most common diagnosis on capsule endoscopy is angiectasia (Figure 10-2) or vascular
lesion, followed by small-bowel tumor (Figure 10-3), NSAID enteropathy, and Crohn’s disease
(Figures 10-4 and 10-5). The definitive outcome of capsule endoscopy is dependent not only on
the type of finding, but also the subsequent therapy employed, such as small-bowel resection for a
tumor or medical treatment for inflammatory Crohn’s disease, as validated by balloon- or device-
assisted enteroscopy.
If positive pathology is identified on CE, then double- or single-balloon enteroscopy can
then be utilized to not only validate the capsule endoscopy findings but also to offer a definitive
treatment for the underlying pathologies, such as argon plasma coagulation treatment of vascular
malformation(s) or removal of small-bowel polyps. Double-balloon enteroscopy has the added
Is There a Role For Enteroscopy?  47

Figure 10-3. Submucosal tumor


(GI stromal tumor) on double
balloon enteroscopy.

Figure 10-4. Deep, linear ulcers in Crohn’s dis-


ease on capsule endoscopy.

Figure 10-5. Ulcerated ileal


Crohn’s strictures.
48  Question 10

Figure 10-6. Diaphragm stricture on


capsule endoscopy.

benefit of achieving total enteroscopy compared to single-balloon enteroscopy or Spirus enteros-


copy6 (Spirus Medical); however, the diagnostic yield is the same. In addition, in a cost-effective
analysis,7 early use of DBE was the most cost-effective method for the diagnosis and management
of obscure GI bleeding when compared to push enteroscopy, intraoperative enteroscopy, and small
bowel imaging.
However, managing some small-bowel lesions, such as angiectasias, do not always lead to a
definitive outcome. In this case, the definitive outcome we expect is the complete resolution of
anemia without the need for iron supplementation. When it comes to angiectasias, outcomes data
on DBE are less promising. Some studies have found that despite DBE and treatment of angiec-
tasias, 30% of patients will rebleed in 1 year and up to 50% in 3 years. This often occurs because
patients continue to use chronic anticoagulant/antiplatelet therapy, and/or the underlying comor-
bid condition, such as valvular heart disease or arrhythmia, still persists.
On the other hand, our patient’s underlying issue is the chronic use of NSAIDs, which can lead
to small-bowel ulcers, and in some cases, small-bowel stricture(s) formation. In this case, DBE
may actually lead to a definitive therapeutic outcome. Specifically, DBE can not only identify
the culprit lesion such as diaphragm strictures (Figure 10-6), but also one can mark or tattoo the
location of the first and the last stricture. This is helpful for the subsequent surgical planning
and preservation of normal small bowel. In addition, only DBE and CE can identify these types
of strictures, as they are not readily seen or appreciated on small-bowel follow-through or other
dedicated small bowel imaging, such as CT enterography or magnetic resonance enterography.
More importantly, because the overlying serosa is normal, the surgeon may not be able to identify
the lesion as well while in the OR. This is where DBE can lead to a definitive change in outcome
in the setting of an obscure GI bleed—complete resolution of iron deficiency anemia. Obviously,
the patient should also be advised to stop taking all NSAIDs.
Is There a Role For Enteroscopy?  49

References
1. Raju GS, Gerson L, Das A, Lewis B; American Gastroenterological Association. American Gastroenterological
Association (AGA) Institute medical position statement on obscure gastrointestinal bleeding. Gastroenterology.
2007;133(5):1694-1696.
2. Fry LC, Belluti M, Neumann H, Malfertheiner P, Monkemuller K. Incidence of bleeding lesions within reach of
conventional upper and lower endoscopes in patients undergoing double balloon enteroscopy for obscure
gastrointestinal bleeding. Aliment Pharmacol Ther. 2009;29(3):342-349.
3. Robinson CA, Jackson C, Condon D, Gerson LB. Impact of inpatient status and gender on small-bowel capsule
endoscopy findings. Gastrointest Endosc. 2011;74(5):1061-1066.
4. Singh A, Marshall C, Chaudhuri B, et al. Timing of video capsule endoscopy relative to overt obscure GI bleeding:
implications from a retrospective study. Gastrointest Endosc. 2013;77(5):761-766.
5. Marmo R, Rotondano G, Piscopo R, Bianco MA, Cipolleta L. Meta-analysis: capsule endoscopy vs. conventional
modalities in diagnosis of small bowel diseases. Aliment Pharmacol Ther. 2005;22(7):595-604.
6. May A, Farber M, Aschmoneit I, et al. Prospective multicenter trial comparing push-and-pull enteroscopy
with the single- and double-balloon techniques in patients with small bowel disorders. Am J Gastroenterol.
2010;105(3):575-581.
7. Gerson L, Kamal A. Cost-effective analysis of management strategies for obscure GI bleeding. Gastrointest Endosc.
2008;68(5):920-936.
11QUESTION

A 59-YEAR-OLD PROFESSOR HAD DYSPEPSIA AND SUBJECTIVE WEIGHT


LOSS. HE WAS TREATED PREVIOUSLY FOR H PYLORI INFECTION. EGD
AND BIOPSIES SHOWED CHRONIC INFLAMMATION WITH INTESTINAL
METAPLASIA AND ONE FOCAL AREA OF LOW-GRADE DYSPLASIA.
WHAT DO I DO?

Chun-Ying Wu, MD, PhD, MPH, LL.M., LL.B. and


Francis K. L. Chan, MD, FACG

Dyspepsia is a nonspecific complaint that may be related to a number of underlying


gastrointestinal problems. It consists of a variety of upper abdominal symptoms, including fullness
and dull aching in the epigastrium and early satiety. Some patients also notice abdominal bloating
associated with frequent belching, whereas others complain of heartburn symptoms. In rare cases,
patients may have nausea and vomiting. It is estimated that 20% to 25% of the general population
has dyspepsia, which could be related to either functional or organic causes. Functional dyspepsia
(FD) can be caused by delayed gastric emptying, complicated longstanding diabetes mellitus,
impaired gastric motility, and hyperalgesia to gastric distension. Organic causes include most
commonly peptic ulcer diseases, gastroesophageal reflux disease (GERD), and intake of gastric
irritants such as nonsteriodal anti-inflammatory drugs (NSAIDs) or other medications. Rarely, it
may be related to underlying gastric malignancy or pancreaticobiliary disorders.1
In general, functional dyspepsia will not give rise to weight loss, so the presence of subjective
weight loss needs to be documented and further investigation is warranted if there is clinical
suspicion of underlying organic lesions. Routine upper endoscopy can rule out a number of upper
gastrointestinal pathologies associated with dyspepsia and weight loss. 2 In addition, an abdominal
ultrasound scan and computed tomography (CT) scan of the abdomen can also rule out significant
pancreaticobiliary pathologies.
The presence of Helicobacter pylori infection and associated gastritis can give rise to dyspeptic
symptoms. The diagnosis can be easily confirmed at the time of upper endoscopy with biopsies. If
the patient is receiving proton pump inhibitor (PPI) therapy, it may be necessary to consider taking
biopsies from the antrum as well as the proximal body of the stomach. Serological testing or stool
studies are alternative tests used to confirm underlying infection. Eradication of H pylori infection
can be accomplished with the use of a 10-day to 2-week course of triple antibiotics in combination
Leung J, Lo SK, eds. Curbside Consultation in Endoscopy:
51 49 Clinical Questions, Second Edition (pp 51-52)
© 2014 SLACK Incorporated
52  Question 11

with a PPI.3 Depending on the combination of drugs used, the clearance rate for the infection can
be as high as 90% to 95% of cases if the medications are taken properly. Inadequate therapy may
be related to drug intolerance or noncompliance. In such cases, the successful eradication rate is
much lower, which can also lead to persistence of symptoms and in some cases, resistant bacteria
and persistence of gastritis. It may be worth keeping patients on a daily low-dose PPI therapy
for acid suppression to control the acid-related symptoms after H pylori eradication. Additional
medications for the control of dyspeptic symptoms include other acid suppressive agents such as
antacids and H2 receptor antagonists, tricyclic antidepressants, and prokinetic drugs. We do not
recommend the use of herbal medications or alternative medicines because the content or active
ingredients in these drugs are often not certain.
H pylori infection often gives rise to nodular gastritis in the stomach, as seen on endoscopy.
However, the presence of intestinal metaplasia on the gastric biopsies raise a concern for risk of
future gastric malignancy. Chronic gastritis is a pathological description indicating the presence of
chronic inflammation cells, including lymphocytes and neutrophils infiltration of the lamina pro-
pria. If the normal glandular structure is lost, it is compatible with chronic atrophic gastritis. The
major concern is that atrophic gastritis with associated intestinal metaplasia is often considered
precancerous conditions because they constitute the background of gastric dysplasia and intestinal
type gastric cancer.
In the presence of intestinal metaplasia with low-grade dysplasia, complete H pylori eradication
is helpful to prevent progression to high-grade dysplasia or gastric cancer. In addition to H pylori,
several factors may increase the risk of gastric cancer development, including a family history of
gastric cancer, elderly male patient, and the presence of H pylori virulence factors, as well as the
host’s genetic predisposition. Among these, a strong family history of gastric cancer is probably
the most important risk factor for development of gastric cancer.4
Presence of low-grade dysplasia on gastric biopsy is not a predetermining factor for progression
to gastric cancer. However, the patient should undergo close surveillance, with repeat endoscopy
and biopsy at regular intervals. It may be necessary to perform systematic biopsies using mapping
to help localize the lesion if biopsy of a particular area in the stomach shows focal low-grade dys-
plasia and to monitor or rule out progression. It is recommended that upper endoscopy be repeated
within 1 year after the initial diagnosis according to the MAPS guidelines.4 If an endoscopically
defined lesion can be seen, endoscopic mucosal resection of the lesion should be performed to
obtain a more accurate histological diagnosis. If the low-grade dysplasia cannot be identified on
subsequent endoscopy, follow-up EGD with random biopsies within 1 year is recommended.4

References
1. Oustamanolakis P, Tack J. Dyspepsia: organic versus functional. J Clin Gastroenterol. 2012;46:175-190.
2. Lacy BE, Talley NJ, Locke GR, 3rd et al. Review article: current treatment options and management of functional
dyspepsia. Aliment Pharmacol Ther. 2012;36:3-15.
3. Malfertheiner P, Megraud F, O’Morain CA et al. Management of Helicobacter pylori infection—the Maastricht IV/
Florence Consensus Report. Gut. 2012;61:646-664.
4. Dinis-Ribeiro M, Areia M, de Vries AC et al. Management of precancerous conditions and lesions in the stomach
(MAPS): guideline from the European Society of Gastrointestinal Endoscopy (ESGE), European Helicobacter Study
Group (EHSG), European Society of Pathology (ESP), and the Sociedade Portuguesa de Endoscopia Digestiva
(SPED). Endoscopy. 2012;44:74-94.
12 QUESTION

A 38-YEAR-OLD MILDLY OBESE FEMALE PRESENTED WITH GERD


SYMPTOMS AND WAS FOUND TO HAVE A PERSISTENTLY ELEVATED
AMYLASE LEVEL AROUND 250 IU/L (NORMAL < 125) BUT
NORMAL LIPASE. HOW SHOULD I MANAGE THIS PATIENT?

Sooraj Tejaswi, MD, MSPH

Amylase is a digestive enzyme mainly derived from the pancreas (isoform P) and the salivary
glands (isoform S), but it can be found in other tissues in smaller quantities. Its main function is
digestion of starch, as suggested by its Greek origin amylone, meaning starch. Amylase is a small
molecule that is easily filtered by the renal glomeruli, and approximately 25% of it is excreted
through the kidneys. The majority (75%) is excreted through the reticuloendothelial system.
Serum amylase should be checked in the clinical setting for patients presenting with abdom-
inal pain and who are suspected of having acute pancreatitis. The serum amylase levels tend to rise
before lipase levels in the acute setting, typically within 2 to 12 hours, peaking at 12 to 72 hours.
It also tends to normalize sooner, typically within 5 days. The serum lipase levels, on the other
hand, rise 4 to 8 hours after the onset of symptoms (ie, abdominal pain), peak at 24 hours, and
stay elevated for 8 to 14 days. Therefore, another clinical utility of serum amylase is to determine
whether it is a recurrent attack of pancreatitis in the acute phase of pancreatitis. However, as a
biochemical marker of acute pancreatitis, it is less specific than serum lipase. Amylase levels can
even be normal in some cases of acute pancreatitis, especially if caused by alcohol ingestion or
hypertriglyceridemia. Alternatively, amylase levels can also be persistently elevated in the absence
of pancreatic inflammation.
Serum amylase levels can be elevated in the presence of several inflammatory conditions within
the abdomen without associated acute pancreatitis, such as perforated peptic ulcer or viscous, mes-
enteric ischemia and infarction, significant bowel obstruction, acute appendicitis, cholecystitis,
liver disease, and acute gastroenteritis.
It can be elevated in the presence of salivary gland diseases, including infection, radiation,
trauma, and ductal obstruction. Gynecological diseases, such as pelvic inflammatory disease and
ruptured ectopic pregnancy, can also cause amylase elevation. In rare circumstances, it can be
elevated in renal failure, severe burns, diabetic ketoacidosis, and after double-balloon enteroscopy.

Leung J, Lo SK, eds. Curbside Consultation in Endoscopy:


53 49 Clinical Questions, Second Edition (pp 53-55)
© 2014 SLACK Incorporated
54  Question 12

An accurate diagnosis of the underlying acute conditions is possible based on a detailed medical
history, thorough physical exam, and appropriate laboratory tests, including liver function tests,
and amylase isoenzymes etc, in addition to radiographic tests, including abdominal and/or pelvic
imaging by ultrasound, computed tomography scan, or magnetic resonance imaging.

Diagnostic Approach to Isolated Persistent


Asymptomatic Amylase Elevation
It is not uncommon to see an isolated and persistent elevated serum amylase in an otherwise
healthy asymptomatic individual. The differential diagnoses would be slightly more obscure and
may include the following conditions: macroamylasemia; solid tumors of the pancreas, ovary, pros-
tate, lung, esophagus, breast, and thymus; and “others”, including alcoholism, celiac disease, preg-
nancy, AIDS, anorexia nervosa and bulimia, and drug-induced and idiopathic amylase elevation.

What Is Macroamylasemia?
Macroamylasemia is a macroenzyme formed when amylase binds to a macromolecule, such as
immunoglobulin A or immunoglobulin G. This is the most common serum immunoglobulin-
enzyme complex in the human body1 and can be seen in up to 9.6% of patients with
hyperamylasemia. Due to its large size, the kidneys are unable to excrete this molecule, resulting
in persistently elevated amylase level.
Macroamylasemia has been reported, albeit infrequently, in cases of diabetes mellitus, celiac
disease, Crohn’s disease, T-cell lymphoma, HIV infection, renal cell cancer, and thyroid cancer.
Macroamylasemia should be suspected in all cases of persistently elevated amylase levels in the
absence of acute gastrointestinal disease in the setting of normal renal function. The simplest way
to diagnose this condition is calculating the amylase–creatinine clearance ratio (ACCR) using the
following formula, based on a 24-hour urine sample2:

Urine Amylase × Serum Creatinine × 100


ACCR =
Urine Creatinine × Serum Amylase

The ratio is < 1% in cases of macroamylasemia. The ratio is > 1% in cases of acute pancreatitis.
Macroamylase has not been shown to have a pathogenic role in any of the diseases with which
it is associated, and neither does it seem to have any prognostic significance.

Conclusion
After a thorough evaluation of this patient, including abdominal imaging such as magnetic
resonance cholangiopancreatography, if no obvious etiology can be found for the persistent isolated
amylase elevation, it is reasonable to repeat the amylase levels every 3 to 6 months. The patient
should also be followed for a few years for the remote possibility of it being a marker for early pan-
creatic malignancy, such as intraductal papillary mucinous neoplasm, pancreatic adenocarcinoma,
or pancreatic neuroendocrine tumor. 3,4
How Should I Manage This Patient With Elevated Amylase Level?   55

References
1. Berk JE, Kizu H, Wilding P, Searcy RL. Macroamylasemia: a newly recognized cause for elevated serum amlyase
activity. N Eng J Med. 1967;277:941-946.
2. Levitt MD. Clinical use of amylase clearance and isoamylase measurements. Mayo Clin Proc. 1979;54:428-431.
3. Mariani A. Chronic asymptomatic pancreatic hyperenzymemia: is it a benign anomaly or a disease? J Pancreas.
2010;11(2):95-98.
4. Pezzilli R, Morselli-Labate AM, Casadei R, et al. Chronic asymptomatic pancreatic hyperenzymemia is a benign
condition in only half the cases: a prospective study. Scand J Gastroenterol. 2009;44(7):888-893.
13 QUESTION

A 35-YEAR-OLD FEMALE PRESENTED WITH EPIGASTRIC PAIN, DIFFICULTY


SWALLOWING, AND SUBJECTIVE FEVER 1 MONTH AFTER GASTRIC BYPASS
SURGERY. SHE WAS SEEN IN THE ER BECAUSE OF RECENT HEMATEMESIS.
WHAT DO I DO?

Andrew Yen, MD, FACG and


Joseph Leung, MD, FRCP, FACP, FASGE, MACG

Laparoscopic Roux-en-Y gastric bypass (LRYGB) is the most common bariatric and metabolic
surgical procedure performed for morbid obesity. Gastroenterologists are often called upon to
evaluate patients with symptoms or suspected complications after LRYGB. The general approach
to the postoperative patient begins with an understanding of the surgical anatomy, delineating the
time course of symptoms since surgery (ie, early or late), and determining the role of endoscopy in
diagnosis and treatment based on the clinical assessment of the patient.
A number of symptoms and complications can occur in the postoperative period after gastric
bypass. Serious early postoperative complications require prompt diagnosis and timely management
to avoid poor clinical outcomes. Such complications include typical postsurgical events, as well
as complications specific to LRYGB, such as anastomotic or staple line leaks, postoperative
hemorrhage, bowel obstruction, and incorrect Roux limb reconstructions. Later complications
that often require endoscopic evaluation include anastomotic strictures, marginal ulceration, fis-
tula formation, weight regain, and nutritional deficiencies.1 Additional clinical problems directly
or indirectly related to the bypass surgery can also be observed 2 (Table 13-1).
In this case, the patient presents with a number of symptoms 1 month following gastric bypass
surgery. She reports abdominal pain and difficulty swallowing, which are symptoms suggestive of
complications related to her recent surgery. The presence of fever is especially worrisome, although
it may or may not be related to a surgical complication, but the differential diagnosis should
include consideration for anastomotic leakage with infection and abscess formation. The recent
onset of hematemesis also suggests upper gastrointestinal hemorrhage, indicating a more alarming
complication that may include bleeding from an anastomotic ulcer or, less likely, bleeding from an
underlying stitch- or surgical-clip associated granulation tissue.
Clinical management should be tailored to the urgency and severity of symptoms. Immediate
management should include resuscitation of the patient if applicable. In addition to IV fluid
replacement, blood should be taken for cross-matching and adequate venous access with 2 large-
bore IV catheters or a central line should be placed. Intravenous proton-pump inhibitor (PPI)
Leung J, Lo SK, eds. Curbside Consultation in Endoscopy:
57 49 Clinical Questions, Second Edition (pp 57-60)
© 2014 SLACK Incorporated
Table 13-1
58  Question 13

Causes of Abdominal Pain After Gastric Bypass


Behavioral, Functional Biliary Disorders Pouch, Remnant Small Intestine Other
Dietary Disorders Disorders Stomach Disorders Disorders
Overeating, rapid Constipation, Cholelithiasis: colic, Ulcer disease Abdominal wall Omental infarction
eating diarrhea, flatus cholecystitis hernias: ventral,
trocar
Food intolerance Irritable bowel Choledocholithiasis: Gastrogastric Adhesions Superior mesenteric
syndrome cholangitis, fistula artery syndrome
pancreatitis
Micronutrient Esophageal GERD Internal hernia
deficiencies motility disorders
Sphincter of Oddi
Micronutrient Dumping Hiatus hernia Intussusception Bezoar
dysfunction
supplementation syndrome Gastrojejunostomy Jejunojejunostomy
stenosis stenosis

Adapted from Greenstein AJ, O’Rourke RW. Abdominal pain following gastric bypass: suspects and solutions. Am J Surg. 2011;201(6):819–827.
Patient Seen in the ER Because of Recent Hematemesis. What Do I Do?   59

therapy should be administered because of the high likelihood that this patient may have an
underlying anastomotic ulcer. After initial assessment and stabilization of the patient, it is impor-
tant for the evaluating gastroenterologist to determine if and when endoscopy should be utilized
for diagnosis and therapy. If deemed necessary, the procedure should be performed in the appro-
priate setting—in the intensive care unit, endoscopy suite, or operating room—depending on the
patient’s clinical status and time since surgery. It is also important for the endoscopist to appreci-
ate the potential need for different endoscopes and accessories based on the patient’s postsurgical
anatomy and anticipated complications to optimize the chance of a successful intervention.
If the patient is not stable, urgent endoscopy would be necessary once the patient is stabilized
to define the source of bleeding. If active bleeding is seen at the time of endoscopy, usu-
ally from an underlying ulcer, therapeutic endoscopy in the form of injection therapy with
1:10000 (0.1 mg/mL) solution epinephrine combined with thermocoagulation using either a
heater probe or BiCAP probe should be used for hemostasis. Alternatively, one or more hemoclips
could be applied to the ulcer base if a visible vessel or active bleeding is observed. This can be done
with or without prior epinephrine injection into the base of the ulcer.
Strictures and stenosis can also develop at the gastrojejunal anastomosis, causing subsequent
narrowing and obstruction with resultant dysphagia. Rarely, this may be related to a very small
gastric remnant; although in such cases, the patient would usually complain of early satiety.
Strictures less commonly occur in the distal GI tract at the jejunojejunal anastomosis or at the level
of the mesocolic window for reconstructions with a retrocolic Roux limb, although strictures in
these areas should not cause dysphagia. The management of dysphagia can include a barium study
and/or upper endoscopy to assess the status of the anastomosis. Strictures can occur in association
with ulcers at the gastrojejunostomy, which usually occur along the small intestinal side of the
anastomosis. If an ulcer is seen at the anastomosis, aggressive medical treatment should be given,
including double-dose PPI therapy and avoidance of risk factors for peptic ulcer disease. These
include cessation of smoking and stopping gastric irritants such as NSAIDs. Some endoscopists
also recommend adding sucralfate to cover the ulcer to promote ulcer healing, although the util-
ity of this is debatable. Such ulcers can be difficult to resolve because of underlying ischemia. If
an anastomotic stricture is seen without any adjacent ulceration, it can be dilated using a dilation
balloon over a guidewire with the goal for symptom reduction. However, it is important to avoid
overdilation of the stoma to prevent counteracting the restrictive properties of the bypass proce-
dure. If there is concern about an increased risk of perforation because the surgery was performed
only 1 month ago, the patient can be managed conservatively with a soft diet, and consideration
for dilation can occur at a later time, although generally, dilation 1 month after surgery is safe and
effective. If concerns exist, discussing the case further with the patient’s surgeon is appropriate.
Balloon dilation is usually postponed in the presence of an anastomotic ulcer until the ulcer has
healed to avoid the risk of perforation.
Fever in the postoperative period could suggest a leak at the anastomosis site secondary to
ischemia or wound dehiscence with infection or even abscess formation. A radiographic contrast
study should be done to evaluate for anastomotic leakage, and a CT scan of the abdomen can help
determine whether there is abscess formation. If there is evidence of leakage, or if a high suspicion
exists even with a reportedly negative contrast study, the patient should be given broad-spectrum
antibiotic coverage for 1 week to 10 days and kept nil by mouth until the infection is controlled and
the leakage is closed off. In certain cases, the patient may require short-term total parenteral nutri-
tion for nutritional support. Corrective surgery may be difficult in such situations, and endoscopic
therapy with placement of a fully covered removable stent across the leak is increasingly being used
for nonoperative management in these cases.
60  Question 13

References
1. PS Griffith, DW Birch, AM Sharma, S Karmali. Managing complications associated with laparoscopic Roux-en-Y
gastric bypass for morbid obesity. Can J Surg. 2012;55(5):329–336.
2. Greenstein AJ, O’Rourke RW. Abdominal pain following gastric bypass: suspects and solutions. Am J Surg.
2011;201(6):819–827.
14QUESTION

A 45-YEAR-OLD ALCOHOLIC WAS ADMITTED WITH ABDOMINAL PAIN,


COFFEE GROUND EMESIS, AND MILD ANEMIA. UPPER ENDOSCOPY
SHOWS A FEW POLYPOID MASSES IN THE GASTRIC BODY AND
PIGMENTATIONS IN THE ANTRUM. WHAT SHOULD I DO?

Sooraj Tejaswi, MD, MSPH

Gastric polyps are encountered incidentally in approximately 6% of esophagogastroduodenos-


copies in the United States.1 By definition, gastric polyp implies an epithelial polyp, and includes
fundic gland polyps, hyperplastic polyps, and adenomas, in order of decreasing prevalence.
Fundic gland polyp has now become the most common type of gastric polyp in the United States
due to widespread use of proton pump inhibitors (PPIs) and low prevalence of Helicobacter pylori
infection.1 Usually seen in the gastric body and fundus, they are small (< .05 cm), sessile, and have
a glassy appearance. PPI users can have multiple, synchronous fundic gland polyps. Histologically,
they consist of cystically dilated oxyntic glands. They are usually benign and do not require further
follow-up.
Hyperplastic, or inflammatory, is currently the second most common type of gastric polyp in the
United States, but in parts of the world with high prevalence of H pylori infection, it is the most
common type of gastric polyp.
Hyperplastic polyps are most often seen in the antrum in the setting of gastric mucosal inflam-
mation and atrophy. They are usually multiple and small in diameter (0.5 to 1.5 cm), with a dome-
shaped surface. They can become larger and pedunculated.
Histologically, they consist of hyperplastic foveolae that are elongated, branched, and dilated.
The lamina propria in which these foveolae lie is edematous, congested, and composed of acute
and chronic inflammatory cells and small, irregularly distributed smooth muscle bundles. 2 Due to
this prominent inflammatory component, this polyp is also termed an inflammatory polyp.
Foveolar hyperplasia is a hyperproliferative response to chronic inflammatory stimuli, such
as H pylori infection or chemical gastropathy caused by NSAID use or bile gastritis. This
hyperproliferative response can also be seen at gastrectomy sites in patients with post-Billroth I
and II gastric stumps and at the edges of ulcers and erosions.

Leung J, Lo SK, eds. Curbside Consultation in Endoscopy:


61 49 Clinical Questions, Second Edition (pp 61-63)
© 2014 SLACK Incorporated
62  Question 14

Larger hyperplastic polyps can develop surface erosions that may bleed, resulting in chronic
blood loss and iron deficiency anemia. Rarely, they can also cause gastric outlet obstruction by
blocking the pylorus.
There is a 0.5% to 7.1% risk of malignancy in hyperplastic polyps, but malignant cells are usu-
ally confined to the mucosa or submucosa. Malignancy develops through the dysplasia–carcinoma
sequence. The risk of malignancy is higher if polyps are greater than 2 cm in size, and if they are
pedunculated.3 In addition to the risk of carcinogenesis within the polyp, hyperplastic polyp is
also a marker for the increased risk of gastric cancer in the surrounding mucosa as a result of the
underlying chronic atrophic gastritis.
Adenomatous polyp is the third and least common type of gastric polyp. Located in the antrum,
these polyps are usually solitary, sessile or pedunculated, with a velvety, lobulated surface.
Histologically, they often exhibit dysplastic epithelium. They should be completely excised, and
they require endoscopic surveillance due to their neoplastic nature.

Management of Hyperplastic Polyps


The first step in the management of any gastric polyp is to establish a histologic diagnosis.
There are marked differences in the prognosis and management based on the type of gastric polyp.
Endoscopic appearance alone cannot differentiate histologic subtypes, and biopsy or polypectomy
is recommended when a polyp is encountered.4
In every case of gastric hyperplastic polyp, the underlying cause should be sought and eradi-
cated. Due to the strong association with H pylori infection, all patients with hyperplastic polyps
should be tested for H pylori and treated if infected. This results in regression of hyperplastic
polyps in up to 70% of patients.5 NSAID use should be sought out and discontinued. Bile gastritis
should be treated if present.
Hyperplastic polyps that are pedunculated, or > 1 to 2 cm in size, should be resected completely
due to the risk of malignant transformation, and because forceps biopsies alone can miss foci of
dysplasia or cancer in a large polyp.6 Due to the increased vascularity of gastric polyps, there is
a higher risk of postpolypectomy bleeding, and hence, prophylactic measures such as endoscopic
clip-closure of polypectomy sites must be considered, especially with larger polyps. If it is found
that dysplasia has spread beyond the confines of the polyp, more aggressive options such as endo-
scopic mucosal resection or subtotal gastrectomy may be required.
There is an increased risk of gastric cancer in the nonpolypoid gastric mucosa in patients with
gastric hyperplastic polyps. This is a direct consequence of the underlying chronic atrophic gastri-
tis in these patients. Therefore, it is recommended to assess the stage of gastritis in the surround-
ing gastric mucosa in terms of gastric cancer risk. One such validated staging system is the OLGA
(Operative Link on Gastritis Assessment) system. It is based on at least 5 biopsy samples: 2 from
antral mucosa, 1 from incisura angularis, and 2 from the oxyntic area. It assigns stages from 0 to
4. The risk of gastric cancer is lowest in stage 0 and highest in stage 4.7

Surveillance Strategies
Surveillance endoscopy after 6 months to 1 year is recommended in cases of hyperplastic polyps
associated with H pylori infection to confirm the eradication of the infection and regression of the
polyps, and to perform gastric mapping for intestinal metaplasia or dysplasia (OLGA staging).
If a hyperplastic polyp is found to have dysplasia or intramucosal carcinoma, surveillance
endoscopy can be performed within 1 year to evaluate for completeness of resection at the site of
polypectomy, and to perform gastric mapping (OLGA staging). If the dysplasia has spread beyond
What Should I Do for Masses and Pigmentations in the Antrum of the Stomach?   63

the confines of the polyp, more aggressive surveillance with extensive gastric mucosal sampling
at 1- to 3-year intervals may be required. Patients with advanced OLGA stage 3 or 4 gastritis
should be considered for surveillance every 1 to 3 years.7 The optimal surveillance strategy in cases
of hyperplastic polyps found at gastroenterostomy sites is not well studied, but given the risk of
cancer, surveillance endoscopies should be considered.

References
1. Carmack SW, Genta RM, Schuler CM, Saboorian MH. The current spectrum of gastric polyps: a 1-year national
study of over 120,000 patients. Am J Gastroenterol. 2009;104:1524-1532.
2. Carmack SW, Genta RM, Graham DY, Lauwers GY. Management of gastric polyps: a pathology-based guide for
gastroenterologists. Nat Rev Gastroenterol Hepatol. 2009;6:331-341.
3. Antonioli DA. Precursors of gastric carcinoma: a critical review with a brief description of early (curable) gastric
cancer. Hum Pathol. 1994;25(10):994-1005.
4. ASGE guideline: the role of endoscopy in surveillance of premalignant conditions of the upper GI tract.
Gastrointest Endosc. 2006;63:570-580.
5. Ohkusa T, Takashimizu I, Fujiki K, et al. Disappearance of hyperplastic polyps in the stomach after eradication of
Helicobacter pylori. A randomized controlled trial. Ann Intern Med. 1998;129:712-715.
6. Muehldorfer SM, Stolte M, Martus P, et al. Diagnostic accuracy of forceps biopsy versus polypectomy for gastric
polyps: a prospective multicenter study. Gut. 2002;50(4):440-465.
7. Rugge M, Meggio A, Pennelli G, et al. Gastritis staging in clinical practice: the OLGA staging system. Gut.
2007;56(5):631-636.
15 QUESTION

A 45-YEAR-OLD PATIENT WITH KNOWN


HEPATITIS C WAS ADMITTED BECAUSE OF MASSIVE
UPPER GI BLEEDING. WHAT SHOULD I DO?
Cara Torruellas, MD, MPH and
Joseph Leung, MD, FRCP, FACP, FASGE, MACG

Patients with massive upper gastrointestinal (GI) bleeding often present with hematemesis
and/or melena. In severe cases, a patient may present with hemodynamic instability and pro-
found anemia. Although peptic ulcer disease is the most common cause of upper GI bleeding,
in a patient with established chronic viral hepatitis and possible cirrhosis, it is important to rule
out bleeding esophageal varices as the etiology of an upper GI bleed. Gastroesophageal varices
are present in approximately 50% of cirrhotic patients; however, the most important predictor of
variceal hemorrhage is the size of the patient’s varices, with the highest risk of first hemorrhage
(15% per year) occurring in patients with large varices. Other predictors of hemorrhage include
decompensated cirrhosis (patients with Child-Pugh class B or C cirrhosis) and red wale signs on
upper endoscopy.1 Thus, in patients presenting with frank hematemesis and evidence of ascites,
jaundice, asterixis, and/or altered mental status on physical examination, esophageal varix bleed-
ing should be strongly suspected.
Patients with a suspected acute variceal hemorrhage should be admitted to the intensive care
unit for close monitoring, management, and resuscitation. Initially, it is important to assess both
the patient’s airway and hemodynamic status. It is critically important to establish adequate
peripheral IV access with 2 large-bore IV catheters for resuscitation of any patient presenting with
an acute upper GI bleed. If the patient is in hypovolemic shock, crystalloid solution or plasma
expander should be administered to maintain the circulatory volume initially. It is also important
to cross match and to have several units of blood available on reserve in the blood bank should
the patient require blood transfusion. Blood volume resuscitation should be initiated promptly,
but with caution, with the goal of maintaining hemodynamic stability and a hemoglobin of
approximately 8 g/dL.2 Vigorous fluid resuscitation or blood transfusion can lead to increased
portal pressure and to recurrent esophageal variceal bleeding. Initial laboratory studies prior to

Leung J, Lo SK, eds. Curbside Consultation in Endoscopy:


65 49 Clinical Questions, Second Edition (pp 65-68)
© 2014 SLACK Incorporated
66  Question 15

the administration of IV fluids or blood products should include a baseline complete blood count,
coagulation screen, and liver and renal function tests.
After the initial resuscitative measures are performed, a detailed history (if the patient is
not confused) and physical examination will provide important information as to the nature of
the bleeding pathology. A prior history of peptic ulcer disease or physical examination showing
stigmata of chronic liver disease may help to elucidate the underlying etiology of bleeding. Patients
with suspected esophageal variceal bleeding should be started immediately on IV proton pump
inhibitor (PPI) therapy, as well as IV octreotide, initially as a .05-mg bolus, followed by continuous
IV infusion of .05 mg every hour for up to 5 days (during which the risk of rebleeding is highest). 3
It is important to correct any underlying coagulopathy associated with chronic liver disease with
transfusion of fresh-frozen plasma, as well as transfusion of platelets as needed in cases of severe
thrombocytopenia. If the patient stabilizes with these measures, endoscopic evaluation and inter-
vention can be performed safely and electively. An indwelling nasogastric tube can be inserted and
the stomach lavaged with 300 mL of sterile saline and siphoned out to determine the status of the
patient’s bleeding, if it remains unclear.
However, if the patient is still actively bleeding despite pharmacologic therapy, a decision
should be made to consider placement of a Sengstaken Blakemore tube (if bleeding esopha-
geal varices is strongly suspected and the patient has recurrent massive hematemesis with
hemodynamic instability), or to perform an emergent endoscopy. Once a decision is made to
place a Blakemore tube, however, alternative treatment to upper endoscopy should be considered,
including evaluation for either shunt surgery (in Child-Pugh A patients) or transjugular intrahe-
patic portosystemic shunt (TIPS) as salvage therapy. Patients who fail both pharmacologic and
endoscopic therapy for bleeding esophageal varices should also undergo shunt surgery or TIPS.4
An urgent endoscopy is indicated if the patient has signs of active bleeding with fresh
hematemesis or passing maroon colored stool with hemodynamic instability. Because bleeding
from ruptured esophageal varices could be massive and the patient can aspirate during the
examination, it is important to consider intubation to protect the airway before performing urgent
endoscopy to minimize the risk of aspiration pneumonitis or pneumonia, particularly if the patient
is encephalopathic. If necessary, the urgent endoscopy can be performed in the emergency depart-
ment given the appropriate support.
With the patient adequately sedated (ideally with IV sedative and propofol), emergency
endoscopy can be performed using a regular upper GI scope, which is more flexible and easier to
manipulate than a therapeutic upper GI endoscope. It can be used to perform injection therapy, for
placement of hemoclips, or banding of esophageal varices. The only drawback is the small-scope
channel size, which limits suction power and will not accommodate a 10-Fr heater probe, which
is preferable in the setting of a bleeding ulcer.
In preparation for the upper endoscopy, many would prefer to perform gastric lavage to help
empty the stomach of blood clots that may impair the endoscopic examination. However, gastric
lavage is not always necessary. With the patient intubated and placed in a left lateral position, most
of the blood or clots will pool in the fundal area and it is possible to examine the upper body of
the stomach and the entire lesser curve and antrum, as well as the duodenum where most of the
benign bleeding pathologies are located. The presence of clots in the fundus may not be seen easily,
but with repositioning of the patient during the procedure, the endoscopist can expose the fundus
of the stomach and help rule out underlying gastric varices. If fresh blood is seen in the stomach,
the endoscopist should quickly examine the duodenum to rule out any underlying duodenal
ulcerations or distal gastric lesions, including ulcers, visible vessels, or malignancy. Retroflexion of
the endoscope will allow examination of the gastric fundus around the cardia to rule out junctional
or gastric varices. This is followed by a close examination of the distal esophagus to assess for the
What Should I Do for Massive Upper GI Bleeding?   67

presence of esophageal varices, particularly for signs of active or recent bleeding, including spurt-
ing or oozing varices, presence of a red dot, or dilated venules on the varix. All of these suggest
active or ongoing bleeding.
Today, banding ligation is the most effective and efficient treatment for bleeding esophageal
varices. A commonly used device is the Six-shooter (Cook Endoscopy). The bands are mounted on
a transparent cap that is attached to the tip of the regular endoscope. The applicator mechanism is
connected to a control placed at the level of the biopsy valve. The scope is reinserted in the esopha-
gus until the tip reaches the gastroesophageal (GE) junction. The stomach is deflated to minimize
the patient’s discomfort and the GE junction is examined closely for evidence of stigmata of recent
hemorrhage on the varices. Banding should start close to the GE junction, and the cap is directed
onto a column of varix. Suction is then applied to collapse the varix into the cap and continuous
suction is applied (preferably to achieve a “red out,” ie, the mucosa is touching the lens). In doing
so, the varix is adequately suctioned and the control turned gently until a band is fired to wrap
around the varix, forming a polypoid structure. The suction is continued for a few seconds after
firing the band to allow the band to properly wrap around the varix. Air is then insufflated and
the cap is pulled back to release the banded varix. The scope is pulled back gently and turned in
a clockwise direction to target another varix column. The procedure is repeated, and the scope is
then pulled back to repeat banding of a different column at a slightly more proximal level, until
all of the dilated veins are ligated. It is important to avoid banding the varices at the same level
because the resultant polypoid structures can cause transient obstruction to the esophageal lumen.
If active bleeding is induced while positioning the banding device, it may be necessary to
change the scope position to capture and suction the bleeding point into the cap and then apply
the band to control the bleeding. Some endoscopists prefer to inject sclerosant into the varix after
banding, but this may not be necessary.
After the banding, it is not advisable to replace the nasogastric tube because it can potentially
dislodge the bands and induce bleeding. Continued monitoring of the patient’s hemodynamic status
will detect whether there is ongoing bleeding or whether the patient has recurrent hematemesis.
To prevent further bleeding, it is advisable to continue with the IV infusion of octreotide for at
least 72 hours and up to 5 days, during which the risk for rebleeding is highest. After the octreo-
tide drip is completed, the patient should be transitioned to a nonselective beta-blocker, such as
propranolol, and titrated to a heart rate of 55 to 60 beats per minute for secondary prophylaxis of
esophageal variceal bleeding. In addition, cirrhotic patients with acute upper GI bleeding are at
increased risk for developing bacterial infections, and the use of short-term prophylactic antibiot-
ics in cirrhotic patients with or without ascites has been shown to increase survival.5 Therefore,
all patients with acute esophageal variceal bleeding should receive the recommended antibiotic
schedule of norfloxacin 400 mg orally twice daily or ceftriaxone 1 g IV every 24 hours for 7 days.6
When the patient is stable, he or she will likely need a repeat endoscopy every 2 to 3 weeks for
repeat variceal band ligation until the varices are completely obliterated.

References
1. The North Italian Endoscopic Club for the Study and Treatment of Esophageal Varices. Prediction of the first
variceal hemorrhage in patients with cirrhosis of the liver and esophageal varices: a prospective multicenter
study. N Engl J Med. 1988;319:983-989.
2. de Franchis R. Evolving consensus in portal hypertension. Report of the Baveno IV consensus workshop on
methodology of diagnosis and therapy in portal hypertension. J Hepatol. 2005;43:167-176.
3. Banares R, Albillos A, Rincon D, et al. Endoscopic treatment versus endoscopic plus pharmacologic treatment for
acute variceal bleeding: a meta-analysis. Hepatology. 2002;305:609-615.
4. McCormick PA, Dick R, Panagou EB, et al. Emergency transjugular intrahepatic portosystemic stent shunting as a
salvage treatment for uncontrolled variceal hemorrhage. Br J Surg. 1994;81:1324-1327.
68  Question 15

5. Bernard B, Grange JD, Khac EN, Amiot X, Opolon P, Poynard T. Antibiotic prophylaxis for the prevention of bacterial
infections in cirrhotic patients with gastrointestinal hemorrhage: a meta-analysis. Hepatology 1999;29:1655-1661.
6. Rimola A, Garcia-Tsao G, Navasa M, et al. Diagnosis, treatment and prophylaxis of spontaneous bacterial perito-
nitis: a consensus document. J Hepatol. 2000;32:142-153.
16 QUESTION

BOUGIENAGE, BALLOON DILATION, CAUTERY CUTTING, STENTING,


AND STEROID INJECTION HAVE ALL BEEN DESCRIBED TO
TREAT BENIGN STRICTURES OF THE GI TRACT.
IS THERE A RIGHT WAY OF DOING THIS?

Jeffrey L. Conklin, MD, FACP

When faced with a symptomatic esophageal stricture, the first thing to do is determine its
etiology, if possible. Is it benign or malignant? If benign, is it peptic, a manifestation of eosino-
philic esophagitis, from caustic injury, or the result of some therapeutic intervention (eg, radiation
injury, surgical anastomosis, or the result of an ablative therapy)? The treatment approach is likely
to vary somewhat depending on these factors. Benign strictures are further classified according
to their physical characteristics. Strictures that are short, symmetrical, and allow easy passage of
a standard adult endoscope are called simple strictures (eg, mild peptic strictures, Schatzki rings,
and webs). Dysphagia caused by simple strictures is, in most cases, successfully treated with 1 to
3 dilations and treatment of underlying causes, such as gastroesophageal reflux. A stricture that is
longer (> 2 cm), is irregular in contour, and constricts the esophageal lumen so the endoscope can-
not easily traverse the narrowing is called a complex stricture. Complex strictures are more difficult
to treat, and their treatment is more likely to be associated with complications.

Esophageal Dilation
The initial therapy for benign esophageal stricture is dilation. Three general types of dila-
tors are used: 2 types of mechanical bougies and through-the-scope (TTS) balloon dilators. The
oldest are mercury- or tungsten-weighted rubber or silicone bougies that come in graded diam-
eters (eg, Hurst dilators that have a rounded tip [Medovations, Inc], or Maloney dilators with a
tapered tip [Medovations, Inc]). These dilators are passed down the esophagus without a guide-
wire. Only simple strictures should be treated with these dilators because the risk of perforating
complex strictures with them is greater than with other types of dilators. For this reason, and

Leung J, Lo SK, eds. Curbside Consultation in Endoscopy:


69 49 Clinical Questions, Second Edition (pp 69-72)
© 2014 SLACK Incorporated
70  Question 16

because physicians feel uncomfortable performing blind dilation, use of these dilators has fallen
out of favor. The most extensively used wire-guided bougie is the Savary-Gilliard dilator (Cook
Medical). It is made of polyvinyl, is tapered at one end, and has a central lumen that allows it to be
advanced over a guidewire. Stricture dilation by both types of mechanical bougie is accomplished
by sequentially passing dilators with progressively larger diameters across the stricture. The TTS
balloon dilator is a sausage-shaped, stiff-walled balloon that is mounted at the end of a relatively
inflexible catheter so that it can be passed into the esophageal lumen via the endoscope’s operating
channel. The balloon is inflated with water in a stepwise fashion. The first step is to determine
the balloon’s initial operating diameter, which is usually close to what the endoscopist judges to
be the stricture’s luminal diameter. From this point, the balloon can be inflated in up to 2 steps
to increase its diameter 2 to 3 mm. Blind and wire-guided bougienage do not differ in efficacy
or safety.
It is generally accepted, but far from proven, that the safest way to perform bougie dilation
of esophageal strictures is to follow “the rule of 3.” The diameter of the first dilator used should
be the known (or estimated) luminal diameter of the stricture. Dilators with progressively larger
diameters are sequentially passed across the stricture until resistance to passage is met. From this
point, no more than 3 consecutive dilations in 1-mm increments should be done before ending the
session. When using TTS balloons to dilate simple strictures, it might be safe to exceed the rule
of 3. Having said this, it is probably always best to be conservative and err on the side of underdi-
lating when the stricture is complex and difficult. This is particularly true when a large mucosal
rent is produced—it is best to stop right there.
There are times when the stricture is too tight to allow passage of the adult upper endoscope.
In this situation, it is often possible to traverse the stricture with a pediatric upper endoscope and
place a guidewire that can be used to direct either the Savary-Gilliard or TTS balloon dilator.
When not even the pediatric endoscopy can pass the stricture, a floppy-tipped guidewire can be
introduced across the stricture and positioned under fluoroscopic guidance.
As stated previously, the first-line therapy for symptomatic, benign esophageal strictures is
dilation with tools and techniques described above. The goal of therapy is to resolve the patient’s
dysphagia. Of course, the goals of therapy, the risks, and the possible therapeutic options should be
discussed with the patient. Adequate resolution of dysphagia usually requires increasing the lumi-
nal diameter of a stricture to the range of 14 mm, but preferably greater, and sustaining it there.
This might require frequent, closely timed dilations for some complex strictures that rapidly return
to their predilation dimensions. Although there is no evidence-based guidance regarding how this
is done, experienced endoscopists might perform up to 5 dilations, spaced at 1-week intervals or
less, before switching to another therapy.

Refractory or Recurrent Strictures


Combined Steroid Injection and Dilation
What are the options if multiple dilations fail to open the stricture adequately or its luminal
diameter is not maintained for an adequate time (at least 1 month)? There are a few treatment
options. Which one is chosen depends on the type of stricture, experience of the endoscopist,
and to some extent, patient preference. Refractory or recurrent peptic esophageal strictures can
be managed by injection of steroids into the stricture, combined with dilation. The purported
mechanism of action for steroid injection is inhibition of inflammation and subsequent decrease
in collagen formation. Four-quadrant intralesional injection of 0.5 mL/quadrant triamcinolone
(40 mg/mL), with accompanying dilation and subsequent acid suppression, reduces the number
of dilations needed and lengthens the time between dilations. Although this approach appears
Is There a Right Way to Treat Benign Strictures of the GI Tract?   71

efficacious, treatment protocols vary among reports, and most trials are small and uncontrolled.
Clinical experience suggests that a therapeutic trial of dilation and steroid injection should end
after 3 sessions if not successful. In addition, steroid injection has been used only to treat peptic
stricture, so its applicability to the management of other types of strictures is not known.

Incisional Therapy
If the recalcitrant narrowing is at a fibrotic, surgical esophagogastric anastomotic stricture
(or Schatzki ring), incisional therapy can be contemplated. This approach is best taken when the
stricture is short (generally < 1 cm) and symmetrical. Reported techniques vary greatly among
practitioners: electrocautery alone or combined with dilation, electrocautery with a polypectomy
snare combined with argon plasma coagulation, or incision with a needle knife. Incising the stric-
ture in 4 quadrants might maximize the luminal opening. This approach appears to be safe and
effective, but no large controlled trials have been done.

Stenting
The latest treatment for recurrent or refractory esophageal strictures is short-term placement
of a self-expanding stent. The presumption is that prolonged dilation allows for resolution of
inflammation so that the stricture can heal. This approach might also allow tissue remodeling
at a greater sustained diameter so stricture recurrence is less likely after stent removal. The type
of stent to be used for benign esophageal diseases remains somewhat controversial. Open and
partially covered metal stents have fallen out of favor because ingrowth and overgrowth of hyper-
plastic tissue imbeds them in the esophageal wall, causing recurrent dysphagia and making the
stent removal difficult, dangerous, or impossible. When left for long periods, they can migrate
through the esophageal wall, cause a fistula, or precipitate bleeding. Use of plastic stents to treat
benign esophageal disease has gained favor because they can be retrieved relatively easily and
their complication rate is lower. The most common complication encountered is stent migration.
Successful resolution of dysphagia after short-term stenting with plastic stents is in the range of
50%. The final option for stenting of intractable esophageal strictures is a biodegradable stent,
which is dissolved and absorbed over a period of 2 to 3 months. It obviates the problems encoun-
tered with metal stents (tissue ingrowth) and plastic stents (migration), and its success rate is also
approximately 50%.
Unfortunately, there is little evidence to guide us in the optimal use of stents to manage refrac-
tory esophageal strictures. Stent removal after 4 to 8 weeks can be adequate therapy for most types
of stricture), but when the underlying pathophysiology is ischemia (eg, radiation injury or surgical
anastomosis), the stent might need to remain longer. The procedure can be repeated if the stric-
ture recurs or if the stent was not left in long enough. It is probably prudent to perform endoscopy
periodically to make sure there is no overgrowth of hyperplastic tissue.
Strictures of the proximal esophagus should not be treated by stenting. In this location, stents
produce severe pain, and the complication rate is high. Instead, dilation and vigorous pursuit of
the underlying pathophysiology, if other than an esophagogastric anastomosis, is probably the best
approach.

Eosinophilic Esophagitis
Eosinophilic esophagitis (EoE) is a special case regarding management of esophageal stric-
tures. Esophageal dilation is a safe and effective treatment for strictures or narrow-caliber
esophagus caused by EoE, but the typical approach is to put off esophageal dilation at the initial
diagnostic endoscopy, unless the esophageal lumen is critically narrowed. Dilation, if needed, is
72  Question 16

usually performed during a second endoscopy done to evaluate the response to a medical or dietary
therapy. How to go about doing dilation for EoE is still not entirely clear. If bougies are chosen,
they should be wire guided. Their major advantage is that they dilate the entire esophageal lumen
in one pass. Their major disadvantage is that the endoscope must be reintroduced after each dilator
is passed to assess mucosal injury. The advantages of TTS balloons are accurate appraisal of lumi-
nal diameter and direct visualization of the dilation process, so the procedure can be terminated
if a mucosal tear occurs. They do not, however, dilate the entire lumen with one inflation. Before
embarking upon esophageal dilation of EoE, it is critical to appraise the patient of its risks and
the very real likelihood of postprocedural chest pain.

Conclusion
Benign esophageal strictures are safely and effectively treated by a number of therapies. Most
respond well to dilation with wire-guided bougies or TTS balloons. Recalcitrant anastomotic
strictures and Schatzki rings often improve with incisional therapy, and peptic strictures improve
with intralesional steroid injection, accompanied by dilation. Refractory strictures can also be
managed with stents, preferably those that are fully covered plastic or biodegradable.

Suggested Readings
De Wijkerslooth LRH, Vleggaar FP, Siersema PD. Endoscopic management of difficult or recurrent esophageal stric-
tures. Am J Gastroenterol. 2011;106:2080-2091.
Dellon ES. Diagnosis and management of eosinophilic esophagitis. Clin Gastroenterol Hepatol. 2012;10:1066-1078.
Kamal A, Vaezi MF. Diagnosis and initial management of gastroesophageal complications. Best Pract Res Clin
Gastroenterol. 2010;24:799-820.
SECTION II
COLONOSCOPY
17 QUESTION

WHICH OF MY PATIENTS WITH ULCERATIVE COLITIS


NEED "SURVEILLANCE" COLONOSCOPY?
HOW SHOULD THE PROCEDURE BE PERFORMED?

Michael F. Picco, MD, PhD and Francis A. Farraye, MD, MSc

Patients with long-standing ulcerative colitis (UC) are at an increased risk for developing
dysplasia and colorectal carcinoma (CRC). This risk approaches 8% by 20 years and 18% by 30
years.1 Recent data suggest that the risk of CRC in patients with UC may be lower than previously
reported.2 Patients with extensive Crohn’s colitis also have an increased risk of CRC and should
undergo regular surveillance. 3 At present, despite a lack of evidence from randomized controlled
trials, surveillance colonoscopy is the best and most widely used method to detect dysplasia and
cancer in inflammatory bowel disease (IBD) patients.4 A number of factors are associated with
an increased risk of developing CRC in IBD. These include a longer duration of colitis, greater
extent of colonic involvement, family history of colorectal cancer, primary sclerosing cholangitis,
and severity of inflammation that is defined endoscopically and histologically.4,5 Using these clini-
cal factors, patients at an increased risk of developing dysplasia and/or CRC can be identified.
Endoscopic findings may also help in the stratification of patients at highest risk of developing
dysplasia or cancer. The presence of strictures (OR: 4.62; 95% CI, 1.03-20.8) and pseudopolyps
(OR: 2.29; 95% CI, 1.28-4.11) were associated with an increased risk of CRC in one study.6
There are several limitations to surveillance colonoscopy, and colonoscopy practices are not
uniform. Multiple biopsies are needed, which is time consuming. It has been estimated that
33 biopsies are required to achieve 90% confidence to detect dysplasia if it is present.4 A recent
study demonstrated that dysplasia is more common in the rectum and sigmoid regions compared
to the more proximal colon, confirming guidelines that recommend additional biopsies be taken
in these areas.7 Table 17-1 is a modification of the American Gastroenterological Association rec-
ommendations for the performance of surveillance colonoscopy and management of endoscopic/
histologic findings.4

Leung J, Lo SK, eds. Curbside Consultation in Endoscopy:


75 49 Clinical Questions, Second Edition (pp 75-79)
© 2014 SLACK Incorporated
76  Question 17

Table 17-1

American Gastroenterological Association Surveillance


Guidelines for Colorectal Neoplasia in IBD4
● All patients, regardless of extent of disease at initial diagnosis, should undergo
a screening colonoscopy a maximum of 8 years after onset of symptoms, with
multiple biopsy specimens obtained throughout the entire colon, to assess the
true microscopic extent of inflammation.
● Patients with ulcerative proctitis or ulcerative proctosigmoiditis are not con-
sidered at increased risk for IBD-related CRC and thus may be managed on the
basis of average risk recommendations.
● Patients with extensive or left sided colitis should begin surveillance 1 to 3
years after the initial screening colonoscopy.
● After 2 negative exams (no dysplasia or cancer), further surveillance exams
should be performed every 1 to 3 years.
● Patients with PSC should begin surveillance colonoscopy at the time of PSC
diagnosis and then yearly.
● Patients with a history of CRC in a first-degree relative, ongoing active endo-
scopic or histologic inflammation, or anatomic abnormalities (shortened colon,
multiple pseudopolyps, or stricture), may benefit from more frequent surveil-
lance colonoscopy.
● Representative biopsy specimens from each anatomic section of the colon
is recommended. Although no prospective trials have determined the opti-
mal number of biopsies to take, one study has recommended a minimum of
33 biopsy specimens
● Surveillance colonoscopy should ideally be performed when the patient is in
remission.
● As the sensitivity for detecting dysplasia by chromoendoscopy is higher than
that of white light endoscopy, chromoendoscopy with targeted biopsies is
recommended as an alternative to random biopsies for endoscopists who have
expertise with this technique.
● These recommendations apply to patients with Crohn s colitis who have dis-
ease involving at least one-third of their colon.

Adapted from Farraye FA, Odze RD, Eaden J, Itzkowitz SH. AGA technical review on the diagnosis and manage-
ment of colorectal neoplasia in inflammatory bowel disease. Gastroenterology. 2010;138(2):746-774.e4.

The finding of flat, high-grade dysplasia (HGD) confirmed by 2 gastrointestinal pathologists


or carcinoma in endoscopic biopsy samples is an indication for colectomy. The management of
low-grade dysplasia (LGD) is more controversial. Strong consideration for colectomy should be
given for flat multifocal LGD (more than one colon location) or LGD found on more than one
colonoscopy because of the relatively high rate of progression to HGD or cancer.4
Dysplasia in IBD may occur in flat mucosa (endoscopically invisible) or as an elevated lesion on
endoscopy. In fact, most dysplasia found in patients with IBD is elevated.8,9 It is now accepted that
endoscopically resectable dysplastic lesions (polypoid dysplasia), also called adenoma-like lesions/
How Should "Surveillance" Colonoscopy Be Performed?   77

Figure 17-1. Approach to dysplasia in UC. 4


(Adapted from Farraye FA, Odze RD, Eaden
J, Itzkowitz SH. AGA technical review on the
diagnosis and management of colorectal
neoplasia in inflammatory bowel disease.
Gastroenterology. 2010;138(2):746-774.e4.)

Figure 17-2. Flat dysplastic lesion on chromo-


endoscopy.

masses (ALMs), without associated flat dysplasia surrounding the lesion or elsewhere in the colon
may be treated adequately by polypectomy and continued surveillance. In contrast, endoscopically
unresectable polypoid dysplastic lesions (nonadenoma-like dysyplasia-associated lesions or masses
[DALMs]), remain an indication for colectomy because of their high association with cancer.4
An approach to the management of dysplastic lesions in IBD, modified from the American
Gastroenterological Association, is summarized in Figure 17-1.
Newer techniques are needed to facilitate the identification of neoplastic lesions in patients
with IBD. Chromoendoscopy is the technique most readily applicable in clinical practice.10
Chromoendoscopy can improve the detection of subtle colonic lesions, raising the sensitivity of
the endoscopic examination and improving lesion characterization, increasing the specificity of
the examination (Figure 17-2). In addition, crypt architecture can be categorized using the pit
pattern, aiding in differentiation between neoplastic and non-neoplastic changes, and enabling the
performance of targeted biopsies. Several different stains have been used, including contrast stains
(indigo carmine) and vital stains (methylene blue). These are typically applied using a spray cath-
eter or through a standard water pump attached to the colonoscope. In a recent meta-analysis of
6 studies involving 1277 patients, the difference in yield of dysplasia between chromoendoscopy
and white light endoscopy was 7% (95% CI, 3.2-11.3) on a per patient analysis with an number
needed to treat (NNT) of 14.3. The difference in proportion of lesions detected by targeted
biopsies was 44% (95% CI, 28.6-59.1) and flat lesions was 27% (95% CI, 11.2-41.9) in favor of
chromoendoscopy.11
78  Question 17

Given these findings, the practice of taking multiple random biopsies of nonpolypoid mucosa
(ie, nontargeted biopsies) during UC surveillance has been called into question. Rutter et al12
found that there was no dysplasia in 2904 nontargeted biopsies performed.12 Marion did find 3
cases (0.09%) of low-grade dysplasia but this was among 3264 nontargeted random biopsies and
prior to chromoendoscopy staining.13 One of these cases had a dysplastic lesion after dye staining
in area of the positive random biopsy. Although random biopsy may find dysplasia, these rates
are extremely low, suggesting that random biopsy may be abandoned if chromoendoscopy is per-
formed, but this remains controversial.
Despite convincing data from the literature, chromoendoscopy has not been universally
accepted. Although the American Gastroenterological Association and the British Society of
Gastroenterology have endorsed the technique, it has not been recommended by the American
College of Gastroenterology for general UC surveillance.4,14,15 The American College of
Gastroenterology chose not to recommend the technique because of lack of knowledge of the
natural history of the lesions detected by chromoendoscopy, but did suggest a possible benefit in
high-risk patients (those with a history of dysplasia). Before chromoendoscopy can be accepted
widely, further studies are needed to determine its feasibility and to develop quality standards for
adopting it into general practice.
Narrow band imaging is a more convenient technique compared with chromoendoscopy and
has shown utility in the assessment of Barrett’s metaplasia. Recent studies did not find a benefit
in the detection of dysplasia. This may be because of difficulties distinguishing dysplasia from
background inflammation with this technique.16,17
Given the inherent difficulties in the performance of surveillance colonoscopy, it has been sug-
gested that chemoprevention be explored as a method to lower the risk of developing dysplasia
and CRC in IBD.18 Chemoprevention refers to the use of drugs to reverse, suppress, or delay the
process of carcinogenesis. Several agents have been suggested as chemopreventive, including folic
acid, NSAIDs, and 5-aminosalicylic acids (5ASAs). It must be made clear that there is insuffi-
cient evidence to modify present screening and surveillance practices in IBD patients taking these
medications and that chemoprevention is not a substitute for surveillance colonoscopy.

References
1. Eaden J. Review article: colorectal carcinoma and inflammatory bowel disease. Aliment Pharmacol Ther.
2004;20(Suppl 4):24-30.
2. Loftus EV. Epidemiology and risk factors for colorectal dysplasia and cancer in ulcerative colitis. Gastroenterol Clin
North Am. 2006;35(3):517-531.
3. Friedman S, Rubin PH, Bodian C, et al. Screening and surveillance colonoscopy in chronic Crohn’s colitis.
Gastroenterology. 2001;120(4):820-826.
4. Farraye FA, Odze RD, Eaden J, Itzkowitz SH. AGA medical position statement on the diagnosis and management
of colorectal neoplasia in inflammatory bowel disease. Gastroenterology. 2010:138:738-745.
5. Rutter M, Saunders B, Wilkinson K, et al. Severity of inflammation is a risk factor for colorectal neoplasia in ulcer-
ative colitis. Gastroenterology. 2004;126(2):451-459.
6. Rutter MD, Saunders BP, Wilkinson KH, et al. Cancer surveillance in longstanding ulcerative colitis: endoscopic
appearances help predict cancer risk. Gut. 2004;53(12):1813-1816.
7. Goldstone R, Itzkowitz S, Harpaz N, Ullman T. Dysplasia is more common in the distal rather than proximal colon
in ulcerative colitis surveillance. Inflamm Bowel Dis. 2012;18:832-37.
8. Rutter MD, Saunders BP, Wilkinson KH, et al. Most dysplasia in ulcerative colitis is visible at colonoscopy.
Gastrointest Endosc. 2004;60(3):334-339.
9. Rubin DT, Rothe JA, Hetzel JT, Cohen RD, Hanauer SB. Are dysplasia and colorectal cancer endoscopically visible
in patients with ulcerative colitis? Gastrointest Endosc. 2007;65(7):998-1004.
10. Kiesslich R, Neurath MF. Chromoendoscopy in inflammatory bowel disease. Gastroenterol Clin North Am.
2012;41(2):291-302.
How Should the Procedure be Performed?   79

11. Subramanian V, Mannath J, Ragunath K, Hawley CJ. Meta-analysis: the diagnostic yield of chromoendoscopy
for detecting dysplasia in patients with colonic inflammatory bowel disease. Aliment Pharmacol Ther. 2011;33:
304-312.
12. Rutter MD, Saunders BP, Schofield G, et al. Pancolonic indigo carmine dye spraying for the detection of dysplasia
in ulcerative colitis. Gut. 2004;53(2):256-260.
13. Marion JF, Waye JD, Present DH, et al. Chromoendoscopy-targeted biopsies are superior to standard colono-
scopic surveillance for detecting dysplasia in inflammatory bowel disease patients: a prospective endoscopic
trial. Amer J Gastroenterol. 2008;103:2342-2349.
14. Cairns SR, Scholefield JH, Steele RJ, et al. Guidelines for colorectal cancer screening and surveillance in moderate
and high risk groups (update from 2002). Gut. 2010;59:666-689.
15. Kornbluth A, Sachar DB. Ulcerative colitis practice guidelines in adults: American College of Gastroenterology,
Practice Parameters Committee. Amer J Gastroenterol. 2010;105:501-523.
16. Van den Broek JC, Fockens P, van Eeden S, et al. Narrow-band imaging versus high-definition endoscopy for the
diagnosis of neoplasia in ulcerative colitis. Endoscopy. 2011;43:108-115.
17. Ignjatovic A, East JE, Subramanian V, et al. Narrow-band imaging for detection of dysplasia in colitis: a random-
ized controlled trial. Am J Gastroenterol. 2012;107(6):885-90. doi:10.1038/ajg.2012.67
18. Subramanian V, Logan RF. Chemoprevention of colorectal cancer in inflammatory bowel disease. Best Pract Res
Clin Gastroenterol. 2011;25(4-5):593-606.
18 QUESTION

I HAD A DIFFICULT COLONOSCOPY AND NO MATTER WHAT I TRIED,


I JUST COULD NOT REACH THE CECUM. I MAXED OUT ON IV SEDATIONS
AND HAVE NO ANESTHESIOLOGY SUPPORT.
CAN YOU HELP ME?

Felix W. Leung, MD, FACG

It sounds like you are describing what most colleagues consider a difficult colonoscopy. From
your comment of “no matter what I tried,” I assume you have already adhered to the use of
minimal air insufflation, appropriate loop reduction, use of abdominal compression, and changing
patient position. If these indispensable maneuvers for optimizing success with any method to aid
insertion have not been used, try them before considering other measures.
I have been interested in difficult colonoscopy, especially in unsedated patients, since 2002. I
found a plethora of reports that I summarized in a systematic review.1 Double-balloon colonoscopy
has salvaged failed conventional colonoscopy. Push-and-pull enteroscopy, using a single-balloon
technique; propofol sedation; and attention to techniques, external straightener, upper endo-
scope; and pediatric colonoscope after guidewire exchange; enteroscope, with or without external
straightener; and Cath-Cam guidewire-directed colonoscopy all were reported to enhance cecal
intubation. Repeat bowel preparation, followed by next day colonoscopy, ensured improved success
of cecal intubation. Scheduling in the morning was associated with better bowel preparation and
higher cecal intubation rate. Transparent hood attached to the tip of colonoscope reduced pain and
enhanced cecal intubation. Carbon dioxide reduced pain after colonoscopy.
However, I found that water-related methods as adjuncts to air insufflation are most promising
because of their simplicity.2 My failure in the unsedated patient indicated that air insufflation was
the main culprit in preventing success. The insufflated air sharpens angulations at flexures and
lengthens the colon, preventing cecal intubation in the unsedated patient. With the help of inter-
ested colleagues, I researched the question of whether colonoscopy augmented by modifications
of water-related methods could be successful without any air insufflation. Maneuvers described
as adjuncts in the water-related methods were combined with air exclusion and water exchange
during insertion. I successfully completed a series of difficult cases using the water exchange

Leung J, Lo SK, eds. Curbside Consultation in Endoscopy:


81 49 Clinical Questions, Second Edition (pp 81-85)
© 2014 SLACK Incorporated
82  Question 18

method in unsedated patients with a real history of, or the potential for, paradoxical agitation due
to chronic narcotic pain medication use. These cases were reported in 2008. 3 Since then, I have
published a post hoc analysis of 2 randomized controlled trials confirming the efficacy of the water
exchange method in difficult colonoscopy—it enhances cecal intubation in unsedated patients
with a history of abdominal surgery.4 Most recently, I summarized my thoughts on this subject in a
hypothesis-generating review of the water exchange method for difficult colonoscopy.5 Therefore,
I believe I am in an excellent position to address your question.
Depending on the skill of the operator and his or her diligence in implementing minimal air
insufflation, loop reduction, use of abdominal compression, and changing patient position, dif-
ficult colonoscopy occurs in up to 10% of cases performed with traditional air insufflation, with
or without the aid of sedation. The mechanism of underlying failure in difficult colonoscopy
is not fully understood, but certain factors are known to be associated.5 Patient characteristics
include female gender, low body mass index (BMI) (≤ 25), female gender with low BMI, female
after hysterectomy, younger age (≤ 40 years), advanced age (> 50 years), older age (> 80 years), and
anxiety and anticipated discomfort. Past history characteristics include abdominal and/or pelvic
surgery, diagnosed left-sided diverticulosis, incomplete colonoscopy (redundant colon, difficult
sigmoid, difficult to sedate), previous unsatisfactory (poor) bowel preparation, history of irritable
bowel syndrome, and end-stage renal disease on hemodialysis. Current colonoscopy characteristics
include difficult anatomy, patient pain or discomfort, symptoms of inflammatory bowel disease,
prolonged insertion time, technically difficult insertion, lower gastrointestinal bleeding, obstruct-
ing malignancy, severe inflammation, poor bowel preparation, afternoon colonoscopy, colonos-
copy following gastroscopy, and unsedated colonoscopy.5
A number of water-related techniques have been described separately as adjuncts to usual air
insufflation 2 to minimize discomfort, reduce intubation time, and improve success rate of cecal
intubation. In line with other experts’ opinions, I have labeled them as “water immersion” tech-
niques because the water infused during insertion is removed predominantly during colonoscope
withdrawal.6,7 In contrast, the water exchange method entails removal of the infused water pre-
dominantly during colonoscope insertion.6,7 The aggregate data in approximately 12 randomized
controlled trials comparing water exchange or water immersion with conventional air insufflation
showed an overall significant reduction of pain scores—53% [IQR = 9.0, 51.5-60.5] with water
exchange and 28% [IQR = 9.5, 24.0-33.5] with water immersion.7 Qualitatively, the difference
is of a higher order of magnitude with water exchange than with water immersion.7 The water
exchange method is also associated with a serendipitous bonus of a higher adenoma detection
rate.7,8 Thus, of all the methods described in the literature for managing difficult colonoscopy,
the most useful one appears to be the water method with water exchange during insertion of the
colonoscope.6-11 Details of the components of the method9-11 are shown in Table 18-1. Acquisition
of the skills is easy, but it does require some practice.12
I Have No Anesthesiology Support. Can You Help Me?   83

Table 18-1

Components of the Water Exchange Method

1. Confirm proper function of the air and water pump on the colonoscope and
the accessory water pump used for delivery of water. Adjust suction to approxi-
mately half maximum and the water pump to almost maximum.
2. The air pump should be turned off to avoid inadvertent air insufflations, which
can elongate the colon.
3. All residual air should be removed when air pockets are encountered to mini-
mize angulations at the flexures and to shorten the colon. Point the tip of
the colonoscope into the air pocket and apply suction to collapse the lumen.
Removal of residual air and maintaining minimal distention of the lumen by
water allow the colon to wrap around the tip of the colonoscope, enhancing
the chance that the tip is pointing at the next lumen.
4. Direct the tip of the colonoscope to abut where the folds converge or the slit-
like opening ahead before starting water infusion.
5. If there is no obvious opening ahead, move the tip of the colonoscope sys-
temically in a large circular fashion while infusing and suctioning water. If the
colonoscope is equipped with separate water and suction channels, these
maneuvers are carried out simultaneously. If the colonoscope is equipped
with a common water and suction channel, these maneuvers are carried out in
rapid succession.
6. Infuse sufficient amount of water to confirm that the lumen ahead opens up
to allow passage. Stop water infusion if the lumen does not open, pull back
and redirect the tip of the colonoscope, and repeat the process. Large jerky
movements are less likely than slow deliberate movements to yield the proper
orientation to the next lumen.
7. Avoid suction of the mucosa by decreasing the level of wall suction. Initiating
water infusion to push the mucosa away just before pressing on the suction
button is helpful. Pointing the suction port (usually at 5 o clock) toward the
center of the lumen is another useful maneuver. You will see more of the muco-
sa on the left side and the upper part of the monitor screen.
8. Be patient if bowel preparation is poor. Remove as much of the suspended
feces as possible and infuse clean water for visualization of the lumen. It is eas-
ier to clean the mucosa in a collapsed water-filled colon during insertion with
water exchange than in a distended air-filled colon during withdrawal with
the water jet followed by suction. In a collapsed lumen, even a small amount
of infused water soaks the entire surrounding mucosa and the turbulence
produced by water exchange dislodges feces adherent to the mucosa for easy
removal by suction.

(continued)
84  Question 18

Table 18-1 (continued)

Components of the Water Exchange Method

9. When the insertion is going smoothly, do not forget to remove the infused
water by suction. Looping can easily develop in a colon filled with water and
increase discomfort for the patient. There is likely to be too much water in the
colon if the appearance of the lumen surrounding the tip of the colonoscope
is round, rather than slit-like and narrowed. More suction than infusion is
called for.
10. Note the underwater appearance of diverticular openings to avoid inappropri-
ate infusion of water into these lumens.
11. The underwater appearances of the appendix orifice (folds appearing as con-
centric rings) and red suction marks in the cecum indicate cecal intubation.
Remove as much of the water in the cecum as possible before insufflating air
to initiate the withdrawal process.
12. Shortening the colonoscope, compressing the abdominal, and changing
patient position are integral components of the water method. Employ these
maneuvers if necessary (eg, when lumen ahead cannot be seen, paradoxical
movement occurs). These maneuvers are necessary from time to time but
may be needed less often than when the air method is used.

Adapted from Leung FW, Leung JW, Mann SK, Friedland S, Ramirez FC. The water method significantly enhanc-
es patient-centered outcomes in sedated and unsedated colonoscopy. Endoscopy. 2011;43(9):816-821 and
Leung FW. Water exchange may be superior to water immersion for colonoscopy. Clin Gastroenterol Hepatol.
2011;9(12):1012-1014.

References
1. Leung FW. Methods of reducing discomfort during colonoscopy. Dig Dis Sci. 2008;53(6):1462-1467.
2. Leung FW. Water-related method for performance of colonoscopy. Dig Dis Sci. 2008;53(11):2847-2850.
3. Leung FW. Unsedated colonoscopy for paradoxical agitation: an unusual practice for an uncommon complication
in US veterans. Am J Gastroenterol. 2008;103(6):1578-1579.
4. Leung FW, Mann SK, Leung JW, Siao-Salera RM, Guy J. The water method is effective in difficult colonoscopy –
it enhances cecal intubation in unsedated patients with a history of abdominal surgery. J Interv Gastroenterol.
2011;1(4):172-176.
5. Leung FW. A hypothesis-generating review of the water method for difficult colonoscopy. Scand J Gastroenterol.
2011;46(5):517-521.
6. Leung FW, Harker JO, Leung JW, et al. Removal of infused water predominantly during insertion (water exchange)
is consistently associated with a greater reduction of pain score—review of randomized controlled trials (RCTs) of
water method colonoscopy. J Interv Gastroenterol. 2011;1(3):114-120.
7. Leung FW, Amato A, Ell C, et al. Water-aided colonoscopy: a systematic review. Gastrointest Endosc. 2012;76(3):657-
666.
8. Leung FW, Harker JO, Leung JW, et al. Removal of infused water predominantly during insertion (water exchange)
is consistently associated with an increase in adenoma detection rate—review of data in randomized controlled
trials (RCTs) of water-related method. J Interv Gastroenterol. 2011;1(3):121-126.
I Have No Anesthesiology Support. Can You Help Me?   85

9. Leung FW, Leung JW, Mann SK, Friedland S, Ramirez FC. The water method significantly enhances patient-
centered outcomes in sedated and unsedated colonoscopy. Endoscopy. 2011;43(9):816-821.
10. Leung FW. Water exchange may be superior to water immersion for colonoscopy. Clin Gastroenterol Hepatol.
2011;9(12):1012-1014.
11. Leung FW, Leung JW, Mann SK, Friedland S, Ramirez FC, Olafsson S. DDW 2011 cutting edge colonoscopy tech-
niques—state of the art lecture master class – warm water infusion/CO2 insufflation for colonoscopy. J Interv
Gastroenterol. 2011;1(2):78-82.
12. Ramirez FC, Leung FW. The water method for aiding colonoscope insertion: the learning curve of an experienced
colonoscopist. J Interv Gastroenterol. 2011;1(3):97-101.
19 QUESTION

A 68-YEAR-OLD MALE WITH SIGNIFICANT COMORBIDITIES PRESENTED


WITH WEIGHT LOSS AND NEW ONSET VOMITING. INVESTIGATION
SHOWED COLONIC OBSTRUCTION WITH POORLY DIFFERENTIATED
ADENOCARCINOMA INVOLVING THE SIGMOID COLON.
DO I CONSIDER STENTING?

Todd H. Baron, MD, FASGE

The short answer to this question is absolutely yes, you should consider placement of a self-
expandable metal stent for this patient. But before we get to that definitive recommendation, you
must first recognize that acute, complete colonic obstruction is a medical emergency and doing
nothing is not an option. It is a medical emergency because colonic perforation may occur from
distension of the cecum and/or ischemia of the right colon.
Your options in this case are to place a colonic stent, surgical ileostomy, or colostomy (with or
without resection of the sigmoid mass), and percutaneous decompression of the right colon by an
interventional radiologist. You could also consider placement of a colonic decompression tube to
relieve the obstruction while deciding on management options. Percutaneous endoscopic colosto-
my (PEC) has been used for treatment of colonic pseudo-obstruction, although this would require
passage of the endoscope through the lesion with risk of perforation, and is the least viable option.
The advantage of colonic stent placement is the restoration of bowel continuity and relatively
low morbidity and mortality. Before undertaking colonic stent placement, you need to ask whether
you or your team have the skills and support needed to successfully place a stent. Although colonic
stent placement can be straightforward, it can also be the most technically challenging type of gas-
trointestinal luminal stent placement. Failure to place a colonic stent can lead to worse outcomes
than proceeding with more invasive approaches, especially if perforation and fecal spillage into
the peritoneum occurs.
The technical difficulties in colonic stent placement are multiple. First, this is a group of
patients who as in this case are both acutely ill and chronically ill. Coupled with abdominal
distension, this makes them tenuous and at high risk for sedation-related adverse events. Thus,
anesthesia support should be strongly considered. Second, the bowel cannot be prepared orally
Leung J, Lo SK, eds. Curbside Consultation in Endoscopy:
87 49 Clinical Questions, Second Edition (pp 87-88)
© 2014 SLACK Incorporated
88  Question 19

in the setting of complete colonic obstruction. Although enemas can be performed, the prep is
often suboptimal. There are also numerous technical challenges to successful stent placement.
The tumor and potential luminal opening are often seen en face when the lesion is located at
turns (descending sigmoid, rectal junction, splenic and hepatic flexures). Thus, guidewire place-
ment, the limiting step to successful stent placement, can be difficult to achieve. We have found
that endoscopists who have experience in pancreaticobiliary interventions, such as endoscopic
retrograde cholangiopancreatography (ERCP), have greater success and lower adverse event rates
than those without such experience. This is related to their knowledge of fluoroscopy, passage of
guidewires, and placement of self-expandable metal stents. Other technical challenges include the
need to limit air insufflation so that the upstream bowel is not perforated. Water infusion can be
used as described for successful colonoscopy. Use of carbon dioxide instead of room air may also
decrease the risk of barotrauma-induced perforation. It is also very important to have assistants
with expertise in complex endoscopic procedures and who are familiar with the deployment
mechanisms of expandable metal stents.
If you or one of your colleagues is not confident that successful stent placement can be achieved
endoscopically, you may wish to consider partnering with an interventional radiology colleague, or
have an interventional radiologist perform the entire procedure. Interventional radiologists have
an understanding of the nuances of traversing difficult strictures. Endoscopists have the ability to
work in close proximity to the lesion. Thus, the team approach of an endoscopist and interven-
tional radiologist may be better than either alone.
Surgical colostomy or ileostomy is not a technically difficult procedure; it can be performed
with low morbidity and mortality and, in some cases, it can be performed laparoscopically.
However, patient quality of life is less than ideal because patients prefer not to have stool exiting
the skin. The advantage of this approach is the durability of relief of colonic obstruction, which
reduces the need for repeat interventions as a result of stent occlusion or migration. In addition,
delayed stent-induced perforation can occur and limits the long-term success of stent placement.
The use of bevacizumab early after stent insertion is associated with a significant increase in per-
foration, and thus, consultation with an oncologist on subsequent treatment planning is important
prior to consideration of stent placement.
Percutaneous colostomy is a nonsurgical option that may also allow subsequent antegrade place-
ment of a colonic stent through the percutaneous tract after bowel decompression.
Overall, if there is a high likelihood that successful endoscopic stent placement can be achieved
with a low adverse event rate, I would recommend that as your first option. Discussion with the
patient and the family about the risks, benefits, and alternatives should be undertaken so that the
patient’s wishes are part of the decision-making process.

Bibliography
Dayyeh BK, Baron TH. Editorial: endoscopic stent placement as a bridge to surgery in malignant colorectal obstruc-
tion: a balance between study validity and real-world applicability. Am J Gastroenterol. 2011;106(12):2181-2182.
Park JS, Kim BG, Chang IT, et al. Placement of stents in proximal colonic obstructions using a percutaneous retroperi-
toneal colostomy. Surg Laparosc Endosc Percutan Tech. 2009;19(5):e202-e205.
Small AJ, Coelho-Prabhu N, Baron TH. Endoscopic placement of self-expandable metal stents for malignant colonic
obstruction: long-term outcomes and complication factors. Gastrointest Endosc. 2010;71(3):560-572.
20 QUESTION

A 58-YEAR-OLD FEMALE UNDERWENT A SCREENING COLONOSCOPY


THAT SHOWED A 2.5-CM FLAT POLYP IN THE PROXIMAL ASCENDING
COLON. I TATTOOED THE AREA AND BIOPSIED THE POLYP, WHICH WAS
INITIALLY REPORTED AS HYPERPLASTIC. WHAT SHOULD I DO?

Michael Bourke, MBBS, FRACP and


Farzan Fahrtash Bahin, MBBS (Hons)

Large, flat proximal colonic lesions are usually not hyperplastic. Serrated lesions (SL) are being
increasingly recognized as lesions with a significant premalignant potential. Due to their flat and
inconspicuous appearance, they can be difficult to detect. We will describe herein the significance
of large serrated lesions and an approach to safe and effective endoscopic treatment.

Lesion Detection and Characterization


The polyp described is most likely an SL. Large, flat polyps > 10 mm in the proximal colon
with histological features suggestive of a hyperplastic polyp (HP) are usually SLs. If a large,
proximal colonic lesion is histologically reported as hyperplastic, it is useful to consider having the
slides reviewed because a correct diagnosis is important, and there are significant therapeutic and
surveillance implications. There are 2 main subtypes of SLs:
1. Sessile serrated adenomas (SSAs) are the most common, often multiple, and usually found in
the proximal colon, particularly in females.
2. Traditional serrated adenomas (TSAs) are < 10% of the total, have a greater risk of malignant
transformation, and are typically located in the left colon.1
Proximal colonic SLs are usually flat and inconspicuous and may be easily overlooked
(Figure 20-1). A mucus cap is a characteristic feature and stool residue may also adhere to the
lesion. The endoscopist must be alert to these aspects because they are useful red flags to improve
SL detection. As a universal standard, endoscopists should aim for complete mucosal inspection
by various techniques, including cleaning by water jet irrigation and aspiration of fluid. This is

Leung J, Lo SK, eds. Curbside Consultation in Endoscopy:


89 49 Clinical Questions, Second Edition (pp 89-94)
© 2014 SLACK Incorporated
90  Question 20

Figure 20-1. Barely perceptible flat lesion at


the hepatic flexure, seen only on retroflex-
ion. (Reprinted with permission from Michael
Bourke, MBBS, FRACP.)

especially important for the detection of SLs because in sections of clean mucosa, an area of adher-
ent mucus or stool may harbor an SL beneath it. The importance of good-quality bowel prepara-
tion and meticulous efforts to identify subtle lesions cannot be overemphasized. These are factors
known to influence both adenoma and SL detection. 2

Lesion Biology and Significance


Twenty percent to 30% of colorectal carcinomas develop through the “serrated pathway,” which
is characterized by widespread gene inactivation via hypermethylation of promoter regions (the
CpG island methylator phenotype [CIMP]), BRAF mutations, and frequent microsatellite insta-
bility (MSI). Phenotypically these lesions arise in the proximal colon and have a greater incidence
in smokers and females. Elevated CIMP and MSI lesions such as those found in inconspicuous
SLs may have a more rapidly progressive transition to invasive carcinoma, and consequently lead
to interval carcinomas. Alternatively, they may be more consistently overlooked at colonoscopy,
despite a large size, as they are most often very flat (Paris classification 0-IIB), and thus can be
hard to recognize, even at the point of transition to invasive disease.1
The detection of this type of polyp may imply the presence of the serrated polyposis syn-
drome (SPS), formerly known as the hyperplastic polyposis syndrome (HPS). The World Health
Organization’s definition of a diagnosis of SPS is fulfilled if any of the following 3 clinical criteria
are met3:
1. More than 5 serrated polyps proximal to the sigmoid colon, 2 of which are greater than
10 mm in size
2. Any number of serrated polyps occurring proximal to the sigmoid colon in an individual who
has a first-degree relative with SPS
3. More than 20 serrated polyps of any size distributed throughout the colon
SPS is underrecognized by endoscopists and confers a greatly increased risk of colorectal car-
cinoma development of approximately 25% to 50%. 2 When a large SL is found, a very careful
interrogation of the colon under optimal conditions is required to examine for synchronous lesions.
What Should I Do for a Hyperplastic Polyp?   91

Refer or Resect?
Lesions larger than 20 mm carry different therapeutic implications and risk profiles, particu-
larly in the right colon. Conventional polypectomy has a risk of serious adverse consequences of
1:1000 to 3000 procedures, whereas wide field endoscopic resection carries a perforation rate of
1% and a clinically significant bleeding risk of 5% to 7%. Bleeding is dependent on the site of the
lesion, being generally 2% to 3% in the left colon and 10% to 12% in the right colon.4 Although
endoscopic resection has comparable efficacy and the most favorable safety and cost profile in
comparison with surgery, the patient needs to be appropriately informed about the potential risks
of the procedure. Performing an unexpected endoscopic resection during a busy day of performing
routine screening colonoscopy is also unlikely to lead to optimal outcomes.
Competent endoscopists should be comfortable with removing lesions up to 20 mm maximal
size in the right colon. Lesions larger than 25 mm require different and more advanced technical
skills and are probably best managed in tertiary care centers. Endoscopists routinely employing
endoscopic resection require the appropriate resources for optimal patient outcomes, including
expert nursing assistance, appropriate equipment, and surgical support.

Tattoo and Biopsy of the Lesion


Because SLs are often difficult to detect and thus potentially hard to identify subsequently,
marking the lesion is useful. Marking is also valuable for scar identification at surveillance.
However, the approach needs to be standardized for optimal outcomes. A 2-step technique for
tattoo is advised. A submucosal injection of saline creates the initial bleb; with the needle tip in
the same plane, the solution is changed to a carbon particle suspension and injection of 2 to 3 mL
is performed. The sterile carbon is not biologically inert and causes a fibroinflammatory response
in the submucosa, which may cause adherence between the mucosa and the muscularis propria,
creating difficulty with later endoscopic resection or increased risk of perforation due to adherence
between these 2 layers. Placing the tattoo at least 3 cm distal to the lesion is advised.
For any resectable lesion where biopsy is considered to be necessary, take 1 to 2 biopsies from
the edge. Avoid tunneling biopsies or the use of diathermy to minimize the subsequent develop-
ment of submucosal fibrosis, which may compromise subsequent endoscopic resection.

Lesion Assessment
The application of standardized lesion assessment tools facilitates optimal lesion assessment
and subsequent resection. Imaging enhancement, such as narrow-band imaging, may improve
characterization of SLs (Figures 20-2 and 20-3). Apply the Paris system of endoscopic classifica-
tion of superficial neoplastic lesions for standardized description of lesion morphology. Most SLs
have IIA or IIB morphology. The Kudo pit pattern and lesion granularity are used mainly for
assessment of adenomatous lesions.5 Photodocumention of the lesion should be performed. Size,
location, adjacent landmarks, and other features that may be relevant to endoscopic resection
should also be recorded.
92  Question 20

Figure 20-2. Lesion seen on narrow band


imaging. (Reprinted with permission from
Michael Bourke, MBBS, FRACP.)

Figure 20-3. Close-up view with narrow band


imaging. The margins of the lesion, which
were difficult to appreciate with white light
endoscopy, are more clearly defined. Note
the sessile shape, surface grooves, hetero-
geneous pit pattern, and low vascular inten-
sity. (Reprinted with permission from Michael
Bourke, MBBS, FRACP.)

Lesion Resection Technique

Injection Solution
Submucosal injection separates the mucosa from the muscularis propria and thus minimizes
the likelihood of deep resection or transmural injury that may lead to colonic perforation. Use of
a biologically inert dye that is taken up only by the submucosa (eg, indigo carmine) in the injec-
tion solution helps to confirm resection is taking place in the right plane and clearly delineates the
lesion’s margins and the extent of the submucosal cushion. This is particularly relevant for SLs
where margins are hard to discern. Use of a colloidal plasma volume expander solution, such as
succinylated gelatin (Gelofusine), in the injection solution creates a sustained and uniform muco-
sal lift. It has been shown to improve resection size and en bloc resection rate, decrease resection
time, and lead to superior clinical outcomes compared with normal saline.4 Hydroxyl ethyl starch
(Voluven) would be an appropriate, readily available, and inexpensive alternative agent to be con-
sidered in the United States where Gelofusine is not accessible.
What Should I Do for a Hyperplastic Polyp?   93

Figure 20-4. After endoscopic resection; note


the clear margins. Although initial biopsy his-
tology showed a serrated morphology, final
histology after resection was a mixed tubu-
lar adenoma and sessile serrated adenoma.
(Reprinted with permission from Michael
Bourke, MBBS, FRACP.)

Resection
The following points pertain to the essentials of endoscopic resection of this lesion (Figure 20-4):
● Start at one edge (generally the most difficult or inaccessible area, as this area will only
become more difficult to access later) and work sequentially from the point of first entry to
the submucosal plane.
● Start the injection before you stab the mucosa with the needle. This technique swiftly
identifies the submucosal plane.
● Elevate only each segment to be resected (sequential inject and resect approach).
● Include a margin of normal tissue (approximately 2 to 3 mm).
● Meticulously place the snare at the edge of the advancing defect.
● Carefully inspect the post endoscopic resection defect.

Postresection
Carefully inspect the post endoscopic resection margin and defect for residual lesion or signs of
deep injury. The “target sign” represents muscularis propria resection, indicating a deeper resec-
tion and, as such, a potential risk for perforation. A similar mirror target sign may be appreciated
on the resected specimen. Appropriate recognition of this sign allows for early closure of the defect
and may avoid the need for surgery or prolonged hospital admission.4,6

Screening and Surveillance Implications


The timing of endoscopic surveillance intervals after endoscopic resection of colonic later-
ally spreading tumors is dictated by the completeness of excision, underlying histology, and the
patient’s clinical context. If the lesion has been completely removed in 1 to 3 pieces, follow-up
colonoscopy at 12 months would be appropriate. Photodocumentation and biopsy of the resection
scar should be performed. The resection scar requires assessment, and the risk of metachronous
lesions is significant, thus a subsequent 12-month surveillance interval is appropriate.
If the patient has SPS, all serrated lesions should be completely excised in due course. In the
case of numerous large lesions (> 50) the risks and benefits of endoscopic resection and compliance
with endoscopic surveillance need to be weighed against surgical intervention and discussed with
94  Question 20

the patient. In the author’s experience, with diligent endoscopic follow-up, it is possible to com-
pletely clear the colon of residual disease. Initially, surveillance colonoscopies should be performed
once or twice per year.
Because first-degree relatives of patients with SPS are at an increased risk for the development
of colorectal cancer, the screening colonoscopy of this patient group should begin at age 40, or
10 years prior to the age at which any advanced colorectal neoplasia has occurred.

References
1. Leggett B, Whitehall V. Role of the serrated pathway in colorectal cancer pathogenesis. Gastroenterology.
2010;138(6):2088-2100.
2. Kahi CJ, Hewett DG, Norton DL, Eckert GJ, Rex DK. Prevalence and variable detection of proximal colon serrated
polyps during screening colonoscopy. Clin Gastroenterol Hepatol. 2011;9(1):42-46.
3. Snover DCA, Burt DJ, Odze RD. Serrated Polyps of the Colon and Rectum and Serrated (“Hyperplastic”) Polyposis.
Berlin: Springer Verlag; 2010.
4. Holt BA, Bourke MJ. Wide field endoscopic resection for advanced colonic mucosal neoplasia: current status and
future directions. Clin Gastroenterol Hepatol. 2012;10(9):969-979.
5. Kudo S, Hirota S, Nakajima T, et al. Colorectal tumours and pit pattern. J Clin Pathol. 1994;47(10):880–885.
6. Bourke M. Endoscopic mucosal resection in the colon: a practical guide. Tech Gastrointest Endosc. 2011;13(1):35-49.
21 QUESTION

WHAT IS THE ROLE OF CHROMOENDOSCOPY IN SCREENING


COLONOSCOPY? WILL IT HELP TO DIFFERENTIATE DIFFERENT TYPES OF
POLYPS, AND IS IT USEFUL FOR PROXIMAL SERRATED POLYPS?

Ihab I. El Hajj, MD, MPH and Charles J. Kahi, MD, MSc

Dye-based colonoscopy with chromoendoscopy (CE), or chromocolonoscopy, consists of pan-


colonic or lesion-targeted application of absorptive or contrast stains to an adequately cleansed
colonic mucosa. This can be done using a special spray catheter during conventional white light
colonoscopy. CE is inexpensive, safe, and relatively straightforward to learn and perform for an
endoscopist who is already experienced in conventional colonoscopy. The superficial structure of
lesions is enhanced by active mucosal uptake of dye (absorptive stains) or by the pooling of dye in
colonic pits and ridges (contrast stains). The spraying of the colon is followed by immediate obser-
vation of mucosal irregularities and pit patterns; this may require the use of magnification. Two
of the most commonly used dyes are methylene blue (absorptive) and indigo carmine (contrast).
CE, combined with screening colonoscopy, has 3 major potential applications:
1. To identify small and flat polyps that may be missed with conventional colonoscopy
2. To allow the demarcation of a neoplastic region identified during conventional colonoscopy
and ensure complete endoscopic resection
3. To help differentiate the histologic type of polyps
The question of whether CE increases the detection of neoplasia during colonoscopy has been
addressed by several randomized clinical trials1-8 (Table 21-1). In these trials, the proportion of
colonoscopies performed for an average-risk screening and/or surveillance indications ranged
between 20% and 100%. Overall, the data show that CE increases the overall detection of ade-
nomas, primarily due to increased yield for small adenomas without high-grade dysplasia. The
detection of hyperplastic and flat lesions, particularly in the proximal colon, is also increased.
However, CE prolongs procedure time, which may negatively affect its routine use with
screening colonoscopy. Our group conducted a multicenter, randomized controlled trial, com-
paring the adenoma yield of high-definition chromocolonoscopy to high-definition white light
colonoscopy in 660 average-risk screening patients undergoing their first screening colonoscopy.7

Leung J, Lo SK, eds. Curbside Consultation in Endoscopy:


95 49 Clinical Questions, Second Edition (pp 95-99)
© 2014 SLACK Incorporated
Table 21-1

Randomized Controlled Trials of Chromoendoscopy


96  Question 21

Author # of Study Indication Adenoma Higher Higher Higher Higher Extubation


(year) Patients Design Detection Yield for Yield for Yield Yield for Time (ET)
CE vs SC TA per HP per for Flat Right-Sided or Total
Patient Patient Lesions Lesions Procedure
per (Type) Time
Patient (TPT)
Brooker 259 Segmental CE Screening 33% vs 25% Ya Ya Ya Y* ET: 9 vs 5 *
et al vs SC (55%), (p = 0.173) (diminutive
(2002) surveillance adenomas,
(20%) HP)
Hursltone 260 Segmental CE Surveillance Not given Ya Ya Ya Y* ET: 17 vs 15
et al vs SC (with NS (80%) (flat and
(2004) spraying) diminutive
adenomas,
HP)
Lapalus 292 First exam: SC Screening 40% vs - Ya Ya Y ET: 27 vs
et al Second exam: (55%) 36% (diminutive 18 *
(2006) CE with struc- Surveillance (p = 0.47) adenomasa
ture enhance- (30%) HP)
ment vs SC
(continued)
Table 21-1 (continued)

Randomized Controlled Trials of Chromoendoscopy

Author # of Study Indication Adenoma Higher Higher Higher Higher Extubation


(year) Patients Design Detection Yield for Yield for Yield Yield for Time (ET)
CE vs SC TA per HP per for Flat Right-Sided or Total
Patient Patient Lesions Lesions Procedure
per (Type) Time
Patient (TPT)
Le Rhun 203 Segmental Screening 39% vs 31% N Ya Ya N ET: 26 vs 8 *
et al high resolution or surveil- (p = NS) (adenomas)
(2006) CE vs SC lance (85%)
Park et al 316 Double Screening 17% vs 5% Ya Ya - Ya ET: 112 vs
(2008) inspection of (35%), (p = 0.001) 108
cecum and surveillance (cecum to
AC with or (20%) HF)
without CE.b
Stoffel et 50 First exam: SC Surveillance 44% vs 17% Ya Ya Ya Ya TPT: 37 vs
al (2008) Second exam: (100%) (p < 0.01) (flat and 27 *
CE vs SC with diminutive
inspection adenomas)
> 20
Kahi et al 660 HD panco- Screening 55.5% vs Ya Ya Ya - TPT: 31 vs
(2010) lonic CE vs HD (100%) 48.4% (p = 22 a
white light SC 0.07)
Pohl et al 1008 Continuous Screening 46.2% vs Ya Ya Ya Y TPT: 17 vs
(2011) pancolonic CE (50%), 36.3% (p = (adenomas) 13 a
vs SC surveillance 0.02)
(20%)
What Is the Role of Chromoendoscopy in Screening Colonoscopy?   97

a
p value is significant; b Study limited to cecum and AC; AC, ascending colon; CE, chromoendoscopy; SC, standard colonoscopy; TA, tubular adenoma; HP, hyper-
plastic polyp; Y, yes; N, no; HF, hepatic flexure; , minutes; ", seconds; NS, normal saline; HD, high definition
98  Question 21

The adenoma detection rate (ADR) (55.5% vs 48.4%, P = .07), and the number of adenomas per
patient (1.3 vs 1.1, P = .07) were marginally higher in the CE group. There were no significant
differences in the number of advanced adenomas per patient. CE allowed the detection of sig-
nificantly more flat adenomas per patient (0.6 vs 0.4, P = .01), adenomas < 5 mm in diameter
per patient (0.8 vs 0.7, P = .03), and non-neoplastic lesions per patient (1.8 vs 1.0, P < .0001). A
recent randomized controlled trial by Pohl et al8 included 1008 patients who were randomized
to pan-colonic CE or conventional colonoscopy (approximately half for screening). The ADR
and mean number of adenomas per patient were significantly higher in the CE group (46.2% vs
36.3%, p = .002, and 0.95 vs 0.66, P < .001). In both studies, procedure time was longer in the
CE group, the increment in adenoma yield was mostly due to small and diminutive lesions with
low-grade dysplasia, and CE appeared to benefit certain endoscopists more than others. These
findings suggest decreased utility for routine chromocolonoscopy in average-risk screening
situations. CE may be better suited to help to improve the ADR of certain endoscopists and in
patients at higher risk for colorectal neoplasia. Current European and US guidelines endorse the
use of CE and targeted biopsies in ulcerative colitis surveillance to increase the detection rate of
dysplastic lesions and to reduce the number of unnecessary random biopsies.
CE can also be used to help better delineate polyp edges and to help ensure complete resec-
tion. CE following polyp resection helps to identify remnant islands of neoplastic tissue. This is
particularly useful for flat and depressed neoplasms, and polyps whose margins may be difficult
to demarcate with white light alone. The use of CE for this purpose likely depends largely on
endoscopist preference; some may elect to use preferentially electronic CE (such as narrow-band
imaging) for practical reasons.
Endoscopic pit pattern interpretation with CE allows the differentiation between non-
neoplastic and neoplastic lesions. However, this requires the concomitant use of high magnifica-
tion, which is not routinely available or used in the United States. The accuracy of CE for the
determination of colonic polyp histology has been addressed by several studies. The sensitivity
and specificity of indigo carmine CE for predicting polyp histology (adenomatous vs hyperplastic)
has ranged from 82% to 95%, and 64% to 95%, respectively.9-11 Relative to standard colonoscopy,
indigo carmine CE with magnification increased the accuracy for polyp histology prediction
from 84% to 96% in one study.11 However, high-resolution indigo carmine CE only marginally
increased the accuracy from 81% to 83% in another study.10 In light of these considerations, CE is
not currently considered a substitute for histologic diagnosis.
The final question is whether CE is useful for proximal serrated polyps. This is an important
issue, as serrated lesions are thought to be a significant contributor to interval colorectal cancers.
Serrated lesions include hyperplastic polyps, sessile serrated adenomas/polyps (SSA/P), and
traditional serrated adenomas. A better framework for this question is whether CE improves
colonoscopy yield for proximal serrated lesions, and whether CE allows for the differentiation
between hyperplastic polyps and the more advanced SSA/P, as these 2 types of serrated lesions
have similar surface characteristics. Neither issue has been well studied. Chromocolonoscopy has
been shown to increase the detection of hyperplastic polyps.1-3 However, a secondary analysis of
our chromocolonoscopy randomized controlled trial showed that the proximal serrated polyp
detection rates of high-definition chromocolonoscopy and high-definition white-light colonos-
copy were not significantly different (17.6% vs 21.9%; P = .34).12 No trial has been conducted
comparing CE to conventional colonoscopy specifically for the detection of nonadenomatous
polyps, and there are few data regarding the use of CE to help distinguish hyperplastic polyps
from SSA/P.
What Is the Role of Chromoendoscopy in Screening Colonoscopy?   99

References
1. Brooker JC, Saunders BP, Shah SG, et al. Total colonic dye-spray increases the detection of diminutive adenomas
during routine colonoscopy: a randomized controlled trial. Gastrointest Endosc. 2002;56(3):333-338.
2. Hurlstone DP, Cross SS, Slater R, et al. Detecting diminutive colorectal lesions at colonoscopy: a randomised con-
trolled trial of pan-colonic versus targeted chromoscopy. Gut. 2004;53(3):376-380.
3. Lapalus MG, Helbert T, Napoleon B, et al. Does chromoendoscopy with structure enhancement improve the
colonoscopic adenoma detection rate? Endoscopy. 2006;38(5):444-448.
4. Le Rhun M, Coron E, Parlier D, et al. High resolution colonoscopy with chromoscopy versus standard colonoscopy
for the detection of colonic neoplasia: a randomized study. Clin Gastroenterol Hepatol. 2006;4(3):349-354.
5. Park SY, Lee SK, Kim BC, et al. Efficacy of chromoendoscopy with indigocarmine for the detection of ascending
colon and cecum lesions. Scand J Gastroenterol. 2008;43(7):878-885.
6. Stoffel EM, Turgeon DK, Stockwell DH, et al. Chromoendoscopy detects more adenomas than colonoscopy using
intensive inspection without dye spraying. Cancer Prev Res (Phila). 2008;1(7):507-513.
7. Kahi CJ, Anderson JC, Waxman I, et al. High-definition chromocolonoscopy vs. high-definition white light colo-
noscopy for average-risk colorectal cancer screening. Am J Gastroenterol. 2010;105(6):1301-1307.
8. Pohl J, Schneider A, Vogell H, et al. Pancolonic chromoendoscopy with indigo carmine versus standard colonos-
copy for detection of neoplastic lesions: a randomised two-centre trial. Gut. 2011;60(4):485-490.
9. Eisen GM, Kim CY, Fleischer DE, et al. High-resolution chromoendoscopy for classifying colonic polyps: a multi-
center study. Gastrointest Endosc. 2002;55(6):687-694.
10. Apel D, Jakobs R, Schilling D, et al. Accuracy of high-resolution chromoendoscopy in prediction of histologic find-
ings in diminutive lesions of the rectosigmoid. Gastrointest Endosc. 2006;63(6):824-828.
11. Fu KI, Sano Y, Kato S, et al. Chromoendoscopy using indigocarmine dye spraying with magnifying observation is
the most reliable method for differential diagnosis between non-neoplastic and neoplastic colorectal lesions; a
prospective study. Endoscopy. 2004;36(12):1089-1093.
12. Kahi CJ, Li X, Eckert GJ, Rex DK. High colonoscopic prevalence of proximal colon serrated polyps in average-risk
men and women. Gastrointest Endosc. 2012;75(3):515-520.
22 QUESTION

I HAVE TROUBLE DOING A COMPLETE COLONOSCOPY IN


SOME OF MY PATIENTS BECAUSE OF POOR BOWEL PREPARATION.
WHAT SHOULD I DO TO INSURE A PROPERLY PREPARED COLON?

Han-Mo Chiu, MD, PhD

With the increasing evidence that screening, either with a fecal occult blood test or colonos-
copy, can prevent colorectal cancer, the demand for colonoscopy has increased remarkably. The
discomfort accompanying an endoscopic examination can be efficiently reduced by conscious seda-
tion, but ingesting purgatives for bowel preparation, according to many examinees, is the worst
part of the colonoscopy experience. Nevertheless, when using PEG-ELS (polyethylene glycol-
electrolyte lavage solution) or sodium phosphate, ingesting a large amount of liquid is inevitable
and necessary for adequate bowel preparation. When patients are informed that insufficient bowel
preparation has resulted in incomplete colonoscopy, they usually feel frustrated about the lack of
results and anxious about the need to repeat the examination.
Adequate bowel preparation is crucial for a complete colonoscopy.1 Previous studies have shown
that insufficient bowel cleansing may prolong the procedure, increase risk of complications, and
most importantly, make the operator more likely to miss significant lesions and resulting interval
cancers.2 It also ensures the need for a repeat examination, which costs the patient more lost time
from work and costs the system for both the repeat endoscopic examination and conscious seda-
tion. Increasing evidence indicates that split-dose cleansing or ingesting a whole dose of laxatives
shortly before the procedure may significantly improve the quality of bowel cleansing, especially in
the proximal colon, making these procedures more effective than a single dose taken the evening
before the procedure. 3

Leung J, Lo SK, eds. Curbside Consultation in Endoscopy:


101 49 Clinical Questions, Second Edition (pp 101-105)
© 2014 SLACK Incorporated
102  Question 22

Table 22-1

Diagnostic Yield of Colonoscopies Following a Morning


(On-the-Day) or Evening (Previous Day) Regimen4
Morning Group (n = 60) Evening Group (n = 58) P Value
Total lesions 2.78 (0.29) 1.90 (0.27) .028
Proximal lesions 1.52 (0.22) 0.97 (0.24) .094
Advanced 0.87 (0.13) 0.55 (0.10) .056
lesions

Reprinted by permission from Macmillian Publishers Ltd: The American Journal of Gastroenterology. Chiu HM,
Lin JT, Wang HP, Lee YC, Wu MS. The impact of colon preparation timing on colonoscopic detection of colorec-
tal neoplasms: a prospective endoscopist-blinded randomized trial. 101(12):2719-2725. 2006

Advantages and Practice of


Split-Dose or On-the-Day Regimen
Previous randomized trials have shown that either split-dose or on-the-day regimen could
achieve better quality of bowel cleansing than conventional preparation on the evening prior to the
procedure. In our unit, where most of the patients or examinees are Chinese or Asian, we use the
on-the-day regimen of 2 L PEG-ELS for colonoscopy and give the cleansing agent 5 to 8 hours
prior to procedure. Although no evidence exists to suggest that the amount of cleansing agent
should be body-size dependent, 4 L of PEG may be necessary for more obese subjects.
Our previous randomized study showed better preparation quality (93% vs 72% with excellent
or good preparation) and more lesion detection (2.78 vs 1.9 adenoma per patient) with on-the-day
regimen than with prior day preparation4 (Table 22-1). Moreover, our recent study also showed
that on-the-day regimen was associated with higher detection of nonpolypoid and proximal
adenoma 2 (Table 22-2). This is of utmost importance because several recent population studies
have shown that colonoscopy is less protective against proximal colon cancer and nonpolypoid
neoplasms, which are subtle and difficult to detect endoscopically, but they grow rapidly and are
prone to be proximally located (Figure 22-1). If these significant lesions remain undetected, some
may very likely turn into invasive cancers.
If your patients received a conventional previous-day bowel preparation for a prior poorly pre-
pared colonoscopy, you may simply advise them to change to the split-dose or on-the-day regimen.
If they tried split-dose or on-the-day bowel preparation in the previous colonoscopy but experi-
enced a failed cleansing, you may check whether they obeyed the diet restriction and give them
a stimulant laxative such as bisacodyl or senna with the cleansing agents, if not contraindicated.
Increasing the volume of PEG-ELS to 1.5 times the usual dose or encouraging oral hydration is
also helpful.
What Should I Do to Insure a Properly Prepared Colon?   103

Table 22-2

Cleansing Levels and Diagnostic Yields by


Preparation Schedule3
On-the-day Prior day P Value
(N = 1552) (N = 1527)
Cleansing Levels, n (%)
Excellent 197 (12.7%) 38 (2.5%) <.001
Good 930 (59.9%) 481 (31.5%)
Fair/adequate 354 (22.8%) 649 (42.5%)
Inadequate or poor 71 (4.6%) 359(23.5%)
Diagnostic Yields, n (%)
Adenoma
Overall 270 (17.4%) 233 (15.3%) .109
Proximal 175 (11.3%) 138 (9.0%) .040
Advanced adenoma
Overall 68 (4.4%) 46 (3.0%) .044
Proximal 34 (2.2%) 25 (1.6%) .263
Nonpolypoid neoplasm
Overall 98 (6.3%) 67 (4.4%) .018
Proximal 71 (4.6%) 40 (2.6%) .004
Advanced 25 (1.6%) 12 (0.8%) 0.036

Reprinted from Chiu HM, Lin JT, Lee YC, et al. Different bowel preparation schedule leads to different diagnos-
tic yield of proximal and nonpolypoid colorectal neoplasm at screening colonoscopy in average-risk popula-
tion. Dis Colon Rectum. 2011;54(12):1570-1577.

Implementation of Split-Dose or
On-the-Day Regimen in an Endoscopic Unit
There may be 2 barriers to popularization of the split-dose or on-the-day regimen. The first
barrier may come from anesthesiologists, who may forbid ingesting cleansing agents shortly before
the procedure because of a concern of increased risk of aspiration. Currently, no evidence shows
that split-dose or on-the-day regimen increases the risk of aspiration, and the American Society of
Anesthesiologists’ guidelines allow for ingestion of clear liquid up until 2 hours before sedation.5
Moreover, a recent study has shown that the volume of residual gastric fluid is almost the same in
patients receiving split-dose as in those receiving previous-evening bowel preparation. The other
barrier comes from patients, in that those whose colonoscopy is scheduled in the morning may
104  Question 22

Figure 22-1. Endoscopic view of colonic mucosa with on-the-day regimen (2 L PEG-ELS): With on-the-day
regimen, the time spent on washing or irrigating colonic mucosa can be largely reduced and endoscopist
can be more concentrated on inspection. A subtle nonpolypoid neoplasm was detected (arrow), which
turned out to be a mucosal cancer (carcinoma in situ). Such lesions are very unlikely to be detected with
inadequate bowel preparation.

be unwilling to get up early to ingest the cleansing agent. In our unit, when we implemented on-
the-day regimen for morning screening colonoscopy in 2008, only half of examinees were willing
to get up early to drink the cleansing agent. This proportion held until the end of 2009, when we
changed the patient education material to describe the benefits of on-the-day regimen, especially
in detecting adenomas. The proportion of those who were compliant began to rise significantly,
so that now nearly 90% of examinees are willing to accept the on-the-day regimen for morning
colonoscopy (Figure 22-2). A recent study also showed that as many as 85% of patients scheduled
to receive morning colonoscopy indicated they would be willing to get up during the night to
take the second dose of a split-dose regimen.6 Together, on-the-day and split-dose regimen are
becoming— and should be—standard practice for bowel preparation before colonoscopy because
of their good patient acceptance, better cleansing, and higher lesion detection rate.
What Should I Do to Insure a Properly Prepared Colon?   105

Figure 22-2. Trend of the proportion of examinees willing to undergo on-the-day regi-
men rather than previous-evening regimen for morning colonoscopy in a screening unit
of National Taiwan University Hospital, 2008-2010. Q1 denotes the first quarter of the year,
i.e. January to March, Q2 is April to June, and so on.

References
1. Cohen LB, Tennyson C. Bowel preparation for colonoscopy: maximizing efficacy, minimizing risk. Gastroenterology
& Endoscopy News, March 2010. http://www.gastroendonews.com/download/maxprep_GEN0310_WM.pdf
2. Froehlich F, Wietlisbach V, Gonvers JJ, Burnand B, Vader JP. Impact of colonic cleansing on quality and diagnostic
yield of colonoscopy: the European Panel of Appropriateness of Gastrointestinal Endoscopy European multi-
center study. Gastrointest Endosc. 2005;61(3):378-384.
3. Chiu HM, Lin JT, Lee YC, et al. Different bowel preparation schedule leads to different diagnostic yield of proximal
and nonpolypoid colorectal neoplasm at screening colonoscopy in average-risk population. Dis Colon Rectum.
2011;54(12):1570-1577.
4. Chiu HM, Lin JT, Wang HP, Lee YC, Wu MS. The impact of colon preparation timing on colonoscopic detection of
colorectal neoplasms: a prospective endoscopist-blinded randomized trial. Am J Gastroenterol. 2006;101(12):2719-
2725.
5. American Society of Anesthesiologists Committee. Practice guidelines for preoperative fasting and the use of
pharmacologic agents to reduce the risk of pulmonary aspiration: application to healthy patients undergoing
elective procedures: an updated report by the american society of anesthesiologists committee on standards
and practice parameters. Anesthesiology. 2011;114(3):495-511.
6. Unger RZ, Amstutz SP, SeoDH, Huffman M, Rex DK. Willingness to undergo split-dose bowel preparation for colo-
noscopy and compliance with split-dose instructions. Dig Dis Sci. 2010;55(7):2030-2034.
SECTION III
ENDOSCOPIC RETROGRADE
CHOLANGIOPANCREATO-
GRAPHY
23 QUESTION

A 55-YEAR-OLD VETERAN HAD A RUPTURED GALLBLADDER


AND A DIFFICULT CHOLECYSTECTOMY. POSTOPERATIVELY, THE PATIENT
HAD PERSISTENT BILE LEAKAGE DESPITE MONTHS OF
MULTIPLE BILIARY STENTING. WHAT CAN I DO?

Joseph Leung, MD, FRCP, FACP, FASGE, MACG and Catherine Ngo, MD

Bile leakage occurs as a result of injury to the biliary system. The cystic duct stump is the most
common cause of a bile leak following cholecystectomy. Less common, but more serious, causes
are injury to the bile duct and (rarely) transection of an aberrant branch duct to the right lobe in
close proximity to the cystic duct. In the latter case, persistent bile leakage occurs from the more
proximal bile duct, which is isolated from the main system. Injury to the bile duct may result in
persistent leakage if there is significant damage to the bile duct wall. In cases with difficult chole-
cystectomy (eg, a perforated necrotic gallbladder), the surgeon may not be able to tie off the cystic
duct or resect the entire gallbladder. The leakage in this situation would be different than leak-
age from the cystic duct stump. In the latter case, ischemia and bile irritation may cause fibrosis
around the smaller cystic duct opening, which favors early sealing off of the leak.
In general, endoscopic biliary stenting is effective in diverting bile through the stent back into
the duodenum and minimizing leakage from the injury site. However, a number of factors may
contribute to persistent leakage, in particular if there is a significant discrepancy between the
diameter of the stent and the damaged bile duct. Because bile tends to flow in the path of least
resistance, a stent with a small lumen may not be sufficient to decompress the biliary system, and
may fail to close off the bile leakage. A large cystic duct stump (because it is bigger than the lumen
of a single stent) and low insertion of the cystic duct (because of the hydrostatic column of bile
built up with a long stent, especially in the absence of a papillotomy) all tend to favor bile leakage.
A patent cystic duct (in cases where the gallbladder is partially removed) also tends to perpetuate
the bile leak.
The best approach to a patient with a complicated cholecystectomy and bile leak is to confirm
the site, and to some extent, the severity of the bile leakage at the time of endoscopic retrograde
cholangiopancreatography (ERCP) (including the use of occlusion cholangiogram if necessary) to
assess the outcome.
Leung J, Lo SK, eds. Curbside Consultation in Endoscopy:
109 49 Clinical Questions, Second Edition (pp 109-112)
© 2014 SLACK Incorporated
110  Question 23

A papillotomy alone may provide drainage for the bile duct and helps to close off a simple leak.
The management of a simple bile leak from the cystic duct stump can be achieved by insertion
of a 10-Fr indwelling plastic stent, with the proximal tip of the stent placed above the cystic duct
insertion. This will insure that the bile is collected by the stent. Although a biliary papillotomy
has been reported to be effective, papillotomy is not necessary for single-stent placement and this
will also avoid the risk of papillotomy-associated complications. A large papillotomy also may
predispose to upwards stent migration because the distal flap may not be able to hold the stent
in place. Other reports have described the use of nasobiliary catheter drainage to decompress the
biliary system, and this is also an effective treatment for a simple bile leak. However, an indwelling
nasobiliary catheter causes patient discomfort and carries a risk of external bile loss and electrolyte
imbalance. Proper anchorage of the nasobiliary catheter is necessary because the drain can be dis-
lodged. Improper anchorage may predispose to unnecessary trauma to the nose.
Leakage from the aberrant ducts of Luschka in the gallbladder bed can be controlled similarly
with an indwelling biliary stent, and they are never the cause of a persistent bile leakage. In rare
situations when the leakage is more peripheral, sometimes the intrabiliary pressure may be main-
tained if only a small stent is used, thus predisposing to persistent leakage. Placement of a longer
large-diameter stent (close to the origin of the branch of right hepatic duct that drains the aber-
rant ducts) is more helpful to divert the bile from the peripheral ducts. A more serious situation
is injury to an aberrant duct that arises from the cystic duct draining part of the right lobe. Injury
can occur when inadvertent ligation and duct transection result in free bile leakage but a failure
to demonstrate the leakage on cholangiogram. Fortunately, this complication is not common.
Management may involve interventional radiology with percutaneous transhepatic cholangiogram
to determine the exact site of ductal injury and leakage. In a complete transection, the injury may
require a surgical bilioenteric anastomosis to control the leak.

Associated Common Duct Injury


If the presence of a bile leak is associated with common bile duct injury, it is important to define
the location and extent of the injury. The most common cause of ductal injury is excess cautery
or thermal injury, often as a result of hemostasis. This may leave a vent in the bile duct wall as a
result of tissue necrosis. Damage may also be a result of clip injury to the bile duct. In both cases,
there is a risk of late bile duct stenosis or stricture formation.
In the management of the bile leak, it is necessary to seal off the area of ductal injury. Simply
providing drainage distally may not be sufficient to control the bile leak. We recommend negotiat-
ing the damaged duct with a guidewire and inserting a 10-Fr stent across the leak, which provides
drainage of bile and also helps to seal off the leak. At the same time, the stent provides splintage
of the damaged bile duct and promotes healing. It may be necessary to keep the stent in longer to
insure proper sealing of the bile leak. In the event of a late ductal stricture, access is maintained
across the stricture, which will facilitate subsequent endoscopic therapy. Balloon dilation is not
recommended in the immediate postsurgical period—even in the presence of possible stenosis—
to avoid extending the ductal injury and leakage. Partial clip injury to the bile duct may create a
stenosis and it may be necessary to resort to percutaneous transhepatic manipulation to try to cross
the area of injury and loosen the clip with balloon dilation. Placement of a stent across the area
of duct injury promotes healing and prevents further leakage. If necessary, repeat balloon dila-
tion and prolonged stenting may be required later to treat the bile duct stenosis. It is important to
consider percutaneous drainage for a significant amount of subhepatic bile collection, or biloma,
to prevent the abscess formation.
Persistent bile leakage may result from a large area of damage to the bile duct wall, a patent
cystic duct (as in cases where a cuff of the gallbladder is left behind), or a large cystic duct stump
What Can I Do for Bile Leakage Despite Months of Multiple Biliary Stenting?   111

Figure 23-1. Cholangiogram showing leakage


of contrast from a gallbladder remnant and
percutaneous drain in position.

A B
Figure 23-2. (A) Plain radiograph showing double biliary stent in the CBD. (B) Endoscopic view of double
stent placed for bile leakage.

that is not closed off properly (Figure 23-1). The relative discrepancy between the size of the leak
and lumen of the stent may be one reason for persistent leakage. If necessary, multiple stents can
be placed to insure adequate drainage from the bile duct (Figure 23-2). In those with a large cystic
duct stump or a low cystic duct insertion, we may need to use long and short stents, in addition to
a papillotomy to divert all of the bile back into the duodenum.
In this patient, who has persistent bile leakage despite several months of stenting, we would
consider the use of a fully covered (and retrievable) metal expandable stent to provide drainage.
112  Question 23

B
Figure 23-3. (A) Radiograph showing fully cov-
ered self-expandable metallic stent (SEMS) within
common bile duct blocking cystic duct insertion.
(B) Distal end of patent SEMS seen in duodenum.
Sludge is seen accumulated around the distal end
of the stent.

The large diameter of the stent lumen insures complete emptying of the bile duct, and the covering
membrane blocks the cystic duct origin and prevents further leakage. A fully covered self-expand-
ing metal stent is removable. We have seen success with this approach in sealing a refractory bile
leak (Figure 23-3).

Bibliography
Leung J. Fundamentals of ERCP. In: Cotton PB, Leung JW, eds. Advanced Digestive Endoscopy: ERCP. Hoboken, NJ:
Wiley-Blackwell; 2008:17-80.
24 QUESTION

A 65-YEAR-OLD MALE ON PLAVIX FOLLOWING A RECENT MI AND


CORONARY STENTING WAS SEEN IN THE ER WITH FEVER, PAIN,
JAUNDICE, AND A WBC OF 18,000. US SHOWED A DILATED BILE DUCT
OF 15 MM AND A GALLBLADDER FULL OF STONES. MRCP REVEALED A
15-MM STONE IN THE DISTAL CBD. WHAT WOULD YOU DO?

Wei-Chih Liao, MD, PhD

This patient presented with Charcot’s triad (right upper quadrant pain, fever, and jaundice),
which is the classic presentation of acute cholangitis. Acute bacterial cholangitis is caused by
obstruction of the common bile duct (CBD) and subsequent bacterial infection, and CBD stone
is the most common cause.1
For a patient with acute cholangitis, you need to obtain blood culture and start antibiotics as
soon as possible. You should also consider giving other supportive measures as needed (eg, oxygen,
intravenous fluid, or even inotropic agents if the patient is in profound septic shock) depending on
the condition of the patient. The antibiotic should be effective against the most common causative
pathogens, such as Escherichia coli, Klebsiella, Pseudomonas, and anaerobes. However, you must
remember that the key to successful treatment is to relieve bile duct obstruction and drain the
infected bile; cholangitis usually subsides promptly with adequate drainage of bile duct.
In mild cases, the infection usually improves after antibiotic treatment, and the CBD stone can
be extracted electively by endoscopic retrograde cholangiopancreatography (ERCP). However, if
the patient presents with hypotension and altered mental status (these together with Charcot’s
triad constitute the Reynolds’ pentad) or does not improve after 6 to 12 hours of medical treat-
ment, he or she may have acute suppurative cholangitis, and the bile duct should be drained imme-
diately, preferably by ERCP. Acute suppurative cholangitis is the most severe form of cholangitis
and is a medical emergency. In this condition, complete ductal obstruction raises pressure inside
the CBD, which prevents penetration of the antibiotic into the bile and facilitates dissemination of
bacteria into the blood stream. Without immediate drainage, acute suppurative cholangitis carries
a high risk of mortality.

Leung J, Lo SK, eds. Curbside Consultation in Endoscopy:


113 49 Clinical Questions, Second Edition (pp 113-116)
© 2014 SLACK Incorporated
114  Question 24

Figure 24-1. Endoscopic sphincterotomy uses a


cutting wire connected to a diathermy unit to cut
the sphincter of Oddi and open the bile duct at the
ampulla.

ERCP is the preferred procedure to extract CBD stones or drain the bile duct. To extract CBD
stones, a sphincterotomy is performed during ERCP, which cuts the sphincter of Oddi to enlarge
the bile duct opening at the duodenum (Figure 24-1) for stone extraction. If the patient is critically
ill and cannot withstand a long procedure, a plastic biliary stent (a plastic tube with one end in the
duodenum and the other in the bile duct across the obstruction site) can be quickly placed to drain
the bile duct without attempting stone extraction in the initial ERCP session. The stone can be
extracted by ERCP electively, after cholangitis subsides and the patient is stabilized.
Although sphincterotomy is the current standard treatment for CBD stones, it carries a 2% risk
of bleeding.2 Plavix (clopidogrel), an antiplatelet agent, may increase the risk of bleeding, and cur-
rent guidelines suggest withholding the drug for at least 5 days before sphincterotomy. 3 However,
stopping Plavix in a patient with a recent heart attack and a newly placed coronary stent carries
the risk of stent occlusion and recurrent acute coronary event. Such a dilemma is not uncommon
today with the increasing use of coronary stents and antithrombotic agents to prevent stent block-
age. In patients with bleeding diathesis from causes such as thrombocytopenia, coagulopathy, and
antithrombotics, endoscopic papillary balloon dilation (EPBD) should be used instead of sphinc-
terotomy to open up the bile duct orifice for stone extraction.2,3
EPBD uses a balloon 10 mm or less in diameter to dilate the sphincter of Oddi (Figure 24-2).
EPBD has a negligible risk of bleeding and can be safely performed in this patient without
withholding Plavix.3 Meta-analyses comparing these 2 treatments for extraction of CBD stones
have demonstrated that sphincterotomy and EPBD have comparable success rates of stone
extraction and overall complications, with EPBD less likely to cause bleeding but more likely to
cause pancreatitis than sphincterotomy2 (Table 24-1).4 Although this suggests that using EPBD
to extract CBD stones avoids bleeding at the cost of a higher pancreatitis risk, recent evidence
indicates that the risk of pancreatitis with EPBD can be significantly reduced if adequate dila-
tion duration, preferably 5 minutes, is used in performing EPBD.4,5 A previous study has shown
that compared with dilating for 5 minutes, dilating for 1 minute achieves less effective sphincter
dilation and results in more difficulty in extracting the stone and higher risk of pancreatitis after
EPBD.5
Therefore, to perform EPBD safely and effectively, you first place a guidewire into the
CBD and then insert the dilating balloon over the wire across the ampulla. Next, the balloon
is slowly inflated with diluted contrast under fluoroscopic monitoring, which enables you to see
the waist of the balloon gradually disappear with balloon inflation, indicating successful dilation
of the sphincter. Inflate the balloon until its waist disappears and the pressure reaches the level
What Would You Do for a 15-mm Stone in the Distal CBD?   115

Figure 24-2. Endoscopic papillary balloon dilation


uses a balloon to dilate the sphincter of Oddi and
open the bile duct at the ampulla.

Table 24-1

Comparison of Sphincterotomy and Endoscopic Papillary


Balloon Dilation for Extracting Bile Duct Stones
Sphincterotomy Endoscopic Papillary
Balloon Dilation
Success of stone 96.5 94.3
extraction (%)
Complications (%)
Overall 10.3 10.5
Bleeding 2 0
Pancreatitis 4.3 7.4
Aspirin Can be continued Can be continued
Plavix (clopidogrel) Withhold ≥ 5 days before Can be continued
procedure
Effient (prasugrel) Withhold ≥ 7 days before Can be continued
procedure

Adapted from Baron TH, Harewood GC. Endoscopic balloon dilation of the biliary sphincter com-
pared to endoscopic biliary sphincterotomy for removal of common bile duct stones during ERCP: a
metaanalysis of randomized, controlled trials. Am J Gastroenterol. 2004;99:1455-1460 and Boustiere C,
Veitch A, Vanbiervliet G, et al. Endoscopy and antiplatelet agents. European Society of Gastrointestinal
Endoscopy (ESGE) Guideline. Endoscopy. 2011;43:445-461.
116  Question 24

recommended by the manufacturer, then keep the balloon for 5 minutes before you deflate it and
proceed with stone extraction.
The best course for this patient is to use EPBD rather than sphincterotomy to avoid both the
risk of bleeding from Plavix and the risk of stent occlusion from stopping Plavix. The risk of
pancreatitis with EPBD can be reduced by using adequate dilation duration for EPBD as previ-
ously mentioned, or you can place a pancreatic stent during ERCP. If the stone is soft, extracting
a 15-mm stone should be easy after EPBD. If the 15-mm stone is hard and cannot be pulled out
from the opening created by EPBD with a 10-mm dilating balloon, you can use a mechanical
lithotripter to break up the stone and then extract all the stone fragments. You may wonder why
we don’t use a 15-mm diameter dilating balloon to make the duct opening bigger and stone extrac-
tion easier (the so-called large-balloon papillary dilation). Using a big balloon is associated with
increased risk of bleeding during and after ERCP. Therefore, although using a bigger balloon for
papillary dilation may obviate the need for a mechanical lithotripter and save time, it should not
be used for similar reasons as sphincterotomy in this particular patient.
Finally, after the stone is extracted and the patient recovers, remember to tell him or her that
the stones in the gallbladder may fall out and drop into bile duct again, causing another similar
episode in the future. A discussion about treating the gallbladder stones by elective cholecystec-
tomy and education about the symptoms of recurrent cholangitis and other related complications,
such as cholecystitis and pancreatitis, should be very useful for the patient.

References
1. Wang D Q-H, Afdhal NH. Gallstone disease. In: Feldman M, Friedman LS, Brandt LJ, eds. Sleisenger & Fordtran’s
Gastrointestinal and Liver Disease. 9th ed. Philadelphia, PA: W.B. Saunders; 2010:1089-1120.
2. Baron TH, Harewood GC. Endoscopic balloon dilation of the biliary sphincter compared to endoscopic biliary
sphincterotomy for removal of common bile duct stones during ERCP: a metaanalysis of randomized, controlled
trials. Am J Gastroenterol. 2004;99:1455-1460.
3. Boustiere C, Veitch A, Vanbiervliet G, et al. Endoscopy and antiplatelet agents. European Society of Gastrointestinal
Endoscopy (ESGE) Guideline. Endoscopy. 2011;43:445-461.
4. Liao WC, Tu YK, Wu MS, et al. Balloon dilation with adequate duration is safer than sphincterotomy for extracting
bile duct stones: a systematic review and meta-analyses. Clin Gastroenterol Hepatol. 2012;10:1101-1109.
5. Liao WC, Lee CT, Chang CY, et al. Randomized trial of 1-minute versus 5-minute endoscopic balloon dilation for
extraction of bile duct stones. Gastrointest Endosc. 2010;72:1154-1162.
25 QUESTION

WHAT IS THE BEST ENDOSCOPIC METHOD TO DIAGNOSE


PANCREATIC CANCER? WHAT IS THE BEST WAY TO SAMPLE
TISSUE TO DIAGNOSE SUSPECTED BILE DUCT CANCER?

John G. Lee, MD

In the United States, the diagnosis of pancreatic cancer requires histological or cytological
confirmation because most physicians and patients will accept only a definitive diagnosis, whereas
in Japan, for example, most cancers are diagnosed on clinical and radiological findings alone. In
addition, a definitive diagnosis is required to initiate treatment because oncologists will not treat
based on a clinical diagnosis (sometimes even in patients with radiographical evidence of metas-
tasis) or on elevated CA 19-9 level, however compelling it may be. Many surgeons also request a
definitive diagnosis before operating because pancreatic resection has significant morbidity, and a
smaller, but very real, mortality rate associated with it, even in the very “best” treatment centers.
We now know that chronic pancreatitis and autoimmune pancreatitis can present virtually identi-
cal to pancreatic cancer, yet they require very different therapies. Therefore, endoscopic diagnosis
of pancreatic cancer requires either biopsy or cytology.
Endoscopic forceps biopsy is possible when pancreatic cancer invades the bowel; however,
apparent duodenal mucosal invasion often just represents inflammatory reaction, decreasing its
positive predictive value well below 100%. Endoscopic biopsy can also be done via the bile duct in
patients with biliary obstruction (Figure 25-1) and/or via the pancreatic duct, especially in patients
with intraductal papillary mucinous neoplasms (IPMN) who have a grossly dilated pancreatic duct.
These methods are useless in patients with small tumors (ie, the patient with the highest chance
of being cured). My feeling is that endoscopic intraductal biopsies are not practical or feasible in
routine clinical practice because of the technical difficulty and low tissue yield. Directed biopsies
using a pancreatoscope should increase the yield but are even more cumbersome, time consuming,
and not widely available. Finally, recent developments allow pancreatoscopy and biopsy through a
19-g endoscopic ultrasound (EUS) and fine needle aspiration (FNA) needle. Thus, the best and
only endoscopic method for obtaining histological diagnosis of pancreatic cancer in most cases is to

Leung J, Lo SK, eds. Curbside Consultation in Endoscopy:


117 49 Clinical Questions, Second Edition (pp 117-120)
© 2014 SLACK Incorporated
118  Question 25

Figure 25-1. “Blind” endoscopic biopsy of


a cholangiocarcinoma using regular biopsy
forceps.

perform a forceps biopsy of the tumor invading into the gastric or duodenal wall. The prevalence
of such invasion is unknown, but it is fairly uncommon in my clinical experience.
Brush cytology from biliary and/or pancreatic stricture can yield a positive cytologic diagno-
sis in approximately one-third of cases.1 The yield in routine clinical practice is probably lower
because results reported in published studies from experienced treatment centers represent the
very best case scenarios. Still, this is the only method available in many units and thus by default
represents the best and only method of diagnosing pancreatic cancer endoscopically.
The most accurate endoscopic method to diagnose pancreatic cancer is by EUS FNA.2 EUS
can easily detect lesions in the 3- to 4-mm range, which is far better than computed tomography,
magnetic resonance imaging, or any other imaging modality, and it is more accurate than surgical
exploration. EUS FNA is easily possible in such small lesions and can be used to obtain a defini-
tive cytologic diagnosis with sensitivity > 90%. In my experience, about half of the patients with
large tumors in EUS present with either a negative or equivocal CT scan. A normal EUS has
an essentially perfect negative predictive value, meaning that a normal EUS rules out pancreatic
cancer.3 EUS is the only method that can be used to detect and mark very small lesions (usually
< 1 cm).4 My approach is to tattoo this small lesion at the time of EUS to facilitate laparoscopic
wedge or partial resection (Figure 25-2). EUS FNA is also far safer than endoscopic retrograde
cholangiopancreatography (ERCP), with a complication rate of around 0.1% to 0.5%, and can also
be used to accurately stage the disease. On the other hand, EUS FNA equipment is not always
available, and technical expertise is even more limited in the United States.
Should you do ERCP and/or EUS? I think the most logical approach is to start with EUS FNA
(the only endoscopic method that can diagnose a tumor without invasion into the bile or pancreatic
duct), followed by ERCP and stenting only in patients with obstructive jaundice.
The best endoscopic method to diagnose pancreatic cancer is EUS FNA. In addition, the
very high negative predictive value of a normal EUS makes it the only modality that I would feel
comfortable using to exclude the possibility of cancer. If unavailable, the patient with suspected
pancreatic cancer should be referred to a center with EUS FNA. If all else fails, I recommend
ERCP with brush cytology and biopsy.
What Is the Best Way to Diagnose Suspected Bile Duct and Pancreatic Cancers?   119

Figure 25-2. (A) EUS shows a 7-mm neuroendo-


crine tumor in the pancreas. (B) EUS-guided tattoo
of the tumor using India ink is performed to facili-
tate intraoperative visualization. Note the hyper-
echoic blush from the injection. (C) Laparoscopic
visualization of the neuroendocrine tumor with
India ink tattoo.

Extrahepatic bile duct cancer with mass (usually around the bile duct but occasionally intrabili-
ary as well) is best diagnosed by EUS FNA, as with pancreatic cancer. EUS poorly visualizes the
bile duct proximal to the hilum, and detecting a mass is dependent on individual anatomy, size,
and location of the lesion; in general, it is less helpful. EUS shows most of the left lobe of the liver,
but only a limited portion of the right liver. A previously placed stent may be helpful for localizing
the site of the biliary mass. Biopsying the point of bile duct dilation can be helpful if a clear mass
is not seen. The yield of EUS FNA is quite low in patients with bile duct wall thickening only.
Therefore, ERCP, brush cytology, and/or biopsy are the only methods for diagnosing bile duct
cancer in most patients, cancer located proximal to the hilum, or cancer without a discrete mass.
The sensitivity of brush cytology is low at around 30% to 40% and can be improved by adding a
120  Question 25

Figure 25-3. Pancreatoscopic visualization of


an adenocarcinoma arising in the setting of
IPMN.

second modality such as biopsy.1 I think it is also reasonable to do multiple brushings (as in EUS
FNA) during a single procedure to increase the diagnostic yield. Biliary biopsy can be performed
under fluoroscopic guidance or under cholangioscopic guidance (Figure 25-3). Fluoroscopic-
guided biopsy can be performed after sphincterotomy using conventional biopsy forceps, but is
limited by the inability to precisely target the biopsy sites with biopsies being done just distal to the
stricture. Cholangioscopic biopsy can target the specific lesion (at least in theory), but all commer-
cially available systems have limitations, especially in the intrahepatic ducts and in the distal duct.
My approach to diagnosing a suspected bile duct is to start with EUS FNA to evaluate the
extrahepatic system and eliminate pancreatic or ampullary cancer. If EUS FNA is negative for
a mass, but shows intrahepatic biliary dilation in the setting of a normal caliber common duct,
then the lesion must be located in the hilum or proximal to it. I then perform ERCP with brush
cytology and biopsy of the biliary stricture. If everything is negative in a patient with high clinical
suspicion, I perform cholangioscopy with directed biopsies. Unfortunately, the results may be neg-
ative, even after repeated biopsies. In such cases, patients with high clinical suspicion and poten-
tially resectable disease should undergo an attempt at a surgical cure and the rest should undergo
close surveillance. I feel confident that a negative surveillance at 3 months almost always excludes
the possibility of pancreatic cancer, but much longer follow-up is required in bile duct cancer, and
I generally recommend repeat EUS and possible ERCP every 3 months for 9 to 12 months before
excluding cancer as a diagnosis.

References
1. Lee JG. Brush cytology and the diagnosis of pancreaticobiliary malignancy during ERCP. Gastrointest Endosc.
2006;63(1):78-80.
2. Chang KJ. State of the art lecture: endoscopic ultrasound (EUS) and FNA in pancreatico-biliary tumors. Endoscopy.
2006;38(Suppl 1):S56-S60.
3. Klapman JB, Chang KJ, Lee JG, Nguyen P. Negative predictive value of endoscopic ultrasound in a large series of
patients with a clinical suspicion of pancreatic cancer. Am J Gastroenterol. 2005;100(12):2658-2661.
4. Root J, Nguyen N, Jones B, et al. Laparoscopic distal pancreatic resection. Am Surg. 2005;71(9):744-749.
26 QUESTION

A 68-YEAR-OLD MAN WITH A KLATSKIN S TUMOR WAS REFERRED FOR


CONSIDERATION OF ERCP AND PALLIATIVE DRAINAGE.
HOW SHOULD I MANAGE THE PATIENT?

Joseph Leung, MD, FRCP, FACG, FASGE, MACG

Patients with malignant hilar obstruction present with obstructive jaundice, which could be
secondary to an underlying cholangiocarinoma (typically a Klatskin’s tumor), or metastatic hilar
lymphadenopathy. The anatomical location and extent of involvement of the intrahepatic ducts
associated with hilar obstruction is important to determine the resectability for cure or stenting
for palliative care. Surgical resection is possible if a major portion of the liver is not involved or if
the liver function is preserved without evidence of vascular involvement.
The extent of ductal involvement is based on the Bismuth-Colette classification, depending on
the level of obstruction. Type I is cancer involving the common hepatic duct within 2 cm of the
bifurcation, but with communication between the main right and left hepatic ducts. Type II is
obstruction at the hilum with involvement of the right and left hepatic ducts, but without involve-
ment of the tertiary ducts. Type III is involvement of tertiary branch ducts limited to the right
(IIIA) or left (IIIB) duct system. Type IV is involvement of tertiary branch ducts bilaterally. Types
I and II are potentially resectable, but Type III and IV lesions are usually unresectable.
Workup of these patients will include imaging to define the level and extent of involvement.
Because of the risk of contamination and stasis with infection, contrast studies, including percu-
taneous transhepatic cholangiogram or endoscopic retrograde cholangiogram (ERCP) are consid-
ered only if there is a plan for drainage. Magnetic resonance cholangiopancreatography (MRCP)
is the best noninvasive diagnostic imaging for initial assessment of the extent of ductal obstruction
and underlying pathology. In patients with multiple segments involvement, contrast injection may
outline the obstructed system, but it carries the risk of sepsis because drainage may not be achieved
or adequate, leading to stasis and infection that can be difficult to control. To minimize the risk
of contamination, a preprocedural MRCP will help define the level of obstruction and the degree
of involvement of the respective ductal system. Indeed, preselection of the intrahepatic system
for drainage and selective cannulation using a guidewire without injection of contrast is feasible,
keeping the risk of infection to a minimum.
Leung J, Lo SK, eds. Curbside Consultation in Endoscopy:
121 49 Clinical Questions, Second Edition (pp 121-124)
© 2014 SLACK Incorporated
122  Question 26

Endoscopic Therapy
ERCP is the main endoscopic procedure for the palliative drainage of patients with hilar
obstruction. As discussed previously, a diagnostic cholangiogram can be attempted, which often
outlines the bile duct below the level of obstruction. Contrast injection is limited to defining the
stricture, including the obstructed right and left hepatic ducts. Excess contrast should be avoided
and a full cholangiogram should not be performed because of the risk of contamination and infec-
tion in the undrained obstructed system. If contrast filled out a limited segment where drainage
is inadequate to improve the liver function, an attempt should be made to cross the stricture and
aspirate bile and contrast to decompress that segment to reduce the risk of infection.
Prior studies have suggested that the recovery of liver function is proportional to the amount of
liver volume/tissue that is successfully drained. Patients’ survival is improved if > 50% of the liver
is drained.1 Ideally, more than one intrahepatic segment should be drained to achieve maximal
benefits, and we need to consider placement of one or more stents for bilateral biliary drainage,
using either plastic or self-expandable metal stents (SEMS).

Single vs Double Stents


Single stent placement is good enough for patients with Type I obstruction because there is
still communication between the right and left hepatic system. Plastic stents should be used if the
patient's tumor is deemed resectable to avoid problems related to the SEMS at the time of surgery.
Placement of a single stent is easy and is similar to conventional stenting after passage of a guide-
wire into part of the intrahepatic system. Because the right hepatic duct branches off after 1 cm
(whereas the left hepatic duct branches off after 2 cm), it would be beneficial to consider selective
cannulation of the left hepatic duct in hopes of prolonging drainage of 2 or more segments of
the liver in the left lobe, even when growth and extension of the tumor involves the bifurcation.
Selective cannulation can be achieved with shaping of the guidewire for easier deflection into the
left hepatic system. Subsequent balloon dilation and brush cytology can be performed. A plastic
or metal stent can be placed across the stricture. A long plastic stent should be used, especially
for the left hepatic system. The usual straight stent is good for the right hepatic system, but a
plastic stent may kink if it is placed in the left hepatic system and may require some preshaping
or modification before it is inserted into the left hepatic duct. A SEMS can be inserted across the
obstruction, but because of the length of the available stents, the stent is usually placed entirely
inside the biliary system.

Double or Bilateral Stents


In patients with Type II or more extensive hilar involvement, it is beneficial to consider drain-
age for both the right and left hepatic systems. This will require the initial placement of 2 guide-
wires, one into the right and left hepatic system, respectively. The guidewires can be anchored
using wire locks or hemostats at the biopsy valve level. Most hilar strictures are tight and will
require balloon dilation (using a 6-mm dilation balloon with contrast to demonstrate obliteration
of the waist formation on the balloon at the stricture level) to facilitate the passage of the stents
across the stricture. This is followed by brush cytology to help confirm the pathological diagnosis
before stent deployment.
How Should I Manage a Patient With a Klatskin s Tumor?   123

Bilateral Plastic Stents


Either plastic or metal stents can be used for drainage. For bilateral stenting, placement of plas-
tic stents is done in a parallel configuration. In most cases, it is preferable to consider stenting the
left side first (using a modified stent), which is more difficult because of the anatomy and axis. The
first plastic stent is placed with the distal tip a little further out in the duodenum. This is followed
by insertion of the right stent, which is technically easier. The stents should be well-lubricated to
reduce friction, and there is a tendency for the second stent to drag the first stent further up the
bile duct during insertion.

Bilateral Self-Expandable Metal Stents


Bilateral SEMS can be placed to drain the right and left hepatic system. Balloon dilation is
necessary to open the stricture, which will facilitate subsequent expansion of the stents. Because of
the design of the SEMS, early stents are usually placed one after another in a parallel configura-
tion, until more recently, when special stents with a wide cell size became available so that stents
can be placed in a Y configuration.
Parallel stents placement is technically difficult if stents are inserted one after another. Because
of the expansion of the distal end of the stent inside the bile duct, advancement of the second
applicator system can be difficult, even with the initial placement of 2 guidewires across the
stricture. Improvement in stent design with the introduction of the 6-Fr delivery system allows
2 smaller stents (Zilver 635, Cook Endoscopy) to be placed simultaneously through a large channel
(4.2 mm) duodenoscope over 2 previously inserted guidewires into the right and left intrahepatic
systems.2 Under fluoroscopic control, both stents can be deployed simultaneously across both
intrahepatic strictures. Considerable lubrication is required to minimize friction, and the deploy-
ment is performed slowly while monitoring for the gradual expansion of the stents under fluoros-
copy. This new stenting system allows for drainage of both the right and left hepatic systems in a
parallel stent fashion.
Recent development of the stent-in-stent, or Y stent system, using stents with a large cell
diameter in the mid-portion allows for the placement of a single stent initially into the left hepatic
system. A second guidewire is used to negotiate the hilar stricture through the large mesh open-
ing of the first stent into the right hepatic system. Over the guidewire, a second stent is inserted
through the mesh of the first stent into the right hepatic system so that the stents overlap at the
distal ends, or as a Y configuration.3
Retrospective and randomized controled studies showed that both the Y configuration and the
smaller 6-Fr stenting system increased the success of bilateral stenting for hilar obstruction. Single
stent placement had a lower incidence of infection.4 In a randomized controlled trial, single SEMS
offered a significantly better clinical outcome with longer survival compared with those drained
with plastic stents.5
In general, plastic stents become blocked more quickly, often as a result of infection and sludge
formation, and will require more frequent stent exchange, whereas the SEMS with a larger lumen
tend to remain patent for a longer period. Stent blockage can be treated with stent exchange by
removal of the blocked stent and replacement with a new stent to provide drainage. The SEMS
become blocked as a result of tissue ingrowth or overgrowth, and the treatment is insertion of a
plastic stent through the obstructed stents to provide drainage.
124  Question 26

Conclusion
Plastic or SEMS stents can be used for the palliative drainage of hilar obstruction. Bilateral
stent placement offers a better chance to decompress the liver and liver function tests can improve
significantly if > 50% of the liver volume is drained. The newly developed SEMS with large cell
diameter allow bilateral stent placement in a Y configuration, whereas the small 6-Fr system
allows simultaneous deployment of 2 stents across the right and left hepatic system in a parallel
fashion.

References
1. Vienne A, Hobeika E, Gouya H, et al. Prediction of drainage effectiveness during endoscopic stenting of malig-
nant hilar strictures: the role of liver volume assessment. Gastrointest Endosc. 2010;72:728-735.
2. Chennat J, Waxman I. Initial performance profile of a new 6 Fr self-expandable metal stent for palliation of malig-
nant hilar biliary obstruction. Gastrointest Endosc. 2010;72:632-636.
3. Hwang JC, Kim JH, Lim SG, et al. Y-shaped endoscopic bilateral metal stent placement for malignant hilar biliary
obstruction: prospective long-term study. Scand J Gastroenterol. 2011;46:326-332.
4. Iwano H, Ryozawa S, Ishigaki N, et al. Unilateral versus bilateral drainage using self-expandable metallic stent for
unresectable hilar biliary obstruction. Dig Endosc. 2011;23:43-48.
5. Sangchan A, Kongkasame W, Pugkhem A, et al. Efficacy of metal and plastic stents in unresectable complex hilar
cholangiocarcinoma: a randomized controlled trial. Gastrointest Endosc. 2012;76:93-99.
27 QUESTION

AN ELDERLY PATIENT WITH MULTIPLE MEDICAL PROBLEMS PRESENTS


WITH ACUTE PANCREATITIS AND STONES IN THE GALLBLADDER. LFTS
SHOW AST OF 90 AND ALT OF 95, BOTH DECLINING. MRCP SHOWS
NO CBD STONES OR DUCTAL DILATION. IS ERCP INDICATED?

Stuart Sherman, MD

This patient has multiple medical problems and clearly has pancreatitis. In deciding whether
an endoscopic retrograde cholangiopancreatography (ERCP) is indicated, I would want to know
whether the pancreatitis is due to bile duct stones. Distinguishing biliary pancreatitis from other
causes may be difficult, and often requires an extensive biochemical and radiologic evaluation.
Finding gallbladder stones, as in this case, is suggestive but not conclusive of a biliary origin. A
serum amylase level greater than 1000 IU/L should suggest a biliary tract origin, although some
overlap exists with other causes. A meta-analysis has suggested that a 3-fold or greater eleva-
tion in the alanine transferase (ALT) in the presence of acute pancreatitis has a predictive value
of 95% in diagnosing gallstone pancreatitis. Certainly, if one finds common duct stones on a
radiologic imaging study, the diagnosis of stone-induced pancreatitis is near certain and ERCP is
indicated (unless a combined laparoscopic cholecystectomy and common duct exploration is pos-
sible). Magnetic resonance cholangiopancreatography (MRCP) has a sensitivity and specificity of
detecting bile duct stones in the range of 90% to 100% and 92% to 100%, respectively. However,
the sensitivity and specificity are reduced for small stones, which more commonly cause pancre-
atitis. Endoscopic ultrasound (EUS), when performed by experts, has comparable if not better
accuracy in detecting bile duct stones than ERCP. When deciding whether to perform ERCP,
single test results are less important than the constellation of findings that make up the overall
clinical presentation. Remember, ERCP, as practiced currently, is used principally as a therapeutic
modality to treat an anatomic obstruction as would be the case for bile duct stones. Based on the
information provided, I can speculate that the patient likely has stone-induced pancreatitis, but
not certainly. Moreover, I have no clear evidence that there are stones in the bile duct by radiologic
imaging tests and, as a result, no absolute reason to do an ERCP. Finally, improving liver chemis-
tries suggest that if indeed a stone was present in the bile duct, it likely has passed.

Leung J, Lo SK, eds. Curbside Consultation in Endoscopy:


125 49 Clinical Questions, Second Edition (pp 125-126)
© 2014 SLACK Incorporated
126  Question 27

After I make a diagnosis of biliary pancreatitis, it is critical that I grade its severity and deter-
mine whether there is evidence of biliary tract obstruction or cholangitis. Addressing these issues
will help to determine the necessity of ERCP. When cholangitis or jaundice complicates pancre-
atitis, the likelihood of finding an obstructing stone is increased. There are now 8 prospective
randomized trials (7 published as full papers and 1 as an abstract) that have attempted to define
the role of early ERCP in gallstone pancreatitis. These studies differ to some degree with regards
to patient entry criteria, timing of the intervention, and grading of the severity of pancreatitis.
However, the consensus of 6 meta-analyses is that performing early ERCP in the absence of coex-
isting cholangitis does not lead to a reduction in mortality and local or systemic complications in
predicted mild or severe disease. Specifically, the data do not support performing ERCP in biliary
pancreatitis unless there is evidence of biliary obstruction or cholangitis. In contrast, at the State-
of-the Science Conference on ERCP held in 2002, the conclusion drawn was that ERCP had a
distinct role in the patient with severe biliary pancreatitis.
In the case presented, it appears that the patient has mild gallstone pancreatitis, so I would
advise against ERCP and recommend early laparoscopic cholecystectomy with intraoperative
cholangiogram. However, there appear to be extenuating circumstances in that the patient has
multiple medical problems and may be a poor surgical candidate. In such a patient, an ERCP
should be considered because performing an empiric biliary sphincterotomy (even in the absence
of finding bile duct stones) will prevent recurrent pancreatitis. If this approach is taken, what is
the risk of leaving the gallbladder in situ? Older data suggested that the lifetime risk of develop-
ing a gallbladder complication (eg, acute cholecystitis) was approximately 10% to 20% in elderly
patients. However, in a more recent randomized trial that compared laparoscopic cholecystectomy
within 6 weeks of ERCP, sphincterotomy, and stone removal to a wait-and-see approach in surgi-
cally fit patients with known gallbladder stones, 47% of patients in the wait-and-see group devel-
oped biliary-related events, compared with 2% in the cholecystectomy group. Thus, if the patient
is a surgical candidate, ERCP should be avoided in favor of the cholecystectomy.
Assuming the magnetic resonance imaging/MRCP shows no other pathology to explain the
pancreatitis and the history and other laboratory tests are supportive of a gallstone origin, I would
diagnose gallstone pancreatitis. An ERCP should not be done because the patient has a mild epi-
sode of pancreatitis, there is no evidence of cholangitis or biliary obstruction, and the improving
liver tests suggest that the stone has passed. Assuming our patient is a surgical candidate, I would
recommend laparoscopic cholecystectomy with intraoperative cholangiogram, preferably during
the current hospitalization. If the patient is a poor surgical candidate, then ERCP with empiric
biliary sphincterotomy should be considered, with the caveat that late gallbladder complications
may occur.

Bibliography
Boerma D, Rauws EA, Keulemans YC, et al. Wait-and-see policy of laparoscopic cholecystectomy after endoscopic
sphincterotomy for bile duct stones: a randomized trial. Lancet. 2002;360:761-765.
Fogel EL, Sherman S. Acute biliary pancreatitis: when should the endoscopist intervene? Gastroenterology.
2003;125:229-235.
Kozarek R. Role of ERCP in acute pancreatitis. Gastrointest Endosc. 2002;56:S231-S236.
Petrov MS, van Santvoort HC, Besselink MGH, et al. Early endoscopic retrograde cholangiography versus conserva-
tive management in acute biliary pancreatitis without cholangitis: a meta-analysis of randomized trials. Ann Surg.
2008;247:250-257.
28 QUESTION

I HAVE A PATIENT WITH BILIARY-TYPE PAIN, BUT NO OTHER EVIDENCE


OF BILE DUCT DISEASE (NORMAL DIAMETER DUCT ON ULTRASOUND,
NORMAL LIVER FUNCTION TESTS).
HOW SHOULD SUCH A PATIENT BE MANAGED?

Martin L. Freeman, MD, FACG, FASGE

Right upper quadrant (RUQ ) pain syndrome is most common in women, particularly those who
are young to middle-aged. The first question we should ask is whether the gallbladder is intact.
Our first efforts should be to thoroughly investigate whether there is any evidence of gallbladder
disease. Computed tomography is generally performed to look for structural abnormalities, but it
is generally negative. A high-quality RUQ ultrasound should be done. If there is any evidence of
gallbladder sludge or stone disease, it is reasonable to offer laparoscopic cholecystectomy. If nega-
tive, then a cholecystokinin (CCK)-stimulated hydroxyindole diaminoacetic acid (HIDA) nuclear
medicine scan can be performed to look for a diminished ejection fraction, indicative of “gallblad-
der dyskinesia.” If abnormal, then cholecystectomy can be recommended. Although I also recom-
mend this approach, I counsel patients that there is an approximately 50% chance or less of clinical
improvement, and very limited supporting data. If the CCK HIDA scan is negative, I recommend
endoscopic ultrasound, which may find subtle evidence of gallbladder sludge, chronic pancreatitis,
ductal abnormalities, or occasionally some other surprise—I once found pericardial effusions sug-
gestive of pericarditis that were not suspected by any other test. In very rare circumstances, I will
recommend consideration of empirical cholecystectomy in the absence of any gallbladder disease,
even by all of the previously mentioned tests. I do not recommend endoscopic retrograde cholan-
giopancreatography (ERCP) or sphincter of Oddi manometry (SOM) for unexplained RUQ pain
in patients with intact gallbladders.
The next, and more difficult, question is what to do if there is persistent, recurrent RUQ pain
after the gallbladder has been removed. Then, the differential narrows down to very rare structural
disorders such as bile duct stones (which are almost never found), motility disorders such as gas-
troparesis, “visceral hyperalgesia,” peritoneal adhesions, minimal change chronic pancreatitis, or
sphincter of Oddi dysfunction (SOD). Where to go next is highly dependent on clinical suspicion

Leung J, Lo SK, eds. Curbside Consultation in Endoscopy:


127 49 Clinical Questions, Second Edition (pp 127-131)
© 2014 SLACK Incorporated
128  Question 28

Figure 28-1. SOM showing elevated


basal pressure (> 40 mm).

and, frankly, is mostly driven by the beliefs of the physician seeing the patient. First and foremost
is to take a careful history. Is the pain consistent and reproducible? Is there any precipitating factor,
such as eating fatty foods? Does movement exacerbate the pain? If so, it may be suggestive of a
musculoskeletal rather than a visceral disorder. Is the pain an isolated symptom or is there nausea,
vomiting, or postprandial bloating, which may suggest gastroparesis? Many patients have multiple
other pain syndromes, such as migraine headaches and fibromyalgia. It is also important to docu-
ment liver and pancreatic chemistries during or after pain attacks on at least 3 separate occasions,
if possible. Abnormal results may only occur sporadically after pain attacks, which may point to a
biliary or pancreatic cause. Very occasionally, laparoscopy can be considered to look for adhesions,
but this almost never yields results. Often, empirical trials of antispasmodics or antidepressants
are given, but these seldom yield a satisfactory response.
If I am still suspecting a structural or functional cause, I generally proceed with endoscopic
ultrasound (EUS) to look for subtle evidence of biliary or pancreatic disease. A number of these
patients will have equivocal or suggestive evidence of small-duct chronic pancreatitis, with
3 to 5 out of 9 possible criteria. The significance of such findings is debatable, but it may be the
only evidence of chronic pancreatitis. If EUS is done before any kind of intervention, the results
are interpretable and will not represent iatrogenic artifact of ERCP or pancreatic stents. I also
generally obtain secretin-enhanced MRCP to look for structural abnormalities such as pancreas
divisum or abnormal augmentation of the pancreatic duct with secretin stimulation. Very rarely,
MRCP will reveal biliary tract disease such as a choledochal cyst. It also will provide a road map,
particularly of the pancreatic duct, that will become critical if ERCP with SOM is contemplated.
Whether to proceed with ERCP with SOM is the most difficult and controversial decision;
it depends on the physician’s belief in SOD. It is widely believed that the likelihood of SOD and
chance of response are mostly determined by the presence or absence of abnormal liver chemis-
tries or dilated bile duct—the “Milwaukee” criteria dividing patients into Type I (dilated bile duct
and abnormal liver chemistries), Type II (either but not both), or Type III (neither present, such
as in the case discussed here), with the probability of existence and response to sphincterotomy
descending from high to none from Type I to Type III.1 This construct is based on the concept
that SOD is a biliary disease. We now know that if it exists, it involves both the pancreatic and
biliary sphincter2 (Figure 28-1).
How Should a Patient With Pain But No Evidence of Disease Be Managed?  129

Figure 28-2. Complete pancreatic and biliary


sphincterotomies at follow-up.

Patients may respond to pancreatic sphincterotomy, in addition to biliary sphincterotomy, where


biliary sphincterotomy alone generally fails to improve symptoms (Figure 28-2). Recent data from
our center suggest that, in fact, outcomes of dual biliary and pancreatic sphincterotomy have little
relation to the Milwaukee type but rather are dependent on clinical factors indicative of a more
global dysfunction, such as dependence on narcotic analgesics or the presence of gastroparesis.3 In
addition, findings at pancreatic manometry are predictive. I no longer believe that the Milwaukee
classification is particularly relevant to outcomes of suspected SOD. My views are different from
the generally held opinion,4 but I believe that further research will bear out the importance of the
“bigger picture” beyond simple indicators of biliary obstruction. Many major questions regard-
ing SOD will be answered by the just completed EPISOD study (ClinicalTrials.gov Identifier:
NCT00688662), which is a multicenter, National Institutes of Health (NIH)-funded, double-
blind randomized controlled trial. Two hundred fourteen patients with suspected SOD Type III
have been randomized to biliary sphincterotomy, biliary plus pancreatic sphincterotomy, or sham.
Results of this trial are eagerly awaited.
We do know that ERCP of any kind is highly risky in patients with suspected SOD, and that
post-ERCP pancreatitis can be greatly reduced by the placement of a temporary, small caliber pan-
creatic stent, which is now considered almost mandatory in such patients5 (Figures 28-3 and 28-4).
So, what would I do in a patient postcholecystectomy with intractable, recurrent RUQ pain
suspected to be of biliary or pancreatic origin with no objective evidence of biliary disease? After
EUS and secretin MRCP, assuming they are normal or show nonspecific changes, discuss the
risk/benefit of ERCP with possible dual biliary and pancreatic sphincterotomy depending on
manometric findings. I tell patients that the outcome depends on the other factors (eg, young age,
narcotic dependence, and/or gastroparesis) that lower the expected response dramatically; if none
of these negative predictors are present, and pancreatic manometry is found to be abnormal, then
the response rate is approximately 60%. I quote a risk of mild post-ERCP pancreatitis of approxi-
mately 15%, with a small but real chance (about 1/200 to 1/300) of severe or even life-threatening
complications. I tell patients that some major centers do not believe in this entity. I let the patient
decide whether to proceed.
At this point, approximately half of the patients I see wish to pursue ERCP. I do ERCP under
general anesthesia, for patient safety and comfort. I start with aspirated pancreatic sphincter
130  Question 28

Figure 28-3. Guidewire in pancreatic duct


after biliary sphincterotomy.

Figure 28-4. Pancreatic stent, draining pan-


creatic juice, placed to reduce risk of post-ERCP
pancreatitis.

manometry. If that is abnormal, I proceed with biliary and pancreatic sphincterotomy. If pancre-
atic manometry is normal, I perform biliary manometry. If agreed in advance, in selective cases,
I may perform biliary sphincterotomy without a manometry, but not pancreatic sphincterotomy.
Regardless of what type of maneuvers are performed, I place a small caliber (currently 4 Fr 2 cm,
single, inner flanged, soft material) pancreatic stent to reduce risk. I admit all of these patients
for inpatient observation, keep them NPO (given nothing by mouth), medicate pain and nausea,
which often occurs independent of the development of pancreatitis, and do not allow food until
amylase or lipase is checked and is less than 2 to 3 times normal or the patient feels fine. The
pancreatic stent is always documented to pass or is removed within 2 weeks.
Some patients with SOD clearly respond to endoscopic intervention, whereas others do not.
Our response rate for Type III SOD is not significantly different than the response rate for
How Should a Patient With Pain But No Evidence of Disease Be Managed?  131

Type I or II, which is approximately 60%.3 Whether any treatment is truly effective will be
answered only by a randomized controlled trial, which is currently being planned by Dr. Cotton
et al and funded by the NIH.
ERCP for abdominal pain syndromes in the absence of clear-cut biliary obstructive disease
should be performed only at major treatment centers with an extensive interest and high degree of
endoscopic expertise in SOM and pancreatic as well as biliary endotherapy, and particularly with
placement of small caliber pancreatic stents with near 100% success regardless of the pancreatic
ductal anatomy. Otherwise, the risk of ERCP clearly outweighs the benefit.

References
1. Sherman S. What is the role of ERCP in the setting of abdominal pain of pancreatic or biliary origin (suspected
sphincter of Oddi dysfunction)? Gastrointest Endosc. 2002;56:S258-S266.
2. Aymerich RR, Prakash C, Aliperti G. Sphincter of oddi manometry: is it necessary to measure both biliary and
pancreatic sphincter pressures? Gastrointest Endosc. 2000;52:183-186.
3. Freeman ML, Gill M, Overby C, Cen Y. Predictors of outcomes after biliary and pancreatic sphincterotomy for
sphincter of Oddi dysfunction. J Clin Gastroenterol. 2007;41:94-102.
4. Cohen S, Bacon BR, Berlin JA, et al. National Institutes of Health State-of-the-Science Conference Statement: ERCP
for diagnosis and therapy, January 14-16, 2002. Gastrointest Endosc. 2002;56:803-809.
5. Freeman ML, Guda NM. Prevention of post-ERCP pancreatitis: a comprehensive review. Gastrointest Endosc.
2004;59:845-864.
29 QUESTION

HOW CAN WE ACCURATELY DETERMINE THE


APPROPRIATE LENGTH OF THE STENT TO USE IN A
PATIENT WITH A BILE DUCT STRICTURE?

Joseph Leung, MD, FRCP, FACP, FASGE, MACG and Erina Foster, MD

The length of a plastic stent refers to the shaft of the stent that traverses a bile duct stricture and
is often indicated by the separation between the proximal and distal side flaps. The flaps are used
to resist stent migration. For pigtail stent design, the length is defined as the separation between
the pigtails. When stents were first invented, they were used mainly in patients with malignant
bile duct obstruction. Indications for biliary stenting have now expanded to include benign stric-
tures and large bile duct stones.
In the management of a bile duct stricture, it is generally accepted that the proximal flap be
placed approximately 1 cm above the upper level of the obstruction (to allow for possible tumor
extension), with the distal flap placed at the level of the papilla. In determining the appropriate
length of a biliary stent, it is important to define the upper level of the stricture and determine
whether there is involvement of the right and left hepatic ducts at the bifurcation.
Different methods are available for measuring the length of a stent. A conventional radiograph
has an average magnification factor of up to 30%, and the distance between the papilla and the
upper level of the stricture, as measured on the cholangiogram, should be corrected for magnifica-
tion, then add 1 cm to give the length of the stent.
An indirect measurement of the stent length can be made by comparing the estimated separa-
tion between the upper level of the bile duct obstruction and the papilla with the diameter of the
duodenoscope (Figure 29-1). The diameter of side-viewing duodenoscopes for endoscopic retro-
grade cholangiopancreatography (ERCP) vary between 11 and 13 mm. The separation between
the papilla and the upper level of the stricture can be estimated by the number of scope diameters
between the 2 points. With an adequate cholangiogram, locating the upper level of the stricture is
often easy, whereas locating the exact level of the papilla could be more difficult. The position of
the papilla can be determined by up-angulation of the scope tip against the papilla. No correction

Leung J, Lo SK, eds. Curbside Consultation in Endoscopy:


133 49 Clinical Questions, Second Edition (pp 133-135)
© 2014 SLACK Incorporated
134  Question 29

Figure 29-1. (A) The length of the stent is


denoted by the separation between the proxi-
mal and distal flaps. (B) The length can be
estimated with reference to the diameter of
the duodenoscope, either on fluoroscopy or
on a plain radiograph.

A
B

for magnification is necessary because the scope is in the same plane as the bile duct. It is, how-
ever, important to realize that the lower end of the column of contrast may not represent the true
papillary orifice because there is a variable length of the intraduodenal papilla and sphincter that
may not be filled by contrast.
Direct measurement of this separation can be obtained by using a guidewire and a catheter
with a radiopaque tip or papillotome. Deep cannulation is achieved, leaving either a catheter or
papillotome above the stricture. The guidewire is pulled back until the tip is located at the upper
level of the stricture, as seen on fluoroscopy. The guidewire is then pulled back until the tip is
seen emerging from the bile duct within the catheter lumen at the level of the papilla. The length
of guidewire pulled out is measured at the proximal end of the catheter or papillotome at the
injection port to give a direct measurement of the separation and, thus, the length of stent to be
used. An alternative is to pull back a catheter or papillotome over a deeply placed guidewire until
the fluoroscopic tip or marker is seen at the level of the papilla. Again, the distance traveled by
the catheter or papillotome is measured at the level of the biopsy valve. This will give an accurate
measurement of the separation between the upper level of the stricture and papilla, and thus the
length of stent to be used.
For those who use the traditional 3-layer system (guidewire, inner catheter, and pusher) for
stent placement, an estimate of the stent length can be made by using the inner catheter with
radiopaque ring markers (such as the Cook stenting system [Cook Endoscopy]) to do the measure-
ment. The inner catheter is positioned across the stricture, and the separation between the papilla
How Can We Determine the Appropriate Length of the Stent to Use?  135

can be estimated by referencing the separation of the markers. However, with the new stenting
systems (eg, OASIS [Cook Endoscopy]) in which the inner catheter and pusher are combined into
one, a proper length stent has to be chosen and loaded onto the delivery system prior to insertion.
This makes the use of the inner catheter with markers useless for stent measurement.
Another way of estimating the length of a stent is by using devices containing markers that
have a fixed separation and comparing that against the separation between the papilla and the
stricture. This is more difficult because the device has to be moved and there is no fixed point of
reference. Such devices include dilation balloons with ring markers (eg, the Titan balloons [Cook
Endoscopy] have ring markers placed 4 cm apart within the balloon or the flexible tip of a cytol-
ogy brush).

Special Considerations
If the stricture is located in the distal bile duct, a short stent may result in easy dislocation. In
general, straight stents have curved or angulated shapes to prevent stent migration. A short stent
placed in the distal bile duct is prone to migration because it may not conform to the shape of the
bile duct. We recommend the use of a slightly longer stent or sometimes shaping the stent so that
it conforms to the contour of the bile duct and provides better anchorage. As a general rule, we use
a 7- to 8-cm stent for distal bile duct strictures. For patients with mid common bile duct obstruc-
tion, a 10-cm stent is used and a longer stent is necessary if the stricture is close to the bifurcation.
For strictures located at the bifurcation, it is important to determine whether the right and left
hepatic system are obstructed separately. If so, we may need to consider stenting of both the right
and left hepatic ducts. For drainage of the right side, it is possible to place a longer stent (such as
a 12 cm), leaving the tip in one of the branch ducts, and allow bile to flow in a retrograde manner
into the anchoring segment and then down the stent.
For a left hepatic duct stricture, it is much more convenient to use the 15-cm stent, leaving a
good portion of the stent within the bile duct. It is not uncommon to see kinking or buckling of
a straight stent when it is deployed in the left hepatic duct. This is due to the configuration of the
stent, which usually conforms to the contour of the common hepatic and the right intrahepatic
system. We advocate the use of the “left hepatic duct” stent, which is z-shaped, allowing the
proximal part of the stent to stay within the left hepatic duct and the remaining shaft of the stent
conforming to the mid and distal portions of the bile duct.

Bibliography
Leung JW. Fundamentals of ERCP. In: Cotton P, Leung JW, eds. Advanced Digestive Endoscopy—ERCP. Boston, MA:
Blackwell Publishing; 2005.
30 QUESTION

A 56-YEAR-OLD FEMALE PRESENTS WITH RECURRENT RUQ PAIN,


IMPAIRED LFTS, ELEVATED ALKP, AND TRANSAMINASES. SHE HAD
A PRIOR ERCP AND POSSIBLE BILIARY PAPILLOTOMY FOR SIMILAR
COMPLAINTS BUT SHE DOES NOT RECALL THE DIAGNOSIS.
WHAT SHOULD I DO?

Joseph Leung, MD, FRCP, FACP, FASGE, MACG

The common complaint of right upper quadrant (RUQ ) pain and impaired liver function tests
(LFTs) could suggest biliary obstruction secondary to ductal stones and possible cholangitis if
there is fever or chills and other constitutional symptoms. Less commonly, it can be related to
attacks of pain associated with sphincter of Oddi dysfunction (SOD). However, late complications
of postpapillotomy stenosis remain a possibility in the differential diagnosis.
The clinical diagnosis of SOD is based on the presence of biliary-type pain and evidence of
bile duct obstruction, ie, dilated biliary system and/or impaired LFTs with elevated transaminases
more than 3 times the normal level. Based on the Milwaukee (Geenen) classification, SOD can
be classified as Type I, where patients present with typical biliary pain with imaging evidence of
a dilated common bile duct (CBD) and abnormal LFTs; Type II, where pain is associated with
either a dilated CBD or abnormal LFTs during attacks of pain; or Type III where there is pain
only, without evidence of ductal dilation on imaging and no LFT abnormalities during attacks of
pain. The clinical diagnosis of the classic Type I case is easy, whereas Type III patients may pose
a challenge to the clinician. Sphincter of Oddi manometry (SOM) can be performed to docu-
ment basal pressure in an otherwise normal or intact papilla. A basal pressure of > 40 mm Hg
is considered abnormal and supports the diagnosis of sphincter dysfunction. This is especially
important in patients with suspected Type III SOD for proper documentation before considering
a papillotomy. The conventional treatment for established SOD is papillotomy, aiming at cutting
the sphincter muscle to relieve biliary obstruction. The likely response will be good in Type I cases
but questionable in Type III patients.
The clinical presentation of bile duct stones can be similar with pain and impaired LFTs if the
stone impacts or obstructs the bile duct. Distal stone obstruction in the common channel can also

Leung J, Lo SK, eds. Curbside Consultation in Endoscopy:


137 49 Clinical Questions, Second Edition (pp 137-141)
© 2014 SLACK Incorporated
138  Question 30

Figure 30-1. Cholangiogram showing distal CBD stone.


Extrinsic compression on common hepatic duct is sugges-
tive of Mirrizzi’s syndrome.

cause acute biliary pancreatitis. The diagnosis of biliary stone(s) is more straightforward if there is
imaging evidence of a dilated duct, presence of ductal stones and/or gallbladder stones with ele-
vated alkaline phosphatase (ALKP), transaminases, and possibly bilirubin. The presence of chol-
angitis is more serious and warrants urgent attention and treatment. Imaging includes abdominal
ultrasound or a computed tomography scan, which can help to define the presence of any ductal
dilation or stones; although the presence of very small stones in the distal bile duct may not be
readily detected, even with magnetic resonance cholangiopancreatography (MRCP). Endoscopic
ultrasound (EUS) remains the most sensitive test for diagnosing distal bile duct stones. Endoscopic
retrograde cholangiopancreatography (ERCP) is more invasive, but offers a therapeutic option if
stones are detected in the bile duct.1 The positive predictive factors for the presence of bile duct
stones include positive imaging results, a dilated bile duct with impaired LFTs, concurrent chol-
angitis, or acute biliary pancreatitis. A past history of biliary pancreatitis may not predict the
presence of bile duct stones because the impacted stone can pass after a few days.
However, the clinical situation may change if the patient has undergone prior biliary papil-
lotomy. One thing to consider is whether the symptoms are related to recurrent bile duct stones,
especially in patients who still retain their gallbladder (Figure 30-1). Ascending reflux with
bacterial contamination can predispose to de novo bile duct stone formation, which can cause
How Should I Manage the Patient With Recurrent RUQ Pain?  139

A B
Figure 30-2. (A) Stenotic biliary orifice and evidence of a prior papillotomy (note periampullary diver-
ticulum). (B) Stone extraction after extension of prior papillotomy (guidewire in place).

subsequent cholangitis. Foreign bodies, including surgical clips, similarly can be a nidus of stone
formation. A less likely possibility is postpapillotomy stenosis secondary to scarring of the papil-
lary orifice (Figure 30-2A). This may present with a similar problem of biliary obstruction with
pain and impaired LFTs, with or without symptoms of infection and cholangitis.
The investigation will be similar to the other conditions, with conventional imaging of the bile
ducts to rule out any obstructing factor and to document the size of the bile ducts. Serial LFTs
can indicate whether there is evidence of persistent biliary obstruction. Ethyl hepatic iminodi-
acetic acid imaging (EHIDA) can demonstrate the patency of the biliary system and flow of the
isotope into the duodenum. Early or urgent intervention is indicated if the patient has signs and
symptoms of cholangitis or if there is evidence to suggest delayed emptying of the biliary system.
The aim of intervention is to provide drainage, and ERCP is the first line of treatment. ERCP can
demonstrate the status of the prior papillotomy, whether it is patent, and the presence or absence
of stones in the biliary system. If stones are demonstrated, the papillotomy can be extended if it
can be performed safely and if the stones can be removed with a basket or balloon (Figure 30-2B).
In the unusual situation of postpapillotomy stenosis, a number of treatment options are avail-
able. A lot depends on the size of the remaining papilla (if a small papillotomy was performed)
or if the orifice is stenosed. Late postpapillotomy stenosis can occur in up to 10% of cases after a
prior papillotomy, especially where the prior papillotomy was suboptimal. A tight pinhole open-
ing may sometimes be seen to represent the biliary orifice (Figure 30-3). Extension of a prior
papillotomy can be risky if there is not much remaining papilla. Further cutting will significantly
increase the risk of perforation. Following a diagnostic cholangiogram to rule out ductal filling
defects, a guidewire can be inserted in the (possible) dilated bile duct. A dilation balloon (size
ranging from 4 to 8 mm) can be selected, inserted over the guidewire, and fully inflated for
3 to 5 minutes to perform dilation of the orifice (sphincteroplasty)2 to improve drainage from the
biliary system (Figure 30-4). Bile should be taken for culture and prophylactic antibiotics should
be given if there is clinical suspicion of cholangitis. Depending on the findings and whether there
140  Question 30

Figure 30-3. A different patient with postpapil-


lotomy stenosis.

A B
Figure 30-4. (A) Six-mm biliary dilation balloon inflated at the stenotic orifice. (B) Wide opened biliary
orifice after balloon dilation.
How Should I Manage the Patient?  141

is any residual edema that may block the biliary orifice, a 7- or 8-cm 10-Fr stent can be inserted
to endure biliary drainage and removed after 1 week when tissue edema settles. Clinical follow-up
is necessary, as we have seen patients returning after a period of months to a year with recurrent
symptoms secondary to restenosis following sphincteroplasty. We have found that the addition of
steroid injection around the dilated orifice (Kenalog 40-mg solution injected in divided doses into
4 quadrants around the stent) helps to delay the onset of restenosis.

References
1. Leung J. Fundamentals of ERCP. In: Cotton P, Leung LW, eds. Advanced Digestive Endoscopy: ERCP. Boston, MA:
Blackwell Science; 2005.
2. Liao WC, Lee CT, Chang CY, et al. Randomized trial of 1-minute versus 5-minute endoscopic balloon dilation for
extraction of bile duct stones. Gastrointest Endosc. 2010;72:1154-1162.
31 QUESTION

MY ACCOUNTANT, WHOSE ULCERATIVE COLITIS IS IN GOOD CONTROL


ON MAINTENANCE 5 ASA THERAPY, CALLED BECAUSE HIS LFTS WERE
ELEVATED. HE HAD NO HISTORY OF JAUNDICE, BUT NEW ONSET OF
ITCHING. WHAT SHOULD I DO?

Andrew Yen, MD and Joseph Leung, MD, FRCP, FACP, FASGE, MACG

The approach to the patient with inflammatory bowel disease (IBD) and abnormal liver func-
tion tests (LFTs) starts with accurate interpretation of liver biochemical tests in the context of a
carefully obtained history and thorough physical examination. The pattern and severity of LFT
abnormalities can provide important clues in the differential diagnosis of underlying disease. A
cholestatic pattern of injury and the development of pruritus in this patient with ulcerative colitis
(UC) suggest the possibility of an underlying pancreaticobiliary disorder, including primary scle-
rosing cholangitis (PSC).
PSC is an uncommon, chronic, progressive disease of the biliary tract characterized by inflam-
mation and fibrosis of the intrahepatic and extrahepatic bile ducts, eventually leading to biliary
cirrhosis. The relationship between IBD and PSC is incompletely understood, but the association
is striking. Therefore, findings of cholestasis in this population should prompt consideration for
the presence of PSC, although the majority of patients with IBD will never develop this disease.
PSC is present in 2.4% to 4.0% of patients with UC and 1.4% to 3.4% of patients with Crohn’s
colitis or ileocolitis.1-4 Conversely, 80% of all patients with PSC have concomitant IBD.5-7 Of
patients with both IBD and PSC, 85% to 90% have UC. The relationship between colitis and
PSC is stronger, with more extensive colonic involvement, although the diseases often progress
independently of one another.
There are no standardized criteria for the diagnosis of PSC because the clinical spectrum of
disease is broad. The diagnosis of PSC is, therefore, based on typical cholangiographic findings
in the setting of consistent clinical, biochemical, serologic, and histologic findings, as well as
exclusion of secondary causes of sclerosing cholangitis. Characteristic cholangiographic findings
include multifocal stricturing and ectasia of the biliary tree. The result is the classic “beaded”
appearance of the bile ducts. Intrahepatic ducts are more commonly affected. In some cases,
there may be a dominant stricture involving the extrahepatic duct. Bifurcation obstruction with

Leung J, Lo SK, eds. Curbside Consultation in Endoscopy:


143 49 Clinical Questions, Second Edition (pp 143-145)
© 2014 SLACK Incorporated
144  Question 31

involvement of the right and left hepatic ducts can also occur and can be challenging to manage.
Notably, patients who have histologic, biochemical, and clinical features of PSC but a normal
cholangiogram may still have small-duct PSC.
If the biochemical profile in this patient demonstrates cholestasis, noninvasive imaging studies
can be first-line tests to assist with diagnosis, to evaluate the extent of disease, and to minimize
the risk of procedural complications. Ultrasonography or computed tomography may be useful
in planning further diagnostic and therapeutic strategies in select patients, but they are usually
insufficient for diagnosis, and normal findings do not exclude PSC. In patients who are unlikely
to require therapeutic intervention, magnetic resonance cholangiopancreatography (MRCP) is
reasonable to consider. It can be used to determine whether there is ductal dilation, strictures
involving intra- or extrahepatic bile ducts, and assist with the diagnosis of other etiologies for the
patient’s abnormal LFTs. Endoscopic retrograde cholangiopancreatography (ERCP) is considered
the gold standard for establishing a diagnosis of PSC and has the advantage of therapeutic inter-
ventions such as brush cytology, intraductal biopsy, balloon or catheter dilation, biliary stent place-
ment, and sphincterotomy with stone removal. Percutaneous transhepatic cholangiography (PTC)
may also yield diagnostic images and allows therapeutic interventions, but requires percutaneous
puncture and may be technically difficult if intrahepatic bile ducts are not dilated.
In asymptomatic individuals, treatment of the radiological changes on cholangiogram is unwar-
ranted because the strictures in PSC are often extensive, and contamination of the intrahepatic
system is likely to give rise to infection and subsequent intrahepatic sludge and stone formation.
If symptoms such as pruritus or jaundice are present, ERCP is indicated to define the underlying
ductal pathology and provide necessary intervention.
Other than liver transplantation, no specific therapy is effective in treating PSC. Objectives of
management include treating complications of the disease, such as bacterial cholangitis, jaundice,
and pruritus, and preventing other sequelae, including osteoporosis and nutritional deficiencies.
Cholangiocarcinoma and liver failure occur in PSC and should be diagnosed as early as possible
to allow the possibility of treatment.
Conservative management includes a combination of noninvasive imaging to document the
degree of involvement and checking serial LFTs, specifically alkaline phosphatase, gamma-
glutamyl transpeptidase, and bilirubin to monitor disease progress. Medical treatment may include
the use of oral deoxycholic acid (Ursodiol) to improve bile flow and for its immunomodulatory
effects, although long-term, high-dose ursodeoxycholic acid therapy is associated with higher rates
of serious adverse events.8,9 Because of the risk of biliary cancer with PSC, it is necessary to have a
high index of suspicion for malignancy, particularly when patients present with rapid deterioration,
worsening jaundice, weight loss, and abdominal pain. Documention of tumor markers, such as
CA19-9 and carcinoembryonic antigen (CEA), is often advised because of the high risk of malig-
nant changes with inflammatory strictures.
Endoscopic interventions are limited to endoscopic balloon dilation of extrahepatic dominant
bile duct strictures, followed by short-term (1 week) plastic stenting. Postdilation stenting is lim-
ited to short intervals to minimize ascending reflux and the risk of contamination and infection
within the intrahepatic biliary system. Repeat dilation and short-term stenting have been sug-
gested as a temporary treatment to avoid progression of the stricture(s) that may lead to secondary
biliary cirrhosis. Isolated intrahepatic duct involvement is often difficult to manage.
Long-term management includes serial evaluation of LFTs and tumor markers, in addi-
tion to annual surveillance colonoscopies in individuals with IBD, given the concurrent risk of
colonic dysplasia and cancer in these patients. Repeat ERCP and dilation may be necessary to
keep strictures open. Bile cultures should be taken at the time of ERCP to monitor the bacterial
flora in the biliary system and to guide antibiotic therapy should the patient develop cholangitis.
What Should I Do for a Patient With UC and Elevated LFTs?   145

It is also important for the patient to be followed by a hepatologist who can discuss alternative
treatments and management of advanced liver disease, including the ultimate decision for liver
transplantation.

References
1. Olsson R, Danielsson A, Järnerot G, et al. Prevalence of primary sclerosing cholangitis in patients with ulcerative
colitis. Gastroenterology. 1991;100(5 Pt 1):1319-1323.
2. Schrumpf E, Elgjo K, Fausa O, Gjone E, Kolmannskog F, Ritland S. Sclerosing cholangitis in ulcerative colitis. Scand
J Gastroenterol. 1980;15(6):689-697.
3. Shepherd HA, Selby WS, Chapman RW, et al. Ulcerative colitis and persistent liver dysfunction. Q J Med.
1983;52(208):503-513.
4. Rasmussen HH, Fallingborg JF, Mortensen PB, Vyberg M, Tage-Jensen U, Rasmussen SN. Hepatobiliary dysfunc-
tion and primary sclerosing cholangitis in patients with Crohn’s disease. Scand J Gastroenterol. 1997;32(6):604-610.
5. Bambha K, Kim WR, Talwalkar J, et al. Incidence, clinical spectrum, and outcomes of primary sclerosing cholangitis
in a United States community. Gastroenterology. 2003;125(5):1364-1369.
6. Chapman RW, Arborgh BA, Rhodes JM, et al. Primary sclerosing cholangitis: a review of its clinical features, chol-
angiography, and hepatic histology. Gut. 1980;21(10):870-877.
7. Wiesner RH, Grambsch PM, Dickson ER, et al. Primary sclerosing cholangitis: natural history, prognostic factors
and survival analysis. Gastroenterology. 2003;125(5):1364-1369.
8. Lindor KD, Kowdley KV, Luketic VA, et al. High-dose ursodeoxycholic acid for the treatment of primary sclerosing
cholangitis. Hepatology. 2009;50(3):808-814.
9. Eaton JE, Silveira MG, Pardi DS, et al. High-dose ursodeoxycholic acid is associated with the development of
colorectal neoplasia in patients with ulcerative colitis and primary sclerosing cholangitis. Am J Gastroenterol.
2011;106(9):1638-1645.
32 QUESTION

MY 35-YEAR-OLD PATIENT WHO HAD A CHOLECYSTECTOMY 5 YEARS


AGO DURING HER GASTRIC BYPASS SURGERY PRESENTS WITH
ABDOMINAL PAIN AND IMPAIRED LFTS. CT SHOWED A 15-MM CBD
WITH A DISTAL CBD STONE. ERCP WAS UNSUCCESSFUL USING A
PEDIATRIC SCOPE. CAN YOU HELP?

Simon K. Lo, MD

On the surface, this appears to be a case of a simple bile duct stone that should be easily treated
with a sphincterotomy and stone extraction during an endoscopic retrograde cholangiopancreatog-
raphy (ERCP). However, this patient has a prior Roux-en-Y gastric bypass surgery, which presents
a big problem in reaching the papilla. A standard duodenoscope may be long enough to get to the
jejunojejunostomy, but negotiating into the afferent jejunal limb with the stiff bending section of
the duodenoscope is difficult and the length of the scope is simply inadequate to approach the
papilla. Similar to performing a colonoscopy, passing the scope through the various anastomoses
and backing it up to the duodenum may stretch the intestinal wall far beyond its natural length.
Even a colonoscope is typically too short to do the job.
There are several ways of performing ERCP in this situation. The most common method is to
insert a duodenoscope into the excluded stomach or jejunum through a small incision, with the
help of a laparoscopic surgeon. It may take a little adjustment from the usual ERCP technique
because the procedure is performed in the supine position and the path to the proximal duodenum
is short and angulated. Performing a procedure in the sterile surgical environment is slightly chal-
lenging, and the setup in the operating room is somewhat foreign to the biliary endoscopist. The
surgical staff may find it inconvenient to wear radiation protection gowns throughout the proce-
dure and have nonsterilized equipment crossing the sterile field. Fluoroscopic examinations neces-
sitate clearing of the staff from direct radiation exposure, which mandates very brief examinations
and invariably lowers the quality and quantity of the radiographs. Patients with ERCP done in this
manner usually need to stay overnight for observation. The greatest challenge in doing the proce-
dure in this manner is to coordinate schedules between the busy surgeon and gastroenterologist,

Leung J, Lo SK, eds. Curbside Consultation in Endoscopy:


147 49 Clinical Questions, Second Edition (pp 147-149)
© 2014 SLACK Incorporated
148  Question 32

as their valuable time may be wasted in waiting for each other to start the case. In spite of these
inconveniences, the success rate of the ERCP done in this manner is very high—approaching
100% in experienced hands.1
If a 1-step, 1-day arrangement with a surgical colleague is not possible, the gastric access can
be created on 1 day while the ERCP can be carried out after the gastric fistula has matured.
Although scheduling 2 separate procedures is relatively easy for the gastroenterologist and surgeon
to arrange, patients are not always happy with undergoing 2 procedures on separate days. The
surgical fistula is kept open with a feeding tube–type device until it is no longer needed, and it
usually has to be dilated prior to insertion of the duodenoscope. Finally, a decision has to be made
as to when the fistula should finally be closed. There are 2 alternative, nonoperative methods to
create a gastric fistula. One way is to use endoscopic ultrasound (EUS) to first access the excluded
stomach for gastric insufflation and guidance of a percutaneous endoscopic gastrostomy (PEG)
placement.2 Another way is to perform an antegrade double-balloon enteroscopy (DBE) to get up
to the excluded stomach, with a gastrostomy tube placed in a manner similar to that of a regular
PEG.3
The recent advances in EUS have led to the possibility of performing transgastric puncture of a
dilated intrahepatic duct to obtain cholangiograms and carry out therapeutic interventions. In the
Roux-en-Y bariatric surgery, the needle is inserted across the small gastric pouch. A guidewire is
passed into the bile duct and across the papilla into the duodenum to facilitate ductal and sphincter
dilation. The bile duct stone can then be pushed distally into the duodenum. For this approach to
work, the transgastric tract has to be dilated and the bile duct cannot have too many stones or an
excessively large stone. It has been reported that this approach may fail in one-third of cases and
must be immediately converted to the internal rendezvous procedure.4 The rendezvous technique
involves leaving the guidewire through the bile duct in the small intestine. A double-balloon scope
is then passed orally into the jejunojejunostomy and up the afferent limb until the wire is found.
The transsphincteric guidewire helps to direct instrument passage for sphincter dilation and stone
extraction. This EUS-guided procedure is obviously complicated and requires very good EUS
and ERCP skills. Although accessing a dilated intrahepatic duct is relatively simple to perform,
advancing an instrument for tract enlargement is very challenging, which results in a high failure
rate (33%). A rescue procedure in the form of a DBE, with a long wire threading through all the
organs while the scope is being advanced, is cumbersome and potentially risky. This procedure
should be performed only in tertiary care centers and when alternative methods are not feasible.
I have tried the various techniques of performing ERCP in patients who had a prior Roux-en-Y
bariatric surgery. In my opinion, the best way to do ERCP for these patients is to do so in one ses-
sion and without the arrangement of the operating room. The only way to achieve this goal is to
use an endoscope that can reliably reach the major papilla. Experience tells us that this can be done
only with the double-balloon or another form of deep enteroscope. But there are many hurdles to
performing ERCP with a double-balloon scope, including identification of the jejunojejunostomy
and correct selection of the right entrance into the afferent limb and then dealing with the techni-
cal difficulty in cannulating the sphincter with limited long accessories.
I start the procedure by passing a therapeutic, 200-cm double-balloon scope into the small
gastric pouch. From there, the scope passes through the gastrojejunostomy into the Roux limb,
which usually is located on the right side. Forward passage generally becomes difficult within a
few centimeters before the jejunojejunostomy. This is also where bile fluid is first noted. At this
point, I slow down the scope passage to avoid missing the anastomosis and inadvertently entering
the efferent limb. At the anastomosis, the first decision is to determine the line where the 2 intes-
tinal segments are connected. This should appear as an oval-shaped ring. The enteroscope has to
cross this ring, beyond which are 2 luminal openings. For reasons unknown, the afferent opening
is usually the easier one to visualize. Another way of finding the afferent limb is by choosing the
center lumen when all 3 possible exits at the anastomosis are lined up linearly. With this approach,
I Attempted an ERCP Using the Pediatric Colonoscope but Failed. Can You Help?  149

I typically find the papilla on first endoscopy passage 95% of the time. I am not certain if it is
necessary, but I always try to first enter the stomach, where I use the inflated scope and overtube
balloons to shorten and straighten the enteroscope.
With the double-balloon scope gently pulled back from the pylorus, the minor papilla will soon
be recognized. If the minor papilla is placed at the 4 o’clock position by scope rotation, the major
papilla should be seen at 6 or 7 o’clock upon further withdrawal. I find it best to perform ERCP
cannulation at this position, which is not easy because of the tangential approach. Some endosco-
pists recommend using a soft hood to keep the papilla in view. However, I have not found it to be
very helpful. Still others recommend retroflexing the enteroscope in the descending duodenum to
facilitate cannulation in a more frontal manner. Although it is occasionally possible to do, there is
typically insufficient space to align the papilla properly. I cannulate the papilla with a long length
sphincterotome or plain ERCP catheter. A hydrophilic guidewire preloaded through this catheter
is very helpful to access the common bile duct. In case of failure, I use a sphincterotome that
has been modified into a needle knife to perform precutting. Finally, a balloon dilator is used to
stretch the sphincter to the desired caliber before extracting the bile duct stone.
Since 2005, I have done more than 140 ERCPs on patients with Roux-en-Y gastric bypass with
the double-balloon scope. To date, our chance of successfully entering the bile duct to perform
therapy is approximately 90%. The average total procedure time is 1.5 hours, with roughly equal
time devoted to DBE and ERCP manipulations. Despite our success, this form of ERCP may not
be applicable in most institutions. It requires the combined expertise of DBE and ERCP and the
complication rate is roughly 15%. Mild pancreatitis and pain without confirmed pancreatitis are
the most common complications, with rare incidence of bleeding or perforation.
For this patient, I would consider performing an ERCP with the use of a double-balloon scope.
In the event of failure, I would consider either a combined procedure in the operating room or
a percutaneous transhepatic cholangiogram (PTC) and passage of a wire into the duodenum. If
the interventional radiologist is comfortable with pushing the stone down into the intestine after
balloon dilation, he or she should simply proceed with it. Otherwise, I can perform a PTC and
DBE rendezvous procedure by inserting the double-balloon scope up to the papilla and then use
the wire to help with the sphincterotomy and stone extraction.

References
1. Schreiner MA, Chang L, Gluck M, et al. Laparoscopy-assisted versus balloon enteroscopy-assisted ERCP in bariatric
post-Roux-en-Y gastric bypass patients. Gastrointest Endosc. 2012;75(4):748-756.
2. Attam R, Leslie D, Freeman M, Ikramuddin S, Andrade R. EUS-assisted, fluoroscopically guided gastrostomy
tube placement in patients with Roux-en-Y gastric bypass: a novel technique for access to the gastric remnant.
Gastrointest Endosc. 2011;74:677-682.
3. Ross AS, Dye C. Double-balloon enteroscopy to facilitate retrograde PEG placement as access for therapeutic
ERCP in patients with long-limb gastric bypass. Gastrointest Endosc. 2007;66(2):419.
4. Weilert F, Binmoeller KF, Marson F, Bhat Y, Shah JN. Endoscopic ultrasound-guided anterograde treatment of bili-
ary stones following gastric bypass. Endoscopy. 2011;43:1105-1108.
33 QUESTION

A 48-YEAR-OLD MALE HAD LIVER TRANSPLANTATION


9 MONTHS AGO FOR CHRONIC HEP C CIRRHOSIS. HE PRESENTED
RECENTLY WITH RECURRENCE OF JAUNDICE AND IMPAIRED LFTS.
HOW SHOULD I MANAGE THIS PATIENT?

Milan Bassan, MBBS, FRACP and Paul Kortan, MD, FRCPC, FASGE, AGAF

Recurrent jaundice and derangement of liver function tests (LFTs) following liver transplanta-
tion can have a number of possible causes. The initial step in the management is to determine
whether the hepatic disturbance is related to parenchymal pathology (such as recurrence of hepa-
titis C, chronic rejection, or drug-induced liver injury) or biliary pathology. This chapter will focus
on the management of posttransplant biliary disease.

Assessment
When faced with a patient who potentially has significant posttransplant biliary pathology
that is causing jaundice, the first step is a thorough clinical assessment with special attention to
factors that may predispose to biliary strictures. These include the type of anastomosis, technical
difficulties at time of surgery (extensive dissection, bleeding requiring cautery, prolonged cold
ischemia time), the presence of postsurgical bile leak, history of hepatic artery thrombosis or other
posttransplant ischemia, ABO mismatch, and underlying primary sclerosing cholangitis (PSC).
The placement of a T-tube (now rarely performed) is associated with a higher rate of bile leak and
cholangitis. Whether a Roux-en-Y biliary reconstruction or duct-to-duct anastomosis is fashioned
probably does not affect complication rates, but it does impact management strategies. Hepatic
artery thrombosis is associated with nonanastomotic stricturing because the hepatic artery supplies
the bile duct. These strictures may occur intra- or extrahepatically.
The initial investigation to assess for biliary pathology following liver transplant is imaging
with abdominal ultrasound and Doppler assessment of the hepatic vessels. This allows for the

Leung J, Lo SK, eds. Curbside Consultation in Endoscopy:


151 49 Clinical Questions, Second Edition (pp 151-158)
© 2014 SLACK Incorporated
152  Question 33

assessment of biliary dilation as well as hepatic artery flow. It must be remembered that the
sensitivity of ultrasound for biliary pathology in this population is relatively low, so the pretest
probability of significant pathology must be taken into account when interpreting a negative
ultrasound study.
If there is evidence of biliary dilation in a clinical context that is suggestive of anastomotic
stricture (especially with a duct-to-duct biliary anastomosis), proceeding directly to endoscopic
retrograde cholangiopancreatography (ERCP) without further imaging is a reasonable approach
to allow for diagnostic cholangiography as well as therapeutic intervention at the same session.
However, for most patients, noninvasive imaging with magnetic resonance cholangiopancreatogra-
phy (MRCP) is preferred because it provides further information on the biliary anatomy, and site
of stricture that can guide endoscopic therapy. On occasion, a patient may present with a high
suspicion for biliary pathology but all imaging is negative. In these patients, a clinical decision
needs to be made about proceeding to ERCP or proceeding to liver biopsy to assess for rejection.

Post-Liver Transplant Biliary Complications


The timing of the onset of jaundice and worsening liver function is often helpful in predicting
the type of pathology likely to be encountered, especially if there is a posttransplant biliary
stricture. Early complications are those occurring within 1 month of liver transplantation and late
are those occuring after this.
Posttransplant bile leaks are strongly associated with subsequent stricture formation if not
treated early. They may also lead to the formation of bilomas. Early bile leaks are usually related
to the anastomosis and are seen more commonly with a duct-to-duct anastomosis. These are best
treated with early ERCP, sphincterotomy, and plastic biliary stenting. Bile leak post-Roux-en-Y
reconstruction can be treated endoscopically (often with balloon-assisted enteroscopy) if local
expertise is available or by a percutaneous approach with percutaneous transhepatic cholangiogram
(PTC) and placement of an external–internal biliary drain. Due to the immunosuppression, tissue
healing will be slow, and stents should be left in place for a minimum of 2 months, and resolution
of the leak should be confirmed at ERCP at the time of stent removal.
Most early strictures are due to technical factors related to the surgery and anastomosis.
Strictures can be characterized as anastomotic (single, focal, short, often tight stricture at or
less than 5 mm proximal to the anastomosis) or nonanastomotic. The majority of posttransplant
strictures are anastomotic and occur within the first 12 months (Figure 33-1). A subset of patients
develop early anastomotic narrowing; these are generally due to postsurgical inflammation and
edema and respond very well to short-term placement of a plastic stent (usually for less than
3 months).
Most anastomotic strictures require multiple sessions of endoscopic therapy, which generally
involve sphincterotomy, balloon dilation of the stricture, and placement of plastic stents
(Figure 33-2). The optimal stenting strategy remains unclear, with debate in the literature about
the number of plastic stents and the timing of stent exchange. Treatment is often long term
(12 to 24 months) with multiple ERCP sessions. There is also some promising literature on the
role of fully covered metal stents to treat anastomotic strictures; however, these have not reached
routine clinical practice in most centers. If used, caution must be taken to ensure that the hilum
is not obstructed. Our approach to anastomotic strictures is based on the caliber of the stricture.
When crossed with a wire, we balloon dilate and place a plastic stent (10 Fr if possible). We then
follow-up with regular ERCP sessions every 2 to 3 months with dilation to greater caliber (with
a target of 8 mm) and placement of 2 to 3 10-Fr plastic stents. Even after successful endoscopic
therapy, these patients require close follow-up because recurrence rates are high. Most recurrences
can be successfully retreated endoscopically.
How Should I Manage a Patient With Liver Transplantation and Recent Jaundice?  153

A B

C D
Figure 33-1. (A) Tight anastomotic stricture 3 months following a cadaveric liver transplant.
(B) Guidewire advanced through the stricture. (C) Balloon dilation. (D) One stent and guidewire
for second stent in place.

Nonanastomotic strictures account for up to a quarter of posttransplant strictures and are


more difficult to treat. They are generally related to ischemia, can occur anywhere proximal to
the anastomosis, and may be multifocal (on occasion resembling PSC). They tend to occur early
(3 to 6 months posttransplant) and are more likely to be associated with sludge or casts. The
general principles of management are similar to anastomotic strictures (serial dilation and stent-
ing), but dilation is usually to a smaller caliber (4 to 6 mm). PTC may be required to treat more
peripheral strictures. Long-term response to endoscopic therapy is often suboptimal and there is a
higher rate of need for surgical intervention and retransplantation.
154  Question 33

A B

C D
Figure 33-2. (A) Tight anastomotic stricture 7 months following a cadaveric liver transplant. (B) Biliary
dilation proximal to anastomotic stricture. (C) Balloon dilation. (D) Placement of 2 10-Fr stents. (continued)

Posttransplant biliary strictures in patients with a Roux-en-Y reconstruction can be difficult to


treat endoscopically and generally require balloon-assisted enteroscopy to perform ERCP. If this
expertise is not available, PTC may be required.
Biliary stones or sludge can cause biliary obstruction in approximately 10% of patients. Most
of these occur late posttransplant and can be managed with ERCP in a similar manner to
How Should I Manage a Patient With Liver Transplantation and Hep C Cirrhosis?  155

E F
Figure 33-2 (continued). (E) Improvement in stricture diameter 6 months later. (F) Good spontaneous
drainage of contrast.

nontransplant patients. The presence of biliary strictures is associated with higher rates of stone
and sludge formation and the underlying stricture needs to be treated to successfully manage
these patients. A number of patients can develop what is referred to as the biliary cast syndrome.
The pathogenesis of this entity is poorly understood. These patients develop multiple casts within
the biliary tree, composed primarily of bile, bile acids, collagen, and cholesterol. The presence of
casts is associated with ischemia and multifocal nonanastomotic structuring and also with a worse
prognosis and high risk of retransplantation. Endoscopic management often requires the use of
multiple interventions, including sphincterotomy, basket and balloon extraction, and stenting at
multiple sessions. Many patients may also require concurrent percutaneous therapy. Even with this
strategy, successful clearance of casts occurs only in 60% of patients.
Immunosuppression is also associated with an increased risk of malignancy, and on occasion
a lesion unrelated to the underlying liver disease may be responsible for biliary obstruction
(eg, pancreatic mass, periportal lymphadenopathy), especially long-term posttransplant.

Living Donor Transplantation


With the chronic shortage of donor organs, the rate of living related donor transplantation is
increasing. Typically, the donor right lobe is transplanted with an anastomosis from the donor
right hepatic duct(s) to the recipient common bile duct. These patients have higher rates of stric-
tures and bile leaks, although this is related in part to the center's experience because complication
rates decrease with increasing volume. Strictures are generally managed similarly to those postca-
daveric transplant; however, smaller stents may be required and close pre-ERCP assessment of the
anatomy should be undertaken because there may be multiple anastomoses (Figure 33-3).
156  Question 33

A B

C D
Figure 33-3. (A) Anastomotic stricture post-live donor liver transplant with ductoplasty of the right
anterior and right posterior ductal system. (B) Stricture crossed initially with guidewire with some dif-
ficulty. (C) Dilation of stricture. (D) Single 10-Fr biliary stent placed. (continued)

Our Patient
Going back to the patient in the vignette, the likelihood of a posttransplant biliary stricture is
high. Assuming parenchymal pathology was excluded, the next step is an abdominal ultrasound
with Doppler assessment of the hepatic vessels. If there was biliary dilation on ultrasound, we
would proceed directly to ERCP. If there is no dilation, the next step is further biliary imaging
with MRCP. If this was negative, the decision on proceeding to ERCP or liver biopsy would be
based on the patient’s course since transplant.
If the patient presented 2 years posttransplant, the likelihood of a biliary complication would
be low and further assessment for parenchymal pathology should be undertaken (mostly to exclude
recurrent hepatitis C and chronic rejection). If hepatobiliary imaging studies were negative for
biliary obstruction, we would proceed with a liver biopsy as the initial investigation.
How Should I Manage a Patient With Liver Transplantation and Hep C Cirrhosis?  157

E F

G H
Figure 33-3 (continued). (E) Persistent stricture
3 months later. (F) Placement of 2 10-Fr stents.
(G) Improved stricture diameter 3 months later.
(H) Placement of 3 stents. (I) Resolution of stric-
ture 1 year following initial presentation.

I
158  Question 33

Bibliography
Ayoub WS, Esquivel CO, Martin P. Biliary complications following liver transplantation. Dig Dis Sci. 2010;55:1540-1546.
Balderramo D, Navasa M, Cardenas A. Current management of biliary complications after liver transplantation:
emphasis on endoscopic therapy. Gastroenterol Hepatol. 2011;34:107-115.
Buxbaum JL, Biggins SW, Bagatelos KC, Ostroff JW. Predictors of endoscopic treatment outcomes in the manage-
ment of biliary problems after liver transplantation at a high-volume academic center. Gastrointest Endosc.
2011;73:37-44.
Hsieh TH, Mekeel KL, Cowell MD, et al. Endoscopic treatment of anastomotic biliary strictures after living donor liver
transplantation: outcomes after maximal stent therapy. Gastrointest Endosc. 2013;77:47-54.
Jorgensen JE, Waljee AK, Volk ML, et al. Is MRCP equivalent to ERCP for diagnosing biliary obstruction in orthotopic
liver transplant recipients? A meta-analysis. Gastrointest Endosc.2011;73:955-962.
Zoepf T, Maldonado de Dechene EJ, Dechene A, et al. Optimized endoscopic treatment of ischaemic-type biliary
lesions after liver transplantation. Gastrointest Endosc. 2012;76:556-563.
34 QUESTION

A 68-YEAR-OLD BANKER WHO HAD A PRIOR WHIPPLE FOR


PERIAMPULLARY CANCER HAD RECURRENT ABDOMINAL PAIN AND FEVER
FOR 3 YEARS. EACH RESOLVED WITH ANTIBIOTICS. LFTS ARE IMPAIRED
AND CT SCAN SHOWS DILATED INTRAHEPATIC DUCTS AND A TUBULAR
STRUCTURE IN THE PANCREAS. WHAT SHOULD I DO?

Jason B. Samarasena, MD and John G. Lee, MD

The differential diagnosis for a patient with abdominal pain after a Whipple procedure is broad
and includes the conditions found in Table 34-1.
The fact that this patient had fever and intrahepatic bile duct dilation associated with his
abdominal pain narrows my focus toward cholangitis due to biliary obstruction. Biliary obstruction
after a Whipple procedure is a relatively rare complication, with the more common causes being
postoperative benign biliary strictures, choledocholithiasis, and malignant stricture from recurrent
cancer. Given the fact that this patient has had recurrent symptoms over the past 3 years, a
malignant stricture from recurrent cancer would be highly unlikely. A postoperative benign biliary
stricture is more likely, although when symptoms develop from strictures, further intervention
beyond antibiotics is typically needed. Choledocholithiasis is another likely possibility in this case.
As part of the Whipple procedure, the gallbladder is removed at the time of the surgery as well
as part of the bile duct; however, patients can still produce stones in the intrahepatic ducts and
postsurgical anatomy of the biliary system can lead to bile stagnation and stone formation.
Based on a computed tomography (CT) scan, it is difficult to determine the etiology of this
patient’s biliary obstruction. A diagnostic test that might help here is a magnetic resonance chol-
angiopancreatography (MRCP). This is a noninvasive test to visualize the biliary and pancreatic
systems. It is essentially a heavily T2-weighted magnetic resonance imaging (MRI) of the abdo-
men that has the effect of making stationary or slow-flowing fluid within the bile and pancreatic
ducts appear very bright in relation to the low signal intensity produced by adjacent solid organs.1
This results in a high quality, noninvasive look at the biliary and pancreatic duct anatomy and

Leung J, Lo SK, eds. Curbside Consultation in Endoscopy:


159 49 Clinical Questions, Second Edition (pp 159-162)
© 2014 SLACK Incorporated
160  Question 34

Table 34-1

Differential Diagnosis for Abdominal Pain in Patients


Who Have Undergone the Whipple Procedure
● Choledocholithias ● Recurrence of cancer
● Benign or malignant biliary stricture ● Afferent loop syndrome
● Cholangitis ● Gastric or intestinal dysmotility
● Pancreatitis ● Small bowel bacterial overgrowth

can be helpful in diagnosing biliary strictures and choledocholithiasis. The radiologist should
be informed of the postsurgical anatomy because we have seen MRCP incorrectly interpreted as
showing distal bile duct stricture due to nonvisualization of the distal bile duct (which of course
had been removed during the Whipple procedure).
Another diagnostic test I might consider is endoscopic ultrasound (EUS). In normal anatomy,
EUS is an excellent test to visualize the biliary system and the pancreas. In post-Whipple anatomy,
however, part of the stomach may be removed, and as a result visualization of choledochojeju-
nal anastamosis and biliary system can be limited. One exception here is a pylorus-preserving
Whipple where the stomach is left intact. This affords a better chance of biliary tract visualization.
Direct cholangiography by endoscopic retrograde cholangiopancreatography (ERCP) and
percutaneous transhepatic cholangiography (PTC) are more invasive diagnostic options, but they
provide better visualization as well as the ability to perform biopsy and therapeutic intervention.
This patient has a tubular structure in the pancreas, which most likely represents a pancreatic
stent. Pancreatic stents are often placed at the time of Whipple procedure when there is concern
that the pancreaticojejunal anastamosis may obstruct due to swelling in the immediate postopera-
tive period. Typically, these stents are removed after a period of 6 to 8 weeks, and the fact that the
stent remains within the pancreas of this patient is unusual. Our recommendation would be for
removal of the pancreatic stent given the concern for stent occlusion or migration, and this is best
performed endoscopically. Because endoscopy will be performed for stent removal, our preference
would be to also perform ERCP to evaluate the biliary system in this patient, as opposed to PTC.
Our approach to endoscopy in this patient would start with the patient receiving anesthesia
monitoring for the procedure. Typically, altered anatomy ERCP can be among the most
challenging of ERCPs and procedures can often be prolonged. We prefer the patient to be in
the left lateral position for ERCP. Our initial approach in post-Whipple patients is to start with
a standard gastroscope due to its flexibility and maneuverability. It is usually long enough for
most cases and lack of an elevator is not a problem because post-Whipple patients do not have an
ampulla. If that endoscope is not long enough, we switch to a pediatric colonoscope.
It is impossible to know with 100% certainty which opening of the gastrojejunostomy is
the afferent loop; the only way to be sure is if you can identify the choledochojejunostomy and
the blind end of the afferent limb. Having said that, the entrance to the afferent limb is more
acutely angled and the limb heads to the right upper quadrant as the endoscope is advanced
(Figure  34-1). Presence of an air cholangiogram and/or surgical clips can be helpful markers;
alternatively we usually place a radioopaque object in the right upper quadrant to clearly mark the
location, especially if the patient is on the left lateral position. Other helpful indicators include
What Should I Do for Prior Whipple Patient?  161

Figure 34-1. Fluoroscopic image of a standard


gastroscope at the choledochojejunostomy
with cholangiogram.

bile in the limb as well as the determination of the other limb as being efferent heading deep into
the pelvis. Tattooing the limb already travelled can be useful if there is a chance of confusion later.
Once the end of the afferent limb is reached, the endoscope is slowly withdrawn while we scan
for the pancreatic and biliary anastomoses. The pancreaticojejunal anastomosis in this patient will
likely be identified by the stent in place. The pancreatic stent can be removed with a rat-tooth for-
ceps or a snare. The choledochojejunal anastomosis is usually located at 5 to 10 cm of the end of the
afferent limb and is often found around the 7 or 8 o’clock position in the endoscopic field, although
this varies from patient to patient. Most patients have a single opening to the common hepatic
duct, but some may have separate openings into the left and right intrahepatic ducts, depending
on the level of the surgical anastomosis. The biliary orifice is usually very obvious because most
patients have a dilated bile duct and thus a large choledochojejunal opening; however, the orifice
can be obscured behind a fold or may be stenotic.
Cannulation is straightforward in most cases due to the large biliary orifice; using a
stone extraction balloon is helpful to occlude the orifice when performing a cholangiogram
(Figure 34-2). Cannulation of a stenotic or tumor infiltrated choledochojejunal orifice is best
done using a hydrophilic guidewire. All ERCP accessories can be used with a gastroscope, but a
pediatric colonoscope will require using enteroscope-length accessories or modification of most
standard ERCP accessories that are too short.
After cannulation is achieved, we perform a balloon occlusion cholangiogram to evaluate the
biliary system. In this particular patient, a biliary stricture or choledocholithiasis is the most
likely etiology. If a biliary stricture is present, we perform brushings to evaluate for evidence of
malignancy, if EUS FNA had not already been done. In our experience, benign choledochojejunal
strictures are usually web-like and respond nicely to balloon dilation and injection of steroid.
Removal of sutures or staples from this area may help prevent buildup of sludge or scar tissue in
the future and can be done using rat-tooth forceps. Multiple stents can be placed in patients with
recurrent strictures, as is done in other patients with postoperative stricture. Malignant strictures
are longer and require stenting. An adult gastroscope only accommodates a 7-Fr stent, so long-
term stenting will require use of a metal stent. Use of a covered metal stent will require switching
the endoscope to a large-channel instrument, which should be done cautiously due to increased
risk of perforation.
Stones can be removed using the usual accessories. It would be unusual to have a large stone
requiring lithotripsy, but if needed, intraductal lithotripsy can be done because a standard
gastroscope will not accept a mechanical lithotripter. Cholangioscopy is usually not difficult and
162  Question 34

Figure 34-2. Endoscopic image of cannulation


of the choledochojejunostomy.

can be done using a pediatric gastroscope. Given that this is a surgical anastomosis, there is no role
for sphincterotomy or balloon sphincteroplasty.

Conclusion
Among the challenges of performing ERCP in patients with altered anatomy is failure to
traverse the afferent limb and reach the biliary or pancreatic orifice. In our experience, the length
of the afferent limb in post-Whipple patients varies, but usually it is not outside of the reach of
a standard upper gastroscope. Occasionally, in cases of a long afferent limb, a pediatric colono-
scope or a deep enteroscopy technique, such as spiral enteroscopy or single- or double-balloon
enteroscopy, can be used. However, due to the lack of accessories available for this length of scope,
any therapeutic interventions are limited to performing cholangiogram, removing small stones,
and placing small-caliber stents. 2

References
1. Bret PM, Reinhold C. Magnetic resonance cholangiopancreatography. Endoscopy. 1997;29(6):472-486.
2. Lopes TL, Wilcox CM. Endoscopic retrograde cholangiopancreatography in patients with Roux-en-Y anatomy.
Gastroenterol Clin North Am. 2010;39(1):99-107.
35 QUESTION

A 78-YEAR-OLD VIETNAMESE WOMAN WHO COLLAPSED AT HOME


WAS FOUND TO BE FEBRILE AND IN SHOCK WITH A SYSTOLIC BP OF
80 MM HG. SHE IS MILDLY JAUNDICED WITH ABDOMINAL GUARDING.
HOW WOULD YOU MANAGE HER?

Joseph Leung, MD, FRCP, FACP, FASGE, MACG and Catherine Ngo, MD

The emergency management of someone in shock includes initial aggressive resuscitation and
identification of immediately reversible causes, including underlying cardiopulmonary events. The
presence of fever, abdominal guarding, and jaundice would suggest an acute abdomen and possible
biliary sepsis as a cause of the hypotension. Difficulty in the management of this elderly patient
may include a language barrier, as she may not speak English, and thus the need to get help from
family members or an interpreter is essential to obtain a detailed history.
Biliary sepsis complicated by shock is a serious medical/surgical condition. This is often pre-
cipitated by infection within an obstructed biliary system, which is most likely caused by biliary
stones. In Western countries, most stones originate from the gallbladder. However, an Asian
patient may have stones that form de novo in the bile duct (brown pigment stones) or originate
from the intrahepatic ducts associated with Oriental cholangiohepatitis or recurrent pyogenic
cholangitis (RPC), in which stones formed as a result of bacterial activities. These stones are more
likely to give rise to biliary sepsis if ductal obstruction occurs. In general, the bacteria causing bili-
ary sepsis are a mixture of gram negative and gram positive bacteria. Gram negative sepsis leads
to endotoxemia in the presence of bile duct obstruction and accounts for the complications of sup-
purative cholangitis; namely, mental confusion, hypotension, renal failure, and sometimes death.
Indeed, suppurative cholangitis carries a significantly higher mortality if not managed properly.
Initial management of the patient in shock in the ER includes resuscitation and stabilization
while attempting to obtain a history from the patient and family. The history and clinical exam
may rule out significant gastrointestinal bleeding as the cause of hypotension. An electrocardio-
gram (EKG) and basic blood tests should be done to evaluate for underlying cardiac conditions
(including a myocardial infarction) as the cause of shock. Although the classical description of
cholangitis has the triad of pain, fever, and jaundice, very often elderly patients with biliary sep-
sis may not develop a fever. The occurrence of mental confusion, together with shock, is a bad
Leung J, Lo SK, eds. Curbside Consultation in Endoscopy:
163 49 Clinical Questions, Second Edition (pp 163-167)
© 2014 SLACK Incorporated
164  Question 35

prognostic indicator. Blood culture should be taken to evaluate for sepsis and routine blood tests
including blood count, liver function, kidney function, and coagulation profile. Initial resuscita-
tion will include adequate IV fluid replacement. Patients with endotoxemia require a large amount
of fluids to expand the circulation. However, caution should be taken to avoid overloading the
patient with fluids, which may lead to heart failure. It is important to monitor the urine output
with adequate fluid replacement. Avoid the use of diuretics in the beginning unless the patient has
signs of fluid overload or heart failure to avoid precipitating renal dysfunction. Urgent imaging
should include an abdominal ultrasound or computed tomography to evaluate for gallstone disease,
any evidence of dilated common bile duct or intrahepatic ducts, or the presence of bile duct stones
causing obstruction.
After obtaining a blood culture, the patient should be given broad-spectrum antibiotics to
guard against possible gram negative and gram positive infections. The most common organ-
isms causing biliary sepsis include Escherichia coli, Klebsiella, Enterococcus, and possible anaerobic
bacteria. In general, we recommend giving a second- or third-generation cephalosporin or a
fluoroquinolone (eg, ciprofloxacin). In very sick patients, we add metronidazole to cover against
possible anaerobic infection.1 Ampicillin or augmentin IV is given if blood culture isolates gram
positive bacteria. Caution should be taken when considering an extended use of ciprofloxacin to
avoid long-term complications.
Patients with acute cholangitis will usually respond to conservative treatment with fluid
replacement and antibiotics therapy. In patients with complete bile duct obstruction associated
with stone impaction, the intrabiliary pressure is elevated, which prevents the effective excretion of
antibiotics into bile to control the infection. Suppurative cholangitis occurs in 20% of patients with
complete obstruction. These patients may fail to respond to conservative management and will
require urgent biliary decompression.2 Clinical signs and symptoms that warrant urgent drainage
include high fever, persistent shock with dropping blood pressure and increasing tachycardia, and
persistence of abdominal pain and tenderness, indicating a raised intrabiliary pressure.
Urgent endoscopic retrograde cholangiopancreatography (ERCP) and endoscopic drainage is
the best approach to the management of patients with suppurative cholangitis because of the lower
morbidity and mortality when compared with emergency surgery or percutaneous transhepatic
drainage.2 Because of an associated bleeding tendency from obstructive jaundice and possible dis-
seminated intravascular coagulation with platelet dysfunction from sepsis, we recommend biliary
decompression using a stent or nasobiliary drain, often without a papillotomy if successful deep
common bile duct (CBD) cannulation can be achieved.3
Elderly patients with sepsis do not tolerate IV sedation well and it is important to avoid overse-
dation. It may be necessary to request anesthesiology support for sedation in critically ill patients
undergoing an emergency biliary drainage procedure. It is important to understand that cholan-
gitis is a result of stasis and infection, and the goal of urgent ERCP is to provide drainage of the
obstructed biliary system and not necessarily complete clearance of the bile duct.
During emergency ERCP for patients with underlying bile duct stones, selective bile duct can-
nulation may be easy because of the patulous papilla as a result of previous stone passage. Minimal
contrast should be injected into the bile duct to define the level of obstruction and detect any
ductal stones. It is important to avoid a full cholangiogram or overfilling of the intrahepatic ducts
because this will increase the intrabiliary pressure and precipitate or worsen the sepsis. Urgent
decompression should be performed if the diagnostic catheter or papillotome can be inserted
beyond the level of the obstruction with the help of a guidewire. Active suction of the infected
bile using a 20-mL syringe connected to the catheter or papillotome should be performed (espe-
cially in very ill patients) to decompress the biliary system. Subsequent exchange of the catheter
can be performed over an indwelling guidewire for either a nasobiliary catheter or a biliary stent
to provide biliary drainage. We prefer to use a 10-Fr straight stent (eg, Cotton Leung stent [Cook
Endoscopy]) because it provides better drainage than the smaller 7- or 8.5-Fr stent, especially
How Would You Manage the Patient With Abdominal Guarding and Jaundice?   165

Figure 35-1. Ten-Fr stent and large CBD stone


in dilated bile duct.

when the bile is infected (Figure 35-1). The straight stent is also better than double pigtail stents
because of the larger proximal side holes. In general, an 8-cm 10-Fr stent will suffice for biliary
drainage in case of CBD stone obstruction (Figure 35-2). Bile should be taken for culture to isolate
the bacteria causing the infection and to monitor the antibiotic sensitivity.
166  Question 35

Figure 35-2. (A) Ten-Fr


stent draining pus from
obstructed biliary system.
(B) Infected bile aspirated
from obstructed biliary sys-
tem.

A B

An impacted stone may sometimes be seen at the papilla, causing a bulging papilla and
obstruction. The impacted stone may prevent deep cannulation with either the catheter or papillo-
tome. In cases where the stone is seen emerging from the papilla, we have successfully removed the
impacted stone using a polypectomy snare by ensnaring the bulging papilla beyond the impacted/
obstructing stone. The snare is closed beyond the stone and gentle tugging of the snare will dis-
lodge the stone from the papilla (Figure 35-3).4 Subsequent cholangiogram should be performed,
and we recommend placement of a biliary stent to insure drainage.
In some cases, the impacted ampullary stone prevents successful or deep CBD cannulation.
In such cases, it may be necessary to perform a needle knife precut papillotomy.5 The impacted
stone causes stretching of the papilla and distal CBD and protects the pancreatic orifice. The
stone serves as a “chopping board” and allows the precut papillotomy to be performed safely. The
precut papillotomy can be completed using a standard papillotome when deep cannulation is suc-
cessful. Alternatively, the precut papillotomy can be further extended using the needle knife until
the impacted stone dislodges spontaneously. In most cases, deep cannulation is successful and
the papillotomy can be extended and completed using a standard papillotome. We recommend
insertion of an indwelling biliary stent to insure adequate decompression of the biliary system to
prevent cholangitis.
With successful biliary drainage, biliary infection will respond to antibiotic therapy. The clini-
cal condition of the patient often improves with hemodynamic stability and resolution of the fever.
We recommend completing a full course of antibiotics for at least 1 week, or for 3 to 5 days after
the fever has subsided. A repeat ERCP is performed 2 to 4 weeks after the patient improves clini-
cally to remove the biliary stent, and subsequent papillotomy is performed (if necessary) to remove
any residual CBD stone.
Subsequent management after complete CBD clearance of stones will depend on the general
condition of the patient and the presence or absence of stones in the gallbladder. We may rec-
ommend a subsequent elective cholecystectomy if the gallbladder contains stones. In high-risk
patients or those without gallstones, the gallbladder can be left alone if complete clearance of the
biliary system is achieved.
How Would You Manage the Patient With Abdominal Guarding and Jaundice?   167

A B

C D
Figure 35-3. (A) Impacted stone at the ampulla. (B) Polypectomy snare applied around the impacted
stone and bulging papilla. (C) Impacted stone removed with snare. (D) Drainage of dark bile after stone
disimpaction.

References
1. JW Leung, Ling TK, Chan RC, et al. Antibiotics, biliary sepsis and bile duct stones. Gastrointest Endosc.
1994;40(6):716-721.
2. Leung JW, Chung SC, Sung J, Banez VP, Li AK. Urgent endoscopic drainage for acute suppurative cholangitis.
Lancet. 1989;i(8650):1307-1309.
3. Leung JW, Cotton PB. Endoscopic nasobiliary catheter drainage in biliary and pancreatic disease. Am J
Gastroenterol. 1990;86(4):389-394.
4. Pan YL, Ngo C, Yen D, Leung JW. A novel method of endoscopic removal of an impacted ampullary stone using a
snare (case report with video). J Interv Gastroenterol. 2011;1(4):177-178.
5. Leung JW, Banez VP, Chung SCS. Precut (needle knife) papillotomy for impacted common bile duct stone at the
ampulla. Am J Gastroenterol. 1990;85(8):991-993.
36 QUESTION

MY 68-YEAR-OLD PATIENT WITH MS PRESENTS WITH DYSPEPTIC


SYMPTOMS AND LOOSE STOOLS. IMAGING SHOWED A DILATED
PANCREATIC DUCT AND EXTENSIVE CALCIFICATION IN THE HEAD OF THE
PANCREAS. SYMPTOMS RESPONDED PARTLY TO PANCREATIC ENZYME
SUPPLEMENT. WHAT SHOULD I DO NEXT?

Mohan Ramchandani, MD, DM and


D. Nageshwar Reddy, MD, DM, FRCP, DSc

Chronic pancreatitis (CP) is a progressive inflammatory disease of the pancreas characterized


by irreversible morphological changes that typically cause pain and damage to the endocrine
and exocrine pancreatic tissues. Clinically, patients with CP present with abdominal pain in the
early stage and with diabetes and maldigestion in the late stage due to endocrine and exocrine
insufficiency, respectively. The diagnosis of CP can be made on the basis of clinical symptoms in
combination with structural and functional criteria. This patient can be easily diagnosed as having
CP because he has dilated pancreatic duct with calcification in the head of the pancreas. In some
patients, the diagnosis of CP can be very difficult to make because structural changes may take
years to develop, and functional tests can be normal or not easily accessible.
Patients with CP are generally classified as having either large-duct or small-duct disease.
Large-duct disease implies substantial abnormalities of the pancreatic duct and/or gland, includ-
ing main pancreatic duct dilation, pancreatic atrophy, or diffuse calcifications that are visible on
routinely performed imaging studies. Small-duct disease implies the absence of these findings.
This distinction has both diagnostic and therapeutic implications. Components essential to the
optimal management of chronic pancreatitis are control of pain, improvement of maldigestion,
management of diabetes, of complications such as cysts or strictures, and alcohol and nicotine
abstinence. Indications of treatment in patients with CP are directed by their symptoms. Patients
with pain are evaluated for structural abnormalities (eg, pancreatic ductal strictures, calculi) that
can be treated endoscopically or surgically. Conservative treatment of pain includes fat-reduced
diet, analgesics, and alcohol and smoking cessation.
This patient with CP is symptomatic for diarrhea and dyspepsia. Common causes of diarrhea
in chronic pancreatitis are pancreatic insufficiency, small-intestine bacterial overgrowth, altered
gut motility related to CP, diabetes mellitus associated with CP in later stages, and neuropathy.

Leung J, Lo SK, eds. Curbside Consultation in Endoscopy:


169 49 Clinical Questions, Second Edition (pp 169-174)
© 2014 SLACK Incorporated
170  Question 36

Exocrine pancreatic insufficiency with maldigestion is a major consequence of chronic pancre-


atitis. Fifty percent of patients with CP develop exocrine pancreatic insufficiency at a median of
10 to 12 years from onset of the disease. The clinical manifestations of exocrine pancreatic insuf-
ficiency are fat malabsorption and steatorrhea. Fat malabsorption is the main cause of weight loss
in patients with CP. It also leads to deficiency of micronutrients and fat-soluble vitamins. Analysis
of stool may be considered in this patient to document the presence of steatorrhea. A 72-hour
stool collection for fat is the gold standard to detect steatorrhea. However, this method is too
cumbersome and unpleasant to be applied widely in clinical practice. Others tests that can be used
in clinical settings are 13C-MTG breath test and qualitative stool stains using a Sudan III stain.
Patients with pancreatic insufficiency are managed medically. Ductal decompression by
removal of stones or treating strictures either endoscopically or surgically have not been shown
to improve the exocrine pancreatic insufficiency. Complete abstinence from alcohol and tobacco
should be encouraged in all patients with chronic pancreatitis. Abstinence from alcohol is asso-
ciated with pain relief in some patients and may also increase gastric lipase secretion. Dietary
modifications are not recommended for fat malabsorption. Medium-chain triglycerides that are
directly absorbed by the intestinal mucosa can be useful in patients with weight loss and in patients
with a poor response to oral pancreatic enzymes. It is generally accepted that patients presenting
with weight loss and those with symptoms of steatorrhea should be treated with pancreatic enzyme
supplementation. The indication of treatment in asymptomatic patients with pancreatic insuffi-
ciency is controversial. Development of fat-soluble vitamins and micronutrient deficiency has been
observed in patients with asymptomatic pancreatic insufficiency. To prevent this, some authors
suggest treatment with pancreatic enzymes in every patient with demonstrated exocrine pancre-
atic insufficiency and steatorrhea, regardless of the presence or absence of symptoms. Efficacy
and safety of pancreatic enzyme supplementation in chronic pancreatitis patients with steatorrhea
has been well studied. Enzyme supplementation improves the coefficient of fat absorption, but
fat malabsorption is not completely abolished. Stool frequency and consistency also improve with
supplementation, but it is unclear whether it is associated with weight gain.
The first step in the treatment of our patient with pancreatic insufficiency is to ensure compli-
ance of adequate enzyme intake during meals (Figure 36-1). The dosage of the enzymes should
be properly timed and divided. One-quarter of the dose should be taken at the beginning of the
meal, half of the dose during the meal, and the remaining quarter at the end of the meal. To
improve malabsorption, the minimum total amount of enzymes that should be postprandially
delivered into the duodenum is 5% to 10% of normal enzyme outputs. This quantity is approxi-
mately 30,000 IU lipase per meal, or 90,000 United States Pharmacopoeia (USP) units. This is
the ideal amount required when there is no inactivation of orally ingested enzymes by gastric acid.
The prescribed dose of pancreatic enzymes should be 20,000 to 40,000 IU of lipase per meal.
However, doses up to 80,000 U of lipase per meal may be required in some cases. At present, there
are 3 types of pancreatic enzymes available: nonenteric-coated, enteric-coated tablets, and micro-
encapsulated enteric-coated preparations. Choice of these enzymes is also important. Nonenteric-
coated enzyme preparations are easily destroyed by gastric acid, hence they are not recommended.
However, these may be used in patients with achlorhydria secondary to atrophic gastritis or post-
gastric surgery. The enteric-coated preparations have been developed to circumvent the problem
of acid inactivation of enzymatic activity during gastroduodenal transit. Enteric-coated enzyme
preparations do not disintegrate or release enzymes until the pH is more than 5.5 to 6.0, which is
assumed to be the pH within the duodenum in patients with exocrine insufficiency. Theoretically,
the preparations should traverse the stomach without being inactivated by acid so to liberate
their enzymatic activity within the duodenum and improve the fat malabsorption. The problem
with enteric-coated enzymes is that they may not release enzymes in the duodenum. Because
abnormally low pancreatic secretion of bicarbonate occurs in cases of chronic pancreatitis, small-
bowel pH may not reach above 5, which is required for release of active lipase from enteric-coated
What Should I Do After Patient Responds to Pancreatic Enzymes Supplement?  171

Figure 36-1. Algorithm for pancreatic enzyme sub-


stitution therapy in patients with pancreatic exo-
crine insufficiency. ECMS indicates enteric-coated
microspheres.

preparations, and the enzymes may release in the ileum. Enteric-coated preparations may not be
simultaneously released from the stomach along with meals. This problem has been partly solved
by the development of 1- to 2-mm enteric-coated microspheres (ECMS), which empty from the
stomach with meals. In addition, these preparations include very low amounts of chymotrypsin to
avoid proteolytic inactivation of lipase. We prefer ECMS in patients with pancreatic insufficiency.
Gastric acid secretion may prevent release of active lipase from enteric-coated granules within
proximal segments of the small intestine. In patients who do not have a satisfactory response to
enzyme therapy, adjunctive acid-suppression therapy should be considered. Inhibition of gastric
acid secretion enhances the effect of orally ingested enzymes, either by avoiding acid-mediated
lipase inactivation or by facilitating enzyme release from the microspheres within the proximal
intestine. For these adjunctive therapies to be successful, postprandial gastric and duodenal pH
must be maintained above 4.0 for at least 60 and 90 minutes, respectively. This pH can be obtained
in most patients by the administration of the standard dose of a proton-pump inhibitor (PPI)
once daily, at least 30 minutes before breakfast. In some cases, double-dose PPI may be needed.
Acid-suppression therapy may also reduce steatorrhea because it corrects bile acid malabsorption
by preventing bile acid precipitation.
If all of the above fail, other digestive conditions that may interfere with intestinal absorption
should be considered, including small-bowel bacterial overgrowth, giardiasis, celiac disease, and
blind loop syndrome after gut surgery. A complete medical assessment is needed in the absence
of clinical improvement.
Although our patient doesn’t have significant abdominal pain, this remains the most common
and the most distressing symptom associated with CP. Relief of pain is the most important goal
of therapy in patients with CP. The initial step of medical therapy usually is supportive treatment.
Analgesic drugs are still the most commonly adopted method for pain relief. Patients with mild to
moderate pain are treated with nonopioid analgesics such as acetaminophen or NSAIDs, whereas
patients with severe pain may require opioids. There may be significant psychiatric, psychologi-
cal, or psychosomatic contributions to the pain syndrome in these patients. Many physicians use
172  Question 36

A B
Figure 36-2. (A) Pancreatogram showing a patient with pancreatic ductal
calculus with upstream dilation of PD. (B) Following ESWL, the calculus was
fragmented and pancreatic duct was cleared.

antidepressant medications as concomitant therapy. As a general rule, additional options should


be considered in patients whose pain significantly reduces their quality of life. Other drugs that
have been shown to be of benefit in relieving pain in these patients are antioxidants and pancreatic
enzymes.
Endoscopy and surgery to provide pancreatic drainage are considered the treatment modalities
of choice in the case of painful obstructive pancreatitis. The aim of both modalities is to reduce
the pancreatic ductal pressure and ensure adequate drainage of pancreatic excretions. Surgical
decompression of the pancreatic duct (PD) with clearance of calculi leads to relief of pain in most
patients. Ductal decompression can also be achieved by endoscopic techniques. Among the avail-
able endoscopic modalities, endoscopic retrograde cholangiopancreatography (ERCP) is used to
treat pancreatic strictures, pancreatic ductal stones, pseudocysts, PD fistulas/leaks, and bile duct
strictures. Small pancreatic ductal calculi can be extracted using a basket after an endoscopic
pancreatic sphincterotomy. Approximately 10% to 25% of pancreatic stones can be removed effec-
tively by standard techniques with a balloon and/or basket. Endoscopic removal of pancreatic
stones is more difficult because pancreatic stones tend to be multiple, hard, and spiculed. They
are also usually stuck to the ductal epithelium or impacted behind strictures. Standard technique
may not be successful for removal of large stones in the main pancreatic duct. The problem can
be overcome by fragmenting the calculi using lithotripsy. There are various lithotripsy techniques
available, including mechanical lithotripsy, electrohydraulic lithotripsy, and extracorporeal shock
wave lithotripsy (ESWL). ESWL (Figure 36-2) is the most commonly used technique for pulver-
izing pancreatic stones. We perform ESWL using a third-generation electromagnetic lithotripter
(Delta Compact, Dornier Med Tech). This has both bidimensional fluoroscopic and ultrasound
targeting facility. All radio-opaque stones are targeted fluoroscopically. Contrast is injected
through the nasopancreatic tube to target the radiolucent calculi. Shockwaves up to a maximum
of 5000 shocks are delivered per sitting. An intensity of 5 to 6 (15,000 to 16,000 kv) on a scale
of 1 to 6 is used with a frequency of 90 shocks/min during the procedure. The time taken for
each session is 60 to 90 minutes. Most procedures are performed under epidural anesthesia (EA).
What Should I Do After Patient Responds to Pancreatic Enzymes Supplement?  173

A B
Figure 36-3. A patient with chronic pancreatitis, (A) pancreatogram showing
a PD stricture (arrow), and (B) PD stent inserted into pancreas bypassing the
stricture with the pigtail end of the stent in the duodenum.

General anesthesia or total intravenous anesthesia is used when EA is contraindicated. The most
suitable patients for endotherapy of stones in chronic pancreatitis are those who have single or
multiple stones located in the main duct or side branches and who do not have any associated
ductal strictures and stones confined to the head and/or body of the pancreas. In such patients,
the success of complete ductal clearance with endotherapy using ESWL can reach 90%. Patients
with stones and downstream strictures, patients with multiple strictures, patients who have stones
localized in the pancreatic tail and proximal strictures, and patients with associated inflammatory
pancreatic head masses or complex ductal disease are not suitable candidates for endotherapy. In
these patients, endotherapy is associated with complications and the number of ESWL and ERCP
sessions required is much higher.
Pancreatic duct strictures can be managed endoscopically. It is important to note the location,
number, and length of the strictures, and the presence of upstream dilation. PD strictures in the
tail of the pancreas and multiple strictures along the length of the main PD are more difficult
to manage by endotherapy. The best outcomes of endotherapy are achieved in patients with focal
head/body strictures with upstream dilation. Patients with complex strictures and associated
stones, pseudocysts, inflammatory mass of the head, or diffuse ductal changes are poor candidates
for endotherapy. The technique for pancreatic stenting is the same as that of biliary stenting. After
pancreatic duct cannulation, a guidewire is maneuvered upstream to the narrowing. Pancreatic
sphincterotomy via the major or minor papilla is performed to facilitate placement of accessories
and stents. Stricture dilation with a graduated dilating catheter or balloon dilators should be con-
sidered in tight strictures. Finally, the PD stent is advanced over the guidewire across the stricture
using a pusher tube. Choice of the PD stent depends on the size of the PD downstream from
the stricture (toward the duodenum). In smaller ducts, 4- to 7-Fr stents are used, whereas 10- to
11.5-Fr stents are used in markedly dilated PDs. In general, the diameter of the stent should
not exceed the downstream duct diameter. We prefer to use a pancreatic stent with an external
pigtail and an internal flange in order to prevent proximal and distal migration, respectively
(Figure 36-3). At our institute, we use the following protocol to manage PD strictures. After ini-
tial placement of the PD stent, the stent is removed after 3 months; if a patient is symptomatic, it
is removed earlier. A check pancreatogram is done to assess the stricture. If stricture persists, bal-
loon dilation of the stricture is performed, followed by insertion of the maximum number of stents
allowed by the stricture tightness and PD diameter. These stents are replaced every 3 months for
174  Question 36

A B C
Figure 36-4. (A) A patient with refractory PD stricture. (B) Fully covered SEMS was placed across the
stricture. (C) The SEMS was removed after 3 months, pancreatogram showing complete resolution of PD
stricture.

1 year. Recently, there have been reports of temporary placement of fully covered self-expandable
metal stents (SEMS) in patients with refractory PD strictures (Figure 36-4). Long-term results
are awaited.
We recommend a stepwise approach to manage pain in patients with chronic pancreatitis.
Patients who are symptomatic should be initially managed medically, and patients who have
appropriate morphologic features should be considered for endotherapy early in disease course.
The subset of patients with higher stone load, complex pancreatic ductal anatomy, or mass lesion
should be recommended for surgery immediately. Overall, the treatment of patients with chronic
pancreatitis should be tailored to the individual patient, and an interdisciplinary approach is the
key to success in management of this complex disease.
This patient has a case of chronic obstructive calcific pancreatitis with multiple sclerosis. His
symptoms of dyspepsia and loose stools have partially responded to pancreatic enzymes. This
patient can be managed by stepping up the dose of pancreatic enzymes or adding PPI. Because the
patient does not have significant pain, he is not a candidate for pancreatic endotherapy or surgery.

Bibliography
Adler DG, Lichtenstein D, Baron TH, et al. The role of endoscopy in patients with chronic pancreatitis. Gastrointest
Endosc. 2006;63:933-937.
Bruno MJ, Haverkort EB, Tytgat GN, van Leeuwen DJ. Maldigestion associated with exocrine pancreatic insufficiency:
implications of gastrointestinal physiology and properties of enzyme preparations for a cause-related and
patient-tailored treatment. Am J Gastroenterol. 1995;90(9):1383-1393.
Costamagna G, Bulajic M, Tringali A, et al. Multiple stenting of refractory pancreatic duct strictures in severe chronic
pancreatitis: long-term results. Endoscopy. 2006;38:254-259.
Domínguez-Muñoz JE. Pancreatic exocrine insufficiency: diagnosis and treatment. J Gastroenterol Hepatol. 2011;26
(Suppl 2):12-16.
37 QUESTION

A 12-YEAR-OLD PATIENT HAS A 5-YEAR HISTORY OF RECURRENT


ACUTE PANCREATITIS, MANAGED CONSERVATIVELY WITH PANCREATIC
ENZYMES SUPPLEMENT. CT REVEALS MILDLY DILATED PANCREATIC DUCT
WITH A POSSIBLE STONE IN THE HEAD OF THE PANCREAS.
WHAT ELSE CAN I DO?

Joseph Leung, MD, FRCP, FACP, FASGE, MACG

The patient has had attacks of pancreatitis since a relatively young age and we have to consider
different etiological factors instead of the usual common causes of pancreatitis.1 That imaging
suspicion of a stone in the pancreatic duct (PD) would suggest underlying chronic pancreatitis
secondary to intrinsic pathology in the pancreas, affecting either the secretion or drainage of pan-
creatic juice. Such conditions include congenital ductal anomalies (eg, pancreas divisum) or cystic
fibrosis with predisposition to thick pancreatic secretions and precipitation of calcium carbonate
(stones) in the PD.2 Although it is possible that the patient may have the usual biliary pancreatitis,
imaging did not suggest gallbladder stones. However, an underlying congenital choledochal cyst
can predispose to common bile duct (CBD) stones, which are often calcium carbonate instead
of calcium bilirubinate or cholesterol stones because of reflux of pancreatic juice into the dilated
CBD. The presence of a long common channel between the distal bile duct and PD could lead
to pancreatic outflow tract obstruction, resulting in acute pancreatitis. The calcium carbonate
precipitates (stones) are relatively soft and amorphous in character when first formed. However,
with time, they can become calcified, especially those that are present in the PD, which makes
subsequent removal more difficult.
Patients with pancreas divisum can develop abdominal pain and acute pancreatitis secondary to
a relative outflow obstruction resulting from the dorsal duct that drains the bulk of the pancreas
through the small minor papilla. I have observed among adult patients another cause of pancreatic
outflow obstruction due to the congenital absence or noncommunication of the Santorini’s duct
with the minor papilla. The absent overflow valve may result in increased pressure within the PD,
causing pancreatic pain or pancreatitis.3

Leung J, Lo SK, eds. Curbside Consultation in Endoscopy:


175 49 Clinical Questions, Second Edition (pp 175-177)
© 2014 SLACK Incorporated
176  Question 37

Investigation of the patient should include noninvasive imaging, such as magnetic resonance
cholangiopancreatography (MRCP) and endoscopic ultrasound (EUS) examination. Endoscopic
retrograde cholangiopancreatography (ERCP) remains the gold standard for the treatment of
underlying PD stones if confirmed on imaging. Depending on the underlying pathologies, ERCP
can demonstrate the characteristic appearance of a long common channel and a dilated bile duct
associated with choledochal cyst, but as mentioned before, the stones are more likely to be pres-
ent in the dilated bile duct and not in the pancreas. Patients with underlying cystic fibrosis can
develop PD stones, which are more likely to present earlier in life if they have coexisting congenital
anomalies such as pancreas divisum or absent or noncommunicating Santorini’s duct.
Initial treatment of this patient should include a low-fat diet, adequate pain control, and pan-
creatic enzymes supplement with meals, which helps to reduce pancreatic secretion and minimize
pancreatic ductal hypertension to reduce pancreatic pain and prevent further attacks of pancre-
atitis. Laboratory testing should include checking for the CFTR gene or the variant, as well as
genetic counseling if the patient is confirmed to have cystic fibrosis.4 ERCP and endoscopic
removal of the PD stone is the first line of treatment. Special precautions should be taken when
performing ERCP in the pediatric age group, including discussion with the parents regarding
the risks and benefits, the immediate outcome, and the long-term prognosis, and management of
the underlying etiological condition. Except for very small size and very young patients, ERCP
can be accomplished using an adult side-viewing duodenoscope. Indeed, it is preferable to use the
therapeutic ERCP scope to accommodate the larger accessories that may be required if the stones
are calcified. Another precaution is to provide shielding of the patient from radiation exposure
to the reproductive organs. The ERCP is usually done under general anesthesia for the pediatric
age group.
The presence of stones and a dilated PD suggest that the stones might have been present
for some time. It is important to take a control film before injection of contrast to determine
whether there are obvious calcified stones that are more difficult to remove. Stone removal will
require pancreatic sphincterotomy and possible dilation (including use of dilation balloon or the
Soehendra stent retriever [Cook Endoscopy] for very tight stricture) of the relative PD stricture or
obstruction to engage the stone. Manipulation of a guidewire across the obstruction is necessary
to facilitate subsequent interventions, and we found that shaping the tip of a hydrophilic tip guide-
wire (eg, Metro Tracer wire [Cook Endoscopy]) or using a Terumo guidewire (Terumo Medical)
(to encourage looping of the guidewire at the tip) helps to negotiate the PD stricture. The size of
the dilation balloon should be adjusted to the diameter of the normal PD (usually downstream)
to avoid excessive trauma to the pancreatic duct and aggravating the pancreatitis. If the stones are
soft, they can be removed using a stone extraction balloon, but care should be taken to avoid over-
inflating the balloon in the PD, again to minimize trauma to the pancreas. If the stones are hard
or calcified, basket extraction may be necessary and caution is taken to assess if there is a signifi-
cant discrepancy between the size of the stone and the PD to avoid stone/basket impaction. Stones
that are located in the head of the pancreas are more likely to be removed successfully. Stones that
are located in the body and tail of the pancreas may be more difficult to remove because of the
discrepancy in stone size and the exit passage. In some cases, extracorporeal shockwave lithotripsy
may be required to fragment large calcified stones to facilitate their removal.
Depending on the nature and character of the PD stone, endoscopic removal may result in trau-
ma and bleeding; insertion of a 5-Fr plastic pancreatic stent is necessary to provide drainage and
to prevent post-ERCP pancreatitis. These stents often migrate spontaneously after 1 to 2 weeks,
and a follow-up radiograph is taken to confirm stent migration. If significant pancreatic ductal
stricture and dilation is present, a larger (7 or 10 Fr) indwelling stent is placed for a longer period
to keep the stricture open after dilation. A repeat ERCP may be necessary for removal of the stent.
Long-term regular follow-up is necessary. Concurrent management with the pediatric service
is often necessary to address issues related to the underlying condition (eg, cystic fibrosis). The
What Else Can I Do for a Patient With a Possible Stone in the Pancreas?   177

patient is advised to continue with pancreatic enzymes supplement with meals, although compli-
ance can be an issue. Appropriate pain control with medication is necessary, but we should try to
keep it to a minimum to avoid drug dependence. Follow-up assessment with repeat MRCP may be
necessary to monitor changes in the pancreas. We found that a repeat ERCP is necessary at yearly
intervals to insure no stone recurrence and, if necessary, to provide ductal clearance to prevent the
stones from becoming calcified.

References
1. Forbes A, Leung JWC, Cotton PB. Relapsing acute and chronic pancreatitis. Arch Dis Child. 1984;59:927-934.
2. Hsu RK, Draganov P, Leung JW, et al. Therapeutic ERCP in the management of pancreatitis in children. Gastrointest
Endosc. 2000;51(4):396-400.
3. Foster EN, Leung JW. Variants of the Santorini duct: a cause of pancreatitis? Pancreas. 2007;35(1):103-105.
4. Lucidi V, Alghisi F, Dall’Oglio L, et al. The etiology of acute recurrent pancreatitis in children: a challenge for pedia-
tricians. Pancreas. 2011;40(4):517-521.
38 QUESTION

A 25-YEAR-OLD FEMALE PRESENTS WITH RECURRENT ABDOMINAL PAIN


AND MILDLY ELEVATED SERUM AMYLASE AND LIPASE LEVELS, NORMAL
LFTS. ABDOMINAL IMAGING SHOWS NORMAL SIZE CBD, PD, AND NO
OBVIOUS STONES IN THE GALLBLADDER. WHAT SHOULD I DO NEXT?

James J. Farrell, MD and Harry Aslanian, MD

Although mild elevations of amylase and lipase may be due to nonpancreatitic causes such as
disorders that inflame the small intestine, including celiac sprue or Crohn’s disease (which could
also account for the patient’s pain), elevations of serum amylase and lipase greater than twice
normal are more specific for acute pancreatitis. The initial workup for a young woman with recur-
rent acute pancreatitis should be focused on identifying the etiology of the acute pancreatitis so
that future episodes could potentially be prevented.1 Alcohol intake and gallstones account for
approximately 80% of cases. Other etiologies include pancreas divisum, metabolic disorders, medi-
cations, autoimmune pancreatitis, sphincter of Oddi dysfunction, hereditary pancreatitis, pancreas
malignancy, and cystic neoplasms. Clarification of the duration and frequency of symptoms is
important, as often young patients will often recount symptoms dating back to childhood in the
absence of documented pancreatitis.
It is crucial to review the history and workup to date for evidence of the 2 most common etiolo-
gies for recurrent pancreatitis: alcohol and gallstones. Patients often underestimate their current
alcohol intake and dismiss episodes of excess alcohol intake (often associated with episodes of
abdominal pain) in the remote past, which they do not attribute to their current clinical symptoms.
A review of the prior imaging to evaluate for gallstones and the assessment of the liver function
tests (LFTs) at the initial presentation of pain episodes is important. Gallstones are best seen
on transabdominal ultrasound and may be missed on computed tomography (CT) or magnetic
resonance imaging (MRI) scans. Retained common bile duct (CBD) stones may be missed by
ultrasound or CT scan approximately half of the time and are best seen on magnetic resonance
cholangiopancreatography (MRCP) or endoscopic ultrasound (EUS). Repeat performance of an
abdominal ultrasound or performance of EUS should be considered based on the level of clinical
suspicion for gallstone disease. Normal LFTs make a gallstone etiology less likely, but not impos-
sible.

Leung J, Lo SK, eds. Curbside Consultation in Endoscopy:


179 49 Clinical Questions, Second Edition (pp 179-181)
© 2014 SLACK Incorporated
180  Question 38

When alcohol intake and gallstones have been convincingly ruled out, attention should turn to
assessing the patient for treatable metabolic causes (eg, hypertriglyceridemia or hypercalcemia). It
is important to document the triglyceride levels in between attacks of pancreatitis because elevated
triglycerides are often seen in the setting of acute pancreatitis. A careful medication history,
including the timing of the initiation of medication, is important. The oral contraceptive pill may
be a cause of pancreatitis; however, it is often very difficult to confirm a particular medication as a
definite cause of recurrent pancreatitis without withdrawing and rechallenging the patient. Often
finding a suitable alternative when one is suspicious may be sufficient.
Pancreas divisum is an important (although somewhat controversial) cause of recurrent acute
pancreatitis in the young patient.2 Although it is present in approximately 7% to 10% of the popu-
lation, only approximately 5% of patients develop symptoms. Hence, the presence of pancreas
divisum does not automatically exclude a search for other, more common, causes for recurrent
pancreatitis. A good quality MRI and MRCP (possibly with secretin infusion) should be suffi-
cient for the diagnosis of pancreas divisum, with ERCP reserved for unclear diagnoses and when
therapy (ie, a minor papilla sphincterotomy) is contemplated.
When to perform a minor duct sphincterotomy in this setting is controversial. The risks of the
procedure, including acute pancreatitis and a potential lack of impact on natural history, need to
be balanced with the evidence supporting the pancreas divisum as the cause for the patient’s recur-
rent pancreatitis. For example, it is likely that the chances of improving outcome by performing a
minor sphincteromy in the setting of established chronic pancreatitis are less than in the setting
of a normal pancreas.
Therefore, a search for a diagnosis of the underlying chronic pancreatitis is important in
patients with recurrent attacks of pancreatitis.3 Not only may it impact therapeutic decisions (eg,
minor sphincterotomy in the setting of pancreas divisum or performance of ERCP with sphincter
of Oddi manometry), it may represent a cause of recurrent attacks of abdominal pain with eleva-
tions in amylase and lipase. A clinical history of weight loss, steatorrhea, chronic abdominal pain
that does not resolve between acute attacks, and biochemical evidence (elevated glucose, elevated
hemoglobin A1c) of pancreatic exocrine or endocrine dysfunction should be sought. Although
noninvasive imaging such as CT or MRI can suggest chronic pancreatitis, there is a strong role for
EUS evaluation, which has increased sensitivity but decreased specificity for diagnosing chronic
pancreatitis.
The role and timing of sphincter of Oddi dysfunction evaluation, including possible biliary
and/or pancreatic duct sphincterotomy, also remains unclear in this scenario. The risks of pan-
creatitis, stricture development, and no guarantee of changing the natural history of this disease
need to be considered. Consideration for elective cholecystectomy for occult gallstones disease is
often advocated as a safer initial potential therapeutic alternative after repeated episodes of recur-
rent abdominal pain.
EUS has become an important test in patients with recurrent abdominal pain because it accu-
rately assesses for the most common anatomic abnormalities (gallstones or sludge, bile duct stone,
pancreas divisum, pancreas mass or cyst, chronic pancreatitis, ampullary adenoma), with low risk
and no risk of acute pancreatitis (as associated with ERCP).
Autoimmune pancreatitis (AIP) is a rare type of chronic pancreatitis that can present with
recurrent abdominal pain and mild elevations in amylase and lipase.4 AIP is treatable with steroids
and is important to consider when searching for a diagnosis. Historical and laboratory evidence
of associated autoimmune diseases can support a diagnosis, and elevated immunoglobulin, espe-
cially IgG4, may be useful, but does not need to be present to make a diagnosis. The presence of
a diffusely swollen pancreas with peripheral enhancement on CT can also be helpful. Ultimately,
EUS diagnosis, including a core biopsy, can be helpful. Occasionally, an empiric trial of steroids
for a patient with a suspicion of having AIP but not having all the absolute criteria can be helpful.
What Should I Do Next for a Patient With Pain and no Stones in Gallbladder?   181

Finally, the role of genetic testing for hereditary pancreatitis in this clinical scenario remains
controversial.5 In the absence of a strong family history of hereditary pancreatitis, the finding of
a genetic mutation may function as a cofactor in the development of pancreatitis and may provide
some explanation to the patient as to why she is having recurrent attacks of pancreatitis. However,
at present, no intervention is available to the patient with hereditary pancreatitis. If a definite diag-
nosis of hereditary pancreatitis is made, then stratification for the risk of developing pancreatic
malignancy may also be performed.

References
1. Pandol SJ. Acute pancreatitis. Curr Opin Gastroenterol. 2006;22:481-486.
2. DiMagno MJ, Wamsteker EJ. Pancreas divisum. Curr Gastroenterol Rep. 2011;13:150-156.
3. Etemad B, Whitcomb DC. Chronic pancreatitis: diagnosis, classification, and new genetic developments.
Gastroenterology. 2001;120:682-707.
4. Sah RP, Chari ST. Autoimmune pancreatitis: an update on classification, diagnosis, natural history and manage-
ment. Curr Gastroenterol Rep. 2012;14:95-105.
5. LaRusch J, Whitcomb DC. Genetics of pancreatitis. Curr Opin Gastroenterol. 2011;27:467-474.
39 QUESTION

WHAT IS THE BEST TREATMENT FOR


DUODENAL OR AMPULLARY ADENOMA, AND
WHAT IS THE CURRENT RECOMMENDATION FOR
SURVEILLANCE AFTER TREATMENT?

Richard A. Kozarek, MD

Ampullary tumors historically presented late, with jaundice, bleeding, relapsing pancreatitis,
or duodenal obstruction, and were treated with radical pancreaticoduodenectomy and, less com-
monly, transduodenal resection and dual sphincteroplasty.1 Endoscopy in this setting was limited
diagnostically to tissue acquisition and therapeutically to biliary stent placement or sphincterotomy
in high surgical risk patients with obstructive jaundice. Currently, the majority of adenomas are
found incidentally during screening for reflux or dyspepsia or follow-up of C-loop adenomas in
patients with Gardner’s syndrome.
Earlier diagnosis allows the potential for endoscopic as opposed to surgical resection, but nei-
ther endoscopic papillectomy nor local tumor ablation should be considered in isolation.1-3 If found
incidentally, multiple biopsies should be performed to rule out malignancy or a tumor with unusu-
al histology such as carcinoid (Figure 39-1). Symptomatic patients, in turn, invariably require
an abdominal computed tomography (CT) scan and measurement of tumor markers to include
CEA and CA 19-9. Assuming that CT and biopsy fail to demonstrate malignancy, there have
been a number of attempts to define endoscopic nonresectability. The latter include extremely
large (4- to 5-cm) lesions, an ulcerated papilla, lesions that extend laterally and encompass greater
than 30% to 50% of the duodenal lumen, and a hard lesion that fails to elevate with a submucosal
injection.2 Likewise, polyps that extend a significant length (0.5 to 1.0 cm) into the bile duct or
pancreas as demonstrated by endoscopic retrograde cholangiopancreatography (ERCP) or mag-
netic resonance cholangiopancreatography (MRCP) are considered by many to be unresectable
endoscopically (Figure 39-2). The role that endoscopic ultrasound (EUS) or intraductal ultrasound
(IDUS) plays in patients with ampullary adenomas is controversial.4 Some endoscopists believe
that EUS should be undertaken in all patients with ampullary adenomas to preclude the risk of
Leung J, Lo SK, eds. Curbside Consultation in Endoscopy:
183 49 Clinical Questions, Second Edition (pp 183-187)
© 2014 SLACK Incorporated
184  Question 39

A B
Figure 39-1. (A) Endoscopy demonstrates a large ampullary adenoma (B) with fistula from tumor into
bile duct (arrow).

Figure 39-2. Arrow delineates adenoma and


copious air in biliary tree.

resection in patients with invasive malignancy. I use EUS selectively and reserve it for patients in
whom pancreas protocol CT fails to show malignancy but in whom local infiltration or the ques-
tion of endoscopic resectability remains uncertain.

Technique
Technically, with the exception of asymptomatic and nonenlarging microadenomas of the
papilla of Vater (which are virtually ubiquitous in Gardner’s patients), ampullary adenomas should
be treated. As noted previously, historical treatment was usually done surgically, either with trans-
duodenal papillectomy with concomitant sphincteroplasty or with a Whipple procedure.
What Is the Best Treatment for Duodenal or Ampullary Adenoma?   185

Endoscopic treatment of ampullary adenomas was initially limited to direct thermal ablation,
most commonly with a Nd-YAG laser, but now is more commonly performed with argon plasma
coagulation (APC). 2 Both treatment modalities have been associated with edematous obstruction
of the pancreaticobiliary outlet with resultant jaundice, cholangitis, or obstructive pancreatitis. As
a consequence, these modalities are currently used primarily to treat residual adenomatous tissue
at the edge of a papillectomy site. Snare papillectomy, in turn, can be done in a piecemeal fashion
for lesions less than 2 to 3 cm but can usually be done with a single resection using a blended
current. I do an ERCP in all patients prior to resection, to assure that there is no tumor growth
into the pancreaticobiliary (PB) tree (see Figure 39-2), as well as to define landmarks to facilitate
postpapillectomy PB drainage.2
Some endoscopists utilize submucosal papilla injection to buffer the duodenal wall prior to
resection, as well as to define a non-“lifting sign” in the setting of infiltrating neoplasm. I do not.
Such injections often blur tissue planes and require considerably more electrical current to facilitate
transection of the papilla.
Following baseline ERCP and papillectomy, I undertake a biliary sphincterotomy, using a
blended or pulsed current, and pancreatic sphincterotomy with pure cut current (Figure 39-3),
placing stents in both ducts to protect against edematous obstruction of the duct orifices
(Figure 39-4). APC treatment of the edges or small bleeding areas is sometimes required. Data
from a multicenter, retrospective series by Catalano et al 5 have noted a 17% incidence of pan-
creatitis in patients in whom no stent was inserted vs 3.3% of patients undergoing prophylactic
pancreatic duct stenting. Moreover, subsequent stenosis of the PD sphincterotomy fell from 18.4%
of patients to 1.1% in patients who had prophylactic stenting. Although early and late biliary com-
plications were comparable between patients who did or did not have biliary stenting, early bouts
of cholangitis and cholestasis have changed my practice patterns to favor prosthesis placement in
most patients.

Results
There are now numerous series that suggest that thermal ablation of the papilla is associated
with a higher side-effect profile, less effective eradication of the ampullary adenoma, and need
for significantly more interventions when compared with papillectomy. Moreover, there appears
to be a 3- to 4-fold increased risk for ultimate development of malignancy when compared with
papillectomy. As such, thermal ablation should play a "bit" part in the treatment of ampullary
adenomas, usually to “touch up” the margins of a resection site or to treat the adenomatous tissue
(micro-adenomas) at the papillectomy site that frequently occur in Gardner’s syndrome.
Han and Kim6 have published a wonderful review of papillectomy. Their review included
800 cases published as case reports, abstracts, and clinical series. Approximately one-quarter of
patients had Gardner’s syndrome and three-quarters had sporadic adenomas. Patients presented
with jaundice, cholangitis, pancreatitis, weight loss, or chronic cholangitis; or more commonly,
were found during screening endoscopies. Lesions larger than 3 cm, as well as patients with
Gardner’s adenomas, were less likely to have long-term successful treatment, although 80% of
total patients had a long-term cure. Up to 10% of patients in some of the series were found to have
underlying malignancy, requiring definitive surgery. Procedural complications approximated 10%,
most commonly bleeding or pancreatitis, and the perforation rate ranged between 0% and 3%.
There was a single procedural mortality in this composite of patient series.
186  Question 39

A B

C D
Figure 39-3. (A) Polypectomy snare applied around the papillary tumor. (B) Cautery is applied after
closing the snare around the papillary tumor to perform papillectomy. (C) Retrieved resected specimen
measuring about 2 cm diameter. (D) Biliary sphincterotomy performed after ampullectomy to ensure
bile duct drainage.

Follow-Up
Not only do patients with ampullary adenomas resected endoscopically need stent retrieval and
papillectomy site inspection and biopsy at 4 to 6 weeks, they need long-term follow-up. Follow-up
includes a baseline colonoscopy to insure that there are not concomitant colon adenomas, as well
as evaluation of the ampullectomy site with a side-viewing scope. Contingent upon the patient
and the assumption of complete eradication of the ampullary adenoma, screening should occur at
least yearly for the first 3 years, and more frequently in patients with underlying genetic disorders.
In the review by Han and Kim,6 a 15% recurrence rate was noted. This rate is an estimate
at best, as “recurrences” between 1 and 3 months postresection are much more likely related to
What Is the Best Treatment For Duodenal or Ampullary Adenoma?   187

A B
Figure 39-4. (A) Following guidewire placement into the pancreatic duct, (B) stents were placed into
the PB tree.

incomplete resection than a true recurrence. Nevertheless, residual or recurrent disease is usually
readily handled endoscopically, and the recurrence rates are probably comparable with the 12.5%
rates reported in patients undergoing open surgical transduodenal papillectomy for ampullary
adenomas.
Although it is clear that endoscopic papillectomy has supplanted surgery in most patients with
amenable lesions, it is also clear that there are lesions that are better handled surgically.1,2 They
include ulcerated or infiltrated lesions, patients who are found to have underlying malignancy in
their resected specimens, Gardner’s patients in whom the papilla may be only one of hundreds of
large and enlarging C-loop adenomas, lesions extending a significant distance into the PB tree,
and those in whom the adenoma encompasses a considerable circumferential component of the
C-loop wall. Figures 39-1 through 39-4 depict papillectomy in a high-risk patient who had sig-
nificant dysplasia but no definite malignancy in his resected specimen.

References
1. Winter JM, Cameron JL, Olino K, et al. Clinicopathologic analysis of ampullary neoplasms in 450 patients: implica-
tions for surgical strategy and long-term prognosis. J Gastrointest Surg. 2010;14:379-387.
2. Irani S, Arai A, Ayub K, et al. Papillectomy for ampullary neoplasm: results of a single referral center from 1997-
2007. Gastrointest Endosc. 2009;70:923-932.
3. Patel R, Varadarajulu S, Wilcox CM. Endoscopic ampullectomy: techniques and outcomes. J Clin Gastroenterol.
2012;46:8-15.
4. Heinzow HS, Lenz P, Lallier S, et al. Ampulla of Vater tumors: impact of intraductal ultrasound and transpapillary
endoscopic biopsies on diagnostic accuracy and therapy. Acta Gastroenterol Belg. 2011;74:509-515.
5. Catalano MF, Linder JD, Chak A, et al. Endoscopic management of adenoma of the major duodenal papilla.
Gastrointest Endosc. 2004;59:225-232.
6. Han J, Kim MH. Endoscopic papillectomy for adenomas of the major duodenal papilla (with video). Review.
Gastrointest Endosc. 2006;63:292-301.
40 QUESTION

WHAT SHOULD BE THE SEQUENCE OF


INVESTIGATIONS FOR A PATIENT WITH
RECURRENT, UNEXPLAINED PANCREATITIS?

Stuart Sherman, MD

Determining the cause of acute pancreatitis is not usually difficult. Alcohol intake and gall-
stones are the 2 most common etiologies and account for 60% to 90% of cases. Alcoholism is
diagnosed by history and gallstones by a combination of demographic characteristics, laboratory
findings, and radiographic imaging studies. In patients in whom acute pancreatitis is due to
hypertriglyceridemia, hypercalcemia, drug reactions, trauma, surgery, or endoscopic retrograde
cholangiopancreatography (ERCP), the relationship of the episode of pancreatitis to the cause is
usually clear. Nevertheless, a cause for the pancreatitis will not be identified in 10% to 30% of
patients after a careful history, physical examination, laboratory studies, transabdominal ultra-
sound, and computed tomography (CT) scanning. These patients are conventionally classified as
having idiopathic acute pancreatitis (IAP). Patients with recurrent episodes of IAP are diagnosed
with idiopathic acute recurrent pancreatitis (IARP). Some reserve the terms IAP and IARP for
those patients who remain undiagnosed after a more detailed endoscopic (ERCP with manom-
etry, endoscopic ultrasound [EUS]), radiologic (eg, magnetic resonance cholangiopancreatography
[MRCP]), laboratory (eg, for autoimmune pancreatitis) and genetic evaluations and use the term
unexplained pancreatitis prior to the more intensive investigation.
Two major concerns prompt the physician to do a more intensive evaluation of the patient with
acute pancreatitis in whom no obvious cause is determined. The first is that the patient may have
an underlying disease that will predispose him or her to further attacks of acute pancreatitis unless
the cause is identified and adequately treated. The second concern is that the pancreatitis is due
to a neoplastic condition. There are no standardized evidence-based algorithms to guide us, and
the approach to the evaluation is often center specific and in part related to the expertise avail-
able. After a single episode of unexplained pancreatitis, the role of invasive evaluation with ERCP
is unsettled, but it is often limited to patients in whom there is suspicion of bile duct stones or a

Leung J, Lo SK, eds. Curbside Consultation in Endoscopy:


189 49 Clinical Questions, Second Edition (pp 189-192)
© 2014 SLACK Incorporated
190  Question 40

Figure 40-1. Algorithm for the recommended evaluation and intervention


in IAP. GB, gallbladder; s, secretin enhanced.

neoplastic process. I remain concerned about this approach in patients older than 40 years (when
neoplasia becomes more common) because the “standard evaluation” might miss an unsuspected
neoplastic process, such as an intraductal papillary mucinous neoplasm (IPMN). With the more
widespread availability and expertise in EUS and MRCP/magnetic resonance imaging (MRI),
ERCP is often relegated to patients found to have a treatable endoscopic disease (eg, bile duct
stones, pancreas divisum) or those found to have normal EUS and/or MRCP/MRI where further
endoscopic investigation is warranted (eg, evaluation by sphincter of Oddi manometry [SOM]).
For this patient, I will make the assumption that a standard evaluation has been done as
described previously and has been negative, including repeat triple-phase abdominal CT scan
with pancreatic protocol and transabdominal ultrasound (US) when the gallbladder is in situ.
It is important to stratify patients by their age and gallbladder status to help to direct your
approach. Certainly younger patients are less likely to have a neoplastic process, and those with
the gallbladder in situ may have microlithiasis as an explanation for the pancreatitis. Given the
results of 2 frequently cited studies suggesting that microlithiasis (often defined as stones < 3 mm
in diameter and frequently missed on transabdominal US) was the cause for the pancreatitis in
67% to 74% of patients, some authorities advocate empiric cholecystectomy. Other studies and my
own experience refute this high prevalence of microlithiasis, and as a result, I do not support this
approach unless there is evidence of stone-related disease by bile sampling, transabdominal US, or
EUS. EUS is perhaps the most sensitive way to detect biliary sludge and microlithiasis.
My initial work-up will include repeat laboratory testing consisting of serum triglyceride and
calcium levels. It is critical to try to obtain the results of these tests during the attack because
they may be normal or near normal when the patient is asymptomatic. Although controversial, I
will usually evaluate the patient’s cystic fibrosis transmembrane conductance regulator (CFTR)
gene, given the relatively high prevalence of cystic fibrosis gene mutations in patients with IARP
(approximately 20% vs 3% in the normal population without pancreatitis). However, a positive
result will not preclude further investigation. Unless there is some suggestion that at least one
family member had an episode of pancreatitis, I do not routinely assess for other known genetic
causes of pancreatitis (PRSS1 gene, SPINK1 gene, and CTRC gene) because of the low yield
and high cost. Serologic (eg, serum IgG 4 level) and confirmatory testing and/or therapy (eg,
pancreatic biopsy or corticosteroids) for autoimmune pancreatitis are done in the appropriate set-
ting when supported by consistent clinical, radiographic, and ERCP findings. My recommenda-
tions for further testing are based on having available expertise in both EUS and MRI/MRCP
(Figure 40-1). In many respects, EUS and secretin-enhanced MRCP/MRI are duplicative and
What Should Be the Investigations for Recurrent, Unexplained Pancreatitis?   191

Table 40-1

Idiopathic Acute Recurrent Pancreatitis̶Diagnostic Yield


of Endoscopic Retrograde Cholangiopancreatography,
Sphincter of Oddi Manometry, and Bile Microscopy*
Diagnosis Number Abnormal
Sphincter of Oddi dysfunction 179 (34%)
Pancreas divisum 70 (13%)
Pancreatic or papillary tumor 46 (9%)
Gallbladder or duct stones 37 (7%)
Pancreatic duct stricture/chronic pancreatitis 37 (7%)
Choledochocele 12 (2%)
Total abnormal 381 (73%)

*4 selected series of 522 patients

may not be additive. EUS can better assess for gallbladder pathology (duodenal collection of bile
after cholecystokinin stimulation can be done simultaneously if the EUS fails to demonstrate
gallbladder pathology), small bile duct stones, chronic pancreatitis, and pancreatic/peripancre-
atic neoplasia, but secretin-enhanced MRCP/MRI gives very detailed images of the pancreatic
duct and biliary tree anatomy. In my experience, secretin enhancement is necessary to obtain the
best quality images of the pancreatic duct. Because of the increasing concern for neoplasia with
increasing age, I will usually recommend an EUS for patients 40 years or older. The diagnostic
yield of EUS in IARP varies from 32% to 88%. There are limited data addressing the yield of
secretin-enhanced MRCP/MRI in this setting, but I suspect that a good quality study would
provide nearly equivalent diagnostic accuracy as an ERCP (MRCP/MRI will often miss ampul-
lary tumors and IPMNs, particularly in normal diameter pancreatic ducts). In younger patients, I
will often obtain a secretin-enhanced MRI/MRCP before considering a more invasive assessment
with ERCP. ERCP should be reserved for those IARP patients who either have a normal EUS
and/or secretin-stimulated MRI/MRCP or where the disease identified is treatable by endoscopic
methods (eg, pancreas divisum). Because sphincter of Oddi dysfunction (SOD) is the most com-
mon cause of IARP identified in ERCP series, the ability to perform SOM at the time of ERCP is
mandatory. There is no role for diagnostic ERCP in this setting, and the endoscopist should have
expertise in managing the diseases uncovered, such as pancreas divisum, SOD, choledochocele,
and pancreatic strictures. It is critical that a detailed pancreatogram be done at ERCP and high-
resolution fluoroscopy be available. I have seen many patients with reportedly normal pancreatic
ducts on EUS, MRCP, and/or prior ERCP but they have very subtle cast-like filling defects in a
normal diameter pancreatic duct, which is highly suggestive of an IPMN. ERCP clearly plays an
important role in the management of patients with IARP. In this setting, the reported diagnos-
tic yield of ERCP, SOM, and bile microscopy varies from 38% to 79%, and the overall yield is
influenced by the presence or absence of the gallbladder and referral bias. Table 40-1 presents a
192  Question 40

compilation of 4 series evaluating the yield of ERCP and ancillary procedures. The large majority
of diseases uncovered are treatable by endoscopic or surgical techniques.

Bibliography
Al-Haddad M, Wallace MB. Diagnostic approach to patients with acute idiopathic and recurrent pancreatitis, what
should be done? World J Gastroenterol. 2008;14:1007-1010.
Evans WB, Draganov P. Is empiric cholecystectomy a reasonable treatment option for idiopathic acute pancreatitis? Nat
Clin Prac Gastroenterol Hepatol. 2006;3:356-357.
Fischer M, Hassan A, Sipe BW, et al. Endoscopic retrograde cholangiopancreatography and manometry findings in
1,241 idiopathic pancreatitis patients. Pancreatology. 2010;10:444-452.
Kaw M, Brodmerkel GJ. ERCP, biliary crystal analysis, and sphincter of Oddi manometry in idiopathic pancreatitis.
Gastrointest Endosc. 2002;55:157-162.
Mariani A, Arcidiacono PG, Curioni S, et al. Diagnostic yield of ERCP and secretin-enhanced MRCP and EUS in patients
with acute recurrent pancreatitis of unknown aetiology. Dig Liver Dis. 2009;41:753-758.
Papachristou GI, Topazian M. Idiopathic recurrent pancreatitis: an EUS-based management approach. Gastrointest
Endosc. 2011;73:1155-1157.
Wilcox CM, Varadarjulu S, Eloubeidi M. Role of endoscopic evaluation in idiopathic pancreatitis: a systematic review.
Gastrointest Endosc. 2006;63:1037-1045.
41 QUESTION

A 56-YEAR-OLD FEMALE WITH NAUSEA AND VOMITING HAS A


MILDLY ELEVATED BILIRUBIN BUT NORMAL AMYLASE AND LIPASE. CT
SHOWS THICKENING OF THE DUODENUM AND MILDLY DILATED CBD.
ATTEMPTED ERCP SHOWS A TIGHT ULCERATIVE STENOSIS OF
1ST/2ND PART OF THE DUODENUM. WHAT SHOULD I DO?

Todd H. Baron, MD, FASGE

This clinical scenario described is both interesting and challenging. First, you need to deter-
mine the diagnosis; second, provide treatment. At the present time, you do not need to be con-
cerned about relief of biliary obstruction, given the lack of cholangitis and pruritus.
The differential diagnosis includes inflammatory, neoplastic, and infectious etiologies. All of
these processes can penetrate into and through the medial wall of the duodenum and surround
the distal bile duct with the resultant thickening of the duodenum and biliary obstruction seen on
computed tomography (CT).
Inflammatory causes in this clinical situation include pancreatitis, ulcers and inflammation due
to Crohn’s disease, ingestion of nonsteroidal anti-inflammatory drugs (NSAIDS), peptic ulcer
disease, and radiation therapy. Pancreatitis is unlikely, given the lack of inflammatory changes
in the pancreas by CT, normal serum amylase and lipase, and lack of an antecedent history of
acute pancreatitis. Crohn’s disease involving the duodenum, with or without associated primary
sclerosing cholangitis, is a possibility, although this would be an unusual initial clinical presenta-
tion of Crohn’s. NSAID-induced ulcers should be apparent by obtaining a history, although some
patients may not admit to NSAID use/abuse. Suggestion of NSAID use is a history of chronic
pain syndromes (migraine headaches, back pain) and rheumatoid arthritis, to name a few. Peptic
ulcer disease can also manifest in this fashion, although it is less likely as the initial presentation.
You must also consider hypersecretory acid states, such as Zollinger-Ellison syndrome, which
could present as aggressive ulcer disease at the onset. Radiation injury to the duodenum can occur
after treatment for retroperitoneal sarcoma and germ cell tumors, and this can be determined by
the history.

Leung J, Lo SK, eds. Curbside Consultation in Endoscopy:


193 49 Clinical Questions, Second Edition (pp 193-195)
© 2014 SLACK Incorporated
194  Question 41

Neoplastic diseases that can result in this clinical scenario include ampullary cancer, primary
duodenal cancer, lymphoma, and metastatic disease (breast cancer, lung cancer, and melanoma).
Thus, it is important to obtain a history of antecedent cancer. Because patients with familial poly-
posis syndrome (FAP) can develop duodenal and ampullary cancer, you should inquire whether
other family members have been affected with colon or duodenal cancer. Pancreatic cancer is less
likely, given the lack of pancreatic ductal dilation and mass, although this is still a possibility
because 2% of pancreatic cancers arise outside the ductal epithelium and do not cause pancreatic
ductal obstruction.
Infectious diseases that could produce this syndrome include viral etiologies, such as cytomega-
lovirus (CMV) and herpes simplex virus (HSV), and bacteria, such as mycobacteria. Viral diseases
would be rare in the absence of immunosuppression. If the patient had traveled to other countries
or had lived previously in underdeveloped countries, bacterial infections, such as tuberculosis,
would be a consideration.
After you obtain a complete history and physical and laboratory testing (including obtaining
serologic markers for inflammatory bowel disease) and re-review the CT with the aforemen-
tioned differential diagnosis in mind, you could consider additional noninvasive imaging such
as magnetic resonance imaging, although a tissue diagnosis needs to be obtained. Repeat upper
endoscopy allows you to obtain tissue from the duodenum, assuming this was not done at the time
of endoscopic retrograde cholangiopancreatography (ERCP). At the same time, you could also
consider obtaining biopsies of the stomach for Helicobacter pylori. If the biopsies are nondiagnostic,
your options are to treat empirically for peptic ulcer disease with high-dose proton pump inhibitors
(PPI) while treating the symptoms of nausea and vomiting, followed by clinical reassessment 4 to
8 weeks later, as well as repeat endoscopy, liver function tests, and CT. You could also consider
colonoscopy with inspection of the terminal ileum to assess for inflammatory changes to support
Crohn’s disease and to evaluate for multiple polyps that would suggest FAP.
Another option would be to perform an endoscopic ultrasound (EUS) to obtain tissue either
by fine needle aspiration or core biopsy and to provide staging information if the process is found
to be malignant. If a diagnosis is made by either routine duodenal biopsies or EUS-guided tissue
sampling, then appropriate treatment is undertaken.
Finally, you could ask for a surgical opinion. Albeit aggressive, surgical exploration could be
undertaken, and I would recommend this only as a last resort and if malignancy was highly sus-
pected but not proven. Surgical and biliary bypass would alleviate refractory symptoms, but would
be undertaken only if unresectable malignant disease or benign disease was found and was not
responsive to medical therapy. A pancreaticoduodenectomy (Whipple procedure) could also be
considered, although this would be an extreme last resort.
If nonoperative relief of biliary obstruction was required, you could achieve this endoscopically
using EUS guidance and a rendezvous procedure to provide internal drainage with biliary stent
placement, or via a percutaneous radiologic approach. The percutaneous approach could be done
alone (external drain) or with an endoscopic rendezvous approach to provide internal drainage.
If acid suppression and conservative management is undertaken, you could initiate supplemen-
tal enteral nutrition by placing a nasojejunal feeding tube.

Conclusion
This is a difficult diagnostic dilemma, but one that can be solved by careful consideration of
the differential diagnoses and potential therapeutic approaches.
What Should I Do for a Patient With Tight Ulcerative Stenosis?  195

Suggested Readings
Flores HB, Zano F, Ang EL, Estanislao N. Duodenal tuberculosis presenting as gastric outlet obstruction: a case report.
World J Gastrointest Endosc. 2011;3(1):16-19.
Jeurnink SM, Steyerberg EW, van Hooft JE, et al; Dutch SUSTENT Study Group. Surgical gastrojejunostomy or endo-
scopic stent placement for the palliation of malignant gastric outlet obstruction (SUSTENT study): a multicenter
randomized trial. Gastrointest Endosc. 2010;71(3):490-499.
Kochhar R, Kochhar S. Endoscopic balloon dilation for benign gastric outlet obstruction in adults. World J Gastrointest
Endosc. 2010;2(1):29-35.
Plerhoples TA, Norton JA. Recurrent duodenal stricture secondary to untreated Crohn’s disease. Dig Dis Sci.
2012;57(10):2516-2518.
SECTION IV
ENDOSCOPIC ULTRASOUND
42 QUESTION

HOW DO WE MANAGE A PATIENT


WITH A 10-CM PSEUDOCYST WHO IS
CURRENTLY ASYMPTOMATIC?

Richard A. Kozarek, MD

Pseudocyst size is less important than the clinical context of the patient. How certain are you
that the cystic lesion noted on computed tomography (CT) scan or ultrasound (US) imaging is
actually a pseudocyst? For instance, a middle-aged female patient with vague abdominal symp-
toms who is found to have a cystic mass of any size on abdominal imaging is much more likely to
have a cystic neoplasm if there has been no antecedent history of pancreatitis. Likewise, patients
with intraductal papillary mucinous neoplasm (IPMN) or mucinous cystadenoma of the pancreas
(a premalignant condition) can occasionally present with obstructive pancreatitis. The presence of
a cystic lesion at the onset of pancreatitis is crucial to distinguishing a neoplastic condition from
a true pseudocyst, which is the consequence of a ductal disruption and the body’s containment of
that disruption by a wall of inflammatory and fibrous tissue.
Pseudocysts that occur in the setting of chronic pancreatitis usually are the consequences of
increased ductal pressure, with or without superimposed acute parenchymal inflammation.1 There
is often a stone or a stricture downstream from the site of ductal disruption. If walled off, a pseu-
docyst occurs. Leaks that are not walled off may be associated with high amylase pleural effusions,
pancreatic ascites, or fistulization into contiguous organs, including the bile duct, small bowel, or
colon. As a result of a fixed ductal obstruction, pseudocysts that occur in a patient with chronic
pancreatitis are less likely to resolve spontaneously (Figures 42-1 and 42-2).
In contrast, most pancreatic fluid collections that occur in the setting of acute pancreatitis are
not pseudocysts, and more than three-quarters resolve over 4 to 6 weeks. Pancreatic necrosis is
also not a pseudocyst, although it too is associated with a ductal leak in most patients. Necrosis
frequently results in a collection of debris-filled pancreatic juice that is anatomically constrained
by the lesser sac. CT scanning notoriously overestimates the liquid component of the collection,
and US or endoscopic ultrasound (EUS) may be required to distinguish this from a pseudocyst.

Leung J, Lo SK, eds. Curbside Consultation in Endoscopy:


199 49 Clinical Questions, Second Edition (pp 199-201)
© 2014 SLACK Incorporated
200  Question 42

Figure 42-1. Abdominal CT demonstrates a


10-cm pseudocyst (arrow) in an asymptom-
atic patient with hereditary pancreatitis.
The patient had previous cystogastrostomy
20 years prior.

Figure 42-2. Endoscopic compression by pseu-


docyst in the patient depicted in Figure 42-1.
Because the patient had splenic and portal
vein thromboses with varices, she was initially
followed conservatively elsewhere.

Other clues include the irregularity and variable thickness of the wall, extension into the pelvic
gutters, nonenhancement of the pancreatic parenchyma on the early arterial phase of a pancreatic
protocol CT, and the stormy clinical course of the patient to include multisystem organ failure
(MSOF) and bacterial translocation from the gut with superinfection of the necrotic tissue and
fluid. Pseudocysts may be a consequence of evolving pancreatic necrosis, but this occurs several
months after a severe attack of pancreatitis and occurs when the majority of necrotic tissue has
liquefied and a true fibrous “rind” has formed around the fluid collection.
Pseudocysts that occur in the setting of acute pancreatitis are usually spherical, take
6 to 8 weeks to “mature,” and historically were felt to be associated with a high complication rate
if not treated. This was at a time that treatment consisted primarily of surgical cystgastrostomy
or Roux-en-Y cystojejunostomy. The maxim was that pseudocysts 6 cm or larger present for
How Do We Manage a Patient With a 10-cm Pseudocyst?   201

more than 6 weeks required drainage. It was in that background that percutaneous drainage of
pseudocysts was popularized in an attempt to avoid surgery. Subsequently, endoscopic drainage
was described as a means to avoid an indwelling percutaneous catheter.1 The latter resulted in a
long-term pancreatic fistula in 10% to 20% of patients, most of whom had a downstream ductal
obstruction or the disconnected pancreatic duct syndrome.
Seminal work from the University of Minnesota randomizing asymptomatic patients with acute
pseudocysts to surgery or expectant follow-up changed my treatment paradigm almost 20 years
ago.1,5 Those investigators found a relatively low risk of pseudocyst complications as long as the
diameter of the pseudocyst was stable or decreasing in size. Additional studies suggested that the
7% to 25% complication rates in patients with pseudocyst occurred early in the clinical course or
in the setting of enlarging fluid collections, almost all of whom were symptomatic.1,4,5 These com-
plications can include bleeding with pseudoaneurysm formation, obstruction of contiguous organs
(bile duct, cholestasis/jaundice; stomach/duodenum, gastric outlet obstruction), cyst infection, and
occasional cyst leak with free rupture, but more commonly fistulization into contiguous organs.
What has changed to allow us to follow a patient with a 10-cm pancreatic fluid collection and
not recommend intervention? Imaging has improved dramatically, and CT scans, secretin mag-
netic resonance cholangiopancreatography (MRCP) scans, 2 and EUS with or without sampling
of the cyst fluid have improved our ability to distinguish cystic neoplasms from pseudocysts,
thereby eliminating patients with potential neoplasms early in the course.3 We also have a better
understanding of acute pancreatic fluid collections because serial CT scans have demonstrated that
most collections associated with acute pancreatitis resolve. 2,4 Moreover, as noted previously, the
majority of patients with an enlarging collection or complication of pseudocyst are not asymptom-
atic. Finally, we now recognize that evolving pancreatic necrosis is not a pseudocyst and that the
morbidity and mortality associated with this condition are at least a log factor higher than in the
setting of an acute or chronic pancreatic pseudocyst.1
So, if I am certain that this 10-cm fluid collection is a pseudocyst, that the patient is eating
and is asymptomatic, and that the cyst is stable or slowly being reabsorbed, why not follow it?
How often the patient needs to be scanned, sounded, or palpated is contingent upon whether the
patient is recovering from acute pancreatitis or if the cyst was found incidentally in a patient with
chronic pancreatitis. In the former setting, repeat imaging every 4 to 6 weeks seems reasonable,
eventually increasing imaging intervals to every 5 to 6 months. In patients with chronic pancre-
atitis, a repeat CT or US should be considered at 1 month and repeated at 3 and 9 to 12 months if
the patient remains asymptomatic. Alternatively, because pseudocysts that occur in patients with
chronic pancreatitis usually have a downstream stone or stricture preventing ultimate pseudocyst
resolution, this latter situation is considered by some practitioners to be an absolute indication for
intervention, regardless of the presence of symptoms.5

References
1. Kozarek RA, Traverso LW. Pancreatic fistulas: etiology, consequences, and treatment. Review. Gastroenterologist.
1996;4:238-244.
2. Thoeni RF. The revised Atlanta classification of acute pancreatitis: its importance for the radiologist and its effect
on treatment. Radiology. 2012;262:751-764.
3. Macari M, Finn ME, Bennett GL, et al. Differentiating pancreatic cystic neoplasms from pancreatic pseudocysts at
MR imaging: value of perceived internal debris. Radiology. 2009;251:77-84.
4. Lankisch PG, Weber-Dany B, Maisonneuve P, Lowenfels AB. Pancreatic pseudocysts: prognostic factors for their
development and their spontaneous resolution in the setting of acute pancreatitis. Pancreatology. 2012;12:85-90.
5. Samuelson AL, Shah RJ. Endoscopic management of pancreatic pseudocysts. Gastroenterol Clin North Am.
2012;41:47-62.
43 QUESTION

DO ALL PANCREATIC CYSTS NEED EUS IMAGING,


AND DO THEY ALL NEED TO BE ASPIRATED FOR
ANALYSIS? ARE THERE EVEN STANDARD CRITERIA TO
DETERMINE THEIR ORIGINS?

Won Jae Yoon, MD and William R. Brugge, MD

Pancreatic cystic lesions often present a dilemma to the clinician. These lesions are being dis-
covered more frequently due to the rise in the use of cross-sectional imaging for evaluation of other
disease processes. A minority of these lesions will present with symptoms. The main concern on
finding any pancreatic cystic lesion is whether or not a malignancy is present; thus, further inves-
tigation is often warranted. However, debate remains on the appropriate evaluation algorithm.
Endoscopic ultrasound (EUS) offers high-resolution imaging of cystic lesions and is able to direct
fine needle aspiration (FNA) for cytology and cyst fluid analysis. EUS may play a critical role in
the evaluation algorithm.
There is a wide differential diagnosis of pancreatic cystic lesions. Lesions can fall into
1 of 3 categories: 1) benign, 2) premalignant, and 3) malignant (Table 43-1).1,2
Upon discovering a pancreatic cystic lesion, the evaluation should focus on differentiating
between benign and malignant cysts, as well as mucinous and nonmucinous cysts.
Before any further imaging or invasive procedures are contemplated, a detailed personal and
family history should be obtained from the patient because often clues to the etiology of the lesion
may be evident. For example, patients with a history of von Hippel-Lindau disease are at risk
for developing serous cystadenomas, as well as cystic neuroendocrine tumors. Individuals with
an inherited genetic mutation, such as that seen in breast cancer type 2 susceptibility protein
(BRCA2) or familial atypical multiple mole/melanoma (FAMM) syndrome, are at risk for the
development of pancreatic cancer. Pseudocysts are common in patients who have had a history of
pancreatitis. However, it must be cautioned that some cystic lesions, particularly if they cause pan-
creatic duct obstruction, can lead to pancreatitis. A review of a patient’s available imaging studies
prior to the onset of pancreatitis can be critical.

Leung J, Lo SK, eds. Curbside Consultation in Endoscopy:


203 49 Clinical Questions, Second Edition (pp 203-206)
© 2014 SLACK Incorporated
204  Question 43

Table 43-1

Types of Pancreatic Cystic Lesions


● Benign
● Pseudocyst
● Serous cystadenoma
● Acinar cell cystadenoma
● Lymphoepithelial cyst
● Premalignant
● Intraductal papillary mucinous neoplasm (IPMN) with low- or intermediate-
grade dysplasia
● IPMN with high-grade dysplasia
● Mucinous cystic neoplasm (MCN) with low- or intermediate-grade dysplasia
● MCN with high-grade dysplasia
● Malignant
● IPMN with an associated invasive carcinoma
● MCN with an associated invasive carcinoma
● Soild-pseudopapillary neoplasm
● Serous cystadenocarcinoma (extremely rare)
● Acinar cell cystadenocarcinoma
● Cystic degeneration of pancreatic ductal adenocarcinoma
● Cystic neuroendocrine neoplasms

In addition to a diagnostic evaluation, the patient’s symptoms should be assessed for possible
linkage with the cystic lesion. A patient who presents with symptoms of abdominal pain and is
found to have a pancreatic cystic lesion can be referred directly to surgery, provided the patient
is an appropriate surgical candidate. On the other hand, an elderly patient with an incidental,
asymptomatic lesion found on computed tomography (CT) imaging will likely benefit from a more
detailed evaluation.
Radiologic characteristics of lesions found on standard CT or magnetic resonance imaging
(MRI) can also assist in discerning the type of lesion and whether malignancy is present. Serous
cystadenomas are characterized as having multiple, small, thin-walled cysts in a honeycomb-like
pattern. The presence of a central scar on radiologic imaging is often thought of as being pathog-
nomonic of these lesions. Mucinous cystic neoplasms (MCNs), on the other hand, are oligocystic
lesions, with larger cystic compartments, that are typically located in the tail of the pancreas.
Radiologic assessment of a lesion’s relation to the pancreatic duct can also help in the diagnos-
tic evaluation. Serous cystadenomas and MCNs do not communicate with the pancreatic duct,
whereas intraductal papillary mucinous neoplasms (IPMNs), as their name implies, involve the
pancreatic duct. Finally, the presence of an associated mass, invasion of adjacent organs or vascu-
lature, and/or the presence of lymphadenopathy are all suggestive of a malignant pancreatic lesion.
Are There Standard Criteria to Determine the Origins of Pancreatic Cysts?   205

Figure 43-1. Decision algorithm for when to perform EUS ± FNA on pancreatic cysts.

Unfortunately, the utility of cross-sectional imaging in detecting pancreatic cystic lesions is not
well matched by its ability to determine the type of lesion. Considerable variability exists in the
CT appearance of serous and mucinous pancreatic lesions, and there can be poor interobserver
agreement on studies, thus making CT an insensitive tool at distinguishing these lesions.3 EUS,
on the other hand, is extremely useful in providing a detailed analysis of cyst characteristics,
particularly in identifying features of malignancy, such as wall thickness, macroseptations, intra-
mural nodules or masses, or cystic dilation of the main pancreatic duct.4 Furthermore, FNA of
cystic lesions under EUS guidance allows for the collection of fluid and tissue samples for further
analysis. Mucinous lesions (ie, MCNs and IPMNs) characteristically contain thick, viscous fluid
as compared with serous lesions that have watery fluid. Cyst fluid is also rich in numerous glyco-
proteins, many of which can serve as tumor markers. A large prospective study demonstrated that
a cyst fluid carcinoembryonic antigen (CEA) level greater than 192 ng/mL was the most accurate
test, when compared with EUS alone or cytology, in differentiating mucinous from nonmucinous
lesions.5 Amylase levels also provide information regarding whether a lesion may have communi-
cation with the pancreatic duct and would thus be expectedly elevated in IPMNs and pseudocysts.
Cytological analysis can provide direct evidence of the presence of malignancy. DNA analysis of
cyst fluid showed that KRAS mutation is highly specific for mucinous cysts.6 Perhaps the most
powerful aspect of FNA is that it is an extremely safe procedure. In a retrospective review of
more than 600 patients having undergone EUS with FNA in our center, 13 complications were
encountered (2.2%), including 6 patients with pancreatitis (1%).7 Furthermore, cyst fluid analysis
can be performed on as little as 1 mL of fluid for amylase and tumor marker analysis, and 0.4 mL
is required for genetic analysis.
The decision to perform EUS-FNA of a pancreatic cystic lesion is often influenced by clinical
factors and radiologic findings, but ultimately, it is the concern that a malignancy exists that leads
to its performance (Figure 43-1). With the scant amount of cyst fluid aspirated during EUS-FNA,
206  Question 43

it is important to decide what test to perform. When there is diagnostic uncertainty, the cyst fluid
should be analyzed for CEA and KRAS. If the clinical question is whether the cyst is malignant
or benign, then the fluid should be sent for cytology analysis.8

Conclusion
Not all pancreatic cysts need EUS imaging or aspiration. However, most lesions are asymp-
tomatic and incidentally found with no relevant antecedent medical history. Furthermore, limita-
tions exist with current cross-sectional imaging in terms of determining cyst type and presence
of malignancy. EUS is a proven, safe modality that can provide a high level of detail as well as an
opportunity to obtain fluid and tissue samples for a more definitive diagnosis in patients with a
pancreatic cystic lesion. Thus, it is our practice to perform EUS with FNA in patients in whom a
diagnostic uncertainty exists and where management options would thus be altered.

References
1. Bosman FT, Carneiro F, Hruban RH, Theise ND, eds. WHO Classification of Tumours of the Digestive System. 4th ed.
Lyon, France: WHO Press; IARC; 2010:322-326.
2. Basturk O, Coban I, Adsay NV. Pancreatic cysts: pathologic classification, differential diagnosis, and clinical impli-
cations. Arch Pathol Lab Med. 2009;133:423-438.
3. Curry CA, Eng J, Horton KM, et al. CT of primary cystic pancreatic neoplasms: can CT be used for patient triage
and treatment? Am J Roentgenol. 2000;175:99-103.
4. Sedlack R, Affi A, Vazquez-Sequeiros E, et al. Utility of EUS in evaluation of cystic pancreatic lesions. Gastrointest
Endosc. 2002;56:543-547.
5. Brugge WR, Lewandrowski K, Lee-Lewandrowski E, et al. Diagnosis of pancreatic cystic neoplasms: a report of the
cooperative pancreatic cyst study. Gastroenterology. 2004;126:1330-1336.
6. Khalid A, Zahid M Finkelstein SD, et al. Pancreatic cyst fluid DNA analysis in evaluating pancreatic cysts: a report
of the PANDA study. Gastrointest Endosc. 2009;69:1095-1102.
7. Lee LS, Saltzman JR, Bounds BC, et al. EUS-guided fine needle aspiration of pancreatic cysts: a retrospective
analysis of complications and their predictors. Clin Gastroenterol Hepatol. 2005;3:231-236.
8. Yoon WJ, WR Brugge. Pancreatic cystic neoplasms: diagnosis and management. Gastroenterol Clin North Am.
2012;103-118.
44 QUESTION

OUR ENDOSONOGRAPHER FREQUENTLY DETECTS VAGUE HYPOECHOIC


PANCREATIC LESIONS, BUT DOES NOT PERFORM FINE NEEDLE
ASPIRATION̶HE RECOMMENDS FOLLOW UP EUS IN 3 MONTHS.
HOW MANY FOLLOW UP EXAMS ARE NEEDED? ARE FINE NEEDLE
ASPIRATIONS UNSAFE?

Jason B. Samarasena, MD and Kenneth J. Chang, MD, FACG, FASGE

First, it is important to describe what a vague hypoechoic lesion of the pancreas means to an
endosonographer (Figure 44-1). Any lesion that is large, well circumscribed, cystic, or distorts
surrounding structures (main or side branch ducts, vessels, etc) does not fall into this category.
The differential diagnosis of a vague hypoechoic lesion of the pancreas includes normal changes,
chronic pancreatitis (including focal), lobularity (normal parenchyma surrounded by fibrous
bands), and, much less likely, a neuroendocrine tumor, lymphoma, metastatic tumor, or early
cancer. The 2 main factors to consider in managing these vague hypoechoic lesions of the pancreas
found on endoscopic ultrasound (EUS) are the pretest probability of a neoplastic lesions and the
risk of fine needle aspiration (FNA) itself.
Let us first consider the risk of EUS-guided FNA. A large prospective single center study
following 355 consecutive patients reported a complication rate of 2.5% (95% CI, 1.17-4.76).1 Acute
pancreatitis occurred in 3 of 355 (0.85%, 95% CI, 0.17-2.45). None of the patients experienced
clinically significant hemorrhage, perforation, or death. Recently a meta-analysis on EUS-guided
FNA of pancreatic lesions included 33 studies and 4984 patients.2 No major complications were
noted in any of the studies and minor complications were between 1% to 2%. In our experience,
the risk of pancreatitis is close to 0% when performing FNA into an obvious pancreatic tumor,
and we do not hesitate in making multiple passes to secure a tissue diagnosis. On the other hand,
we think twice before passing a needle through normal parenchyma or into a vague lesion with a
low clinical suspicion for malignancy. In this scenario, the risk of pancreatitis, in our opinion, is
higher and the yield of FNA is much lower.
This brings us to consider the pretest probability of a neoplastic lesion. If the pancreas is being
examined as part of a routine EUS exam (nonpancreatic indication) and a vague hypoechoic lesion
Leung J, Lo SK, eds. Curbside Consultation in Endoscopy:
207 49 Clinical Questions, Second Edition (pp 207-209)
© 2014 SLACK Incorporated
208  Question 44

Figure 44-1. EUS image showing a “vague”


5.4 x 5.1 mm hypoechoic lesion in a patient
with a strong family history of pancreatic can-
cer.

is detected, we would most likely not perform FNA. The only follow-up we would consider would
be a repeat EUS in 6 months. In this setting, 3 months is probably too soon to repeat the EUS.
If this vague lesion remains unchanged at 6 months, we would not repeat imaging unless clinical
symptoms or risk factors change. Other imaging modalities, such as computed tomography (CT)
or magnetic resonance imaging (MRI) in this setting would most likely not detect an abnormality
and hence further follow-up with these modalities would likely result in low yield.
However, if the indication for EUS of the pancreas is based on relevant symptoms or findings
(pancreas abnormality seen on other imaging, weight loss, diarrhea, pancreatic-type pain,
hypoglycemia, hyperinsulinism, hypergastrinemia, etc) where the pretest probability is higher for
a clinically significant lesion, we would be more enthusiastic to perform an EUS-guided FNA. We
would try to minimize the number of FNA (1 to 2 passes) and have “real time” dialogue with the
cytopathologist in between passes. If the FNA was nondiagnostic, we would repeat a EUS with
possible FNA in 3 months. If it is negative again, we would increase the next interval to 6 months
(no FNA unless the lesion increases in size).
Finally, you need to consider the special case of a patient with a strong family history of
pancreatic cancer. Individuals who have 2 or more first-degree relatives with pancreatic cancer
have a lifetime risk of approximately 16% of developing pancreatic cancer. This is even higher
among patients with Peutz-Jeghers syndrome or hereditary pancreatitis, and patients with BRCA2
mutations also have increased risk. In these high-risk individuals, a vague hypoechoic lesion in
the pancreas has a completely different meaning. These may indicate precancerous changes, 3 such
as pancreatic intraepithelial neoplasms (Pan-IN) or intraductal papillary mucinous neoplasm
(IPMN), and one should proceed with FNA and consider partial pancreatectomy. If surgery is not
performed, close surveillance with EUS (every 6 months) is indicated.

Conclusion
As summarized in Table 44-1, the indication for FNA and the necessity/timing of repeat EUS
in patients found to have vague hypoechoic lesions in the pancreas depend on the clinical scenario
(pretest probability).
Are Fine Needle Aspirations Unsafe?  209

Table 44-1

Management of Vague Hypoechoic


Lesion on Pancreatic Endoscopic Ultrasound
Pretest Probability Enthusiasm for Performing Recommended
of Neoplastic Lesion EUS-Guided FNA Surveillance
Low Low Repeat EUS in 6 months If
no change, no further tests
Medium-high High Repeat EUS in 3 months
If no change, increase
interval to 6 months, etc
Familial pancreatic Very high Consider partial
cancer pancreatectomy; otherwise
repeat EUS every 6 months

References
1. Eloubeidi MA, Tamhane A, Varadarajulu S, Wilcox CM. Frequency of major complications after EUS-guided FNA of
solid pancreatic masses: a prospective evaluation. Gastrointest Endosc. 2006;63(4):622-629.
2. Hewitt MJ, McPhail MJ, Possamai L, et al. EUS-guided diagnosis of solid pancreatic neoplasms: a meta-analysis.
Gastrointest Endosc. 2012;75:319-331.
3. Canto MI, Goggins M, Yeo CJ, et al. Screening for pancreatic neoplasia in high-risk individuals: an EUS-based
approach. Clin Gastroenterol Hepatol. 2004;2(7):606-621.
45 QUESTION

WHEN IS EUS NECESSARY FOR A


NEWLY DIAGNOSED CANCER OF THE
ESOPHAGUS, STOMACH, COLON, OR PANCREAS?

Thomas J. Savides, MD

Gastrointestinal (GI) endoscopic ultrasound (EUS) is an important tool for cancer staging
because many treatment algorithms are determined by EUS tumor staging. Computed tomog-
raphy (CT) scan should generally be obtained before EUS to determine whether there is any
metastatic disease that would make the patient a nonsurgical candidate. If there is no evidence
of metastatic disease, then EUS should be performed for locoregional staging. The tumor-node-
metastases (TNM) staging system is used for luminal GI and pancreatic cancer. The EUS accura-
cy for staging GI and pancreatic cancer is approximately 85% for T-staging and 75% for N-staging.
EUS-guided fine needle aspiration (EUS FNA) can be used to obtain tissue diagnosis of tumors
as well as peritumoral metastatic disease.
The time to use EUS in cancer staging is before the patient receives any chemoradiation.
Staging accuracy significantly decreases after chemoradiation because EUS cannot distinguish
between peritumoral inflammation/edema and the actual tumor.

Esophageal Cancer
If EUS shows that a tumor is limited to the mucosal layer (tumor in-situ) and without any
adjacent lymph nodes (N0), then it is potentially amenable to endoscopic resection. Endoscopic
mucosal resection is the only accurate way to know for certain if the tumor is limited to the muco-
sal layer. Esophageal tumors that invade into the submucosal layer have an approximate 15% risk
of metastatic disease to regional lymph nodes and should generally undergo surgical resection with
lymph node dissection.

Leung J, Lo SK, eds. Curbside Consultation in Endoscopy:


211 49 Clinical Questions, Second Edition (pp 211-214)
© 2014 SLACK Incorporated
212  Question 45

Figure 45-1. Gastric MALT lymphoma. Note


the thickening of the mucosal and submucosal
layers.

If EUS reveals there is invasion into the periesophageal fat (T3) or periesophageal regional
lymph nodes (N1), then the patient is generally referred for preoperative chemoradiation. Patients
with involvement into adjacent organs such as the aorta, heart, or trachea (T4) are usually not
considered surgical candidates. I do not perform EUS restaging after chemoradiation because it is
not accurate and there is no established algorithm for treatment management.
EUS FNA can be performed to increase the lymph node staging accuracy in esophageal can-
cer.1 However, often this is not possible due to the lymph nodes being located in a position whereby
the needle would need to pass through the tumor to reach the lymph node.

Gastric Cancer
The utility of EUS is limited to evaluating superficial cancer to determine whether it is poten-
tially amenable to endoscopic resection. Patients with tumors limited to the mucosa/submucosa
(T1) and without adjacent lymph nodes (N0) are candidates for endoscopic resection. For more
advanced tumors, there is no need for EUS because these patients will generally undergo surgi-
cal resection. Occasionally, it may be worthwhile to do an EUS in advanced tumors to determine
whether there is invasion into an adjacent organ, such as the pancreas. In the rare cases of gastric
mucosa-associated lymphoid tissue (MALT) lymphoma, EUS can identify the tumors limited
to the mucosa/submucosa, which are most likely to possibly respond to antibiotic therapy for
Helicobacter pylori infection (Figure 45-1).2

Colorectal Cancer
EUS is useful only in rectal cancer; it is not helpful in colon cancer. 3 This is because for colon
cancer, the surgeon can obtain wide distal, proximal, and lateral margins. In contrast, the location
of rectal cancer in the pelvis precludes extended longitudinal or circumferential surgical margins,
which results in a higher risk of locally recurrent tumor after resection. In addition, because of the
fixed position of the rectum in the pelvis, rectal cancer is amenable to radiation therapy.
If EUS shows that the rectal tumor involves only the mucosa/submucosa (T1, N0), then it is
amenable to surgical transanal resection. If the tumor extends into the perirectal fat (T3) and/or
When Is EUS Necessary for a Newly Diagnosed Cancer?  213

Figure 45-2. Rectal cancer—stage T3, N1.


Note that the tumor extends into the perirec-
tal fat (T3), and there is a malignant-appearing
lymph node adjacent to the mass (N1).

Figure 45-3. Pancreatic cancer. Note that the


mass involves both the common bile duct
(CBD) and portal vein (PV).

has associated malignant-appearing lymph nodes (N1), then generally patients are offered preop-
erative chemoradiation (Figure 45-2). EUS is not routinely performed after chemoradiation for
rectal cancer. If a patient undergoes transanal resection of a rectal cancer, then I perform follow-up
transrectal ultrasound every 6 months for a total of 2 years to detect any local recurrence.

Pancreatic Cancer
The utility of EUS is somewhat less important for staging pancreatic cancer than esophageal
or rectal cancer. This is because initially EUS was better than old-generation CT scanners for
determining locally advanced pancreatic cancer, although more recent multidetector CT scans
have similar staging accuracies as EUS.4 In addition, there are no agreed-upon criteria for locally
unresectable pancreatic cancer. Invasion of the portal vein (Figure 45-3), superior mesenteric vein,
or superior mesenteric artery is generally considered a contraindication to surgery; however, many
experienced pancreatic surgeons can often peel pancreatic tumors off blood vessels and perform
vascular reconstructions for locally invasive cancer. Therefore, in my center, I usually obtain a
214  Question 45

pancreatic protocol multidetector CT scan rather than an EUS to determine whether there are
any absolute contraindications to surgery, such as significant encasement of the celiac artery or
superior mesenteric artery. There are some centers where more emphasis is placed on the EUS
assessment of vascular involvement, although I suspect that multidetector CT scans and MRI
scans will continue to be as good as or better than EUS for predicting respectability. In the end,
the only true way to know whether a pancreatic tumor is resectable is by attempted surgical resec-
tion by an expert pancreatic surgeon.
I believe the most important role of EUS in pancreatic cancer is actually visualizing the pan-
creatic mass (sometimes CT shows only bile duct obstruction or fullness in the pancreatic masses)
and for obtaining FNA cytology tissue diagnosis of malignancy. In the future, EUS-guided fine
needle injection may be used for pancreatic cancer treatment, either by directly injecting antitumor
drugs or by injecting radiopaque markers into the tumor to assist with radiation therapy.

References
1. DeWitt J, Devereaux B, Chriswell M, et al. Comparison of endoscopic ultrasonography and multidetector
computed tomography for detecting and staging pancreatic cancer. Ann Intern Med. 2004;141(10):753-763.
2. Sackmann M, Morgner A, Rudolph B, et al. Regression of gastric MALT lymphoma after eradication of
Helicobacter pylori is predicted by endosonographic staging. MALT Lymphoma Study Group. Gastroenterology.
1997;113(4):1087-1090.
3. Savides TJ, Master SS. EUS in rectal cancer. Gastrointest Endosc. 2002;56(4 Suppl):S12-S18.
4. Vazquez-Sequeiros E, Wiersema MJ, Clain JE, et al. Impact of lymph node staging on therapy of esophageal
carcinoma. Gastroenterology. 2003;125(6):1626-1635.
46 QUESTION

A 35-YEAR-OLD MECHANIC IS COMPLAINING OF NEW ONSET


EPIGASTRIC PAIN AFTER TAKING MOTRIN. EGD SHOWS MILD,
NONEROSIVE ANTRAL GASTRITIS, AND A 1-CM SUBMUCOSAL MASS IN
THE PROXIMAL ANTRUM. BIOPSIES SHOWED CHRONIC INFLAMMATION.
WHAT SHOULD I DO?

Laith H. Jamil, MD

When you see a gastric submucosal/subepithelial mass/bulge on endoscopy, you have to wonder
whether it is arising from the gastrointestinal wall (intramural) or whether this is an extramural
lesion causing compression. Unfortunately, endoscopy alone cannot distinguish between them
adequately.1,2 If this patient had cross sectional imaging that showed normal structures, it does
not rule out the fact that this lesion could still be secondary to extrinsic compression by a normal
structure, or that it’s a true lesion that will not be picked up by imaging, especially small (< 2 cm)
lesion. Examples of true submucosal lesions are gastrointestinal stromal tumors (GIST), carci-
noids, lipomas, pancreatic rests, leiomyoma, metastatis lesions, etc. Examples of normal extramu-
ral structures that can cause gastric indentations and give the appearance of a submucosal mass
are the spleen,1,2 splenic vessels, liver, gall bladder, colon, and pancreas. Other extramural lesions
that can cause a similar appearance include tumors; pancreatic, renal, and liver lesions; aneurysms;
and lymph nodes. Prior to sampling the lesion, it would be better to probe it with a closed-biopsy
forceps. If you suspect the lesion to be vascular or cystic in nature on probing, avoid sampling
initially until the lesion is further evaluated. If on probing the mass is mobile, soft, and indents
(pillow sign), then the lesion is likely to be a lipoma. If the lesion is firm, further evaluation is
definitely warranted.
The best way to further evaluate this finding is by performing an endoscopic ultrasound (EUS),
which can reliably differentiate between an intramural and extramural lesion. EUS will further
allow you to better assess the size, layer of origin, margins, echogenicity, and vasculature of the
mass, and assess it for lymphadenopathy. This will help to narrow the differential diagnosis, but
depending on EUS features alone, it is usually not adequate.1,2
When performing EUS for these lesions, I first use a 7.5 MHz frequency on a radial echoen-
doscope, to determine whether it is an extramural or intramural lesion. Occasionally, I may have
to instill de-aerated water into the lumen and inflate the water-filled balloon to achieve better
Leung J, Lo SK, eds. Curbside Consultation in Endoscopy:
215 49 Clinical Questions, Second Edition (pp 215-217)
© 2014 SLACK Incorporated
216  Question 46

acoustic coupling. If the lesion turns out to be smaller than 1 cm or more flat, and I am unable to
assess it adequately, then I would go to a higher frequency, such as 10 MHz of the electronic radial
echoendoscope, or even higher on the mechanical radial echoendoscope, or use a miniprobe, a
through-the-scope high-frequency catheter ultrasound probe. They come in different frequencies,
and I prefer using a 12- or 20-MHz frequency probe.

Ultrasound Characteristics
Identifying layers of origin of the submucosal mass is important, and helps to narrow the dif-
ferential diagnosis, but it is the echogenicity that is more important.
If the lesion is anechoic, then it’s either the gallbladder, a varix/vessel (which would likely be
doppler positive), or a duplication cyst. No further work-up or sampling would be required if the
patient is asymptomatic.
If the lesion is hyperechoic, then it’s likely a lipoma, especially if it is intensely hyperechoic or
an inflammatory fibroid polyp (which can be hypoechoic also). If it’s hypoechoic, then it can be a
host of things, ranging from benign lesions, such as leiomyoma or a pancreatic rest, to malignant
lesions, such as gastrointestinal stromal tumor (GIST) and lymphoma. Thus, sampling becomes
very important.

Marking the Lesion


When identifying a suspected subepithelial lesion, in particular a small (< 2 cm) one, it would
be very helpful to place a tattoo mark. This is not only helpful for follow up, but also if the patient
is referred for an ultrasound, and subsequently to a surgeon. These lesions are not always easy to
identify, especially if they are caused by an extrinsic compression. It is better to place a tattoo mark
about 2 cm proximal (if the lesion is in the gastric body or antrum) or distal (if the lesion is in the
fundus) to the lesion. This will minimize interference with ultrasound evaluation, and endoscopic
resection if attempted. It is also important to document the tattoo mark in relation to the lesion.

Sampling
There are several methods for sampling, but when the lesion is this small, it is challenging,
especially when the lesion arises from the muscularis propria. Tunnel biopsies, or bite on bite, can
be diagnostic up to 42% of the time, but have a potential complication rate of 2.8%. 3 If the lesion
is in the submucosa, endoscopic resection with an endoscopic electrosurgical snare or cap-fitted
endoscopic mucosal resection device can yield a tissue diagnosis up to 87%.4 EUS-guided fine
needle aspiration (FNA)/biopsy should be attempted, especially if the lesion is arising from the
muscularis mucosa or muscularis propria. Again, the yield is likely not to be high, especially with
such a small lesion. If the clinical and endosonographic suspicion is a GIST, EUS FNA can be
diagnostic in approximately 62% of patients and suspicious in another 22%.5 What I do depends
largely on the size, layer of origin, and echogenicity. If the lesion is hypoechoic, arising from the
muscularis mucosa, has minimal invasion into the submucosa, or if it is a submucosal lesion with
clear margins from the muscularis propria and it is less than 2 cm, then I attempt to resect it,
after performing a submucosal injection to lift the lesion. I then resect it with a snare, cap-assisted
endoscopic mucosal resection, or with a Duette-assisted endoscopic mucosal resection. If the
lesion is arising from the muscularis propria, then I attempt EUS FNA or biopsy.
How Should I Manage the Patient With Pain After Taking Motrin?   217

Management
If there is a tissue diagnosis of this lesion, then management is dependent on the diagnosis. If
the lesion is malignant, or potentially malignant, such as a GIST, glomus tumor, carcinoid, lym-
phoma, or a metastatic lesion, then surgery and oncology evaluation is warranted. GISTs between
1 to 2 cm that are asymptomatic and have no high-risk features (eg, irregular borders, cystic spaces,
ulceration, echogenic foci, and heterogeneity in appearance) may be followed at 6- to 12-month
surveillance interval6 vs surgical resection.
If the lesion is benign, and was not excised, then management depends on the diagnosis. If it is
a lipoma or pancreatic rest, no further follow-up is warranted unless it became symptomatic. Other
benign diagnoses, such as leiomyoma and granular cell tumor, usually require an initial follow-up
examination in 1 year, which can then be lengthened if no change occurs.

Follow-Up of Undiagnosed Lesions


I found that most subepithelial lesions < 3 cm do not change during a median of 23 months.7
Many of these subepithelial lesions arising from the muscularis propria or muscularis mucosa
prove to be GISTs, which can have malignancy potential. Thus, we would follow up on this 1-cm
lesion with an EGD/EUS with sampling in 1 year. If there is no change, we would increase the
surveillance interval. If the lesion becomes symptomatic, increases in size, or shows worrisome
changes, especially on EUS, such as irregular borders, cystic spaces, and heterogeneity in appear-
ance, we would recommend endoscopic or surgical resection.

References
1. Rosch T, Kapfer B, Will U, et al. Accuracy of endoscopic ultrasonography in upper gastrointestinal submucosal
lesions: a prospective multicenter study. Scand J Gastroenterol. 2002;37(7):856-862.
2. Hwang JH, Saunders MD, Rulyak SJ, Shaw S, Nietsch H, Kimmey MB. A prospective study comparing endoscopy
and EUS in the evaluation of GI subepithelial masses. Gastrointest Endosc. 2005;62(2):202-208.
3. Hunt GC, Smith PP, Faigel D.O. Yield of tissue sampling for submucosal lesions evaluated by EUS. Gastrointest
Endosc. 2003;57(1): 68-72.
4. Cantor MJ, Davila RE, Faigel DO. Yield of tissue sampling for subepithelial lesions evaluated by EUS: a comparison
between forceps biopsies and endoscopic submucosal resection. Gastrointest Endosc. 2006;64(1): 29-34.
5. Hoda KM, Rodriguez SA, Faigel DO. EUS-guided sampling of suspected GI stromal tumors. Gastrointest Endosc.
2009;69(7):1218-1223.
6. Sepe PS, Brugge WR. A guide for the diagnosis and management of gastrointestinal stromal cell tumors. Nat Rev
Gastroenterol Hepatol. 2009;6(6):363-371.
7. Gill KR, Camellini L, Conigliaro R, et al. The natural history of upper gastrointestinal subepithelial tumors: a multi-
center endoscopic ultrasound survey. J Clin Gastroenterol. 2009;43(8):723-726.
SECTION V
CAPSULE ENDOSCOPY
47 QUESTION

IN TRUE OBSCURE GI BLEEDING, WHAT SHOULD


I DO IF A CAPSULE ENDOSCOPY AND UPPER AND
LOWER ENDOSCOPIES ARE UNREVEALING?

Anupam Singh, MD and David R. Cave, MD, PhD

Obscure gastrointestinal bleeding (OGIB) is defined as bleeding from the gastrointestinal (GI)
tract that persists or recurs without an obvious etiology after an upper endoscopy and colonoscopy.
It accounts for approximately 5% of all GI bleeding and identifying a source can be time inten-
sive and frustrating for both the clinician and the patient. OGIB is further classified as obscure
occult or obscure overt bleeding. Obscure overt bleeding manifests as visible blood (hematochezia
or melena), whereas obscure occult bleeding manifests as a positive fecal occult blood test or iron
deficiency anemia. In this chapter, we will consider the former condition.
Before discussing the techniques involved in localizing OGIB, it is worth emphasizing a few
points about capsule interpretation. First, the current generation of capsule endoscopies do not
visualize the entire small bowel mucosal surface. A repeat capsule reveals an approximate 10%
increase in clinically significant lesions. Second, the duodenum needs very careful examination
for angioectasias and plumes of blood, which may be visible on only a single frame; therefore, read
the video by advancing the video frame by frame using the mouse scroll wheel, at least through
the duodenum. Third, if the video shows red blood or clots at several points along the length of
the small intestine, interspersed by normal mucosa or melena, it does not reflect several bleeding
points. It reflects the fact that bleeding is intermittent and that the more blood there is in the
lumen, the slower it gets degraded to black hematin. The small bowel is analogous to a peristaltic
pump. The likely source of bleeding is at the very beginning of the bleeding seen on video capsule
endoscopy (VCE).
In approximately 90% of patients with obscure overt GI bleeding, the source is in the small
bowel. The remainder of cases are due to missed lesions in either the upper or lower GI tract.
Therefore, a repeat evaluation with esophagogastroduodenoscopy (EGD) and colonoscopy may

Leung J, Lo SK, eds. Curbside Consultation in Endoscopy:


221 49 Clinical Questions, Second Edition (pp 221-223)
© 2014 SLACK Incorporated
222  Question 47

be worthwhile. This is particularly true if the patient is referred from a source that is not familiar
to your center. The missed lesions may be Cameron’s erosions, ulcers in the duodenal fornices,
angioectasias, and gastric Dieulafoy’s lesions. Gastric varices may be missed because they may
have been collapsed if the initial endoscopy was performed without complete resuscitation. Other
rare causes include hemobilia, hemosuccus pancreaticus, and aortoenteric fistulas (particularly in
patients with a history of abdominal aortic aneurysm repair), and the use of a side-viewing duode-
noscope may increase the diagnostic yield in the appropriate clinical setting.
If the cause of bleeding remains elusive, we would then recommend a repeat VCE, ideally as
an urgent procedure as soon as there is evidence of a recurrent overt bleeding episode. Patients
should be instructed to go to the emergency room or contact the primary gastroenterologist at the
onset of a bleeding episode so that a VCE can be placed quickly and before a further EGD and
colonoscopy are performed. It is the delay caused by doing these procedures that provides time for
the bleeding to stop, hence a negative VCE. Three recent studies1,2,3 have demonstrated a higher
yield of the VCE when it is placed closer to the time of bleeding, resulting in a higher rate of
diagnostic localization and successful therapeutic interventions.
When the etiology or anatomical location of the bleed remains elusive, the next step in the diag-
nostic algorithm should be a multidetector-row computed tomography angiography (MDCTA).
This is supported by various previous studies and also a recent 5-year prospective study4 in which
the overall sensitivity, specificity, and accuracy of MDCTA in patients with active GI bleed was
86%, 100%, and 88.5%, respectively. Depending on the results of the MDCTA, further therapeu-
tic interventions with conventional mesenteric angiography or deep enteroscopy can be planned.
Angiography will localize the bleeding lesion if the rate of bleeding is at least 0.5 mL/minute and
can offer therapy with vasopressin infusion or coil embolization. The latter offers a long-term solu-
tion, but there is a risk of creating small bowel ischemia and perforation.
Deep enteroscopy should be done if there is a positive finding noted on VCE or MDCTA.
Deep enteroscopy can be performed by double-balloon endoscopy (DBE), single-balloon endos-
copy (SBE), or the newer method of spiral enteroscopy (SE). The techniques are based on dif-
ferent designs of overtube, which fit over a newer generation of flexible thin enteroscopes. Each
is designed to minimize looping of the small bowel, while pleating it back over the scope and
overtube, analogous to a shower curtain on a rod. The method of choice is center-specific and is
dependent on the availability of equipment and physician experience. A recent review that focused
on the analysis of 68 studies5 found that the procedural characteristics (mean insertion depth,
diagnostic yield, and adverse events) were comparable for DBE, SBE, and SE. In our center,
we prefer the antegrade SE, when clinically indicated, due to the shorter procedure time and its
relative ease to use. Also, a recent prospective study6 has shown that SE is safe and effective in
reducing the incidence of overt bleeding, with a decrease in blood transfusion requirements and
decrease in need for additional therapeutic procedures. The 118-cm overtube is compatible with
enteroscopes that are 200 cm long and between 9.1 and 9.5 mm in diameter. The overtube has a
coupling device that affixes it to the scope, while permitting rotation of the overtube. By rotat-
ing the overtube clockwise, the small bowel is pleated onto the overtube and the enteroscope is
advanced.
Intraoperative enteroscopy (IOE) is used as a method of last resort in conjunction with the
findings on VCE if deep enteroscopy fails to identify the bleeding lesion. This is supported by
a study from the American Journal of Surgery from 20097 in which the authors noted that among
the 15 patients with VCE positive for small-bowel lesions, laparotomy and IOE yielded localiza-
tion and treatment (surgical n = 11 and endoscopic n = 2) guidance for 13 of 15 (87%) lesions.
At median 19-month follow-up, 3 bleeding recurrences (3 of 15 [20%]) were recorded, resulting
in a 73% therapeutic efficacy of IOE. Although some endoscopists use a peroral approach, we
advocate introducing the endoscope via a small-bowel enterotomy, using a sterilized enteroscope
sheathed in a plastic sleeve to maintain a sterile operative field. One significant drawback of IOE
What Should I Do if Endoscopies Are Unrevealing?   223

is mucosal trauma and bleeding caused by pleating of the bowel over the endoscope, thus mak-
ing differentiation of artifact vs culprit lesions difficult. Also, given the inherent higher risk of
IOE complications, including mesenteric tears, perforation, hematoma, infection and ileus, we
typically recommend IOE in cases where other diagnostic and therapeutic measures have already
been attempted and failed. IOE and surgical intervention should not be used if no target has been
defined by imaging methods.
The role of small-bowel follow through and enteroclysis has largely been replaced by the mul-
tiphase CT enterography, which is increasingly being used in the evaluation of obscure GI bleed-
ing. It is particularly useful in identifying and localizing small bowel masses8 and in patients with
suspected small bowel stricturing due to Crohn’s disease, radiation injury, or use of nonsteroidal
anti-inflammatory drugs (NSAIDs).
If the above measures do not localize the source of the bleeding, the origin may be very hard
to localize. It is most likely due to a submucosal vessel/s that intermittently ruptures and is visible
only when actively bleeding; we refer to these as punctate bleeding sites. These are histologically
distinct from Dieulafoy’s lesions and can be found only by repeated urgent VCE. Treatment is by
endoscopic means or resection.

Conclusion
Obscure bleeding is not as obscure as it was a decade ago, but it remains challenging for patient
and clinician alike.

References
1. Lecleire S, Iwanicki-Caron I, Di-Fiore A, et al. Yield and impact of emergency capsule enteroscopy in severe
obscure-overt gastrointestinal bleeding. Endoscopy. 2012;44(4):337-342.
2. Yamada A, Watabe H, Kobayashi Y, Yamaji Y, Yoshida H, Koike K. Timing of capsule endoscopy influences the
diagnosis and outcome in obscure-overt gastrointestinal bleeding. Hepatogastroenterology. 2012;59(115):
676-679.
3. Singh A, Marshall C, Chaudhuri B, et al. Timing of video capsule endoscopy relative to overt obscure GI bleeding:
implications from a retrospective study. Gastrointest Endosc. 2013 May;77(5):761-766.
4. Sun H, Jin Z, Li X, et al. Detection and localization of active gastrointestinal bleeding with multidetector row
computed tomography angiography: a 5-year prospective study in one medical center. J Clin Gastroenterol.
2012;46(1):31-41.
5. Lenz P, Domagk D. Double- vs. single-balloon vs. spiral enteroscopy. Best Pract Res Clin Gastroenterol. 2012;26(3):303-
313.
6. Williamson JB, Judah JR, Gaidos JK, et al. Prospective evaluation of the long-term outcomes after deep
small-bowel spiral enteroscopy in patients with obscure GI bleeding. Gastrointest Endosc. 2012;76(4):771-778.
doi: 10.1016/j.gie.2012.05.025.
7. Douard R, Wind P, Berger A, et al. Role of intraoperative enteroscopy in the management of obscure gastointes-
tinal bleeding at the time of video-capsule endoscopy. Am J Surg. 2009;198(1):6-11.
8. Huprich JE, Fletcher JG, Fidler JL, et al. Prospective blinded comparison of wireless capsule endoscopy and multi-
phase CT enterography in obscure gastrointestinal bleeding. Radiology. 2011;260(3):744-751.
48 QUESTION

WHAT IS A RELIABLE METHOD TO ESTIMATE


THE LOCATION OF A LESION FOUND ON
CAPSULE ENDOSCOPY?

Lucinda A. Harris, MS, MD and Jonathan A. Leighton, MD

There is a feeling of intense satisfaction when reading a video capsule endoscopy (VCE) study
when you locate the lesion(s) in the small bowel that have been causing obscure gastrointestinal
bleeding or in discovering ulcerations consistent with Crohn’s disease to refine a patient’s therapy.
However, that feeling can soon be replaced by a sinking feeling in the pit of your stomach when
you realize that it is necessary to localize this finding within the 600 cm of small intestine. At
that moment, figuring out exactly where that small bowel lesion is can feel a bit overwhelming.
However, localizing the area of interest can be a key step in planning further studies, medical
therapy, or surgical intervention.
There are 4 CE systems FDA-approved in the United States: the PillCam SB2 (Given Imaging
Ltd), the Endo Capsule (Olympus America, Inc), the OMOM capsule (Jinshan Science and
Technology Co, Ltd), and the MiroCam (IntroMedic). A fifth system, CapsoVision (CapsoVision,
Inc) is only approved in Europe and is in the process of seeking FDA approval. Only the PillCam
SB2 has a “localization module.”1 Therefore, localization of lesions identified by VCE can be
extremely challenging, and there has not been a significant advance in this area that is widely
clinically available. However, plans exist for future developments. So what currently helps us to
determine where a capsule is in the GI tract? There are several methods for estimating localiza-
tion of a lesion. Anatomic landmarks and keeping track of the elapsed transit time with relation
to viewing the landmarks are the 2 features from a technological point of view that can be very
important. Certainly lesions that are close to the duodenal bulb or close to the ileum and the
ileocecal valve are easiest to locate. Generally, lesions seen within 30 to 60 minutes of the pylorus
are usually able to be seen by push enteroscopy or antegrade double-balloon enteroscopy (DBE). 2,3

Leung J, Lo SK, eds. Curbside Consultation in Endoscopy:


225 49 Clinical Questions, Second Edition (pp 225-228)
© 2014 SLACK Incorporated
226  Question 48

Figure 48-1. SensorBelt system for the Given Imaging capsule system. (Images courtesy of Given
Imaging Ltd. PillCam and RAPID are trademarks of Given Imaging Ltd.)

Likewise, if the lesion is seen just prior to the capsule entering the ileocecal valve, it will be reach-
able by retrograde DBE or possibly by colonoscopy.
What happens if the lesion is not near an anatomic landmark? The Given capsule (PillCam2)
possesses a “localization module.” In this capsule, the software has a built-in localization method.
To understand this, it is important to understand the sensor array system. In the past, the sensor
array system was applied to the patient’s abdomen—8 antennas taped to the belly in a predeter-
mined distribution. Each antenna picked up signals from the capsule and carried it to the recording
device. This system has been replaced by the SensorBelt (Given Imaging Ltd) (Figure 48-1). The
SensorBelt is a belt worn around the patient’s waist over clothing. It employs easy-fasten straps for
quick adjustment and removal. The sensors incorporated within the belt eliminate the need for the
sensor sleeves and reduce patient prep time and equipment maintenance. The recorder, equipped
with a battery pack, is then placed in a belt around the patient’s waist. The patient can go about
his or her daily activities and return in 8 hours so that the sensor belt and recording device can be
removed. Signals are then downloaded to the workstation and are ready for physician interpreta-
tion. The sensor belt requires that the patient’s body mass index (BMI) be < 43.3 kg.
The localization module assists physicians in locating the physical findings identified by the
capsule. The camera chip in the capsule obtains 2 images/second and transmits its data via radio-
frequency to a recording device.4 The strength of the signal emitted by the capsule and received by
the 8 antennas in the sensor belt help in determining the true localization of the visible finding.
A graphic trajectory of the capsule as it passes through the gastrointestinal (GI) tract is calculated
from the output of the localization module and appears in the lower left-hand corner of the read-
ing software. The module, in calculations reminiscent of college calculus, calculates and presents
GI transit times, based on the labeled transit times of certain anatomical features, eg, entrance
into the stomach and pylorus and passage into the ileocecal valve as well as the strength of the
radiofrequency (RF) signal. The image data and the levels of the signals received from the sensor
array are recorded together. The sensors are arranged in a predefined manner and the theory of
localization is that the closest sensor receives the strongest signal and the location is determined
relative to the umbilicus, as well as using the sensors with the 3 strongest signals to triangulate
the position of the capsule over the abdominal wall to determine into which of the 4 quadrants the
capsule is located. Figure 48-2 demonstrates a RAPID (Given Imaging Ltd) main screen with the
image of the finding plus the localization feature in the lower left-hand corner. The localization
process is a 2-dimensional (2D) estimated location of the capsule based also on the time. As the
various anatomic landmarks are seen and captured by the reader, the color of the line on the figure
What Is a Reliable Method to Locate a Lesion Found on Endoscopy?  227

Figure 48-2. RAPID main screen picture


of image of a small bowel polyp with the
localization figure in the lower left hand cor-
ner. (Images courtesy of Given Imaging Ltd.
PillCam and RAPID are trademarks of Given
Imaging Ltd.)

changes. The thumbnail feature of capturing the image allows you to label the apparent location of
the finding. In the RAPID imaging system, the stomach is light blue, the small intestine is green,
and the colon is brown (see Figure 48-2).
Although the localization module may have limitations for individuals in whom the capsule
remains too long in the stomach, the majority of studies have verified the accuracy of this soft-
ware. Initially, the software was tested by studying healthy volunteers. In a study that compared
measurements on 17 healthy volunteers (at least 4 repetitions of capturing capsule images in
each individual), the localization calculation was compared with the location of the capsule on
fluoroscopic image.5 Both calculations were made relative to the umbilicus. At least 62% of the
calculations were within 4 cm or better of the fluoroscopic image. The cumulative percentage of
better than 6-cm accuracy was 87%, and only 5% of the samples were at a significantly higher
distance than predicted by the localization module. Another study of 75 patients with a variety of
small-bowel lesions (predominantly obscure gastrointestinal bleeding, but also Crohn’s disease and
other miscellaneous lesions) also verified the accuracy of the localization software. Soquet et al
compared an upright abdominal radiograph localization at 2 hours after swallowing the capsule
with the localization algorithm.6 In this case, there was 65% agreement between the radiograph
and the automatic device for the localization of the capsule. The most common discrepancy in
18 patients was that on the localization software, the capsule appeared to be in a lower position
than on the abdominal x-ray. Yet another study verified that the capsule was useful at determining
whether DBE should be performed and what approach would be most fruitful (oral vs transanal).7
In this study of 164 patients with primarily obscure GI bleeding, the lesions detected by capsule
were reached by DBE except in 2 cases, giving a positive predictive value of almost 95% for capsule
to make a correct indication for push and pull enteroscopy (PPE). The choice of the insertion route
of the endoscope for PPE was determined according to the value of the time in minutes for transit
of the capsule from the pylorus to the lesion of interest divided by its total transit time through the
small bowel, which in turn was defined as the time elapsing between the passage of the capsule
through the pylorus and its arrival into the cecum as determined by the software. This number
yielded an “index of location” and index numbers closer to 1 were correlated with lower lesions. A
value of 0.75 was determined to be the cutoff for determining whether patients needed approach
from the anal as opposed to the oral route. Follow-up of these patients at 9 months demonstrated
that doing DBE had positively influenced the management of greater than 90% of the patients.
As outlined previously, accuracy of the current localization algorithm is very good when com-
bined with time measurements and identification of landmarks, but enhancements are needed.
Recently, researchers have described the use of color image analysis in differentiating between
normal healthy esophagus, stomach, small intestine, and colon.8 This technology would not only
228  Question 48

aid localization but also help facilitate and actually speed up review of the capsule study. Further
advances are being tested in a capsule that incorporates a small magnet and creates a static mag-
netic field. This model also uses 3-axis sensors that allow for a better 3-dimensional, rather than
2-dimensional, localization.9 Also, both Given Imaging and Olympus are exploring external
magnet-directed capsule maneuvering.10 In these models, the capsule is steered via a remote com-
puter and this technique has allowed localization of the capsule in the gut to within 1 to 2 mm. A
very exciting development is the recent tagging module, real-time localization tool.11 This module
consists of a biocompatible micro tag, compressed spring with a string latch, and thermal igniter.
The module can be integrated with the wireless capsule endoscopy and activated using an external
trigger signal. When activated, the micro tag releases instantly and penetrates the mucosal layer of
GI tract in the region of interest. Radiograph imaging can then be used to detect the location of
micro tag embedded in GI tract wall. The radiopaque micro tags provide the position information
of the involved area to facilitate further clinical procedures. These systems are still in the develop-
ment phase, but hold much promise for further accuracy in localization of the lesions found on
capsule endoscopy.

References
1. Fischer D, Schreiber R, Levi D, Eliakim R. Capsule endoscopy: the localization system. Gastrointest Endscopy Clin N
Amer. 2004;14:25-31.
2. Lewis BS. The utility of capsule endoscopy in obscure gastrointestinal bleeding. Tech Gastrointest Endoscopy.
2003;5:115-120.
3. Given Imaging Ltd. Frequently Asked Questions. Available at http://www.givenimaging.com. Accessed March
6th, 2014.
4. Qureshi WA, Willingham F, Anand B. Localizing the lesion by capsule endoscopy: newer techniques in improving
accuracy. Am J Gastroenterol. 2004;99:S67 (Ab 202).
5. Costamgna G, Shah SK, Riccioni ME, et al. A prospective trial comparing small bowel radiographs and video cap-
sule endoscopy for suspected small bowel disease. Gastroenterol. 2002;123:999-1005.
6. Souquet JC, Bellecose S, Belbouab S, et al. Prospective evaluation of the automatic localization system of the
videocapsule during small intestine exploration. 4th International Conference on Capsule Endoscopy. 2005;
Abstract 118:237.
7. Gay G, Delvaux M, Fassler I. Outcome of capsule endoscopy in determining indication and route for push-and-pull
enterocopsy. Endoscopy. 2006;38:49-58.
8. Mackiewicz M, Berens J, Fisher M. Wireless capsule endoscopy color video segmentation. IEEE Trans Med Imaging.
2008;27:1769-1781.
9. Hu C, Meng HQ-H, Mandal M. Efficient linear algorithm for magnetic localization and orientation in capsule
endoscopy. Conf Proc IEEE Eng Med Biol Soci. 2005;7:7143-7146.
10. Fisher L, Hassler WL. New vision in video capsule endosopy: current status and future directions. Nat Rev
Gastroenterol Hepatol. 2012;9(7):392-405.
11. Chandrappan J, Ruiqi L, Su N, et al. Tagging module for lesion localization in capsule endoscopy. Conf Proc IEEE
Eng Med Biol Soc. 2010;1890-1893.
49 QUESTION

IF I SUSPECT A SMALL BOWEL LESION, HOW DO I CHOOSE


AMONG SMALL-BOWEL SERIES, CAPSULE ENDOSCOPY,
CT ENTEROGRAPHY, AND DOUBLE-BALLOON ENTEROSCOPY
AS THE DIAGNOSTIC TEST?

Ravi K. Ghanta, MD and


Jamie S. Barkin, MD, MACP, MACG, FASGE, AGAF

The classic approach to a patient with a suspected small bowel lesion has been to use small
bowel series imaging as an initial diagnostic choice. However, recent advances in radiological
and endoscopic technologies have provided a wider range of choices. Each test or procedure has
its own availabilities, limitations, benefits, and drawbacks; therefore, the appropriate diagnostic
choice should depend on the suspected problem, as well as the resources available to the clinician.
The most common causes of obscure gastrointestinal bleeding (OGIB) in Western countries are
vascular lesions, including angioectasias, Dieulafoy lesions, and varices. The common neoplastic
lesions in the small bowel include adenocarcinoma, carcinoid, gastrointestinal stromal tumor
(GIST), lymphoma, adenoma, and metastasis. Inflammatory lesions of the small bowel include
celiac, Crohn’s, and drug-induced mucosal injury.
A common choice in initial diagnostic imaging is the small-bowel series (SBS), which is widely
available and easy to perform. However, because the orally ingested contrast is primarily designed
to create a radiological mucosal relief pattern, its diagnostic usefulness is limited to large endolu-
minal growths or irregularities. Patients in whom large masses or cancers, strictures, or ulcerations
are suspected are the best candidates for this study. Even in this group, its overall diagnostic yield
is low; therefore, a negative study does not rule out the possibility that a lesion or stricture might
still exist. For example, a recent meta-analysis comparing the yield of SBS to capsule endoscopy
for nonstricturing small bowel Crohn’s disease found that SBS had a diagnostic yield of only 23%.1
Newer radiological choices for small-bowel imaging include computed tomography enterog-
raphy (CTE) and magnetic resonance enterography (MRE). Enteroclysis is less favored over
enterography by patients because it involves placement of nasojejunal tube for insertion of contrast.
CTE uses both a neutral oral contrast and an intravenous contrast to create cross-sectional images
of the abdomen, highlighting the small bowel. CTE provides excellent views of the mucosa of the
small bowel, the bowel wall, and the adjacent organs. In addition, CTE’s capability for multiphasic
imaging also allows the radiologist to evaluate for vascular anomalies. Consequently, CTE can

Leung J, Lo SK, eds. Curbside Consultation in Endoscopy:


229 49 Clinical Questions, Second Edition (pp 229-233)
© 2014 SLACK Incorporated
230  Question 49

be used in patients with both suspected endoluminal masses and suspected vascular anomalies.
However, CTE is not without its drawbacks. First, its availability is usually limited to tertiary
centers. Second, CTE is costly and involves significant radiation exposure to the patient. Finally,
CTE may not be a consideration in patients with iodine allergies or in whom vascular access is
unavailable. The sensitivity and specificity of CTE and CT enteroclysis are comparable. CT
enteroclysis was compared with enteroscopy and surgery in the detection of small bowel tumors.
CT enteroclysis was found to have a sensitivity of 84.7% and a specificity of 96.9%, making it an
excellent imaging tool. 2
An advantage of MRE or MR enteroclysis is that it is not associated with significant radiation
exposure compared to CTE, and its results are comparable. MR enteroclysis had a sensitivity of
91% and specificity of 95%, respectively, when it was performed either to identify or exclude small
bowel neoplasm. The presence of long solitary nonpedunculated lesions, mesenteric fat infiltra-
tion and enlarged mesenteric lymphadenopathy were associated with malignancy.3 Both CTE and
MRE can identify mucosal changes, but not arteriovenous malformations, which are a common
source of bleeding within the small bowel.
The endoscopic procedures for evaluating small-bowel lesions include wireless capsule endos-
copy (WCE), single-balloon enteroscopy (SBE), double-balloon enteroscopy (DBE), and spiral
enteroscopy (SE). WCE is a relatively noninvasive procedure that provides direct mucosal visu-
alization of the small bowel. Therefore, it is useful in evaluating endoluminal lesions, vascular
anomalies, as well as subtle mucosal changes such as villous blunting. WCE is now widely avail-
able. The presence of an implanted defibrillator or a suspected small-bowel obstruction is no lon-
ger a relative contraindication to perform WCE.4 Pregnancy is the only absolute contraindication
for performing WCE. Overall, for a diagnostic small bowel mucosal or endoluminal evaluation,
WCE provides a superior yield to both traditional imaging and push enteroscopy. Thus, we feel
that WCE is the gold standard and may be the initial test of choice in selected patients with
OGIB and in patients with a suspected mucosal small-bowel lesion. Numerous studies have been
conducted that support this assertion, but perhaps the most interesting are those that compare
the diagnostic yields of different tests in the investigation of patients with OGIB. WCE has been
reported to allow visualization of the entire small bowel in 79% to 90% of patients with a diag-
nostic yield of 38% to 83% in OGIB.5 The major advantages of WCE are that it is safe, simple,
noninvasive, and may allow visualization of the entire small bowel. The drawbacks of WCE are
the need for bowel preparation prior to the procedure, operator dependence, low overall diagnostic
yield, and inability to offer treatment at the same time. Although WCE cannot provide therapeu-
tic intervention at the time of the procedure, it can help to decide on the preferred route of balloon
enteroscopy, either orally or anally, to approach the identified lesion.6
Triester et al1 performed a meta-analysis of the diagnostic procedures for obscure GI bleeding
using 20 different studies involving over 500 patients. They found that WCE was superior to both
push enteroscopy and small-bowel barium radiography for diagnosing significant small-intestine
disease. The incremental yield or IY (defined as the yield of WCE minus the yield of the compara-
tive modality) of WCE over push enteroscopy and small-bowel radiography was greater than or
equal to 30% for each. Interestingly, the Triester et al article also included studies that compared
WCE to nontraditional imaging modalities, such as CTE, mesenteric angiogram, and small
bowel magnetic resonance imaging (MRI) (one study for each different modality). Of these, only
the study involving small-bowel MRI rose to the level of statistical significance, and in that one
study WCE was again found to be superior (IY = 36%).
Direct visualization of the small bowel can be achieved by push enteroscopy, single- or double-
balloon-assisted enteroscopy (BAE), or spiral enteroscopy. Balloon enteroscopy involves sequen-
tially inflating 1 or 2 separate balloons on an endoscope to allow the physician to telescope the
small bowel over the scope, thereby advancing the enteroscope through the small bowel. Balloon
enteroscopy can be performed either orally or anally, depending on the result of the preliminary
If I Suspect a Small Bowel Lesion, How Do I Choose Among Diagnostic Tests?   231

study that localizes the lesion. In the evaluation of OBIG, the diagnostic yield of BAE ranged
from 43% to 81% and the therapeutic success rate was 43% to 84%.7 The major advantage of
BAE is its therapeutic ability. However, its major disadvantage is that the entire small bowel is
not visualized in a single procedure. Overall, the rate of complication has been noted to be 1.2%,
including pancreatitis and perforation.8 Most perforations are noted in patients who underwent
polypectomy during DBE. The availability of DBE is limited and it is an expensive and lengthy
procedure, often lasting up to 3 hours. Another drawback of BAE is uncertainty over the depth
of insertion during the procedure. The efficacy of DBE was compared with capsule endoscopy
(CE) in a retrospective analysis of patients who underwent both procedures for evaluating small
bowel mass lesions causing OGIB. DBE is superior in identifying small bowel lesions missed by
capsule endoscopy.9
The data on spiral enteroscopy are limited, but initial reports suggest decreased procedural
time, similar depth of small bowel evaluation, and lower complication rate compared with BAE.
Diagnostic yield of SE was compared with WCE by Buscaglia et al.10 SE was moderately effective
(57%) in identifying small-bowel pathology. Arteriovenous malformations observed on capsule
endoscopy were seen in 60% of the SE.
OGIB is a common presentation of small-bowel pathology. OGIB is defined as bleeding of
unknown origin after an initial esophagogastroduodenoscopy (EGD) and colonoscopy. OGIB is
further divided into obscure overt (visible) and obscure occult (without evidence of bleeding). Various
algorithms have been published for the evaluation of OGIB. In the evaluation of obscure occult
GIB, we recommend CE in asymptomatic patients as an initial test of choice, followed by deep
enteroscopy if CE is positive or if anemia persists. Repeating either colonoscopy or EGD, depend-
ing on the presentation, appears to be a reasonable initial choice, followed by CE in the evaluation
of overt OGIB. This was demonstrated in a study by Fry et al, in which a non-small bowel lesion
definitely explaining the source of bleeding was identified in 24% undergoing DBE. The small-
bowel lesion identified was within reach of traditional endoscopy.11

Conclusion
Our choice of initial modality for diagnosing small-bowel lesions depends on clinical suspicion
as to the type of lesion (Figure 49-1). Radiological and endoscopic procedures, including CE, are
complimentary procedures. Radiological studies should be considered more strongly when there is
a concern for lesions causing symptoms other than bleeding, especially when potentially obstruct-
ing lesions are suspected. Endoscopic studies should be considered when the direct visualization
of the mucosa is desired or if a therapeutic option is needed. In a retrospective study from Honda
et al, the combination of contrast-enhanced CT scan and WCE had a diagnostic yield similar to
DBE for evaluation of small-bowel tumors. In addition, DBE allowed for histological diagnosis
and therapy in the majority of cases.12 Further workup will depend on the results of the initial
diagnostic modality, and a combination of modalities will improve diagnostic yield in most
patients.
232  Question 49

Figure 49 -1. Approach to diagnosing and treating patients with OGIB.

References
1. Triester S, Leighton JA, Leonitiadis GI, et al. A meta-analysis of the yield of capsule endoscopy compared to
other diagnostic modalities in patients with non-stricturing small bowel Crohn’s disease. Am J Gastroenterol.
2006;101:954-964.
2. Pilleul F, Penigaud M, Milot L, Saurin JC, Chayvialle JA, Valette PJ. Possible small bowel neoplasms: contrast-
enhanced and water-enhanced multidetector CT enteroclysis. Radiology. 2006;241(3):796-801.
3. Van Weyenberg SJ, Meijerink MR, Jacobs MA, et al. MR enteroclysis in the diagnosis of small-bowel neoplasms.
Radiology. 2010;254(3):765-773.
4. Vanderveldt HS, Barkin JS. Capsule endoscopy: a primer for the endoscopist: ten ways to prevent capsule
retention and delayed passage. Tech Gastrointest Endosc. 2006;8:164-168.
5. Rondonotti E, Villa F, Mulder CJ, Jacobs MA, de Franchis R. Small bowel capsule endoscopy in 2007: indications,
risks and limitations. World J Gastroenterol. 2007;13:6140-6149.
6. Li X, Chen H, Dai J, Gao Y, Ge Z. Predictive role of capsule endoscopy on the insertion route of double-balloon
enteroscopy. Endoscopy. 2009;41:762-766.
7. ASGE Standards of Practice Committee, Fisher L, Lee Krinsky M, Anderson MA, et al. The role of endoscopy in the
management of obscure GI Bleeding. Gastrointest Endosc. 2010;72(3):471-479.
8. Möschler O, May A, Müller MK, Ell C; German DBE Study Group. Complications in and performance of DBE: results
from a large prospective DBE database in Germany. Endoscopy. 2011;43:484-489.
9. Ross A, Mehdizadeh S, Tokar J, et al. Double balloon enteroscopy detects small bowel mass lesions missed by
capsule endoscopy. Dig Dis Sci. 2008;53:2140-2143.
10. Buscaglia JM, Richards R, Wilkinson MN, et al. Diagnostic yield of spiral enteroscopy when performed for the
evaluation of abnormal capsule endoscopy findings. J Clin Gastroenterol. 2011;45(4):342.
11. Fry LC, Bellutti M, Neumann H, Malfertheiner P, Mönkemüller K. Incidence of bleeding lesions within reach of
conventional upper and lower endoscopes in patients undergoing double-balloon enteroscopy for obscure
gastrointestinal bleeding. Aliment Pharmacol Ther. 2009;29:342-349.
If I Suspect a Small Bowel Lesion, How Do I Choose Among Small Bowel Series?   233

12. Honda W, Ohmiya N, Hirooka Y, et al. Enteroscopic and radiologic diagnoses, treatment, and prognoses of small-
bowel tumors. Gastrointest Endosc. 2012;76(2):344-354.
FINANCIAL DISCLOSURES

Dr. Harry Aslanian has no financial or proprietary interest in the materials presented herein.

Dr. Farzan Fahrtash Bahin has no financial or proprietary interest in the materials presented
herein.

Dr. Jamie S. Barkin is a member of the speaker's bureau for TAP Pharmaceuticals, Novartis, and
AstraZeneca.

Dr. Todd H. Baron has not disclosed any relevant financial relationships.

Dr. Milan Bassan has no financial or proprietary interest in the materials presented herein.

Dr. Michael Bourke has no financial or proprietary interest in the materials presented herein.

Dr. William R. Brugge was a consultant for Boston Scientific Corporation and a consultant for
ERBE USA, Incorporated.

Dr. David R. Cave was a consultant and has received research support from Olympus America,
Inc. He he also received research support from CapsoVision, Inc.

Dr. Francis K. L. Chan was a consultant for Lee’s Pharmaceutical Holdings. He was a consultant,
received speaker’s honoraria, and was on the steering committee for Pfizer Inc. He was also
on the Advisory Board for Elsevier (Singapore) Pte. Ltd and received speaker’s honoraria from
AstraZeneca and Takeda Pharmaceuticals (HK) Ltd.

Dr. Kenneth J. Chang has not disclosed any relevant financial relationships.

Dr. Han-Mo Chiu has no financial or proprietary interest in the materials presented herein.

Dr. Jeffrey L. Conklin has no financial or proprietary interest in the materials presented herein.

Dr. Ihab I. El Hajj has no financial or proprietary interest in the materials presented herein.
235
236  Financial Disclosures

Dr. Brintha K. Enestvedt has no financial or proprietary interest in the materials presented herein.

Dr. Francis A. Farraye has no financial or proprietary interest in the materials presented herein.

Dr. James J. Farrell has no financial or proprietary interest in the materials presented herein.

Dr. Erina Foster has no financial or proprietary interest in the materials presented herein.

Dr. Martin L. Freeman received speaking honoraria from Cook Medical Inc. and was a consultant
for Boston Scientific.

Dr. Ravi K. Ghanta has no financial or proprietary interest in the materials presented herein.

Dr. Emmanuel C. Gorospe has no financial or proprietary interest in the materials presented herein.

Dr. Rajesh Gupta has not disclosed any relevant financial relationships.

Dr. Gregory Haber has not disclosed any relevant financial relationships.

Dr. Lucinda A. Harris has no financial or proprietary interest in the materials presented herein.

Dr. Drew Ingram has no financial or proprietary interest in the materials presented herein.

Dr. Laith H. Jamil has no financial or proprietary interest in the materials presented herein.

Dr. Charles J. Kahi has no financial or proprietary interest in the materials presented herein.

Dr. Michael L. Kochman was a consultant for BSC, Covidien, Dark Canyon Labs, and Cook
Medical Inc. He has sold patents to Cook Medical Inc. and his spouse has Merck salary and stock.

Dr. Paul Kortan has no financial or proprietary interest in the materials presented herein.

Dr. Richard A. Kozarek has no financial or proprietary interest in the materials presented herein.

Dr. John G. Lee has received honorarium from Cook Medical Inc. and Novartis.

Dr. Jonathan A. Leighton was a consultant and researcher for Given Imaging Ltd, a consultant for
Olympus America, Inc, and a researcher for CapsoVision, Inc.

Dr. Felix W. Leung has received support from the Veterans Affairs Medical Research Funds
at Veterans Affairs Greater Los Angeles Healthcare System and an American College of
Gastroenterology Clinical Research Award (FWL).

Dr. Joseph Leung receives research and educational support from Cook Endoscopy and Olympus
America. He and Dr. Peter Cotton designed the Cotton-Leung biliary stent marketed by Cook
Endoscopy.

Dr. Zhao-shen Li has no financial or proprietary interest in the materials presented herein.

Dr. Wei-Chih Liao has no financial or proprietary interest in the materials presented herein.

Dr. Simon K. Lo has not disclosed any relevant financial relationships.

Dr. Fauze Maluf-Filho has no financial or proprietary interest in the materials presented herein.

Dr. Neel K. Mann has no financial or proprietary interest in the materials presented herein.
Financial Disclosures  237

Dr. Catherine Ngo has no financial or proprietary interest in the materials presented herein.

Dr. Michael F. Picco has no financial or proprietary interest in the materials presented herein.

Dr. Mohan Ramchandani has no financial or proprietary interest in the materials presented herein.

Dr. D. Nageshwar Reddy has no financial or proprietary interest in the materials presented herein.

Dr. Paulo Sakai has no financial or proprietary interest in the materials presented herein.

Dr. Jason B. Samarasena has no financial or proprietary interest in the materials presented herein.

Dr. Thomas J. Savides has no financial or proprietary interest in the materials presented herein.

Dr. Stuart Sherman has not disclosed any relevant financial relationships.

Dr. Anupam Singh has no financial or proprietary interest in the materials presented herein.

Dr. Sooraj Tejaswi has no financial or proprietary interest in the materials presented herein.

Dr. Anne Thai has not disclosed any relevant financial relationships.

Dr. Cara Torruellas has no financial or proprietary interest in the materials presented herein.

Dr. Kenneth K. Wang has no financial or proprietary interest in the materials presented herein.

Dr. Luo-wei Wang has no financial or proprietary interest in the materials presented herein.

Dr. Chun-Ying Wu has no financial or proprietary interest in the materials presented herein.

Dr. Andrew Yen has no financial or proprietary interest in the materials presented herein.

Dr. Won Jae Yoon has no financial or proprietary interest in the materials presented herein.

You might also like